0% found this document useful (0 votes)
44 views553 pages

2019 Sec 4 Pure Physics-12s 6

The document is an examination paper for Secondary 4 Express Physics, dated 2019, containing instructions for students and a series of multiple-choice questions covering various physics concepts. It includes details about the exam format, scoring, and specific topics such as forces, energy, and thermodynamics. The paper consists of 16 printed pages with a total of 40 questions that students are required to answer.

Uploaded by

lee.zaccary
Copyright
© © All Rights Reserved
We take content rights seriously. If you suspect this is your content, claim it here.
Available Formats
Download as PDF, TXT or read online on Scribd
0% found this document useful (0 votes)
44 views553 pages

2019 Sec 4 Pure Physics-12s 6

The document is an examination paper for Secondary 4 Express Physics, dated 2019, containing instructions for students and a series of multiple-choice questions covering various physics concepts. It includes details about the exam format, scoring, and specific topics such as forces, energy, and thermodynamics. The paper consists of 16 printed pages with a total of 40 questions that students are required to answer.

Uploaded by

lee.zaccary
Copyright
© © All Rights Reserved
We take content rights seriously. If you suspect this is your content, claim it here.
Available Formats
Download as PDF, TXT or read online on Scribd
You are on page 1/ 553

2 0 1 9

SECONDARY 4
Express Exam Paper
Pure Physics
1 Anglo-Chinese Ind SA1
2 Admiralty Sec SA2
SA2
3 Bartley Sec SA2
SA2
4 Bendemeer Sec
Sec SA2
SA2
5 CHIJ St Nicholas
Nichholas SA2
6 Dunmann High
High School
Schoool SA2
7 Methodist
Metthodist Girls
Girls SA2
8 North
Nortth Vista
Vista Sec
Sec SA2
9 Spore
Sp
pore Chinese
Chinese Girls SA2
10
1 0 Tanjong
Tanjong Katong Girl SA2
11 Unity
Unity Sec SA2
12 Whitley
Whitley Sec SA2

www.KiasuExamPaper.com
www.KiasuExamPaper.com
2
MID-YEAR EXAMINATION 2019
YEAR 4 EXPRESS
6091/01

PHYSICS PAPER 1

Thursday 13 May 2019 1 hour

INSTRUCTIONS TO STUDENTS
Write your index number in the spaces provided on the Answer Sheet.
Write in soft pencil.
Do not open this booklet until you are told to do so.
There are forty questions on this paper. Answer all questions.
For each question there are four possible answers A, B, C and D.
Choose the one you consider correct and record your choice in soft pencil on the
separate Answer Sheet.
Each correct answer will score one mark. A mark will not be deducted for a wrong
answer.
Any rough working should be done in this booklet.
Calculators are allowed for this paper.

Take g to be 10 m s-2.

This paper consists of 16 printed pages.

www.KiasuExamPaper.com
3
NAME: NO: CLASS: 4E1
ADMIRALTY SECONDARY SCHOOL

PRELIMINARY EXAMINATION 2019

SUBJECT : Physics
CODE/PAPER : 6091 / 1
LEVEL/STREAM : Secondary 4 Express
DATE : 3 September 2019
TIME : 1130h – 1230h
DURATION : 1 hour
Instructions to candidates:
Write your Name, Class and Index Number on all the work you hand in.
Do not use staples, paper clips, highlighters, glue or correction fluid.

Answer all questions.


For each question there are four possible answers A, B, C and D.
Choose the one you consider correct and record your choice on the OTAS sheet.

Marks
Section A 40

DO NOT TURN OVER THIS PAPER UNTIL YOU ARE TOLD TO DO SO.

This question paper consists of 19 printed pages including this cover page
www.KiasuExamPaper.com
4
2

Answer all questions.

1 The diagram below shows two identical spheres placed next to each other.
spheres

mm

What is the radius of each sphere?


A 2 mm B 4 mm C 8 mm D 16 mm

2 It takes 1.5 s for the pendulum to swing from X to Y.

How many complete oscillations are there in 1 minute?


A 10 B 20 C 40 D 80

www.KiasuExamPaper.com
5
3

3 :KLFKRIWKHIROORZLQJGLVWDQFHíWLPH dít) graphs represent deceleration?

4 A stone is thrown upwards from the top of a building. Which row describes the
acceleration and the velocity of the stone when it reaches maximum height?
acceleration / m/s2 velocity/ m/s
A 0 0
B 0 10
C 10 0
D 10 10

www.KiasuExamPaper.com
6
4

5 An object falls from a height of 120 m. How much time does it take to reach the
ground?

A 4.9 s B 10.0 s C 12.0 s D 15.6 s


.
6 The following diagram shows all the horizontal forces acting on a moving truck.
truck

500 N 500 N

300 N

Which of the following best describes the motion of the truck?


A The truck will stop.
B The truck will move to the right.
C The truck will move to the left.
D The truck will accelerate to the right.

7 The diagram below shows how the weight varies with mass on Planets P and Q

weight / N

mass/ kg

An object weighs 400 N on Planet P. The object is then taken to Planet Q.


Which of the following is correct?

mass of object on Planet Q / kg weight of object on Planet Q / N


A 40 200
B 40 400
C 80 200
D 80 400

www.KiasuExamPaper.com
7
5

8 A small box of mass 2.0 kg moves along a track as shown in the figure. The speeds
of the objects at point A and B are 4.0 m/s and 1.0 m/s respectively. The total
distance between A and B is 2.5 m.

A 1.4 m
1.0 m

What is the average friction acting on the box as it moves from A to B?


A 2.8 N B 5.6 N C 6.0 N D 8.0 N

9 In the diagram below, the uniform metre rule is pivoted at X and held up at the point
Y by a rope.

rope
Y X

15 cm 95 cm

Given that the weight of the metre rule is 4.0 N, calculate the tension in the rope that
is needed to ensure that the rule stays horizontal.

A 1.8 N B 2.3 N C 3.1 N D 5.1 N

10 An object is slightly displaced by an external force. When the external force is


removed, the object returns to its original position.
What state of equilibrium is the object in?

A stable B neutral C unstable D rotational

www.KiasuExamPaper.com
8
6

11 Two blocks are stacked on top of one another on a table.

top block

bottom block table

The weight of the top block is Q and the weight of the bottom block is R.
The base area of top block is X and the base area of bottom block is Y.
What is the pressure acting on the table by the blocks?

A (Q+R) / X B (Q+R) / Y C (Q-R) / X D (Q-R) / Y

12 Four holes, A, B, C and D are made on a uniform lamina. The centre of gravity of the
lamina is at G. Which one of the following shows correctly the lamina hanging freely
about each of the holes?

A B

www.KiasuExamPaper.com
9
7

13 The diagram shows four containers containing water or oil. Oil floats on water. Which
of the container will have the higher pressure at the base of the container?
A B C D

water oil

water oil

14 The diagram below shows a simple hydraulic system, where a 20 N force is acting
on piston P. Piston P has an area of 5.0 cm2 and piston Q has an area of 30.0 cm2.

piston Q

F
20 N

piston P
oil
What is the magnitude of force F?

A 4.0 N B 20 N C 120 N D 3000 N

www.KiasuExamPaper.com
10
8

15 A mercury barometer and mercury manometer are placed in the same room placed
on a top of a mountain. The manometer is connected to a gas cylinder.

metre rule in cm

connection to gas

What is the pressure of the gas?


A 35 cm Hg B 45 cm Hg C 60 cm Hg D 95 cm Hg

16 The diagram shows a thermocouple when junction X and Y are placed in melting ice
and liquid at 35 °C respectively.

+1.50 mV

X Y
Melting 35 °C
ice liquid

What is the voltmeter reading when junction X is replaced by boiling water at 100 °C?
A -2.79 mV B -1.92 mV C +1.92 mV D +2.79 mV

www.KiasuExamPaper.com
11
9

17 Which diagram represents the change in arrangement of the particles of water when it
freezes?

18 The diagram shows a container used to keep food warm for delivery.

plastic cover
and box

black inner
walls
vacuum

Which of the following explanation is incorrect?


A Plastic is a poor conductor of heat, hence heat loss will be reduced through
conduction.
B Plastic cover reduces the formation of convection current, preventing the
cooling of food contents in the container.
C Vacuum reduces heat loss due to conduction as it does not have a medium to
transfer the heat.
D Black inner wall is a poor absorber of infra-red radiation, hence does not absorb
heat from the food.

www.KiasuExamPaper.com
12
10

19 A hot piece of heating rod is immersed into a beaker of water.

heating rod

Bubbles are observed in the water at the surface area in contact with the heating rod.
Which of the following statement best explains the observation?

A Conduction of water in contact with the rod causes bubbles to be formed.


B Convection current caused by the hot rod causes bubbles to be formed.
C Evaporation of water next to the heating rod causes bubbles to be formed.
D Radiation to the water cause water to boil, causing bubbles to be formed.

20 An electrical heater is used to heat a 2 kg piece of metal from 30 °C to 40 °C. The


specific heat capacity of the metal is 720 J / kgK. The heater was turned on for 20 s
and it is known that 20 % of the energy supplied is lost to the surrounding.
What is the power rating of the heater?

A 720 W B 864 W C 900 W D 14400 W

21 Two liquids, P and Q, of the same masses are placed in a room for cooling. Their
cooling curves are shown in the diagram below.

temperature / °C

0 time / min

Which of the following statement correctly describes the two liquids, P and Q?
A Both P and Q has the same specific latent heat of fusion.
B Both P and Q have the same freezing point.
C P has a higher specific latent heat of fusion than Q.
D Q has a lower freezing point than P.

www.KiasuExamPaper.com
13
11

22 A plane mirror is inclined at 40 º to the table top. An incident ray parallel to the table
top strikes the mirror and a reflected ray is formed.
mirror

incident
ray
40° table top
What is the angle of reflection?

A 20 ° B 40 ° C 50 ° D 100 °

23 Three rays of light are incident between a glass block and air. The diagram is not
drawn to scale.

Which of the following is most likely to be the critical angle of the glass?

A 25° B 35° C 45° D 55°

24 An object is placed 35 cm from a converging lens and a real image of the same size
as the object is formed. The object is now moved so that it is 20 cm from the lens.
Which statement is correct?

A The new image is bigger and its distance from the lens is less than 35 cm
B The new image is smaller and its distance from the lens is less than 35 cm.
C The new image is bigger and its distance from the lens is greater than 35 cm
D The new image is smaller and its distance from the lens is greater than 35 cm.

www.KiasuExamPaper.com
14
12

25 A ball floats on water in a swimming pool.


wave moving to the right
ball

When the wave reaches the ball, how will the ball be displaced?

A upwards B downwards C to the left D to the right

26 Which of the following is not an application of infrared radiation?

A remote control
B ear thermometer
C night vision goggles
D sunbeds used for skin tanning

27 The diagram shows the relationship between the frequency of electromagnetic


radiation and the wavelength of the waves

frequency

wavelength

Which of the following relationship can be interpreted from the graph?

A The velocity of EM waves is a constant.


B The higher the wavelength, the higher the energy of the wave.
C Frequency is directly proportional to the energy of waves.
D Waves of higher frequency travel faster than waves of lower frequency.

www.KiasuExamPaper.com
15
13

28 A starting pistol is fired at the starting line of a race and the echo from the wall is heard
0.5 s later. The speed of sound in air is 330 m/s.
What is the distance between the starting line and the wall?

A 82.5 m B 165 m C 330 m D 660 m

29 The electric field patterns produced by three charged spheres are as shown.

P Q R

What are the charges on spheres P, Q and R?

P Q R
A negative negative positive
B positive positive negative
C positive negative negative
D negative positive negative

30 A battery drives 100 C of charge around a closed circuit.


The total work done is 750 J. What is the electromotive force of the battery?

A 0.13 V B 0.75 V C 7.5 V D 75 kV

31 7KHUHVLVWDQFHRIDZLUHLVȍ
A second wire is made of the same material but has twice the length and half the
diameter. What is the resistance of the second wire?

A ȍ B 2.0 ȍ C 4.0 ȍ D ȍ

www.KiasuExamPaper.com
16
14

32 $9SRZHUVXSSO\LVFRQQHFWHGWRDFLUFXLWFRQVLVWLQJRIWKUHHȍUHVLVWRUV

What is the amount of work done by the battery in 1 minute?


A 18 J B 48.6 J C 108 J D 778 J

33 An electrician installed two switches, S1 and S2, to control two lamps, L1 and L2,
in the following circuit.

Which of the following configuration will switch on both L1 and L2 at the same
time?

A S1 open, S2 closed
B S1 open, S2 open
C S1 closed, S2 closed
D S1 closed, S2 open

www.KiasuExamPaper.com
17
15

34 A magnet is placed on the top of a paper matchbox cover. The set-up is then
placed on the tray filled with paper clips. A large number of paper clips are
attracted to the based on the matchbox cover.
magnet
matchbox
cover

paper clips

Metal sheets of different materials are thereafter placed inside the matchbox cover.
When sheet X is placed inside, the paper clips remained; when sheet Y is placed
inside, the paper clips fell off.
metal sheet

Which of the following metals are the sheets made of?


X Y
A Aluminum Copper
B Copper Iron
C Iron Aluminum
D Iron Copper

www.KiasuExamPaper.com
18
16

35 In which of the circuits will the filament lamp be less bright if the temperature of the
component G decreases?

www.KiasuExamPaper.com
19
17

36 The diagram shows the cross-section of a cable lying on the ground. There is a
direct current in the cable. The Earth’s magnetic field is in the direction shown.

In which direction does the electromagnetic force act on the cable?


Earth’s
magnetic field

37 The diagram below shows a simple d.c. motor. Which of the following labelled parts
indicates the commutator?
A

www.KiasuExamPaper.com
20
18

38 The diagram below shows a current passing from X to Y.


There is an upward force on the wire.

current

force

magnet magnet

Y
What will be observed if the direction of current reverses?
A The wire will move upwards.
B The wire will move downwards.
C The wire will move towards the left.
D The wire will move towards the right.

39 Which of the following does not affect the magnitude of induced e.m.f in a simple
a.c. generator?

A speed of rotation of coil


B distance between magnet and rectangular coil
C number of turns of coil per unit length
D resistance of rectangular coil

www.KiasuExamPaper.com
21
19

40 Two straight electrical conductors are parallel to each other. Each conductor carries
a current, one into the plane of the paper, and one out of the plane of the paper.

Which diagram accurately represent the magnetic field around the two wires?

END OF PAPER

www.KiasuExamPaper.com
22
NAME : NO: CLASS: 4E1

ADMIRALTY SECONDARY SCHOOL

PRELIMINARY EXAMINATION 2019


SUBJECT : Physics
CODE : 6091 / 2
LEVEL/STREAM : Secondary 4 Express
DATE : 30 August 2019
TIME : 0800-0945 hrs
DURATION : 1 hour 45 minutes

Instructions to candidates:
Write your Name, Class and Index Number on all the work you hand in.
Write in dark blue or black pen on both sides of the paper.
You may use a soft pencil for any diagrams, graphs, tables or rough working.
Do not use staples, paper clips, highlighters, glue or correction fluid.

Section A: Short answer questions [50 marks]


Answer all questions. Write your answers in the spaces provided in the question paper.

Section B: Structured questions [30 marks]


Answer all questions. Write your answers in the spaces provided in the question paper.
Question 13 has a choice of either / or.

Candidates are reminded that all quantitative answers should include appropriate units.
The use of an approved scientific calculator is expected, where appropriate.
The number of marks is given in brackets [ ] at the end of each question or part
question.
.
For Examiner’s Use
Section A / 50
Section B / 30
Total / 80

DO NOT TURN OVER THIS PAPER UNTIL YOU ARE TOLD TO DO SO.

www.KiasuExamPaper.com
This question paper consists of 19 printed pages including this cover page.
23
2

Section A
Answer all questions.

1 Fig. 1 shows the velocity time graph of a 150 kg unmanned rocket launched from
the surface of planet X. The planet has no atmospheric layer.

It rises vertically upwards with constant acceleration and after some time, a
malfunction causes the rocket’s engine to cut off suddenly. On its downward
journey to the ground, the rocket falls with negligible air resistance.

Fig. 1

(a) Using Fig. 1 , determine the total distance travelled upwards from the
surface of planet X.

upwards distance travelled = ……………………… [2]

(b) What is the gravitational field strength of planet X?

gravitational field strength = …………………………………. [2]

www.KiasuExamPaper.com
24
3

(c) Hence, or otherwise, determine the weight of the unmanned rocket on planet
X.

weight = …………………………………. [2]

2 Fig. 2 shows the flood prevention system at Marina Barrage. Water is pumped
out from Marina reservoir to the sea in instances of heavy rain.
barrage

1.2 m sea

reservoir

Fig. 2
(a) Water is released from the reservoir to the sea. The height difference of the
reservoir and the sea is 1.2m, and 200 m3 of water is released per second to
maintain the reservoir’s height.
Calculate the power required by the pump if it is 80% efficient.
(take density of water as 1000 kg / m3)

power = …………………………………. [3]

(b) Explain why there is a need for a higher power input than the answer in (a)
in real life.

………………………………………………………………………………….....

………………………………………………………………………………….....

………………………………………………………………………………….....

…………………………………………………………………………………..... [2]

www.KiasuExamPaper.com
25
4

3 A steam engine uses the energy from steam to turn the turbine. Fig. 3 shows a
safety valve that is fitted to a steam engine. When the pressure of steam rises
above the safety level, the safety valve opens to release steam.

75 cm
10 cm movable weight
400 N
pivot
area = 100 cm2
safety valve

steam steam engine

Fig. 3
(a) Explain, in terms of moments, how the safety valve works.

…………………………………………………………………………………........

…………………………………………………………………………………........

…………………………………………………………………………………........

…………………………………………………………………………………........

…………………………………………………………………………………........

…………………………………………………………………………………........ [3]

(b) Calculate the minimum force acting on the safety valve before it opens.

force = …………………………………. [2]

(c) Suggest how the safety valve can be adjusted so that it would release
steam at a lower pressure.

…………………………………………………………………………………........

…………………………………………………………………………………........

…………………………………………………………………………………........

…………………………………………………………………………………........ [2]

www.KiasuExamPaper.com
26
5

4 Two liquids A and B are contained in a U-tube, as shown in Fig. 4.


The liquids do not mix.

3 cm2 cross
sectional area

liquid A
density
1200 kg / m3

X Y

liquid B

Fig. 4
Point X is at the junction between liquids A and B.
Point Y is at the same horizontal level in liquids B. The surface of liquid A is
15 cm above point X. The surface of liquid B is 24 cm above point Y. The
density of liquid A is 1200 kg / m3. Assume gravitational field strength is 10 N/kg.

(a) Calculate the pressure at point X due to liquid A.

pressure = …………………………………. [2]

(b) Hence, determine the density of liquid B.

density = …………………………………. [2]

www.KiasuExamPaper.com
27
6

5 Fig. 5 shows a side view and a plane view of a container used to serve hot drinks.

Layer of corrugated
cardboard stuck to a
layer of smooth
cardboard, with gaps
between
Fig. 5

(a) Explain how the design allows the person to safely hold the cup with hot
drinks.

………………………………………………………………………….................

………………………………………………………………………….................

………………………………………………………………………….................
[2]
………………………………………………………………………….................

(b) Two cups of coffee, one in a corrugated cardboard cup, another in a ceramic
mug, contains coffee of the same temperature.
After 5 minutes, predict which container will contain coffee with the higher
temperature. Explain your answer.

………………………………………………………………………….................

………………………………………………………………………….................

………………………………………………………………………….................

…………………………………………………………………………................. [2]

www.KiasuExamPaper.com
28
7

6 Fig. 6 shows a pool of water of depth 1.00 m. Rays of light travel in water from
an underwater lamp.

not drawn to scale

air

water 1.0 m θc
underwater
lamp

Fig. 6
Given that the refractive index of water in the pool is 1.33.
(a) Determine the speed of light in water.

speed of light in water = …………………………………. [2]

(b) Calculate critical angle, θc and complete ray R on Fig. 6

θc = …………………………………. [3]

www.KiasuExamPaper.com
29
8

7 Fig. 7 shows an application of electrostatic charges known as electrostatic


coating. A nozzle produces paint droplets, all of which are given a positive
charge. The metal panel is given a negative charge.

metal panel
charged
negatively

positively
charged
paint
droplets

nozzle

Fig. 7
(a) Explain how this method reduces the amount of paint needed to paint a
large surface.

………………………………………………………………………….................

………………………………………………………………………….................

………………………………………………………………………….................

…………………………………………………………………………................. [2]

(b) A student suggests that the metal panel can be of neutral charge as the
positively charged paint droplets would be attracted to neutral objects too.
Explain why his suggestion is not feasible in this context.

………………………………………………………………………….................

………………………………………………………………………….................

………………………………………………………………………….................

…………………………………………………………………………................. [2]

www.KiasuExamPaper.com
30
9

8 Fig. 8 shows the circuit of an electric iron with a metal casing connected to the
live (L), neutral (N) and earth (E) terminals.
metal casing

Fig. 8

(a) Explain how the earth wire protects the user from an electrical shock.

………………………………………………………………………….................

………………………………………………………………………….................

………………………………………………………………………….................

………………………………………………………………………….................

[2]
(b) There is a hazard in the circuitry in Fig. 8.
Identify the hazard and explain why it is unsafe.

………………………………………………………………………….................

………………………………………………………………………….................

………………………………………………………………………….................

…………………………………………………………………………................. [2]

www.KiasuExamPaper.com
31
10

(c) The power rating of the iron is 220 V, 2400 W.


(i) The iron is used for 10 minutes daily for 30 days in a month. How
much does it cost if each unit of electricity is $0.20?

cost = …………………………………. [2]

(ii) Determine the power of the iron when it is plugged into a 110 V
power socket.

cost = …………………………………. [2]

www.KiasuExamPaper.com
32
11

9 Fig. 9 shows the representation of a step-up transformer.

iron core

primary secondary
coil coil

Fig. 9
(a) State one way to increase the secondary voltage of a step-up transformer.
……………………………………………………………………………....... [1]

(i) Explain the function of the iron core and why it cannot be replaced by
steel.

…………………………………………………………………………….......…

………………………………………………………………………….......……

………………………………………………………………………..................
[2]
(b) A step up transformer has an input voltage of 240 V, 13 A, with a turns ratio
of 200.

(i) Calculate the output current of the transformer.

Output current = ……………….. [2]


(ii) Explain why step-up transformers are usually used before electricity is
transmitted by transmisson cables.

…………………………………………………………………………………..

…………………………………………………………………………………..

…………………………………………………………………………………..

………………………………………………………………………………….. [2]

www.KiasuExamPaper.com
33
12

Section B
Answer all the questions in this section.
Question 12 is in the form Either / Or.

10 Fig. 10.1 shows some information from the manufacturer of a car.


The kerb mass refers to the mass of the car without passengers and cargo.
The gross mass refers to the mass of the car with passengers and/or cargo.

kerb mass 850 kg


gross mass (with a 70 kg driver) 920 kg
gross mass (with a 70 kg driver and a 70 kg passenger) 990 kg

with one driver only with one driver and passenger


maximum 3.50 m/s2 3.25 m/s2
acceleration
maximum speed 50.0 m/s 50.0 m/s

Fig. 10.1
(a) When the mass of the people in the car doubles from 70 kg to 140 kg, there
is only a slight decrease in the maximum acceleration.

Explain why the acceleration did not decrease by half when the mass of
people doubled.

………………………………………………………………………………………

……………………………………………………………………………………… [1]

(b) Calculate the shortest time for the car, with a driver and a passenger inside,
to achieve the maximum speed from rest.

time = ……………………… [2]

(c) Ignoring air resistance, calculate the maximum forward thrust of the car
engine.

[2]
maximum thrust = ……………………..

www.KiasuExamPaper.com
34
13

(d) The driver takes the car for a test drive without any passenger. While
travelling at the maximum speed, the driver sees a police car ahead and
applies the brake after 2.0 s. The car decelerates uniformly and comes to
rest a short distance away from the police car.

Fig. 10.2 shows how the speed of the car varies with time after the driver
sees the police car.

time / s speed / m/s


0.0 50.0
2.0 50.0
4.0 40.0
6.0 30.0
8.0 20.0
10.0 10.0
12.0 0.0
Fig. 10.2

(i) State what it means by “decelerates uniformly”.

………………………………………………………………………………..

...…………………………………………………………………………… [1]

(ii) Calculate the distance travelled by the car during the deceleration.

distance = ……………………. [2]

(iii) Describe how the distance between the car and the police car changes
before and after the driver applies the brake.

………………………………………………………………………………

………………………………………………………………………………

……………………………………………………………………………….

……………………………………………………………………….……….. [2]

www.KiasuExamPaper.com
35
14

11 (a) Water is in liquid state at room temperature of 25 °C.

(i) Using kinetic theory of matter, describe the change that occurs when
water evaporates.

………………………………………………………………………………

………………………………………………………………………………

………………………………………………………………………………

……………………………………………………………………………… [2]

(ii) State one difference between the molecules in steam and molecules
in water.

……………………………………………………………………………….

……………………………………………………………………………….. [1]

(b) Fig. 11.1 shows liquid nitrogen, below its boiling point, stored in a vacuum
flask. The boiling point of nitrogen is -196 °C.

glass walls
liquid nitrogen

vacuum

Fig. 11.1

The flask has 2 glass walls with a vacuum between them.


State and explain the purpose of having a vacuum between the two layers
of glass walls

…………………………………………………………………………...........…….

……………………………………………………………………...........………….

………………………………………………………………..................................

…………………………………………………………………………...................

…………………………………………………………………………............……

…………………………………………………………………….......................... [3]

www.KiasuExamPaper.com
36
15

(c) The liquid nitrogen reaches its boiling point of -196 °C. A small piece of metal
at 20 °C is lowered slowly into the liquid nitrogen. Bubbles form within the
liquid nitrogen as it boils. The small piece of metal has a mass of 50 g. When
it is lowered into the liquid nitrogen, the metal cools to -196 °C. The specific
heat capacity of the metal is 0.39 J/(gK). The specific latent heat of
vaporisation of nitrogen is 200 J/g.

(i) Calculate the thermal energy lost by the metal as it cools.

thermal energy lost = ……………………. [2]

(ii) Hence, determine the mass of nitrogen that boils away.

mass of nitrogen = ……………………. [2]

www.KiasuExamPaper.com
37
16

12 Either

A circuit is set up as shown in Fig. 12.1. The resistance of the LDR varies from
800 Ω to 2400 Ω under different brightness.
6.0 V

150 Ω 150 Ω

1200 Ω

Fig. 12.1

(a) State the voltmeter reading on V1.

V1= ……………………. [1]

(b) Calculate the reading shown on the ammeter A1 when the LDR’s
resistance is 800 Ω.

[4]
reading on A1= …………………….

www.KiasuExamPaper.com
38
17

(c) In Fig. 12.2, sketch the graph to show how the resistance of
the LDR varies with light intensity

R/Ω

light intensity [1]


Fig. 12.2
(d) (i) Calculate the voltmeter reading V2 when the light intensity is
low.

[2]
V2= …………………….
(ii) An ammeter A2 is connected across X and Y as shown by Fig.
12.3. State the direction of current flow and explain your
answer.
6.0 V

150 Ω 150 Ω

1200 Ω

Fig. 12.3

………………………………………………………………………

………………………………………………………………………

………………………………………………………………………

……………………………………………………………………… [2]

www.KiasuExamPaper.com
39
18

12 Or
Fig. 12.4 shows how the vibrations of a modern electric guitar string are picked
up by a small coil of wire wound around a cylindrical magnet. The string, which
is made of steel, causes an electrical signal to be generated and detected.

signal

bar magnet

wire coil

Fig. 12.4
(a) State Faraday’s Law of Electromagnetic Induction.

………………………………………………………………………….................

………………………………………………………………………….................

…………………………………………………………………………................. [1]

(b) Using Faraday’s Law, explain how the electrical signal is generated.

………………………………………………………………………….................

………………………………………………………………………….................

………………………………………………………………………….................

………………………………………………………………………….................

………………………………………………………………………….................

…………………………………………………………………………................. [3]

www.KiasuExamPaper.com
40
19

(c) State and explain if any signal would be generated if the string of the
electrical guitar is changed to nylon.

………………………………………………………………………….................

………………………………………………………………………….................

………………………………………………………………………….................

…………………………………………………………………………................. [2]

(d) Fig. 12.5 shows the display on a screen when the signal of a note is being
detected. The horizontal scale indicating the time base is set to 2.0 ms/cm.

1 cm

1 cm

Fig. 12.5
(i) Calculate the frequency of the note being played.

frequency = …………………………………. [2]

(ii) State what happens to the frequency and amplitude of the sound
wave when it is played through a loudspeaker.

………………………………………………………………………….................

………………………………………………………………………….................

………………………………………………………………………….................

…………………………………………………………………………................. [2]
End of Paper

www.KiasuExamPaper.com
41
www.KiasuExamPaper.com
42
ADMIRALTY SECONDARY SCHOOL

4E PPhy 6091 Prelims


2019 Marking Scheme
1. B 2. A 3. B 4. C 5. A 6. D 7. A 8. A 9. B 10. A
11. B 12. A 13. A 14. C 15. D 16. A 17. D 18. D 19. C 20. C
21. C 22. C 23. C 24. C 25. A 26. D 27. A 28. A 29. B 30. C
31. D 32. B 33. A 34. B 35. B 36. A 37. D 38. B 39. D 40. C
Paper 2 Section A
Qn No Answers mk
s
1 (a) Total distance = area under graph
= 0.5 x 64 x 50 [1]
= 1600 m [1]
(b) Gravitational field strength = acceleration on planet anet X
= 64/ 32 (or equivalent for gradient of graph) ph) [1]
= 2.0 N/kg or 2.0 ms-2 [1]
(c) Weight = mass x gravitational field ld strength h
= 150 x 2 [1]
= 300 N [1]
2 (a) Gravitational potential ial en
energy
ne errgy = mgh
mg gh
h
= (200 x 1000) x 10 0 x 1.1.2
1 .2 [1]
= 2400 000 0J [1]
Powerr requ uir
ired = 2 400
required 40000 000
00000 x 100
10
00 0 / 80 0
= 3 MW [1]
(b)) Wa
Water
ate
er needs
n ed
ne ds too be
be raised
raais
ised
ed higher
hig
ighe
he
h er than
er tha
th an
n the
th
he
e sea level
lev
evel
el to
to be discharged [2]
W
Wa aterr cannot
Water ca
annnott be
be drawn
drra
d aw wn from
frro
omm the
th
hee ssurface
urfface of
of th
the wate
water from the reservoir
/ water
w ter is drawn
wa dra
awn
w from m be elo
low the
below the water
th wate er surface
surf
su rfac
rf ace
E
Enner
ergy
Energygy m ay be
ay
may be lost
llo
os
stt to
to friction
f iction in
fr n th
thee process
p oces
pr

H
Heencce th
Hence he h
the heeiig
gh
heighthtt differe
renc
rence
nc
differencee is
is greater
gre than 1.2m, resulting in the higher
power
po
owweer input
inp
in ut needed.
pu needed
ne ed..
ed
3 (a) Mo
Movable
ova
vable
blle wweight
eig
ight
ig ht exerts
exe
xerts a clockwise moments and keeps the safety valve [1]
closed.
lossed
e .
hen
When n tthe
he steam is excessive, it will exert a large force on the safety [1]
e, exerting
valve, exert an anti-clockwise moment.
When thet anti-clockwise moment from the steam is greater than the [1]
clockwise moment, steam releases from the safety valve.
(b) Clockwise moments = anti-clockwise moments
400 x 75 = F x 10 [1]
F = 3000 N [1]
(c) Lower the weight of the movable weight [1]
Move the movable weight closer to the pivot
Increase the surface area of the safety valve
Shift the pivot to the right (any 1)
[1]
To reduce the clockwise moments
4 (a) 3 KȡJ
= 0.15 x 1200 x 10 [1]
= 1800 Pa [1]
(b) 3 KȡJ
1800 = 0.24 x ȡ[ [1]
www.KiasuExamPaper.com
43
2
3
750 kg /m = ȡ [1]

5 (a) Layer of air in the gap [1]


Air is a poor conductor of heat, hence reducing heat conducted to hand [1]
Or
less contact area with the hand.
less surface area results in less conduction from the cup to the hand,
hence the person holding the cup will feel less hot.
(b) Coffee in corrugated cardboard [1]
Poor conductor of heat compared to ceramic, where the molecules are
more closely packed together as solid, than gas. [1]
6 (a) v=c/n
= 3 x 108 / 1.33 [1]
= 2.26 x 108 m/s [1]
(b)
[1]

air not drawn to scale

water
șc
lamp
lam

n=1 / si ssin
nc
1.
.33
3 = 1/ sin
1.33 sin
nc [1]
F Û ÛÛ
 [1]
7 (a)
(a
a) P
Pa
Paint
aint dr droplets
ro
op
pleetss repelel one
onnee another
ano
no
n o
oth
tth
heerr [1]
Spreads
S
Sppre
readads ouoout
ut uniformly
un
nififor
orml
mly to
t coverr moremorore
e area with same amount of paint [1]
(b)
b) M tall panel
Me
Metal pane el would
wo
w oul
uld bebe positively
possittiv
ivel
ely charged
el char due to conduction [1]
and re
an
and rrepel
epepel ot th
he
er paint
other p in
pa nt droplets.
drop
dr ople
op lets.
le
[1]

8 (a) Earth
arthh wires
wire
wires connects
con to the metal casing. In the event that the live wire
cidentally touches the metal casing, causing it to go ‘live’, the earth wire
accidentally [1]
uct away to earth, preventing electric shock from the user who
conducts
touches the metal casing [1]
(b) Fuse should not be at neutral / should be at live wire [1]
In case of excessive current, the fuse may blow but the appliance might [1]
still be ‘live’.
(c)i Total energy = 2.4 x 10/60 x 30
= 12 kWh [1]
Cost = 12 x $0.2 = $2.40 [1]
2
(c)ii P = V /R 2
2400= 220 /R
5 Ÿ [1]

P = 110 2 /20.2
= 600 W [1]

www.KiasuExamPaper.com
44
3
9 (a)i Increase number of coil on the secondary coil [1]

(a)ii Iron is easily magnetised and demagnetised [1]


Ensuring better magnetic flux linkage between the 2 coils. Steel does not
magnetise or demagnetise easily.
It is the change of magnetic flux that is able to induce a current, and
hence an induced emf, in the secondary coil. [1]
(b)i V p I p =V s I s
240 x 13 = (240 x 100) x I s [1]
I s = 0.13 A [1]
(b)ii Reduce energy loss during
2
transmission [1]
Since heat loss is P = I R, the lower the current, the lower the energy loss [1]
during transmission.

www.KiasuExamPaper.com
45
4

Section B Structured Answers


10 (a) Although the mass of the people in the car doubled, the total mass [1]
only increased slightly, hence by F = ma, the acceleration did not
decrease by half.
10 (b) Time = speed / acceleration
=50 / 3.25 [1]
= 15.4 s [1]
(c) F = 990 x 3.25 = 3217.5 N OR [2]
F = 920 x 3.50 = 3220 N
(d)i Rate of decrease of velocity is constant. [1]

(d)ii Distance travelled = 0.5 x 50 x (12-2) [1]


= 250 m [1]
(d)iii Before applying the brakes, the distance between the h car and the [1]
police car decreases by 50m per second.
After applying the brakes, the distance between en the e car
car and the police [1]
car decreases by less than 50m per second. ond.
11 (a)i When water evaporates, the liquid molecul molecules
ulles vvibrate
ibra
ibratte
ra te randomly
rand at [1]
different speeds,
Molecules at the surface that hat have enougheno
no
ouggh energy
en
e nergyy to
to overcome
ove
ov ercom the [1]
downward attractive forces rces ofof other
o he
ot er molecules
mo
m
mole
ole
ecu
cule
les will leave
lea
eave
v the surfa
surface.
(a)ii Molecules in steam am movemo ov
ve att highhig
igh
h speed
spe
sp ee
e
edd while
wh
w hilile
e molecules
mole
mo lecu
le cules in water
cu wat [1]
moves at slow w speed.
sppeeeed.
d.
OR
eculess in
Molecules i steam
ste ea
amm is
is atat boiling
boi
oiliing
ng p o ntt, while molecules
oi
point, molecule
mo es in water
water in
i not
at boiling g point
pooiinntt
OR
O R
M
Mo le
ecu
Moleculesuless iin n ssteam
te
eaamm a re
aree far
far
ar apart
apa
partt ffrom
romm each
eac othe
other but molecules in
waterr areare closely
ar c osely packed
cl pac
pa
pack
ck ked
ed ttogether
oget
og ether
(b)i
)ii Re
R ed du
Reduceuce eh e t gain
ea
heat gaiin
ga n by
by liquid
liquid
id nitrogen
nitiro
roge
gen
ge n through
throu conduction. [1]
Va accu
Vacuumuum
u d oe
o
does es no n
nott co
cconduct
nducuctt a
uc ass it removes the medium, which is [1]
rre
equ
q irre
required ed
d for
for
or conduction
cono duct ctio
ct ion
io n toto take
tak place, hence keeping the liquid [1]
ni
nitrro
oggen
nitrogenen in
in lilliquid
iquidid
d sstate.
tate
tate.
te
(c)i 4 PF¨ș
PF
4 PF¨ș
= 50
50 x 0.39
0.3
.399 x (-196-20)
(-196 [1]
42
= 4212212 J [1]

(c)ii Q = ml v
4212= m (200) [1]
M = 21.1 g [1]

12 Either

(a) 3.0 V [1]

(b)
1 1 1 [1]
= +
‫ܴ ݈ܽݐ݋ݐ‬ 150 + 150 800 + 1200
5 Ÿ [1]

V = IR
6= I (261) [1]

www.KiasuExamPaper.com
46
5
I = 0.023 A [1]

(c)

2400
5Ÿ

800

light intensity [1]


Fig. 12.2

(d)i V = 2400 / 3600 [1]


= 4.0 V [1]
(d)ii X to Y [1]
0 V than Y att 2.0
X has a greater potential of 3.0 2.0 V
V.. [1]

www.KiasuExamPaper.com
47
6

12 Or

(a) Faraday’s Law of electromagnetic induction states that the [1]


magnitude of the induced EMF in a circuit is directly proportionate to
the rate of change of magnetic flux in the current.
(b) When the string vibrations, the magnetic flux linking the coil [1]
changes. [1]
The change in magnetic flux generates an induced current. [1]
The induced current creates an induced emf in the coil, causing the
signal to be generated.
(c) No signal will be generated. [1]
There is no change of magnetic flux as nylon is not an electrical [1]
conductor.
(d)i Period = 8 ms [1]
Frequency = 1 / 0.008
= 125 Hz [1]

(d)ii Frequency remains the same [1]


ncreases.
Amplitude of the sound wave increases. [1]

www.KiasuExamPaper.com
48
7
Paper 3

1 (a)i L = 2.5 cm
d = 1.4 cm [1]
(a)ii It is the diameter of a circle, and it is difficult to ensure that the [1]
measurement passes through the center of the circle, to take repeated
reading from different sides and take the average.

Or

Difficult to determine the inner diameter, can use a inner jaw of Vernier
calipers

(a) iii Disagree. [1]


ʌ G 2 is the area of the circle, not circumference,
nce, so the
the total uncoiled [1]
length equation is not correct.
(b) i L 1 = 8.0 cm [1]
e = 5.5 cm [1]
(b)ii T = 0.17 x 5.5 = 0.935 N [1]

(c)i A = 60.0 cm [1]


B = 40.0 cm
C = 80.0 0 cm
(c)ii W r (0.600
(0.600)
0) + 0.9 (0.400)
(0.4
(0 400
00) = 0.935
0 93
0. 935 [1]
W r = 0.958
0.95
58 N [1]
((a
acttua
ual mass
(actual ma as
ss
s is
is 100g)
10
1000gg)
2 (a)
(a
a) 6
60.0
0.0 ccmm (accept
((a
acccept 58
5
58.0cm
8.0
.0cm
0cm
cm tto
o 62.0cm,
62
6 2.0
.0cm,, mu
must
st be 1 dp) [1]

(b)i
b)i 2
27
7.1
1s ((a
27.1s accep
ep
pt 25.0s
(accept 25.0
25.0s – 30.0s)
30.0
0s) [1]

(b)ii T = 1.36s
1..36s
1 36
36s [1]
2
6))2 = 1.85
T = (1.36)
(1
1.336 1..85 s2 [1]
(c) Human
Hu
uma
m n rereaction
reac
eac
acti
tion
tio timtime of starting and stopping of the stopwatch. [1]

(d) Constant ant variable:


onsstta va mass of bob, distance between cork and table [1]
Description
scri of experiment that will fulfil requirements [1]
Accurate results: take more readings then obtain the average / take time [1]
for more oscillations and then take the period.
Graph of T2 vs h [1]
m is the gradient of the graph. [1]
H can be determined from substitution of values from a point on the best
fit line
3 (a) 3.0 V or measured value [1]

(c) d = 10.0 cm,


I = measured value to precision of 2d.p. (precision of given ammeter) [1]
(d) Measured value as per correct precision and units.
d / mm , I / A , V / V , 1/I / A-1 [4]

www.KiasuExamPaper.com
49
8
(e) Graph of d/mm (y-axis) vs 1/I / A-1 (x-axis) - axes [1]
Plot - plot [1]
Best fit line - graph [1]
[1]
More than half the page - scale
(i) Gradient = (y 2 -y 1 ) / (x 2 -x 1 ) [1]
= 300 (accepted 200-500) [1]
(ii) Using linear law, y=mx+c [1]
Sub d = 0 (meaning y = 0 for the graph), use a selected point on the [1]
best fit line to find the I 0 .
(iii) 500 [1]

(d)i R = 100 E / I 0 d 0 (using values found above) [1]


Ÿ DSSUR[ [1]
(d)ii Kinks in the resistance wire [1]

(d)iii not have


Take readings from portions of the wire that does not have kinks. [1]

(e) Graph with a steeper gradient [1]

www.KiasuExamPaper.com
50
www.KiasuExamPaper.com
51
Class Register Number Name

BARTLEY SECONDARY SCHOOL

GCE O-LEVEL PRELIMINARY EXAMINATIONS

PHYSICS 6091/01
Sec 4 Express
Paper 1 Multiple Choice 4 Sep 2019
1 hour
Candidates answer on the Multiple Choice Answer Sheet.
Additional Materials: Multiple Choice Answer Sheet

READ THESE INSTRUCTIONS FIRST

Write your class, register number and name on all the work you hand in.
Do not use staples, paper clips, highlighters, glue or correction fluid.

There are forty questions in this paper. Answer all questions. For each question there are four possible
answers, A, B, C and D.
Choose the one you consider correct and record your choice in soft pencil on the separate
Multiple Choice Answer Sheet.

Read the instructions on the Answer Sheet very carefully.

Each correct answer will score one mark. A mark will not be deducted for a wrong answer.
Any rough working should be done in this booklet.
The use of an approved scientific calculator is expected, where appropriate.

At the end of the examination, submit the Multiple Choice Answer Sheet.

This document consists of 20 printed pages.

Set by: LHL [Turn over

www.KiasuExamPaper.com
52
2

Answer all questions.

For each question, there are four possible answers A, B, C and D.

Choose the one you consider correct and shade your choice on the Multiple Choice Answer Sheet.

1 Which of the following shows the best estimate of the diameter of Earth?

A 1.3 x 103 km B 1.3 x 104 km


C 1.3 x 105 km D 1.3 x 106 km

2 A micrometer is used to measure the thickness of a metal sheet.

Diagram 1 shows the reading on the micrometer when it is tightened with nothing between the
jaws. Diagram 2 shows the reading taken with the metal sheet between the jaws.

diagram 1 diagram 2

40 40
0

30 30
40

What is the thickness of the metal sheet?

A 1.82 mm B 1.86 mm
C 3.32 mm D 3.36 mm

BSS/2019/Prelim Exam/4E Physics Paper 1


www.KiasuExamPaper.com
53
3

3 Three boys P, Q, and R participated in a race against one other. They started from the same
starting point. The three diagrams show their speed-time graphs.

P Q R

speed / ms-1 speed / ms-1 speed / ms-1

6
5
4

time / s time / s time / s


0 5 10 0 4 10 0 2 10

Which statement about the positions of the boys after 10 s of the race is correct?

A P leads Q and R
B Q leads P and R
C R leads P and Q
D P and R share first place

4 A skydiver falls at terminal velocity. He then opens his parachute.

Which row gives the direction of the resultant force on the skydiver and the direction of the
acceleration of the skydiver, immediately after the parachute opens?

resultant force acceleration

A downwards downwards

B downwards upwards

C upwards downwards

D upwards upwards

BSS/2019/Prelim Exam/4E Physics Paper 1 [Turn over


www.KiasuExamPaper.com
54
4

5 A sledge of mass 15 kg is pulled across a level ground with a constant speed of 0.50 ms-1 by a
horizontal force of 30 N.

0.50 ms-1

30 N

What is the frictional force experienced by the sledge?

A 7.5 N B 23 N
C 30 N D 120 N

6 A rock is hung from a rope and g


given a sideways
y push close to the surface of the Earth.

push

surface of Earth

If the same arrangement were to be given the same sideways push close to the surface of the
Moon, what would happen to the rock?

A The rock will move with more ease as it has less mass.
B The rock will move with more ease as it has less weight.
C The rock will move with the same ease as it has the same mass.
D The rock will move with the same ease as it has the same weight.

BSS/2019/Prelim Exam/4E Physics Paper 1


www.KiasuExamPaper.com
55
5

7 An astronaut conducted an experiment on Mars in which she placed a rock on a spring balance
and then on a beam balance. The gravitational field strength of Mars is larger than that of the
Moon.

Which set of results is correct when the same experiment with the same rock was conducted on
the Moon?

spring balance reading beam balance reading

A greater than in Mars less than in Mars

B greater than in Mars same as in Mars

C smaller than in Mars less than in Mars

D smaller than in Mars same as in Mars

8 Two immiscible liquids, P and Q, are poured into a beaker. After the liquids have settled, an
object R is placed in the beaker. The diagram below shows the final position of object R.

liquid P
object R

liquid Q

What conclusion can be made?

A Object R is denser than both liquids P and Q.


B Object R is denser than liquid P but less dense than liquid Q.
C Object R is denser than liquid Q but less dense than liquid P.
D Object R is less dense than both liquids P and Q.

BSS/2019/Prelim Exam/4E Physics Paper 1 [Turn over


www.KiasuExamPaper.com
56
6

9 The diagram shows a toy balanced on the edge of a table and at rest. The toy has a metal ball
attached to it.

Where is the likely centre of gravity of the toy?

toy

A
table
B
C
D

metal ball

10 Three identical hollow pipes X, Y and Z have one or two identical weights attached to their inner
surfaces as shown below.

X Y Z

Which row best describes the stability of the pipes?

X Y Z

A neutral equilibrium stable equilibrium unstable equilibrium

B neutral equilibrium unstable equilibrium stable equilibrium

C stable equilibrium neutral equilibrium unstable equilibrium

D stable equilibrium unstable equilibrium neutral equilibrium

BSS/2019/Prelim Exam/4E Physics Paper 1


www.KiasuExamPaper.com
57
7

11 A block was pulled up along a slope.

distance moved
along slope 25 m

pull 1 600 N
hm
frictional force
400 N

weight of block 5 000 N

What is the magnitude of h, the increase in vertical height of the block?

A 2.0 m B 6.0 m
C 8.0 m D 10 m

12 Alan, Darrel, Jake and Kent squat on concrete steps as shown. Each person carries a sphere
which is of the same size but of different mass. They drop the spheres onto a smooth level sand
pit.

Kent carries
100 g

Jake carries
150 g

Darrel carries
200 g

Alan carries
300 g
4.0 m
3.0 m

2.0 m

1.0 m
sand pit

Who carries the sphere which will sink the deepest in the sand?

A Alan
B Darrel
C Jake
D Kent

BSS/2019/Prelim Exam/4E Physics Paper 1 [Turn over


www.KiasuExamPaper.com
58
8

13 A simple mercury barometer is used to measure atmospheric pressure as shown below.

80 cm

15 cm
X 0 cm

What is the pressure at X?


A 150 mm Hg B 650 mm Hg
C 800 mm Hg D 950 mm Hg

14 A glass vessel is connected to a side-arm through a tap.

With the tap closed, the depth of water in the vessel is h. When the tap is opened, water flows
into the side-arm. The depth of water in the vessel falls.

The diagram on the right shows the water levels before they have settled.

glass vessel

side-arm

h y
x

tap closed
tap open

When the levels have settled, which statement is true?

A h=x B y=x
C h=y+x D h=y–x

BSS/2019/Prelim Exam/4E Physics Paper 1


www.KiasuExamPaper.com
59
9

15 A fixed mass of gas is compressed while kept at constant temperature.

How will the properties of the molecules of the gas change?

frequency of collisions
average distance between the molecules average speed of
between molecules and the walls of the molecules
container

A decreased decreased increased

B decreased increased no change

C increased decreased no change

D no change increased decreased

16 A centimetre scale is fixed next to an unmarked mercury-in-glass thermometer. At ice point the
length of the mercury thread is 1.5 cm. At 60 °C, the length of the mercury thread increases to
9.5 cm.

What is the length of the mercury thread at steam point?

A 13.3 cm B 14.3 cm
C 14.8 cm D 15.8 cm

17 A Brownian motion experiment involving smoke particles in air was conducted.

What best explains why heavy particles settle down quickly but very small particles remain
suspended for long period of time?

A Air pressure has a greater effect on smaller particles.


B Random molecular bombardment by air molecules keeps the small particles suspended.
C The earth’s gravitational field does not act on very small particles.
D The small smoke particles have the same density as air.

BSS/2019/Prelim Exam/4E Physics Paper 1 [Turn over


www.KiasuExamPaper.com
60
10

18 A student filled an upright ring-shaped container completely with warm water. He then placed a
pack of dry ice at a corner of the container.

lid

pack of dry ice


warm water

Which diagram correctly illustrates the convection current that was set up?

A B

C D

BSS/2019/Prelim Exam/4E Physics Paper 1


www.KiasuExamPaper.com
61
11

19 A metal box has four different surfaces of equal area: dull black, shiny black, dull silver and shiny
silver.

The box is filled with boiling water so that each surface is at the same temperature. A sensor
measures the amount of radiation from each surface.

sensor

dull silver
meter
shiny silver

Which surface emits radiation at the slowest rate and which surface emits radiation at the fastest
rate?

emits radiation slowest emits radiation fastest

A dull black shiny silver

B dull silver shiny black

C shiny black dull silver

D shiny silver dull black

20 Two liquids X and Y of equal weight are put in identical vessels, and thermal energy is supplied
to them at the same rate. The temperature-time graphs are as shown.

temperature X
Y

time

Which statement(s) is(are) true?

I Y has a higher freezing point than X.


II Y at liquid state has a larger specific heat capacity compared to X at liquid state.
III Y has a larger specific latent heat of vaporisation than X.

A II only
B III only
C I and II only
D II and III only

BSS/2019/Prelim Exam/4E Physics Paper 1 [Turn over


www.KiasuExamPaper.com
62
12

21 Some wax in a test tube was heated till it melted. It was then allowed to cool. The temperature-
time graph during the cooling process is shown.
temperature / °C

Q R

S
time / min

Which statement is correct?

A Along PQ, the molecules gain internal energy.


B Along QR, the molecules lose kinetic energy.
C Along QR, the molecules lose potential energy.
D Along RS, the molecules gain potential energy.

22 Air is blown into ether through the glass tube as shown in the diagram.
After some time, it is observed that the film of water freezes into ice.

blown air

glass tube

a film of water
ether

wooden board

Which option best describes the processes that result from blowing the air?

rate of evaporation of ether temperature of ether heat transfer

A decreases falls from ether to water

B decreases rises from ether to water

C increases falls from water to ether

D increases rises from water to ether

BSS/2019/Prelim Exam/4E Physics Paper 1


www.KiasuExamPaper.com
63
13

23 Water waves can be used to demonstrate refraction by passing them in a trough of water of
different depths.

wave deep water shallow water


direction

wavefronts

Which statement describes why the water wave changes direction as it passes into shallow
water?

A The frequency of the wave decreases.


B The frequency of the wave increases.
C The speed of the wave decreases.
D The speed of the wave increases.

24 Which statement about the components of the electromagnetic spectrum is not correct?

A Microwaves have shorter wavelengths compared to those of radio waves and they are used
in airport security to detect metallic objects in passengers’ baggage.
B Radio waves have lower frequencies compared to those of gamma rays and they are used
in television broadcasts.
C Visible light have higher frequencies compared to those of microwaves and they are used
in lasers to weld metals together.
D X-rays have shorter wavelengths compared to those of ultraviolet rays and they are used
to check flaws in metal welds.

25 A student holds a sheet of paper with letters on it facing a plane mirror. The letters on the paper
are shown below.

What does the student see in the mirror?

A B C D

BSS/2019/Prelim Exam/4E Physics Paper 1 [Turn over


www.KiasuExamPaper.com
64
14

26 The figure shows a semi-circular glass slab with centre O. The glass has a critical angle of 45°.

R
45°
O

What happens when a ray of light, perpendicular to the diameter POQ, is incident at R?
A The light ray emerges at O.
B The light ray emerges at some point between O and P.
C Total internal reflection occurs at O.
D Total internal reflection occurs at some point between O and P.

27 The converging lens in a camera is used to make an image on a film.

At which point could an object be placed so that it forms a diminished image?

focal length
of lens film

A B C D

2 x focal length
of lens

28 The diagram shows the trace on a cathode-ray oscilloscope when a microphone which is
connected to it picks up a sound.

Which trace is obtained when the sound wave is changed to one that has the same loudness
but of higher pitch?

A B C D

BSS/2019/Prelim Exam/4E Physics Paper 1


www.KiasuExamPaper.com
65
15

29 An attendant is filling up a car with petrol at a petrol station.

Which of the following describes how electrostatic charges becomes a hazard?

A As the car is filled with petrol, the petrol vapour gets ignited by the sparks between the
attendant and the car when they discharge accidentally.
B As the car is not earthed, charges on the car ignite the fuel inside the car’s petrol tank.
C Charges on the car flow through the hose to the underground petrol reservoir, causing the
petrol to catch fire.
D The attendant gains charges due to friction of his clothes and he gets an electric shock
when he touches the car.

30 The graph shows how the current flowing through a 12 V lamp varies with the potential difference
(p.d.) across the lamp.

current / A

4
3

0 2 4 6 8 10 12 p.d. / V

From the graph, which statement correctly describes the resistance of the lamp as the p.d.
increases?

A The resistance of the lamp decreases throughout the voltage range.


B The resistance of the lamp increases at first and then decreases.
C The resistance of the lamp increases throughout the voltage range.
D The resistance of the lamp remains constant.

BSS/2019/Prelim Exam/4E Physics Paper 1 [Turn over


www.KiasuExamPaper.com
66
16

31 A piece of wire initLDOO\KDVDUHVLVWDQFHRIȍThis wire is then stretched evenly until it is twice


its initial length as shown below. The volume of the wire remains unchanged.

What is the new resistance of the wire?

A ȍ B ȍ
C ȍ D ȍ

32 A filament lamp is connected to a 12 V battery as shown.

When the circuit is switched on for 2.0 minutes, 4.0 C of charges flow through the circuit.

What is the amount of electrical energy that is converted into light and thermal energy in the
lamp?

A 6.0 J B 24 J
C 48 J D 96 J

BSS/2019/Prelim Exam/4E Physics Paper 1


www.KiasuExamPaper.com
67
17

33 A d.c. circuit is set up as shown. The electromotive force of the battery is 12 V.

What is meant by the electromotive force of the battery is 12 V?

A It takes 1.0 s to drive 12 C of charge around the circuit.


B It takes 12 s to drive 1.0 C of charge around the circuit.
C It takes 1.0 J of energy to drive 12 C of charge around the circuit.
D It takes 12 J of energy to drive 1.0 C of charge around the circuit.

34 Each lamp in the circuit below is rated 9.0 V, 36 W. A, B, C and D are 6 A fuses.

Which fuse will blow when the switch is closed?

9.0 V

B
C

BSS/2019/Prelim Exam/4E Physics Paper 1 [Turn over


www.KiasuExamPaper.com
68
18

35 The diagram shows the circuit formed when a rice-cooker is plugged into a mains socket.

resistance of heating
HOHPHQW Ÿ
neutral wire

230 V mains
supply

live wire

10 A fuse

metal case

What is the fault in this circuit arrangement?

A The earth wire is connected wrongly.


B The fuse is connected to the wrong wire.
C The fuse rating is too low.
D The switch is connected to the wrong wire.

36 The diagram shows the direction of the compass needle when placed near two bar magnets.

X Y

What are the likely poles at X and Y?

pole at X pole at Y

A North South

B North North

C South North

D South South

BSS/2019/Prelim Exam/4E Physics Paper 1


www.KiasuExamPaper.com
69
19

37 Four metal rods are placed, one at a time, inside a coil of copper wire.

direction of current
in coil of copper
wire

metal rod

paper clips

The table below gives the results of the experiment.

Which rod would be the most suitable to use for the core of a coil in a circuit breaker?

metal number of paper clips picked up number of paper clips still attracted
rod when there is current in the coil when the current is switched off

A 1 0

B 20 2

C 35 0

D 35 30

38 The diagram below shows a positive charge entering a magnetic field directed into the paper.

Which path correctly illustrates the motion of the positive charge?

X X X X X
X X X X X
B
X X X X X
+
X X X X X
C
X X X X X
X X X X X
magnetic field D

BSS/2019/Prelim Exam/4E Physics Paper 1 [Turn over


www.KiasuExamPaper.com
70
20

39 A solenoid is placed in between a magnet and an iron nail that are freely suspended from the
ceiling. The magnet is then moved away from the solenoid as shown.

S N
movement of magnet

What is the direction of the current flowing through the fixed resistor and the direction that the
iron nail moves?

direction of current
direction of movement of iron nail
through the resistor

A to the left away from solenoid

B to the left towards solenoid

C to the right away from solenoid

D to the right towards solenoid

40 A supply of peak value 5.0 V and of frequency 50 Hz is connected to the Y-input terminals of a
cathode ray oscilloscope. The Y-gain and time-base settings are set at 5.0 V per division and
10 ms per division respectively.

Which trace is obtained?

A B

C D

BSS/2019/Prelim Exam/4E Physics Paper 1


www.KiasuExamPaper.com
71
21

Q1 Q2 Q3 Q4 Q5 Q6 Q7 Q8 Q9 Q10

B B A D C C D B B C

Q11 Q12 Q13 Q14 Q15 Q16 Q17 Q18 Q19 Q20

B C C B B C B B D D

Q21 Q22 Q23 Q24 Q25 Q26 Q27 Q28 Q29 Q30

C C C A B B A A A C

Q31 Q32 Q33 Q34 Q35 Q36 Q37 Q38 Q39 Q40

C C D B D A C A B D

BSS/2019/Prelim Exam/4E Physics Paper 1 [Turn over


www.KiasuExamPaper.com
72
Class Register Number Name

BARTLEY SECONDARY SCHOOL

GCE O-LEVEL PRELIMINARY EXAMINATIONS

PHYSICS 6091/02
Sec 4 Express

Paper 2 Theory 20 Sep 2019

1 hours 45 minutes
Candidates answer on the Question Paper.
Additional Materials: NIL

READ THESE INSTRUCTIONS FIRST

Write your class, register number and name on all the work you hand in.
Write in dark blue or black pen on both sides of the paper.
You may use a soft pencil for any diagrams and graphs.
Do not use staples, paper clips, highlighters, glue or correction fluid.

Section A
Answer all questions.

Section B
Answer all questions. Question 12 has a choice of parts to answer.

Candidates are reminded that all quantitative answers should include appropriate units.
The use of an approved scientific calculator is expected, where appropriate.
Candidates are advised to show all their working in a clear and
orderly manner, as more marks are awarded for sound use of
Physics than for correct answers. For Examiner’s Use

Section A
At the end of the examination, submit this question paper.
The number of marks is given in brackets [ ] at the end of each
Section B
question or part question.
Total

This document consists of 21 printed pages and 1 blank page.

Set by: LHL [Turn over

www.KiasuExamPaper.com
73
2
Section A

Answer all the questions in the spaces provided.

1 When out in the open sea, ship X and ship Y use a steel cable to transfer cargo from one to the
other. Fig. 1.1 shows how the steel cable looks like when a cargo of mass 130 kg is exactly in
between the two ships. The ends of the cable connected to each ship are positioned at the same
height above the surface of the sea. The gravitational field strength is 10 N/kg.

20° 20°
TA TB ship Y
ship X
130 kg
ship X ship Y

Fig. 1.1

(a) Calculate the weight exerted by the cargo.

weight of cargo = [1]

(b) By means of a scaled drawing, find the tension in the steel cable.

tension in the cable = [3]

BSS/2019/Prelim Exam/4E Physics Paper 2


www.KiasuExamPaper.com
74
3

2 (a) State the principle of moments.

[2]

(b) Fig. 2.1 shows part of the boiler of a steam engine. Thermal energy is transferred to the
water in the boiler by conduction.

lever arm mass M


10 m 5.0 m
pivot

valve

water

thermal energy

Fig. 2.1

The boiler has a valve which allows steam to escape if the pressure inside the boiler is too
high. The pressure in the boiler is controlled by the valve connected to a pivoted lever arm.
The uniform lever arm has a movable mass M which is used to adjust the pressure at which
the valve opens.

(i) The surface area of the valve in contact with the steam is 4.0 u 10-2 m2.

Calculate the upward force on the valve when the pressure in the boiler is 20 kPa.

force = [2]

BSS/2019/Prelim Exam/4E Physics Paper 2 [Turn over


www.KiasuExamPaper.com
75
4
(ii) The length of the uniform lever arm is 20 m and the weight of the lever arm is 100 N.
By placing mass M 10 m away from the valve, the valve opens when the pressure in
the boiler is 20 kPa.

Calculate the weight of M.

weight = [2]

(iii) State how the mass M should be moved so that the valve of the boiler is opened at a
pressure higher than 20 kPa. Explain your answer.

[2]

BSS/2019/Prelim Exam/4E Physics Paper 2


www.KiasuExamPaper.com
76
5

3 (a) Fig. 3.1 shows a soccer ball on the ground being stepped on by a foot.

foot
soccer ball

Fig. 3.1

It is observed that the volume of the soccer ball decreases while the temperature of the air
in the ball remains the same.

Using the kinetic model of matter, state and explain how pressure of the air in the ball
changes.

[3]

(b) Fig. 3.2 shows a mercury manometer that is being used to measure the pressure in a
chamber. The pressure due to the air trapped in the chamber is found to be 810 mm Hg.
The density of mercury is 13 600 kg/m3.

air

h = 50 mm
chamber

Fig. 3.2

Determine the atmospheric pressure, leaving your answer in Pa.

pressure = Pa [2]

BSS/2019/Prelim Exam/4E Physics Paper 2 [Turn over


www.KiasuExamPaper.com
77
6
4 Fig. 4.1 shows an igloo, which is a small dome-shaped house built by Eskimos from blocks of
hard snow. The Eskimos live in very cold regions near to the North pole.

The entrance is dug lower than the sleeping area to create a cold sink where cold air flows to.
The cross-section of an igloo is shown in Fig. 4.2.

Eskimo

sleeping sleeping
area area
igloo
cold sink
Fig. 4.1 Fig. 4.2

(a) State one thermal property of the hard snow blocks that makes them suitable for building
igloos.

[1]

(b) Suggest one way by which the Eskimos can reduce their rate of heat loss by radiation when
they sleep. Explain your answer.

[2]

(c) Explain the process of how the heat from the Eskimos keep the igloo warm inside even if it
is cold outside.

[2]

BSS/2019/Prelim Exam/4E Physics Paper 2


www.KiasuExamPaper.com
78
7

5 Geothermal energy makes use of heat from the Earth’s interior to generate power. Fig. 5.1
shows 1 200 kg of cold water at a temperature of 25 °C is pumped down to the hot rocks of the
Earth’s interior. 750 kg of it returns as hot water and the rest returns as steam, both at a
temperature of 100 °C.

Earth’s surface
hot water cold water
and steam at 25 °C
at 100 °C
cracks in
hot rocks

Fig. 5.1

The specific heat capacity of water is 4 200 J / (kg °C) and the specific latent heat of vaporisation
of water is 2.3 × 106 J / kg.

(a) Distinguish between the specific heat capacity of water and the specific latent heat of
vaporisation of water.

[2]

(b) Calculate the energy needed to heat 1 200 kg of water from 25 °C to 100 °C.

energy = [2]

(c) Calculate the energy needed to produce steam at 100 °C.

energy = [2]

BSS/2019/Prelim Exam/4E Physics Paper 2 [Turn over


www.KiasuExamPaper.com
79
8
6 (a) State what is meant by critical angle.

[1]

(b) Fig. 6.1 shows the cross-section of an optical fibre. The optical fibre has a very thin glass
FRUHZLWKDGLDPHWHURIȝP7KHUHIUDFWLYHLQGH[ of the glass core is 1.65.

plastic cladding

air
i
glass core

Fig. 6.1

(i) The glass core is surrounded by a plastic cladding.

State and explain which material has a higher refractive index and explain your answer.

[2]

(ii) The light ray enters the air-glass interface at 45°.

Calculate the angle of refraction in the glass core.

angle of refraction = [2]

(iii) Prove that the minimum value for i is 37.3°.

[2]

BSS/2019/Prelim Exam/4E Physics Paper 2


www.KiasuExamPaper.com
80
9

7 Fig. 7.1 shows a potential divider made from a thermistor DQG D  Nȍ fixed resistor. The
potential divider is connected in series with a 12 V d.c. power supply. A voltmeter is connected
DFURVVWKHNȍUHVLVWRU

12 V

NŸ

Fig. 7.1

(a) At the current room temperature of 30 °C, the resistance of the thermistor LVNȍ

Calculate the reading on the voltmeter.

voltmeter reading = [2]

(b) The temperature of the room is gradually decreased.

State and explain what happens to the reading on the voltmeter.

[2]

BSS/2019/Prelim Exam/4E Physics Paper 2 [Turn over


www.KiasuExamPaper.com
81
10
8 Fig. 8.1 shows an electric heater used in a shower.

device W wire X
wire Y
heating element wire Z

coils of
copper pipe

shower plastic
head case

water supply from tap

Fig. 8.1

(a) Wires X, Y and Z are connected to the mains power supply. Identify these wires and device
W.

wire X:

wire Y:

wire Z:

device W:
[2]

(b) In a faulty water heater, an electric current flows from the heating element to the copper
pipe and the water that flows inside. This puts the user at risk of electric shock when the
shower is used.

(i) Explain how wire Z protects the user from electric shock.

[2]

(ii) State the colour of the insulation for wire Z.

[1]

BSS/2019/Prelim Exam/4E Physics Paper 2


www.KiasuExamPaper.com
82
11

(c) The water heater consumes 3.0 kW of power as it operates on 240 V a.c. Assume that the
efficiency of the water heater is 100%.

The cost of electricity supplied is $0.30 per kWh.

Calculate the total cost of using the water heater for 30 minutes a day for 30 days.

total cost = [2]

BSS/2019/Prelim Exam/4E Physics Paper 2 [Turn over


www.KiasuExamPaper.com
83
12
9 Fig. 9.1 shows a type of electric door lock.

coil iron bolt

iron core door

door opens in
spring this direction

Fig. 9.1

The lock is closed when the position of the iron bolt is as shown in Fig. 9.1.

(a) Explain how closing the switch in the circuit allows the door to be opened.

[2]

(b) The door’s iron bolt is changed to a thicker piece of iron. When the switch is closed, the lock
remains closed.

Without doing further changes to the bolt, suggest two other changes that could be made
in order to open the lock.

[2]

BSS/2019/Prelim Exam/4E Physics Paper 2


www.KiasuExamPaper.com
84
13

Section B (30 marks)

Answer all the questions in this section.

Answer only one of the two alternative questions in Question 12.

10 An engineer designs a device that can be used to monitor the thickness of pipes as shown in
Fig. 10.1.

to c.r.o.

transmitter receiver
device
d wall of
the pipe

chemical
solution

metal pipe
Fig. 10.1

The device emits an ultrasound pulse of frequency 4.0 x 106 Hz. The pulse travels through the
wall of the metal pipe and is reflected by the inner wall back to the device. The returning pulse
is picked up by the receiver. Both the transmitter and the receiver are connected to a cathode-
ray oscilloscope (c.r.o.)

The metal pipe is made of steel. Table 10.1 shows the speed of sound in different media.

Table 10.1

medium speed of sound / ms-1

air 340

glass 4 000

steel 6 100

water 1 400

(a) (i) Determine the wavelength of the ultrasound pulse in the wall of the pipe.

wavelength = [2]

BSS/2019/Prelim Exam/4E Physics Paper 2 [Turn over


www.KiasuExamPaper.com
85
14
(ii) At one point of time, the series of compressions and rarefactions of the ultrasound wave
in the wall of the pipe is as shown in Fig. 10.2. Points A, B, C and D are at the centres
of regions of compression.

A B C D

Fig. 10.2 (not to scale)

Determine the distance between points A and D.

distance = [1]

(b) The thickness of the pipe is 4.0 mm.

(i) Determine the time interval, in μs, between the emitted pulse and the reflected pulse.

time interval = μs [2]

(ii) The c.r.o. has a time base setting of 100 ns / div. Fig. 10.3 shows the emitted pulse on
the c.r.o. On Fig. 10.3, draw the reflected pulse.

emitted
pulse

100 ns

Fig. 10.3
[1]

BSS/2019/Prelim Exam/4E Physics Paper 2


www.KiasuExamPaper.com
86
15

(c) Suggest and explain how your answer in (b)(ii) will change if the chemical solution corrodes
the metal pipe.

[2]

(d) The engineer’s colleague suggests that the device should be placed at the bottom of the
pipe in order to better monitor the thickness of the pipe. Explain how this can better monitor
the thickness of the pipe.

[2]

BSS/2019/Prelim Exam/4E Physics Paper 2 [Turn over


www.KiasuExamPaper.com
87
16
11 (a) Fig. 11.1 shows a simple setup that can be used to detect seismic waves from earthquakes.
The setup consists of a bar magnet suspended from a spring hanging from a metal rod. The
metal rod transmits vibrations from the earth and the magnet moves in and out of the coil
when there is an earthquake. The coil is connected to a cathode-ray oscilloscope (c.r.o.)
that monitors the e.m.f across the coil.

metal rod

c.r.o.
spring

N bar magnet

coil

Fig. 11.1 Fig. 11.2

Fig. 11.2 shows the trace that was displayed on the c.r.o. during a particular earthquake.
Each complete oscillation of the same magnitude represents one tremor.

(i) Describe and explain how a trace shown on the c.r.o. in Fig. 11.2 is obtained when
there is an earthquake.

[4]

(ii) On Fig. 11.1, indicate the direction of the current in the coil when the south pole of the
magnet is moving into the coil. [1]

BSS/2019/Prelim Exam/4E Physics Paper 2


www.KiasuExamPaper.com
88
17

(b) An output voltage of 2.0 V from a generator is connected to the primary coil of a step-up
transformer with a turns ratio of 50 : 1. The current in the secondary coil is 2.4 mA. The
transformer is 75% efficient.

(i) State the metal used for the core of a transformer.

[1]

(ii) Calculate the current in the primary coil.

current = [2]

(iii) State two reasons why a typical transformer is not 100% efficient.

[2]

BSS/2019/Prelim Exam/4E Physics Paper 2 [Turn over


www.KiasuExamPaper.com
89
18
12 EITHER

A car travelled along a smooth and straight road with a uniform speed of 20 ms-1 for 4.0 s. It
then experienced uniform deceleration until it came to a stop 5.0 s later. It remained stationary
for 2.0 s before it started to travel in the opposite direction. Its speed increased at a decreasing
rate to reach 25 ms-1 after 8.0 s. After that it travelled at a uniform speed for another 6.0 s.

(a) (i) In the space below, sketch the velocity-time graph of the car.

[3]

(ii) State what is meant by uniform deceleration.

[1]

(iii) Calculate the deceleration of the car.

deceleration = [2]

(b) Given that the mass of the car was 1 500 kg, determine the retarding force that was acting
on the car from 4.0 s to 9.0 s.

retarding force = [2]

BSS/2019/Prelim Exam/4E Physics Paper 2


www.KiasuExamPaper.com
90
19

(c) When the car was accelerating in the opposite direction, the driving force produced by the
car engine was constant throughout its journey.

Explain, in terms of forces acting on the car, why the velocity was increasing at a
decreasing rate, even though the driving force was constant.

[2]

BSS/2019/Prelim Exam/4E Physics Paper 2 [Turn over


www.KiasuExamPaper.com
91
20
12 OR

A golf club hits a stationary golf ball. Fig. 12.1 shows three stages in the process.

before impact

during impact

after impact

Fig. 12.1

(a) Explain how the principle of conservation of energy applies during the impact.

[3]

(b) Using ideas about energy, explain why the speed of the golf ball does not depend on its
mass.

[2]

BSS/2019/Prelim Exam/4E Physics Paper 2


www.KiasuExamPaper.com
92
21

(c) The golf ball rises from the ground at A to a vertical height of 16 m at B before landing on
the ground at C, as shown in Fig. 12.2. You may assume that there is no air resistance as
the ball travels from A to B to C.

A C

Fig. 12.2 (not to scale)

The mass of the ball is 0.045 kg. The gravitational field strength g is 10 N / kg.

(i) Calculate the increase in gravitational potential energy of the ball between A and B.

increase in potential energy = [2]

(ii) At B, the kinetic energy of the ball is 2.5 J.

Calculate the kinetic energy of the ball at A.

kinetic energy = [1]

(iii) Calculate the speed of the golf ball just before it lands at C.

speed = [2]

BSS/2019/Prelim Exam/4E Physics Paper 2 [Turn over


www.KiasuExamPaper.com
93
22

BLANK PAGE

BSS/2019/Prelim Exam/4E Physics Paper 2


www.KiasuExamPaper.com
94
www.KiasuExamPaper.com
95
4E1 Phy Prelim Exam 2019 Suggested Marking Scheme

Paper 1 [40 marks]

Q1 Q2 Q3 Q4 Q5 Q6 Q7 Q8 Q9 Q10

B B A D C C D B B C

Q11 Q12 Q13 Q14 Q15 Q16 Q17 Q18 Q19 Q20

B C C B B C B B D D

Q21 Q22 Q23 Q24 Q25 Q26 Q27 Q28 Q29 Q30

C C C A B B A A A C

Q31 Q32 Q33 Q34 Q35 Q36 Q37 Q38 Q39 Q40

C C D B D A C A B D

Paper 2 Section A [50 marks]


1)(a) Weight of cargo = 130 x 10 = 1 300
0N A1
(b) Using a min. scale of 1 cm: 200 N B1
Diagram correctly drawn (either
wn (e
eitthe parallelogram
h rp
pa
ara
r llelog
og
gra method
am me
m eth orr ti
thod o ttip-to-tail
tip
ip-to-ta method)
tailil m ethod)
d) B1

TB 70
70 °

1300 N
130

TA 70 °

Allow ECF
CF for
for weight
wei of cargo from (a)
TA = TB = 1 900 N (accept values from 1 800 to 2 000 N) A1

2(a) For any system to be in equilibrium, B½


total clockwise moments is equal to total anticlockwise moments B1
about the same pivot B½
(b)(i) F = PuA
= 20000 u 4.0 u 10-2 m2 C1
= 800 N A1
(b)(ii) By Principle of Moments, Clockwise moments = anticlockwise moments
800 u 5 = 100 u 10 + W u 15 Allow ECF from (b)(i) C1
W = 200 N A1
1
www.KiasuExamPaper.com
96
4E1 Phy Prelim Exam 2019 Suggested Marking Scheme

(b)(iii) Mass M should be moved further from the pivot. B1


With a larger release pressure, a larger force is produced at the valve, which means the
clockwise moment is increased. Hence a larger anticlockwise moment is needed to
balance this increased clockwise moment. B1

3(a) As the volume of the ball decreases, the number of air particles per unit volume
increases. B1
Air particles in the ball collide more frequently with the inner walls of the ball. B1
The air particles then exert a larger force on the inner walls of the ball. Since pressure is
the force per unit area, the pressure exerted increases. B1

(b) P atm = 810 – 50 = 760 mm Hg C1


= 0.76 x 10 x 13 600
= 103 360 Pa § 103 000 PA (3 s.f.) A1

4(a) They are poor heat conductors / poor emitter


mitter off radiation
em rad
adia
ia
ati
tio
tion / poor
on absorber
or abso
bs orber of radiation
ra
hence heat transfer into and
nd outt of
of igloo
o iss slower.
slo
lowe
wer. B1

(b) Any one of the following


g ssets
ets of
of ans
answers:
nsswe
w rs:
x The Eskimoss can
ca
an curll ttheir
he
heir
ir llegs
eg
gs close
c osse to
cl t the
he
e body
body to
o reduce
red
e uce
e their
thei
theirr surface
ei surfa area B1
exposed
expo
po
pose
ed to
t the
he
e surroundings,
surrro
rou
un
nd
diing
ngs,
s, so
s heat
hea
ea
eat loss
lo
os
sss by
by radiation
radiatio
on from
from
m their
the bodies is slower. B1
x The
Th
he Eski
Eskimos
im
moos ca
ccan
an wearr ssilver
illv
veer / white
er whitte coloured
wh
whit coloured
co d cl
clot
clothes,
othe
othes, as
he a such clothes are B1
poorer
po
poor
orer
er emitters
emiitt
tter
errs of
o rad
radiation
diati
iati
iation
on so that
th
ha
at he
heat
eat loss
los
osss from their bodies is slower B1

(c) When
Wh
hen
en air around
nd
nd the
the
he eskimos
eskim
sk
s kiim
mos is
is warmed,
warm
warm
med, it becomes less dense and rises B½
Hot
ot ai
air
ir is
i ttrapped
rap
rapp
ped
ed a
att the to
top
p of tthe
he igl
igloo. B½
The cold
daair
ir w
which
hich
h iis
s de
denser ssinks into the cold sink and flows out of the igloo. B1

5(a) Specific heat capacity


ca is the amount of thermal energy required to raise the temperature
of 1.0 kg of water by 1.0 °C. B1
Specific latent heat of vaporisation is the amount of thermal energy required to change
1.0 kg of water into steam without a change of temperature. B1

(b) Heat needed = mFǻ7


= 1 200 x 4200 x (100 – 25) C1
8
= 3.78 x 10 J A1

(c) Heat needed = ml v


= 450 x 2.3 x 106 C1
= 1.04 x 109 J A1

2
www.KiasuExamPaper.com
97
4E1 Phy Prelim Exam 2019 Suggested Marking Scheme

6(a) Critical angle is the angle of incidence in the optically denser medium for which the angle
of refraction in the optically less dense medium is 90°. B1

(b)(i) In order for total internal reflection to occur, light ray must travel from an optically denser
medium to an optically less dense medium. B1
Therefore, glass core has a higher refractive index than plastic cladding. B1

sin 45°
(b)(ii) = 1.65 C1
sin r
r = 25.4° A1

1
(b)(iii) = 1.65 M1
sin c
c = 37.3° A1

6
7(a) voltmeter reading = x 12 C1
7
= 10.3 V (3 s.f.)
s.f.
f.)
f. A1

6
(b) The SRWHQWLDOGLIIHUHQFHDFURVVWKHNŸUHVLVWRULVJLYHQE\
IIHUHQFH
FH DFU
FH FURV
RVVWK
WKH NŸ
WK NŸ
NŸUHVLVWRULV
LV
V JLYHQ
Q E\ x 12 where
wh
6 + RTTH
H

R TH is the
he resista
resistance
tta
anc
nce of the
h thermistor.
therm
misstor. B1
As the rresistance
e re sist
sist
sistaance
eooff tthe
he
he thermistor
th
he
erm
rmiissto inFUHDVHVWKHSRWHQWLDOGLIIHUHQFHDFURVVWKHNŸ
tor in
nF
FUUHD
HDVH
VHV
HV
V WKHSRWHQ
KH
KH QWL
WLDO
DO GLILIIHUH
rresistor
re
esist
sttor
s o will d
de
decrease
ecr
ce ea
ase and
dvvo
voltmeter
oltltme
mette
er sh
sshows
ho
ow
ws a smaller
smal
sm alle
allerr reading.
le readin B1
A
Al
Alternative
te
ern
nattiv
ive explanation
ex
xp
pllan
anatio
on
As tthe
he resi
he resistance
is
sttance
t e of
of tthe
he tthermistor
he hermisto
he or in
inc
increases,
crea
e ses the total resistance of the circuit increases
and
nd the
th
he current
c rrrren
cu ent in
in tthe
he circuit
he circu
uit d
decreases.
ecr
c ease B1
Since
e SRWHQWLDOGLIIHUHQFHDFURVVWKHNŸresistor
SR
RWWH
HQWWLD
LDOGLIIIHU
HUHQ
HQFH
HQFHDFURV
FH is given by V = IR, if current
decreases,
ases,, potential
pot
oten
enti
ential difference
ti diffe decreases and voltmeter shows a smaller reading. B1

8(a) Wire X: Live wire


Wire Y: Neutral wire B2
1 mark for every two correct answers
Wire Z: Earth wire
Device W: fuse

(b)(i) The large current flows through the earth wire to the ground. B½
The fuse will melt (or circuit breaker will trip) and break the circuit. B½
Thus the high voltage source is disconnected from the water heater and the water inside
will no longer be live, preventing electric shock. B1

(b)(ii) green and yellow B1

3
www.KiasuExamPaper.com
98
4E1 Phy Prelim Exam 2019 Suggested Marking Scheme

(c) Total kWh used per month = 3.0 x 0.5 x 30


= 45 C1
Total cost = 45 x $0.30
= $13.50 A1

9(a) When the switch is closed, current flows through the coil and the iron core becomes an
electromagnet. B1
The electromagnet attracts the iron bolt, causing it to move to the left, allowing the door
to be opened. B1

(b) Any two of the following answers: B2


x Increase the number of turns of the coil of wire
x Increase the current flowing through the coil of wire.
re.
x Move the iron core closer to the iron bolt.

v
10(a)(i) Ȝ =
f
6 100
= C1
4.0 ×106
= 0.00153
0. 00153 m (3
.00 (3 s.f.)
.) or
s.ff.) or 1.53
1..53
1 53 mm
mm A1

(a)(ii)) distance
dist
sttan
s a ce
e = 3 x 0.01525
0.0
0. 01
152
525 = 0..0
0 00
0458 m (3 s.f.)
0.00458 s.f.) A1
2d
d
(b))(i)
(i) s =
t
2 × 0.004
00
0.00
04
6
6100
100 = C1
t
t = 0.000001311
0.00
000
0001
0 31
01311
= ȝV

  ȝV A1
(b)(ii) A shorter
er pulse
orte pulse 13
1 spaces after the reflected pulse. B1

(c) Either the emitted and transmitted pulses will be closer


OR the time interval between emitted and refelcted pulses will be shorter B1
The ultrasound travels a shorter distance at the same speed before it is reflected. B1

(d) The chemical solution will always be in contact with the bottom wall. B1
Any corrosion of the wall will be detected at the bottom wall first. B1

4
www.KiasuExamPaper.com
99
4E1 Phy Prelim Exam 2019 Suggested Marking Scheme

11 (a)(i) During an earthquake, the magnet moves in and out of coil, producing a change in
magnetic flux linking (in) the coil, thus inducing an electromotive force (e.m.f.) at the
solenoid. B1
The direction of the e.m.f. changes when the magnet moves in and out of the coil,
hence an alternating trace is produced. B1
The magnitude of the induced emf is proportional to the rate of change of magnetic
flux linkage, B1
hence a larger tremor will produce a trace with a higher amplitude. B1

(a)(ii)

(b)(i) Soft iron


oft iro
on B1

(ii) Vs = 50 x 2.0
50 2.0 = 100
10
00 V
0.75
0.75
75 x V p I p = VsIs
0.
0
0.75
.75
5x2
2.0
.0 x I p = 100
0 x 0.0024
0.00
0. 0024
2 C1
Ip = 0
0.
0.16
16 A A1

(iii) Any two


o answers
answ
answers from
f the following: B2
ƒ There is energy loss due to eddy currents formed in the core of the transformer.
x There is heat loss due to the resistance in the primary / secondary coils.
x There is magnetic flux leakage between the primary and secondary coil.

5
www.KiasuExamPaper.com
100
4E1 Phy Prelim Exam 2019 Suggested Marking Scheme

12 EITHER
(a)(i)
velocity / ms-1

20

time / s
0 4.0 9.0 11 19 25

– 25

x B1 mark for correct timings


ngs
x B1 mark for all correct shapes
sh
s
shap
hap
apes
x B1 mark for axes
ess ccorrectly
orrre
o rect labelled.
c lyy lab
ab
abelled.
d

(a)(ii) The velocity


ve
eloc ty of
oci of the
he car
tth decreases
car de
ca decr
creea
ases att a constant
se const
onst
on nt rate.
stan B1
v ି u
(iii) a =
t
0 ି 20
0
= C1
5
5.
.0
5.0
= 4.0
–4
4.
.0
deceleration
ece
elle
era
atiio
on
n = m 2
4.0 m/s A1

(b) F = ma
= 1 500 x 4.0 Allow ECF from (a)(iii) C1
= 6 000 N Do not accept – 6 000 N A1

(c) The opposing force such as air resistance acting on the car increases as velocity of
the car increases. B1
This causes the resultant force acting on the car to decrease, resulting in a decrease
in its acceleration. B1

6
www.KiasuExamPaper.com
101
4E1 Phy Prelim Exam 2019 Suggested Marking Scheme

12 OR
(a) During impact, part of the kinetic energy of the golf club is converted into kinetic energy
of the ball and sound and thermal energy as the club hits the ball. B1
The golf club continues moving with a smaller amount of kinetic energy. B1
The amount of total energy remains constant before and during impact. B1

(b) As the golf ball travels, its gravitational potential energy is changed into kinetic energy and
vice-versa. B1
This means that mgh = ½mv2 or v2 = 2gh or speed is independent of mass. B1

(c)(i) Increase in GPE = mgh


= 0.045 x 10 x 16 C1
= 7.2 J A1

(ii) KE at A + GPE at A = KE at B and GPE at B


= 2.5
5 + 7.2
2 Allow
A low
Al w ECF
ECF from (c)(i)
= 9.7
7J A1

(iii) KE a
att C = KE att B and
KE a nd GPE
an G E at
GP a B
0 045 x v2
½ x 0.045
0. = 9
9.
9.77 Allow
A lo
Allow
w ECF
ECF from
fr (c)(ii) C1
v = 20.8
8mm/s
/s
/s A1

7
www.KiasuExamPaper.com
102
www.KiasuExamPaper.com
103
Register No. Class
Name

emeer Secondary School Bendemeer Secondary School Bendemeer Secondary School Bendemeer Secondary School Bendemeer Secondary School Bendemeer Secondary School
emeer Secondary
ondary School Bendem Bendemeer Secondary School Bendemeer Secondary School Bendemeer Secondary School Bendemeer Secondary School Bendemeer Secondary School
emeer Secondary
onddaarry S
emeer Secondary
ond
ndaarry S
nd
Sc
School
Sc
chho
School
oo
chooo
oll B
oll B
BENDEMEER SECONDARY SCHOOL
Bendemeer
een ndem Secondary School Bendemeer Secondary School Bendemeer Secondary School Bendemeer Secondary School Bendemeer Secondary School
Bendemeer
endem Secondary School Bendemeer Secondary School Bendemeer Secondary School Bendemeer Secondary School Bendemeer Secondary School
en
emeer Secondary
on
nd
n daarry Scho
emeer Secondary
on
nd
n
S
Sc
School
daarry S
Sc
cho
hool
School
chool
ol B
ol
hoool
ol B 2019 PRELIMINARY EXAMINATION
Bendemeer
e d
en deem Secondary School Bendemeer Secondary School Bendemeer Secondary School Bendemeer Secondary School Bendemeer Secondary School
Bendemeer
end
en
end deem Secondary School Bendemeer Secondary School Bendemeer Secondary School Bendemeer Secondary School Bendemeer Secondary School

SECONDARY 4 EXPRESS
emeer Secondary
on
ndaary
ry SSchool
cchhoo
ool BBe
Bendemeer
en nd
dem Secondary School Bendemeer Secondary School Bendemeer Secondary School Bendemeer Secondary School Bendemeer Secondary School
emeer Secondary
on
nd
n dary S Sc
School
chhooo
oll B
Bendemeer
eennde
dem Secondary School Bendemeer Secondary School Bendemeer Secondary School Bendemeer Secondary School Bendemeer Secondary School
emeer Secondary
on
nd
n daarry S
Sc
School
cch
hoo
ho oll B
Bendemeer
end
en deem Secondary School Bendemeer Secondary School Bendemeer Secondary School Bendemeer Secondary School Bendemeer Secondary School
emeer Secondary
on
nd
n dary S
emeer Secondary
ond
Sc
School
daarry S
Sc
cho
hool
School
cho
hoo
ol B
oll B
PHYSICS PAPER 1
Bendemeer
een
ndem Secondary School Bendemeer Secondary School Bendemeer Secondary School Bendemeer Secondary School Bendemeer Secondary School
Bendemeer
endem Secondary School Bendemeer Secondary School Bendemeer Secondary School Bendemeer Secondary School Bendemeer Secondary School
en
emeer Secondary
ondary
emeer Secondary
ySSc
School
cho
hool
hool
ol B
ondary School Bendem
6091/01
Bendemeer
e dem Secondary School Bendemeer Secondary School Bendemeer Secondary School Bendemeer Secondary School Bendemeer Secondary School
en
Bendemeer Secondary School Bendemeer Secondary School Bendemeer Secondary School Bendemeer Secondary School Bendemeer Secondary School
emeer Secondary School Bendemeer Secondary School Bendemeer Secondary School Bendemeer Secondary School Bendemeer Secondary School Bendemeer Secondary School

DATE : 3 Sep 2019


DURATION : 1 hour

READ THESE INSTRUCTIONS FIRST

Write in 2B pencil.
Do not use paper clips, glue or correction fluid.
Write your name, class and register number on the question paper and OTAS sheet in the
spaces provided.

There are forty questions on this paper. Answer all questions. For each question there are
four possible answers A, B, C and D.
Choose the one you consider correct and record your choice in 2B pencil on the OTAS
sheet.

Read the instructions on the OTAS sheet very carefully.

Each correct answer will score one mark. A mark will not be deducted for a wrong answer.
Any rough working should be done in this booklet.
The use of an approved scientific calculator is expected, where appropriate.

40
This document consists of 17 printed pages.

[Turn over

www.KiasuExamPaper.com
104
2

1 A pipe has an approximate length of 60 cm and an approximate internal diameter of


3 cm.

Which instruments are the most suitable for measuring accurately the internal diameter
and the length?

A calipers and micrometer


B calipers and rule
C rule and micrometer
D rule and tape

2 A student measures the thickness of 20 sheets of paper with a micrometer.

The diagram shows the reading on the micrometer.

What is the average thickness of one sheet of paper?

A 0.119 mm B 0.144 mm C 0.169 mm D 0.171 mm

3 A body accelerates from rest at 4 m/s2 for 5 s.

What is its average speed?

A 0.8 m/s B 2.0 m/s C 10.0 m/s D 20.0 m/s

www.KiasuExamPaper.com [Turn over


105
3

4 A free-fall parachutist falls at a constant speed. He then opens his parachute and
continues to fall to Earth at a lower, constant speed.

Which diagram shows how the displacement of the parachutist varies with time?

5 The following chairs are drawn to the same scale.

Which one is the most stable?

A B C D

www.KiasuExamPaper.com [Turn over


106
4

6 A coin and a feather are released from rest in vacuum as shown in the diagram. It is
observed that both the coin and the feather reach the bottom of the cylinder at the same
time.

coin
feather

vacuum

Which of the following is/are correct deduction(s) from this experiment?

I The masses of the coin and the feather are identical in vacuum.
II The coin and the feather fall with the same acceleration in vacuum.
III The gravitational forces acting on the coin and the feather in vacuum are
identical.

A I only B II only C II and III D III only

7 Two balls are dropped one after another from the same height. Assuming that the air
resistance is negligible, which of the following statements is true?

A The two balls drop with a constant distance between them.


B The two balls drop with a constant speed.
C The two balls get closer as they descend.
D The two balls get further away as they descend.

8 5000 kg of iron is melted and mixed with 2.0 m3 of molten copper.

If the density of molten iron and molten copper are 7500 kg/m3 and 9000 kg/m3
respectively, what is the approximate density of the mixture?

A 7.5 g/cm3 B 8.3 kg/m3 C 8300 kg/m3 D 8600 kg/m3

www.KiasuExamPaper.com [Turn over


107
5

9 Two forces act on a rod as shown in the diagram.

Which effect will be produced by these two forces?

A both rotation and movement in a straight line


B movement in a straight line only
C no effect, because the forces are balanced
D rotation only

10 An object is experiencing a pressure of 800 mmHg.

Express this pressure in Pa, given that the density of mercury is 13,600 kg/m3.

A 108,800 Pa
B 170,000 Pa
C 170,000,000 Pa
D 108,800,000 Pa

11 An engineer designs a dam wall for a reservoir.

Which factor determines the pressure at X?

A the depth of the water in the reservoir


B the length of the reservoir
C the surface area of the reservoir
D the thickness of the dam wall

www.KiasuExamPaper.com [Turn over


108
6

12 A parachutist opens his parachute and falls to Earth at constant speed.

What is the principal energy conversion taking place as he falls?

A kinetic energy Î potential energy


B kinetic energy Î thermal energy (heat)
C potential energy Î kinetic energy
D potential energy Î thermal energy (heat)

13 A girl of weight 500 N runs up a flight of stairs in 10 seconds. The vertical height of the
stairs is 5 m.

What is the average useful power developed by the girl?

A 50 W B 100 W C 250 W D 1000 W

14 Which change is condensation?

www.KiasuExamPaper.com [Turn over


109
7

15 A fixed mass of gas is trapped in a cylinder with a movable piston. The piston is pushed
inward slowly to decrease the volume and yet maintain a constant temperature in the
cylinder.

Which of the following about the gas molecules is correct?

average speed average distance apart


A increase decrease
B increase no change
C no change decrease
D no change no change

16 How is heat transferred from the Sun to Earth?

I conduction
II convection
III radiation

A I only B III only C I and II D I and III

17 In the diagram, two copper cans X and Y with outer surface of different colours and
textures are filled with the same amount of water at room temperature and heated by
heaters of the same power.

immersion
temperature heater
polished
probe dull black chrome
surface surface
X Y

Which of the following statements is correct?

A Water in both cans take the same amount of time to boil because the texture
of the outer surface will not affect the rate of energy absorbed by the water.
B Water in X boils faster because the dull black surface is a good absorber of
radiant heat.
C Water in Y boils faster because the polished chrome surface is a poor absorber
of radiant heat.
D Water in Y boils faster because the polished chrome surface is a poor radiator.

18 An electric kettle contains 1500 g of liquid and is powered by a 0.5 kW electric element. If
the temperature rises at 5qC every minute, what is the specific heat capacity of the liquid?

A 6.7 PJ/(kgqC) B 4 mJ/(kgqC) C 4000 J/(kgqC) D 4200 J/(kgqC)

www.KiasuExamPaper.com [Turn over


110
8

19 According to the kinetic theory, matter is made up of very small particles in a constant
state of motion.

Which row best describes the particle behaviour in the liquid state?

forces between
motion of particles
particles
A strong move randomly at high speeds
B strong vibrate but can move freely
C strong vibrate to and from around a fixed position
D weak move randomly at high speed

20 A liquid evaporates rapidly.

Why does this cause it to cool?

A Air molecules remove heat by contact with the liquid surface.


B Energy is lost by convection currents.
C Some of the most energetic molecules leave the liquid.
D The molecules have less room to move around.

21 The same transverse wave is represented by the following graphs.


displacement displacement
a
b

time distance

Which of the following gives the speed of the wave?

a b 1
A ab B C D
b a ab

www.KiasuExamPaper.com [Turn over


111
9

22 Astronaut X is hammering on one side of a satellite.

Y
X

Astronaut Y on the other side of the satellite will not hear the hammering because

A the satellite is blocking the sound travel.


B the sound is refracted away from him.
C the sound cannot be produced in space.
D the sound cannot travel in space.

23 The diagram shows a ray of light from one point on a lamp striking a plane mirror.

The image of the point on the lamp formed by the mirror is

A at P and is real.
B at P and is virtual.
C at R and is real.
D at R and is virtual.

www.KiasuExamPaper.com [Turn over


112
10

24 A piece of glass was placed on top of a polished mirrored surface as shown in the diagram
below.

What is the critical angle of the glass?

A 28.1° B 41.8° C 45.0° D 56.3°

25 Which of the following uses microwaves?

I oven
II mobile phone
III metal detector

A I and II B I and III C II and III D I, II and III

26 A student stands 50 m from a wall and knocks two wooden blocks together. When the
frequency of knocking is 3 knocks per second, the echo of a knock is heard at the instant
of the next one.

What is the speed of sound in air?

A 150 m/s B 200 m/s C 300 m/s D 350 m/s

www.KiasuExamPaper.com [Turn over


113
11

27 The diagrams represent two different sound waves.

How do the frequency and pitch of P compare with the frequency and pitch of Q?

frequency of P pitch of P
A greater than Q higher than Q
B greater than Q same as Q
C same as Q higher than Q
D same as Q same as Q

28 A piece of polythene is rubbed with a cloth duster. The polythene becomes negatively
charged and the cloth becomes positively charged.

What happens to the polythene and to the cloth to cause this?

polythene cloth
A gains electrons gains protons
B gains electrons loses electrons
C loses protons gains protons
D loses protons loses electrons

www.KiasuExamPaper.com [Turn over


114
12

29 The diagram below shows the pattern of an electric field produced by three charged
spheres.

Which of the following correctly shows the charge on each sphere?

P Q R
A – – +
B + – +
C – + –
D + + –

30 60 C of charge passes through a resistor in 120 seconds.

The energy converted in the resistor is 5 J every second.

What is the potential difference across the resistor?

A 5V B 10 V C 12 V D 24 V

31 Three wires X, Y and Z are made from the same metal. Their dimensions are listed below.

length / m cross-sectional area / mm2


X 4 2
Y 5 1
Z 8 6

Arrange them in ascending order of their resistances.

A X, Y, Z B X, Z, Y C Y, X, Z D Z, X, Y

www.KiasuExamPaper.com [Turn over


115
13

32 When a filament lamp is switched on, there is a current in the lamp. As the temperature of
the filament rises, its resistance changes.

Which pair of graphs shows how the resistance R of the filament and the current I vary
with time after the lamp is switched on?

33 Three identical lamps and three identical ammeters are connected as shown.

The readings on the ammeters are I 1 , I 2 and I 3 .

How are the readings related?

A I1 = I2 = I3
B I1 > I 2 and I 2 = I 3
C I1 > I3 > I2
D I1 > I2 > I3

www.KiasuExamPaper.com [Turn over


116
14

34 The circuit diagram shows a variable resistor R connected in parallel to the lower half of a
potential divider.

The resistance of R increases.

What happens to the voltmeter readings?

reading on V 1 reading on V 2
A decreases decreases
B decreases increases
C increases decreases
D increases increases

35 The table below shows the typical daily electrical usage in an office.

Appliance Quantity Number of hours used per day


40 W lamp 10 15
3 kW air-conditioner 2 12

If the cost of electricity is $0.20 per kWh, calculate the total electrical bill per day.

A $12.14 B $15.60 C $1214.40 D $15 600

www.KiasuExamPaper.com [Turn over


117
15

36 The diagram shows an old vacuum cleaner with plastic wheels and a metal case.

The plug of the vacuum cleaner is wrongly wired as shown.

What is the effect of using the plug wired this way?

A The fuse in the plug blows.


B The metal case becomes live.
C The vacuum cleaner catches fire.
D The vacuum cleaner does not work.

37 Which material is used for the needle of a plotting compass?

A aluminium B brass C iron D steel

www.KiasuExamPaper.com [Turn over


118
16

38 An alternating supply with a period of 0.020 s is connected to a cathode-ray oscilloscope


(c.r.o.).

What is the time-base setting of the c.r.o.?

A 0.2 ms/cm B 0.5 ms/cm C 2 ms/cm D 5 ms/cm

39 An output voltage is produced as the coil in the diagram rotates.

One side of the coil is labelled P.

During the rotation, when is the output voltage zero?

orientation of coil position of P


A horizontal near the N-pole only
B horizontal near the N-pole or near the S-pole
C vertical at the top only
D vertical at the top or bottom

www.KiasuExamPaper.com [Turn over


119
17

40 Which statement about a transformer is correct?

A The core of the transformer is made of iron because iron is a good electrical
conductor.
B The direction of the induced e.m.f. in the secondary coil opposes the change
that produces it.
C The transformer converts alternating current to direct current.
D The transformer converts direct current to alternating current.

END OF PAPER

www.KiasuExamPaper.com [Turn over


120
Register No. Class
Name

emeer Secondary School Bendemeer Secondary School Bendemeer Secondary School Bendemeer Secondary School Bendemeer Secondary School Bendemeer Secondary School
emeer Secondary
ondary School Bendem Bendemeer Secondary School Bendemeer Secondary School Bendemeer Secondary School Bendemeer Secondary School Bendemeer Secondary School
emeer Secondary
onddaarry S
emeer Secondary
ond
ndaarry S
nd
Sc
School
Sc
chho
School
oo
chooo
oll B
oll B
BENDEMEER SECONDARY SCHOOL
Bendemeer
een ndem Secondary School Bendemeer Secondary School Bendemeer Secondary School Bendemeer Secondary School Bendemeer Secondary School
Bendemeer
endem Secondary School Bendemeer Secondary School Bendemeer Secondary School Bendemeer Secondary School Bendemeer Secondary School
en
emeer Secondary
on
nd
n daarry Scho
emeer Secondary
on
nd
n
S
Sc
School
daarry S
Sc
cho
hool
School
chool
ol B
ol
hoool
ol B 2019 PRELIMINARY EXAMINATION
Bendemeer
e d
en deem Secondary School Bendemeer Secondary School Bendemeer Secondary School Bendemeer Secondary School Bendemeer Secondary School
Bendemeer
end
en
end deem Secondary School Bendemeer Secondary School Bendemeer Secondary School Bendemeer Secondary School Bendemeer Secondary School

SECONDARY 4 EXPRESS
emeer Secondary
on
ndaary
ry SSchool
cchhoo
ool BBe
Bendemeer
en nd
dem Secondary School Bendemeer Secondary School Bendemeer Secondary School Bendemeer Secondary School Bendemeer Secondary School
emeer Secondary
on
nd
n dary S Sc
School
chhooo
oll B
Bendemeer
eennde
dem Secondary School Bendemeer Secondary School Bendemeer Secondary School Bendemeer Secondary School Bendemeer Secondary School
emeer Secondary
on
nd
n daarry S
Sc
School
cch
hoo
ho oll B
Bendemeer
end
en deem Secondary School Bendemeer Secondary School Bendemeer Secondary School Bendemeer Secondary School Bendemeer Secondary School
emeer Secondary
on
nd
n dary S
emeer Secondary
ond
Sc
School
daarry S
Sc
cho
hool
School
cho
hoo
ol B
oll B
PHYSICS PAPER 2
Bendemeer
een
ndem Secondary School Bendemeer Secondary School Bendemeer Secondary School Bendemeer Secondary School Bendemeer Secondary School
Bendemeer
endem Secondary School Bendemeer Secondary School Bendemeer Secondary School Bendemeer Secondary School Bendemeer Secondary School
en
emeer Secondary
ondary
emeer Secondary
ySSc
School
cho
hool
hool
ol B
ondary School Bendem
6091/02
Bendemeer
e dem Secondary School Bendemeer Secondary School Bendemeer Secondary School Bendemeer Secondary School Bendemeer Secondary School
en
Bendemeer Secondary School Bendemeer Secondary School Bendemeer Secondary School Bendemeer Secondary School Bendemeer Secondary School
emeer Secondary School Bendemeer Secondary School Bendemeer Secondary School Bendemeer Secondary School Bendemeer Secondary School Bendemeer Secondary School

DATE : 2 Sep 2019


DURATION : 1 hour 45 minutes

READ THESE INSTRUCTIONS FIRST

Write your name, class and register number on the work you hand in.
Write in dark blue or black pen.
You may use a 2B pencil for any diagrams or graphs.
Do not use paper clips, glue or correction fluid.

Section A
Answer all questions.

Section B
Answer all questions. Question 11 has a choice of parts to answer.

Candidates are reminded that all quantitative answers should include appropriate units.
The use of an approved scientific calculator is expected, where appropriate.
Candidates are advised to show all their working in a clear and orderly manner, as more
marks are awarded for sound use of Physics than for correct answers.

At the end of the examination, fasten all your work securely together.
The number of marks is given in brackets [ ] at the end of each question or part question.

80
This document consists of 21 printed pages.

[Turn over

www.KiasuExamPaper.com
121
2

Section A

Answer all the questions in this section.

1 The contractor of the school’s upgrading project often uses a crane to lift construction
materials from the ground to a higher-level floor.

On one occasion, the crane is used to lift a long and heavy metal bar. Fig. 1.1 shows
part of the lifting mechanism comprising a main cable AB, two other cables BC and
BD, and the metal bar. BC and BD make an angle of 100°.

3000 N

100o
C D
metal bar

Fig. 1.1

When the metal bar is being lifted vertically at a constant speed, the tension in the
main cable AB is 3000 N. Take gravitational field strength g to be 10 N/kg.

(a) Given that the tension of cables BC and BD are equal, use a scaled drawing to
determine the tension in each of these two cables.

Scale: ……………………………………

tension = ….….……….…………….………….. [4]

www.KiasuExamPaper.com [Turn over


122
3

(b) Calculate the total mass of the three cables and metal bar.

total mass = ….….……….…………….………….. [2]

2 An MRT train took 7 minutes to travel from Sembawang station to Admiralty station.
During this time, the train was travelling in a straight line and it reached a top speed
of 80 km/h.

Fig 2.1 shows the velocity-time graph of the train.

velocity / km/h
80

60

40

20

0 time / min
1 2 3 4 5 6 7
Fig. 2.1

(a) Calculate the average speed of the train (in km/h) from 0 to 7 minutes.

average speed = ….….……….…………….…… km/h [2]

www.KiasuExamPaper.com [Turn over


123
4

(b) Calculate the deceleration of the train (in km/h2) from 5 to 7 minutes.

deceleration = ….….……….………...……… km/h2 [2]

(c) During the time interval of 1 to 5 minutes, a passenger in the train stood without
holding onto the rails or leaning on anything.

What are the forces acting on the passenger during this time interval?

………………………………………………………………………………………

………………………………………………………………………………………

……………………………………………………………………………………… [2]

3 Fig. 3.1 shows an optical fibre cable probe used in medical procedures.

outer coating

optical fibre core

Fig. 3.1

www.KiasuExamPaper.com [Turn over


124
5

Fig. 3.2 shows the cross-section of one part of the cable probe with a ray of light
entering the fibre core at point X.

optical fibre cable probe


outer coating
50° optical
X 30.8° fibre
core

Y outer coating

Fig. 3.2

(a) Calculate the refractive index of the optical fibre.

refractive index = ….….……….…………….………….. [2]

(b) Calculate the critical angle c for this optical fibre.

critical angle c = ….….……….…………….………….. [2]

(c) State and explain what will happen to the ray at point Y.

………………………………………………………………………………………

………………………………………………………………………………………

……………………………………………………………………………………… [2]

www.KiasuExamPaper.com [Turn over


125
6

4 Fig. 4.1 shows an incorrect electromagnetic spectrum drawn by a student. The parts
of the spectrum and the wavelengths are in the wrong order. The values of the
wavelengths do not match the correct parts of the spectrum.

short long
wavelength wavelength
radio gamma
microwaves ultraviolet infra-red X-rays visible
waves rays
103 m 10-14 m 10-10 m 10-8 m 10-2 m 10-6 m 10-5 m

Fig. 4.1

(a) On Fig. 4.2, complete the table of the electromagnetic spectrum. Radio waves
and their correct wavelength have been inserted for you.

short long
wavelength wavelength
radio
waves
103 m

Fig. 4.2

[3]

(b) State the speed of all electromagnetic waves in vacuum.

……………………………………………………………………………………… [1]

(c) State two uses of infra-red radiation.

1. …………………………………………………………………………………

2. ………………………………………………………………………………… [2]

www.KiasuExamPaper.com [Turn over


126
7

5 Fig. 5.1 shows a student touching the metal dome of a Van de Graaff generator. When
the generator is switched on, the metal dome becomes negatively charged.

Van de
Graaff
generator

earthed metal
dome

polystyrene
box

Fig. 5.1

(a) Explain why the student’s hair stands on end when the generator is switched on.

………………………………………………………………………………………

………………………………………………………………………………………

……………………………………………………………………………………… [2]

(b) (i) When the potential difference between the student and a nearby earthed
metal dome reaches 15 kV, a spark jumps between the student and the
earthed dome. The spark transforms 0.3 J of energy into heat, light and
sound.

Calculate the charge carried by the spark.

charge = ….….……….…………….………….. [2]

(ii) State the physical quantity that represents the rate of transfer of charge.

……………………………………………………………………………………… [1]

www.KiasuExamPaper.com [Turn over


127
8

6 Fig. 6.1 shows a piece of video tape passing under the recording head of a video
recorder. An alternating current is passed through the coil. The video tape is coated
with a magnetic material which becomes magnetised.

Fig. 6.1

(a) (i) Explain why the tape becomes magnetised.

……………………………………………………………………………….

……………………………………………………………………………….

………………………………………………………………………………. [1]

(ii) Fig. 6.1 shows that the sections of the video tape are magnetised in
opposite directions. Explain the cause of this occurrence.

……………………………………………………………………………….

………………………………………………………………………………. [1]

(iii) The tape is moved faster past the recording head. State how this changes
the pattern on the tape.

……………………………………………………………………………….

………………………………………………………………………………. [1]

(b) (i) Explain why the coating must be of a permanent magnetic material.

……………………………………………………………………………….

………………………………………………………………………………. [1]

www.KiasuExamPaper.com [Turn over


128
9

(ii) State the name of a permanent magnetic material.

………………………………………………………………………………. [1]

7 Fig. 7.1 shows a circuit in which all switches S1, S2 and S3 are open.

Fig. 7.1

(a) Calculate the effective resistance between points P and Q when S1, S2 and S3
are closed.

effective resistance = ….….……….…………….………….. [2]

(b) Calculate the resistance of the lamp when S1, S2 and S3 are closed and the
ammeter reads 2.0 A

resistance = ….….……….…………….………….. [2]

www.KiasuExamPaper.com [Turn over


129
10

(c) Calculate the energy dissipated by the lamp in 2 min when S1, S2 and S3 are
closed.

energy dissipated = ….….……….…………….………….. [2]

8 Fig. 8.1 shows two coils wound on an iron ring. One coil is connected in series to a
switch and a d.c. supply, and the other is connected to a very sensitive centre-zero
voltmeter.

Fig. 8.1

At first the switch is open, as shown in Fig. 8.1.

(a) The following actions are taken in turn.

Describe and explain what happens to the reading on the voltmeter in each
case.

(i) The switch is closed.

……………………………………………………………………………….

……………………………………………………………………………….

……………………………………………………………………………….

………………………………………………………………………………. [4]

www.KiasuExamPaper.com [Turn over


130
11

(ii) The switch is left closed for a long time.

……………………………………………………………………………….

………………………………………………………………………………. [1]

(iii) The switch is opened.

……………………………………………………………………………….

……………………………………………………………………………….

………………………………………………………………………………. [2]

(b) State why an a.c. supply, rather than a d.c. supply, is used for a transformer.

………………………………………………………………………………………

……………………………………………………………………………………… [1]

(c) State two ways how the turning effect on a current-carrying coil in a d.c. motor
can be increased.

1. …………………………………………………………………………………

2. ………………………………………………………………………………… [2]

www.KiasuExamPaper.com [Turn over


131
12

Section B

Answer all the questions from this section.


Answer only one of the two alternative questions in Question 11.

9 In an experiment, different sized metal pellets are fired from an air rifle towards an
8.0 kg block of plastic suspended from the top of a ceiling. The initial position of the
block is shown in Fig. 9.1, and when the pellet hits the plastic block, the block is
displaced as shown in Fig. 9.2.

Fig. 9.1 Fig. 9.2

The information obtained from the experiment is shown in Fig. 9.3.

time taken for maximum


speed of pellet depth of
mass of pellet to increase in
just before it hits penetration by
pellet, m come to a height of plastic
plastic block, v pellet, d
stop, t block, h
0.050 kg 40 m/s 0.15 m 0.025 s 0.348 m
0.025 kg 56 m/s 0.12 m 0.020 s 0.292 m
0.020 kg 62 m/s 0.11 m 0.018 s 0.274 m

Fig. 9.3

(a) Calculate the kinetic energy of the 0.025 kg pellet just before it hits the block of
plastic.

kinetic energy = ….….……….…………….………….. [2]

www.KiasuExamPaper.com [Turn over


132
13

(b) Calculate the deceleration and hence the resistive force acting on the 0.025 kg
pellet.

deceleration = ….….……….…………….………….. [2]

resistive force = ….….……….…………….………….. [2]

(c) Calculate the work done by the 0.025 kg pellet against friction.

work done = ….….……….…………….………….. [2]

(d) Show that there is a discrepancy between the experimental and theoretical
values for the increase in height of the plastic block.

[2]

www.KiasuExamPaper.com [Turn over


133
14

10 Fig. 10.1 shows a refrigerator. The refrigerator walls are made of smooth white metal
with a layer of polystyrene foam between the inside and outside walls.

Fig. 10.1

(a) (i) Describe how the polystyrene foam insulates the refrigerator.

……………………………………………………………………………….

……………………………………………………………………………….

……………………………………………………………………………….

………………………………………………………………………………. [2]

(ii) Explain how the cooling unit at the top cools all the contents of the
refrigerator.

……………………………………………………………………………….

……………………………………………………………………………….

……………………………………………………………………………….

……………………………………………………………………………….

……………………………………………………………………………….

………………………………………………………………………………. [2]

www.KiasuExamPaper.com [Turn over


134
15

(iii) The inside wall radiates a small amount of thermal energy (infra-red
radiation). State how the colours of the surface affects the amount of
energy radiated.

……………………………………………………………………………….

………………………………………………………………………………. [1]

(b) A beaker contains 100 g of water at temperature of 50°C. An ice cube is removed
from a refrigerator and dropped into the water. The ice cube is initially all solid
at 0°C and has a mass of 3.0 g.

When the ice has melted, the water is stirred and has a temperature of 46°C.

The specific heat capacity of water is 4.2 J/(g°C).

In this question, ignore heat loss to the beaker and surroundings.

(i) Calculate the energy lost by the water as it cools from 50°C to 46°C.

energy = ….….……….…………….………….. [2]

(ii) The melted ice (water) from the ice cube gains energy as it warms from
0°C to 46°C. Calculate the thermal energy needed for this rise in
temperature.

energy = ….….……….…………….………….. [1]

www.KiasuExamPaper.com [Turn over


135
16

(iii) Use your answers to (i) and (ii) to determine the latent heat needed to melt
the ice cube.

energy = ….….……….…………….………….. [1]

(iv) Determine the specific latent heat of fusion of water.

specific latent heat of fusion = ….….……….…………….………….. [1]

www.KiasuExamPaper.com [Turn over


136
17

11 EITHER

Fig. 11.1 shows a man using an exercise machine.

F end of horizontal steel bar


0.25 m
padding for bar

pivot
metal frame
0.28 m
C
250 N
weights
0.2 m

250 N

Fig. 11.1

(a) State the Principle of Moments.

………………………………………………………………………………………

………………………………………………………………………………………

………………………………………………………………………………………

……………………………………………………………………………………… [2]

(b) Define the moment of a force and state its SI unit.

………………………………………………………………………………………

………………………………………………………………………………………

………………………………………………………………………………………

……………………………………………………………………………………… [2]

www.KiasuExamPaper.com [Turn over


137
18

(c) The heels press against the pad with a force F and cause a turning effect about
the pivot. The weight of the 250 N weights acts through the centre of gravity C.

Calculate the

(i) number of weights supported at C if each piece has a mass of 5 kg;

number of weights = ….….……….…………….………….. [2]

(ii) moment due to the 250 N weights about the pivot;

moment = ….….……….…………….………….. [2]

(iii) force F.

F = ….….……….…………….………….. [2]

www.KiasuExamPaper.com [Turn over


138
19

OR

An appliance is connected to the live, neutral and earth conductors of the mains
supply.

The current in the circuit is 4.0 A and the rating of the fuse is 5 A.

(a) Explain what is meant by

(i) live;

……………………………………………………………………………….

………………………………………………………………………………. [1]

(ii) neutral.

……………………………………………………………………………….

………………………………………………………………………………. [1]

(b) When a fault occurs in the appliance, no damage or injury is caused provided
that the correct fuse is used and the metal case is connected to earth.

(i) The 5 A fuse is replaced by a 30 A fuse.

Explain why this presents a risk of damage or injury.

……………………………………………………………………………….

……………………………………………………………………………….

……………………………………………………………………………….

……………………………………………………………………………….

………………………………………………………………………………. [2]

www.KiasuExamPaper.com [Turn over


139
20

(ii) The earth conductor is not connected to the metal case.

Explain why this presents a risk of damage or injury.

……………………………………………………………………………….

……………………………………………………………………………….

……………………………………………………………………………….

……………………………………………………………………………….

………………………………………………………………………………. [2]

(c) State one advantage of using a circuit breaker rather than a fuse to protect the
appliance.

………………………………………………………………………………………

……………………………………………………………………………………… [1]

www.KiasuExamPaper.com [Turn over


140
21

(d) Fig. 11.2 shows a circuit connected to a mains voltage of 220 V.

Fig. 11.2

Name the wires X, Y, and Z.

Wire X …………………………………………….

Wire Y …………………………………………….

Wire Z ……………………………………………. [3]

END OF PAPER

www.KiasuExamPaper.com [Turn over


141
www.KiasuExamPaper.com
142
Answer Key

2019 Prelim Examination


Sec 4 Express
Physics
Paper 1

1 B 11 A 21 C 31 D
2 B 12 D 22 D 32
3 B
3 C 13 C 233 B 33 D
4 B 14 D 24 B 34
3 B
5 B 15 C 25 A 35
35 B
6 B 16 B 266 C 36 B
7 D 17 D 27
27 D 37 D
8 D 18 C 288 B 38 D
9 D 19
19 B 29 D 39 D
10 A 20
20 C 30
3 B 40
4 B

www.KiasuExamPaper.com
143
Answer Key

2019 Preliminary Examination


Sec 4 Express
Physics
Paper 2

Section A

1 (a)

[1 forr sc
[1]
[1 sscale
ale ((1
al 1 cmcm : 1100000
0 00 N))
[1]
1] forr co
[[1 correct
orr
rrec ddrawing
eecct dr raw
a ing g of
of forces
fo
orrcees
[1]
1] for
[1 fo correct
co
o labelling
laabe
bell
llin
ing
Tension
[1] T
Teens
nsiio
on – ac aaccept 2200
200 N to 22400
ccceepptt 220
20 400
400N

(b)
b) W = mg mg
3000
330
00 0 = m x 10
00 [1]
300
m = 3000 kg
g [1]

2 (a) Total
T
To tal distance
di = area under speed-time graph
= 00.5 x (7+4) x 80 / 60
= 7.333 km [1]
average speed = 7.333 / (7/60)
= 62.9 km/h [1]

(b) a = (v – u)/t
= (0 – 80) / (2/60) [1]
= –2400 km/h2

Deceleration = 2400 km/h2 [1]

(c) Weight and [1]


normal reaction force / contact force [1]

www.KiasuExamPaper.com
144
3 (a) sin ݅
ߟ=
sin ‫ݎ‬
ୱ୧୬ ହ଴ι
= ୱ୧୬ ଷ଴.଼ι
[1]
= 1.50 (3sf)
[1]
(b) 1
ߟ=
sin ܿ
1
sin ܿ =
1.50 [1]

c = 41.8° (3sf) [1]

(c) ce at point
Total internal reflection will take place nt Y
Y.. [1]

Angle of incidence at pointt Y = 180q  990q


0q  330.8q . 2q
0 8q = 559.2q
0. 9.2

Explanation: angle
gle of incidence
inc ncee at
n iden at point
po
p nt Y is
oiin is greater
grrea
g terr than
eate
te h critical [1]
n the
than the
angle

4 (a) correc ct or
correct rde
d r off powerss of
order o ten [1]
ccorrect
co
correc
o ct oor rd
deer of
order o sspectrum
peectru
p ctru
ct rum [2]
(exc
(exchaang
ngin
gin
ing ttw
(exchanging wo pa
two parts oror m ovin
ov ing
movingng on
oone
ne part
paart produces
pro
rodu
d ces correct order
[1
1])
[1])])

short
rt long
wavelength
gth wavelength
gamma rayss X-
X
X-rays
-rays ultr
ul
ultraviolet
traviole
tr visible infra-red microwaves radio waves
10-14 m 0-10
10 10
m 110-8 m 10-6 m 10-5 m 10-2 m 103 m

(b) 3 x 108 m/s [1]

(c) any TWO from cooking, intruder (accept burglar, motion, security) [2]
alarms,
any specific sensor or medical use, remote controls, night vision,
heating (e.g. just heating or heating a greenhouse, heating a solar
panel),
detect temp., see in fog,
detect hot bodies, IR astronomy, distance measurement

www.KiasuExamPaper.com
145
5 (a) Excess electrons from the negatively charged Van de Graaff
generator flows to the student, causing his whole body to be
negatively charged. [1]
As his hair strands attain a negative charge, they are repelled from
the body as like charges repel, thus making the hair stand on ends. [1]

(b) (i) V=W/Q


15,000 = 0.3 / Q [1]
Q = 2 x 10-5 C [1]

(ii) Current [1]

6 (a) (i) gnet field in


When current is flowing in the coil, it generates a magnetic
and around the coil.
Hence the recording head is magnetised as an elect tro
roma
maagn
g et which
electromagnet,
in turn magnetises the tape. [1]

(ii) magnetism / magnetic field orr current or ppoles


o es on
ol on head
ad reverses
rev
ever
ersses /
er
changes direction [1]

(iii) each direction / one ccycle


yccle
y l longer
longe
geer (o
g on tta
(on ap
pee)
tape) [[1]

(b) (i) d to keep record


need reeccord
d / tape
pee stored
sto
t red
d [1]

(ii) s eel et
st
steel eetc.
cc.. [1]

7 (a)
(a) 1 1 1
= +
ܴ ܴଵ ܴଶ

=଺+ଷ
ଵ [1]

[1]
5 ȍ
5 ȍ

(b) V = IR
12 = 2 x R
Total R = 6 ȍ [1]

Resistance of lamp = 6 – 2 = 4 ȍ [1]

(c) E = Pt = I2R x t
= 22 x 4 x (2 x 60) [1]
= 1920 J [1]

www.KiasuExamPaper.com
146
8 (a) (i) meter deflects one way then returns to zero [1]
(current in left coil creates a) magnetic field [1]
magnetic field / flux cuts right-hand coil or changes (and induces a [1]
voltage)
meter returns to zero because no more change in flux [1]

(ii) meter remains at / returns to zero and no change in flux / no flux


cuts coil [1]

(iii) meter deflects in opposite direction [1]


field change in opposite direction or field / flux cuts in opposite
direction [1]

(b) direction of current / magnetic field constantly


tly changes
chaang
nges
es [1]

(c) inserting a soft iron core into the coil


oil any
increasing the number of turnsns in the coil two
increasing the current in the coil [2]

Section B

9 (a) KE = ½ m
K mvv2
= 0.
00.5
5 x 0.
00.025
.02
025 x 5
5662 [1]
= 39
39.2 J [1]

(b)
(b
b) a = (v(v-u)/t
v-u)
u /t
= (0
( – 56 556)/0.02
56)/
6))//00..022 [1]
= –2
––2800
280 m/s
00 m/
m / s2
Deceleration
D
De cceele
cele
lerationn = 22800800 m/s2
80 [1]

F = ma
= 0.
00.025
025 x 2800 [1]
= 70 N [1]

(c) work done = F x d


= 70 x 0.12 [1]
= 8.4 J [1]

(d) By the Principle of Conservation of Energy


KE = W + GPE
39.2 = 8.4 + GPE [1]
GPE = 30.8 J = mgh
h = (30.8) / (0.025+8.0)(10)
= 0.384 m (theoretical value) [1]

www.KiasuExamPaper.com
147
10 (a) (i) The polystyrene foam which traps air is a good heat insulator. [1]
It is a good insulator and it prevents heat conduction. [1]

(ii) The cooling unit cools the air at the top.


As the cool air contracts and sinks, the warm air rises. [1]
This movement of air sets up a convention current.
This convection current cools all the contents of the refrigerator. [1]

(iii) Shiny and smooth surfaces are poorer emitters compared to black
and dull surfaces.
Hence only a small amount of energy is radiated by thee in
inside wall. [1]

(b) (i) ( PFǻș


= 100 x 4.2 x (50 – 46) [1]
= 1680 J [1]

(ii) ( PFǻș
= 3.0 x 4.2 x (46 – 0)
= 580 J (3sf) [1]
[

(iii) Latent
nt heat required d = 1680
reequuired 168
68
6 80 – 580
58
80
= 1100
000
0J [1]

(iv)
v)) specific
(iiv spec
sp e iffiicc llatent
aten
at
ten heat
ent h
he at ooff ffu
eat fusion
usion
on l f = L f / m
on
= 11
1100 0 / 3.03.0
= 367
367 JJ/
36 J/g
/g [1]

EITHER

11 (a) Principle
Th Prin
The
Th nci
cipl
plee of M
pl Moments states that when a body is in
equilibrium,
eq
qui
uili
libr
li ium, the sum of the clockwise moments about a pivot is
briu
briu [1]
equal
eq
qu the sum of the anticlockwise moments about the same
uaal to th
pivot.
pivot [1]

(b) The moment of a force is the product of the force and the
perpendicular distance from the pivot to the line of action of the
force. [1]

SI unit: Nm [1]

(c) (i) Weight of each piece W = m x g


= 5 x 10
= 50 N [1]

No. of weights at C = 250 / 50


=5 [1]

www.KiasuExamPaper.com
148
(ii) moment = F x d
= 250 x 0.2 [1]
= 50 Nm [1]

(iii) By the Principle of Moments


sum of clockwise moments = sum of the anticlockwise moments
50 = F x 0.25 [1]
F = 200 N [1]

OR

11 (a) (i) Live conductors allow current to flow through andd reach th
the
appliance. [1]

(ii) Neutral conductors allow the current w back


ent to flow bac
ack to tthe
k to he source and
hence complete the circuit. [1]

(b) (i) The fuse rating is much high higher than


heerr tha tthe
haan th he amount
am
mou n of current
ount curr
curren
rrentt flowing
en
through the circuit.
rcuit. [[1]
When excessive ve ccurrent
cessive
ve rent fflows
urrre l ws tthrough
lo hrough the
hr the appliance,
app
pp
pliiancee, th may
thee fusee m ay
y
not melt and disconnect
dissconnect th tthe circuit.
he circ cu t As a result,
uiit. lt,, it
resuult it mayy cause
ay cauuse hharm
am
ar [1]
to thee user
ser or damage
use dam
amaag ge to
to the
hee appliance.
th apppplliianc
ncce.
n e

(ii)
(iii)) The
T em
Th metal
me ccase
ettaal caasee bbecomes
ecom
ec live
omes liviiv when
ve w
wh n tthe
heen hee livee wire
h wire iiss damaged
dam and [1]
touches
toouche
ucc es tth the metal
he me case.
ettaal cca
ase
se. [1]
Ass a rresult
essu
ullt it mayay y ccause
ausee hharm
au to the
aarrm to the user
s r or ddamage to the
use
aappliance.
appp
plliiaanc
n e.

(c)) Th
TThe circuit
he cci uit breaker
irrccui breake
br works
kerr wo
ke rks faster
work
rk fast in disconnecting all the switches in Any
the
he house.
tth house.
ho one
circuit
Th circ
The
Th cui breaker only needs to be reset after activated whereas
uitt br
thee fuse
fuse needs
needs to be replaced after melting. [1]

(d) Wire X: Live [1]

Wire Y: Neutral [1]

Wire Z: Earth [1]

www.KiasuExamPaper.com
149
www.KiasuExamPaper.com
150
1

Name: _________________________ ( ) Class: _____________


PRELIMINARY EXAMINATION
GENERAL CERTIFICATE OF EDUCATION ORDINARY LEVEL

PHYSICS 6091/01
Paper 1 Multiple Choice Tuesday 3 September 2019
1 hour

READ THESE INSTRUCTIONS FIRST

Write in soft pencil.


Do not use staples, paper clips, highlighters, glue or correction fluid.
Write your name, index number and class on the Answer Sheet in the spaces provided.

There are forty questions on this paper. Answer all questions. For each question there
are four possible answers A, B, C and D.
Choose the one you consider correct and record your choice in soft pencil on the
separate Answer Sheet.

Read instructions on the Answer Sheet very carefully.

Each correct answer will score one mark. A mark will not be deducted for a wrong
answer. Any rough working should be done on this booklet.
The use of an approved scientific calculator is expected, where appropriate.

This document consists of 19 printed pages and 1 blank page.

>7XUQRYHU

2019 SNGS Sec 4 Physics Prelim P1

www.KiasuExamPaper.com
151
2

1 A manufacturer measures the three dimensions of a floor tile using three different
instruments. The approximate dimensions of the tile are shown.

Which instruments are used to measure accurately each of these dimensions?

length width thickness


A calipers micrometer metre rule
B metre rule micrometer calipers
C metre rule calipers micrometer
D micrometer metre rule calipers

2 Two forces, X and Y, act on an object and produce a resultant force. The diagram
represents the sizes and directions of forces X and Y.

(not to scale)

Force Z balances the resultant force due to X and Y and keeps the object stationary.

Which arrow represents force Z?


A B

C D

2019 SNGS Sec 4 Physics Prelim P1

www.KiasuExamPaper.com
152
3

3 A student used a micrometer screw gauge to measure the thickness of a metal sheet.

40
5
35

30

25

What is the thickness of the metal sheet?

A 5.83 mm B 7.33 mm C 7.83 mm D 10.33 mm

4 Two blocks are joined together as shown below.


1.0 cm

1.0 cm
density density
6.0 g/cm3 9.0 g/cm3
1.0 cm 2.0 cm
One block has a density of 6.0 g/cm and the other has a density of 9.0 g/cm 3.
3

What is the overall density of the two blocks joined together?

A 7.0 g/cm3
B 7.5 g/cm3
C 8.0 g/cm3
D 15 g/cm3

5 A trolley with a ticker-tape attached to it, moves down a runway. The ticker-tape timer
operates at a frequency of 50 Hz on the tape. The diagram below shows two sections
from tape P and Q, separated by 25 spaces.

25 spaces

P QQ
P

44.0
cmcm 8 cmcm
10.0

Calculate the average acceleration of the trolley.

A 0.4 m/s2
B 0.6 m/s2
C 0.8 m/s2
D 1.2 m/s2

2019 SNGS Sec 4 Physics Prelim P1

www.KiasuExamPaper.com
153
4

6 The distance travelled by a car is increasing uniformly as it is driven along a straight


road up a hill.

Which quantity of the car is constant but not zero?

A acceleration
B displacement
C gravitational potential energy
D kinetic energy

7 Which moving body has a resultant force acting on it?

A a parachutist descending vertically at terminal velocity


B a diver rising vertically through water at constant speed
C an aircraft circling an airport at constant speed
D a train going up a straight incline at constant speed

8 An engine pulls a truck at constant speed on a level track.

The link between the engine and the truck breaks. The driving force on the engine
remains constant.

What effect does this have on the engine and the truck?

engine truck
A speed stays constant slows down
B speeds up slows down
C speed stays constant stops immediately
D speeds up stops immediately

2019 SNGS Sec 4 Physics Prelim P1

www.KiasuExamPaper.com
154
5

9 A free-fall skydiver jumps from a plane. As he falls, there is a force acting upwards
and a force acting downwards on his body. These produce a resultant force.

Before he reaches terminal velocity, how do the sizes of the forces change?

downward force upward force resultant force


A decreases decreases stays the same
B increases stays the same decreases
C stays the same increases decreases
D stays the same increases increases

10 The diagram shows an axle fixed to a wheel. The axle of radius 5.0 cm is pulled by
a force F so that the wheel of radius 30.0 cm turns clockwise. The wheel moves
forward at constant speed and experience a frictional force of 1000 N between the
wheel and the floor. The mass of the wheel and axle is 200 kg.

1000 N

What is the force F acting on the axle?

A 167 N
B 1 000 N
C 6 000 N
D 8 000 N

2019 SNGS Sec 4 Physics Prelim P1

www.KiasuExamPaper.com
155
6

11 A force F acts on an L-shaped object pivoted at point P. Which of the following will
produce the largest turning moment about pivot P?

12 A car of mass 1500 kg has a speed of 20 m/s. It accelerates until its speed is 25 m/s.
What is the increase in the kinetic energy of the car?

A 19 kJ
B 38 kJ
C 170 kJ
D 340 kJ

13 A man exerts a horizontal force of 600 N on a box as shown in the diagram. A


frictional force of 200 N acts in the opposite direction. The box moves 3.0 m in 5.0 s.

What is the useful power in pushing the box forward?

A 120 W
B 240 W
C 360 W
D 480 W

2019 SNGS Sec 4 Physics Prelim P1

www.KiasuExamPaper.com
156
7

14 The energy supplied to an electric motor is E, and, in the same time, the energy
wasted by the motor is W.

What is the efficiency of the motor?


A


B

ሺாିௐሻ
C

ሺாିௐሻ
D

15 Which graph shows the total external pressure acting on a submarine at different
depths below the surface of the sea? Assume the change in density of sea water is
negligible.

2019 SNGS Sec 4 Physics Prelim P1

www.KiasuExamPaper.com
157
8

16 The diagram shows a simple hydraulic jack. A downward force of 20 N is exerted on


piston X, causing it to move down by 30 cm.

What is the upward force on piston Y and the distance moved by piston Y raised?

upward force on piston Y distance moved by piston Y


A 40 N 15 cm
B 40 N 20 cm
C 80 N 10 cm
D 80 N 15 cm

17 The diagram shows a U-tube manometer containing three liquids: mercury, liquid X
and liquid Y. Neither liquid X or liquid Y mixes with mercury.

Which correctly describes the densities of X and Y, and the pressures they exert on
the mercury?

pressure exerted on the mercury densities of X and Y


A pressure of X is greater than Y density of X is greater than Y
B pressure of Y is greater than X density of Y is greater than X
C pressure of X and of Y are the same density of X is greater than Y
D pressure of X and of Y are the same density of Y is greater than X

2019 SNGS Sec 4 Physics Prelim P1

www.KiasuExamPaper.com
158
9

18 Air is trapped in a cylinder by a piston. The pressure of the air is p and the length of
the air column is 20 cm. The piston is moved outwards until the length of the air
column has increased by 40 cm. The temperature of the air remains constant.

What is the new air pressure?

A p/2
B p/3
C 2p
D 3p

19 Some gas is trapped in a closed container. The gas is cooled and the volume of the
container is kept constant.

What happens to the gas molecules?

A They collide with the walls more often.


B They contract.
C They get closer together.
D They move more slowly.

20 A column of dry air is trapped by a pellet of mercury in a capillary tube. The capillary
tube is held in different positions as shown in the diagrams.

P2
P3

P1

Compare the air pressures P1, P2 and P3.

A P1 > P 2 > P 3
B P3 > P2 > P 1
C P1 > P3 > P 2
D P1 = P2 = P 3

2019 SNGS Sec 4 Physics Prelim P1

www.KiasuExamPaper.com
159
10

21 A 125 g piece of solid lead at room temperature is heated. It completely melted after
22 s. The graph shows how its temperature varies with time.

The power of the heater is 400 W. What is the specific latent heat of fusion of lead, in
J/kg?

଻Ǥ଴ൈସ଴଴
A
଴Ǥଵଶହൈଷ଴଴

ଶଶൈସ଴଴
B
଴Ǥଵଶହൈଷ଴଴

଻Ǥ଴ൈସ଴଴
C
଴Ǥଵଶହ

ଶଶൈସ଴଴
D
଴Ǥଵଶହ

22 A slice of bread is placed under a red-hot electric grill to make toast.

How does heat reach the bread?

A conduction and convection


B conduction and radiation
C convection and radiation
D radiation only

2019 SNGS Sec 4 Physics Prelim P1

www.KiasuExamPaper.com
160
11

23 A thermometer is used to measure a temperature of 80 °C.

Which thermometer is the most sensitive?

24 The graph shows a water wave with frequency 10 Hz traveling from left to right. The
displacement of particle P on the wave at this instant is zero.

How long later would particle Q be at zero displacement?

A 0.025 s
B 0.050 s
C 0.10 s
D 2.5 s

25 Which device uses ultrasound?

A an optical fibre
B a pre-natal scanner
C a steriliser
D a sunbed

2019 SNGS Sec 4 Physics Prelim P1

www.KiasuExamPaper.com
161
12

26 Which statement about microwaves is correct?

A Microwaves are longitudinal waves.


B In vacuum, the speed of microwaves is equal to speed of visible light.
C The frequencies of microwaves are greater than the frequencies of visible light
D The wavelengths of microwaves are smaller than the wavelengths of infra-red.

27 A solid plastic cylinder is immersed in a liquid of refractive index 1.4. Light travelling
in the plastic cylinder strikes the inside surface at an angle of incidence of 70 o. The
light undergoes total internal reflection.

What are the values of the critical angle in the plastic and the refractive index of the
plastic?

critical angle in the plastic refractive index of the plastic


A greater than 70o greater than 1.4
B greater than 70o less than 1.4
C less than 70o greater than 1.4
D less than 70o less than 1.4

2019 SNGS Sec 4 Physics Prelim P1

www.KiasuExamPaper.com
162
13

28 The ray diagrams 1 and 2 show two ways in which a thin converging lens produces
an image that is larger than the object.

diagram 1 diagram 2

Which devices use lenses in the ways as shown in diagrams 1 and 2?

diagram 1 diagram 2
A camera magnifying glass
B magnifying glass projector
C photographic enlarger camera
D photographic enlarger projector

29 The diagram shows rays of light from a ray-box passing through three lenses placed
at positions 1, 2 and 3.

What type of lens is used at each position?

position 1 position 2 position 3


A converging converging converging
B converging converging diverging
C diverging converging diverging
D diverging diverging converging

2019 SNGS Sec 4 Physics Prelim P1

www.KiasuExamPaper.com
163
14

30 A positively charged metal sphere is placed midway between two previously


uncharged metal rods, one of which is connected to earth.

Which diagram shows the charges on the rods?

31 The diagram shows two charged parallel plates. Four negatively charged identical
particles, P, Q, R and S are placed in between the plates.

P _
+
R
Q
S

Which particles experienced the same force due to electric field?

A P and Q
B R and S
C All of them
D None of them

2019 SNGS Sec 4 Physics Prelim P1

www.KiasuExamPaper.com
164
15

32 A piece of wire has a resistance of 16 Ω. The wire is 20 cm long and has a


cross-sectional area of 2.0 mm2.

Which wire of the same material has a resistance of 8.0 Ω?

length cross-sectional area


A 10 cm 0.5 mm2
B 10 cm 1.0 mm2
C 20 cm 0.5 mm2
D 20 cm 4.0 mm2

33 A defibrillator is a device that is used to give an electric shock to a patient’s heart.


It supplies an electric shock of energy 300 J at an average voltage of 2000 V for
10 ms.

What is the average current it supplies?

A 0.015 A
B 0.67 A
C 6.7 A
D 15 A

34 The diagram shows the circuit for a hair-dryer.

The fan motor has a power rating of 0.10 kW and the heaters each have a rating of
0.40 kW. The cost of electricity is 18 cents per kWh.

What is the cost of running the hair-dryer for two hours with switches P and Q closed
and switch R open?

A 9 cents
B 18 cents
C 24 cents
D 32 cents

2019 SNGS Sec 4 Physics Prelim P1

www.KiasuExamPaper.com
165
16

35 The current in a kettle is 10 A and the kettle is protected by a 13 A fuse. The owner
of the kettle replaces the 13 A fuse with a 3 A fuse.

What happens when the kettle is switched on?

A The fuse does not melt and the kettle works correctly.
B The fuse does not melt but the kettle fails to work.
C The fuse melts and the kettle is undamaged.
D The fuse melts and the kettle might be damaged.

36 An old lamp is found and a new filament bulb with a power rating of 500 W is inserted.
When the lamp is plugged into the mains and switched on, it does not light up.

What is a possible cause of this?

A The earth wire in the plug is disconnected.


B The fuse in the circuit has too high a rating.
C The neutral wire in the plug is disconnected.
D The lamp is doubly insulated.

37 A beam of electrons is travelling towards the right. The electrons subsequently move
past a solenoid connected to a d.c. source.

+ +

_
electrons

Assuming that the electrons have negligible mass, in which direction will the beam of
electrons be deflected?

A downwards
B into the page
C out of the page
D upwards

2019 SNGS Sec 4 Physics Prelim P1

www.KiasuExamPaper.com
166
17

38 A magnet moves up and down above a coil of wire. The bottom of the magnet moves
up and down between P and R.

Where is the bottom of the magnet when the induced electromotive force (e.m.f.) in
the coil is maximum?

A P
B Q
C R
D P and R

2019 SNGS Sec 4 Physics Prelim P1

www.KiasuExamPaper.com
167
18

39 The graph shows the voltage induced by an a.c. generator that varies with time.

If the number of turns in the coil is increased four times and the speed of the rotation
of the coil is halved, what will be the new waveform shown on the CRO?

2019 SNGS Sec 4 Physics Prelim P1

www.KiasuExamPaper.com
168
19

40 The diagram shows many small identical bulbs connected to an ideal transformer of
turns ratio 1:4. The bulbs are rated at 2.0 V, 2.0 W.

To operate the bulbs at normal brightness, how many bulbs can be connected to the
secondary side of the transformer and what is the primary current?

number of bulbs primary current / A


A 30 0.25
B 30 0.50
C 60 0.50
D 120 4.00

2019 SNGS Sec 4 Physics Prelim P1

www.KiasuExamPaper.com
169
1

Name ___________________________ ( ) Class ______________

PRELIMINARY EXAMINATION
GENERAL CERTIFICATE OF EDUCATION ORDINARY LEVEL

PHYSICS 6091/02
Paper 2 20 August 2019
1 hour 45 minutes

READ THESE INSTRUCTIONS

Write your name and index number on all the work you hand in. Write in dark blue
or black pen in the spaces provided on the Question Paper. You may use a soft
pencil for any diagrams or graphs. You may use geometrical instruments and
electronic calculators. Do not use highlighters, correction fluid or correction tape.

Section A [50 marks]


Answer ALL questions.

Section B [30 marks]


Answer ALL questions. Question 11 has a choice of parts to answer.

Information for students:

Students are reminded that all quantitative answers should include appropriate units
and should be given to a sensible number of significant figures. Errors in units and
numbers of significant figures will be penalised. If working is needed for any
question, it must be shown in the space provided. Omission of essential working
will result in loss of marks.

The number of marks is given in brackets [ ] For Examiner’s Use


at the end of each question or part question.
Section A (50 Marks)

Section B (30 Marks)

Total (80 Marks)

This document consists of 21 printed pages and 1 blank page.

[Turn over]

SNGS Sec 4 Physics Prelim P2 2019


www.KiasuExamPaper.com
170
2

Section A (50 marks)


Answer all the questions in this section in the spaces provided.

1 Fig 1.1 shows a crane, with a “wing” attachment on its side, lifting a load.

load

“wing” attachment

Fig 1.1

(a) Explain the purpose of this “wing” attachment.

………………………………………………………………………………..

………………………………………………………………………………..

……………………………………………………………………………….. [2]

(b) On a windy day, the 25 kN load experiences a force of 50 kN to the


east, as shown in Fig 1.2.

steel cable

50 kN
load

Fig 1.2
Using a scaled vector diagram, determine the tension in the steel
cable.

tension = ……………………. [3]

SNGS Sec 4 Physics Prelim P2 2019


www.KiasuExamPaper.com
171
3

2 A piece of ice, with a mass of 22 g, at a temperature of -12.0 ιC is placed


in a container containing water at 22.0 ιC.

(a) Given that the specific heat capacity of ice is 2100 J/(kgK) and the
latent heat of fusion of ice is 3.3 x 105 J/kg, calculate the heat needed
(i) to raise the temperature of ice from -12.0 ιC to 0 ιC.

heat = …………………... [2]

(ii) to change the ice at 0 ιC to water at 0 ιC.

heat = ……………………. [2]

(b) The temperature of the water in the container falls after ice has been
added.

(i) Given that the specific heat capacity of water = 4200 J/(kgK),
calculate the initial mass of water in the container if the final
temperature of the mixture is 8.0 ιC.

initial mass of water = …………………... [2]

(ii) State an assumption you have made in (b)(i).

..……………………………………………………………………..……….

………………………………………………………………………………. [1]

SNGS Sec 4 Physics Prelim P2 2019


www.KiasuExamPaper.com
172
4

3 A light dependent resistor (LDR) is used to turn on a lamp in another circuit


when it gets dark. Part of the circuit is shown in Fig 3.1.

2000 Ω

12 V

to lamp in
another circuit

Fig 3.1

(a) The light intensity decreases. State and explain what happens to the
potential difference across the LDR.

………………………………………………………………………………

………………………………………………………………………………

……………………………………………………………………………… [2]

(b) Calculate the resistance of the LDR when the current flowing through
it is 4.0 mA.

resistance = ……………………. [2]

SNGS Sec 4 Physics Prelim P2 2019


www.KiasuExamPaper.com
173
5

(c) The electrical circuit in Fig 3.1 is now altered such that the LDR is
removed and 3 other resistors are added, as shown in Fig 3.2.

2000 Ω 1000 Ω

P Q
12 V

2000 Ω 2000 Ω

Fig 3.2

(i) Calculate the effective resistance of the circuit in Fig 3.2.

effective resistance = ……………………. [2]

(ii) Compare the current flowing through point P and point Q.


Explain your answer.

…………………………………………………………………………

…………………………………………………………………………

…………………………………………………………………………

………………………………………………………………………… [2]

SNGS Sec 4 Physics Prelim P2 2019


www.KiasuExamPaper.com
174
6

4 Jamie bought a tea kettle with a dull black external surface, as shown in Fig
4.1. The tea kettle uses an electric element to heat water and its power
output is 2.8 kW.

Fig 4.1

(a) State the energy change that takes place when the tea kettle is used
to heat up the water.

……………………………………………………………………………….. [1]

(b) Suggest a modification which can be made to the tea kettle such that
the water in the kettle can remain hot for a longer period of time.

………………………………………………………………………………..

……………………………………………………………………………….. [1]

(c) An electricity retailer passes the cost due to transmission loss to the
consumers. The transmission loss factor for the current year is 1.03.

Given that the tea kettle is used for 10 minutes daily and the cost per
unit of electricity charged by this electricity retailer is 18 cents,
calculate the cost of using this tea kettle daily for one week.

cost = …………………… [2]

SNGS Sec 4 Physics Prelim P2 2019


www.KiasuExamPaper.com
175
7

(d) The mains voltage supply for the tea kettle is 230 V. Suggest a suitable
fuse rating for the tea kettle.

fuse rating = …………………… [2]

5 Fig. 5.1 shows a method to paint a metal panel using electrostatic charges.
The nozzle of the spray gun is connected to a high voltage electrode which
applies positive charges to the paint droplets. The metal panel is connected
to earth.

metal panel
connected to
earth

paint droplets
with positive
charges
nozzle to high voltage

Fig 5.1

(a) Explain why the paint droplets spread out as they leave the nozzle.

.……………………………………………………………………..………

..…….……………………………………………………………………… [1]

(b) Explain why this method of painting reduces the amount of paint lost.

.……………………………………………………………………..……….

……….………………………………………………………………………

.……………………………………………………………………..……….

……….……………………………………………………………………… [2]

SNGS Sec 4 Physics Prelim P2 2019


www.KiasuExamPaper.com
176
8

(c) Another type of spray gun does not make use of electrical voltage to
charge the paint. However, the paint droplets are still charged
positively as they leave the nozzle. Suggest how the paint droplets
become positively charged as they leave the nozzle.

……………………………………………………………………………….

……….………………………………………………………………………

………………………………………………………………………………. [2]

6 (a) Fig 6.1 shows a series of lines representing a longitudinal wave set up
in a long spring (slinky).

Fig 6.1

(i) Describe how a longitudinal wave could be set up in the spring.

..........................................................................................................................

.......................................................................................................................... [1]

(ii) On Fig 6.1, mark


1. a position of compression and rarefaction with the letter “C”
and “R” respectively and [1]
2. a distance to represent the wavelength of the wave with an
arrow. [1]

(b) Fig 6.2 represents a cross-section of the water waves.

direction of
wave travel

18 cm

Fig 6.2

SNGS Sec 4 Physics Prelim P2 2019


www.KiasuExamPaper.com
177
9

The water waves has a frequency of 5.0 Hz.

(i) Calculate the speed of the water waves in cm/s.

speed = …………………… [2]

(ii) The wave as shown in Fig 6.2 travels into a shallow region.
State and explain what happens to
1. frequency and
2. wavelength of the wave in the shallow region.

1. ………………………………………………………………….

………………………………………………………............... [1]

2. ………………………………………………………………….

………………………………………………………………….

…………………………………………………………………. [1]

7 Fig 7.1 shows white light incident on the top face of a diamond. The white
light is dispersed into its various rays. Only the red (R) and blue (B) light
rays are shown in Fig 7.1.

white light

49q

crown

18q
B R

23q pavilion

Fig 7.1 (not to scale)

SNGS Sec 4 Physics Prelim P2 2019


www.KiasuExamPaper.com
178
10

(a) Explain why white light disperses in diamond, as shown in Fig 7.1.

..............................................................................................................

..............................................................................................................

.............................................................................................................. [2]

(b) (i) Calculate the refractive index of diamond for blue light.

refractive index = …………………… [2]

(ii) Given that the speed of light in air is 3.0 x 108 m/s, calculate the
speed of blue light in the diamond.

speed of blue light in diamond = …………………… [1]

(c) The diamond is now submerged in water. If the white light is still
incident on the top face of the diamond at the same angle, explain
what happens to the path of the blue light ray in the diamond.

..............................................................................................................

..............................................................................................................

..............................................................................................................

.............................................................................................................. [2]

SNGS Sec 4 Physics Prelim P2 2019


www.KiasuExamPaper.com
179
11

8 (a) A student has 3 identical metal bars. Two of the metal bars are
magnets and one is not. Explain, with the aid of a diagram, how the
student can identify the two magnets without using any other
apparatus.

..............................................................................................................

..............................................................................................................

..............................................................................................................

.............................................................................................................. [2]

(b) State a type of metal that can be used to make a permanent magnet.

.............................................................................................................. [1]

(c) (i) Fig 8.1 shows a vertical wire passing through a horizontal piece
of card.

Fig 8.1

There is a direct current (d.c.) in the wire, which produces a


magnetic field around it. Suggest how the magnetic field
produced by the current-carrying wire can be investigated.

....................................................................................................

................................................................................................... [1]

SNGS Sec 4 Physics Prelim P2 2019


www.KiasuExamPaper.com
180
12

(ii) Fig 8.2 shows the wire and the card viewed from above.

Fig 8.2

On Fig 8.2, draw three complete field lines produced by the


current-carrying wire. [1]

SNGS Sec 4 Physics Prelim P2 2019


www.KiasuExamPaper.com
181
13

Section B (30 marks)


Answer all the questions in the spaces provided.
Answer only one of the two alternative questions in Q11.
9 Fig 9.1 shows a rollercoaster train being pulled horizontally to the right by a
cable.

cable

Fig 9.1

Fig 9.2 shows information on the train.

speed / 8.3 8.3 8.3 16.6 24.9 33.2 41.5


m/s
time / s 0.0 0.5 1.0 1.5 2.0 2.5 3.0

Mass of empty train: 1500 kg


Mass of fully loaded train : 1800 kg
Total length: 10.5 m
Number of wheels: 16

Fig 9.2

(a) Describe the motion of the train from t = 0.0 s to t = 3.0 s.

…………………………………………………………………………….…….

……………………………………………………………………….………….

…………………………………………………………………….…………….

………………………………………………………………………………….. [2]

SNGS Sec 4 Physics Prelim P2 2019


www.KiasuExamPaper.com
182
14

(b) (i) Determine the average tension in the cable pulling a fully loaded
train during t = 1.0 s to t = 3.0 s.

average tension = ……………………. [3]

(ii) Is the actual average tension higher, lower or the same


compared to your answer in (b)(i)? Explain your answer.

…………………………………………………………………………….

……………………………………………………………………….……

…………………………………………………………………….………

……………………………………………………………………………. [2]

(c) Calculate the distance the train travelled from t = 0.0 s to t = 3.0 s.

distance = ……………………. [2]

(d) One of the passenger claims that if the tension in the cable remains
constant and the number of passengers is halved, the acceleration of the
train will be doubled. Explain whether the claim is true.

……………………………………………………………………………………

…………………………………………………………………………………… [1]

SNGS Sec 4 Physics Prelim P2 2019


www.KiasuExamPaper.com
183
15

10 (a) Fig 10.1 shows the structure of a simple a.c. generator.

Magnet Direction of rotation


N B
Coil of wire

C
S
Magnet
Slip rings A
D

Y
Carbon brushes X
Fig 10.1

The two ends of the wires, X and Y are the output terminals which are
connected to a cathode ray oscilloscope (CRO). The coil is turned by a
water wheel at 50 Hz. The peak voltage of the output is 9.0 V.

The time base of CRO is switched on at 10 ms/cm and Y-gain is set at


3 V/cm.

(i) Sketch the trace of the output from the generator on CRO.

1 cm

1 cm [2]

SNGS Sec 4 Physics Prelim P2 2019


www.KiasuExamPaper.com
184
16

(ii) Explain how the generator gives the output in (a)(i).

…………………………………………………………………………….

……………………………………………………………………….……

…………………………………………………………………….………

……………………………………………………………………………

……………………………………………………………………………

…………………………………………………………………………… [3]

(iii) Sketch the new trace of the output display when the slip rings are
replaced by a split-ring commutator.

1 cm
[1]

1 cm

SNGS Sec 4 Physics Prelim P2 2019


www.KiasuExamPaper.com
185
17

(b) Fig 10.2 shows an electric guitar, with strings that are already magnetised.
A coil is placed near each string. Fig 10.3 shows that the coil is connected
to a loudspeaker, which produces the sound of the guitar.

pickups

to coil of
Fig 10.2 loudspeaker

coil

loudspeaker

Fig 10.3

(i) Explain how the coil in Fig 10.2 can detect the vibration in the
magnetised string when it is plucked.

…………………………………………………………………………….

…………………………………………………………………………….

…………………………………………………………………….……… [2]

(ii) Explain how the loudspeaker, a moving-coil device, produces the


sound of the electric guitar.

……………………………………………………………………….……

…………………………………………………………………………….

……………………………………………………………………………. [2]

SNGS Sec 4 Physics Prelim P2 2019


www.KiasuExamPaper.com
186
18

11 EITHER
(a) Fig 11.1 shows a simple d.c. motor. The ends of the single loop coil ABCD are
soldered to the split-ring commutators. Two batteries are connected in the
external circuit.

Fig 11.1

(i) State the direction of rotation of the coil when viewed from the
commutator.

…………………………………………………………………………………… [1]

(ii) Explain how this d.c. motor works and the purpose of the split-ring
commutator.

……………………………………………………………………………………

……………………………………………………………………….…………...

…………………………………………………………………….……………...

……………………………………………………………………………………

……………………………………………………………………………………

…………………………………………………………………………………… [3]

SNGS Sec 4 Physics Prelim P2 2019


www.KiasuExamPaper.com
187
19

(b) Jamie designs a simple doorbell using materials found in the Physics laboratory,
as shown in Fig 11.2. When switch S is pressed and then released, two sound
notes of identical frequency are produced.

switch S

wire wound
around
soft iron core soft iron spring
attached to hammer

metal plate metal plate

Fig 11.2

(i) Explain how the two sound notes are produced.

……………………………………………………………………………………

……………………………………………………………………….…………...

…………………………………………………………………….……………...

…………………………………………………………………………………… [2]

(ii) Jamie makes the following comment: “The bell will still work if the battery
is replaced with an a.c. source.”
Do you agree? Explain your answer.

……………………………………………………………………………………

……………………………………………………………………….…………...

…………………………………………………………………….……………...

…………………………………………………………………………………… [2]

(iii) Suggest two ways to increase the loudness of the bell.

……………………………………………………………………………………

……………………………………………………………………….…………... [2]

SNGS Sec 4 Physics Prelim P2 2019


www.KiasuExamPaper.com
188
20

11 OR
Fig 11.3 shows a diving bell with a hatch of outer surface area 0.50 m2. While the
bell is on the surface of the sea, the hatch is closed and sealed.
air

sea

30 m

trapped
air

hatch

Fig 11.3 (not to scale)

(a) The bell is lowered from the surface of the sea to a depth of 30 m. The
pressure of the atmosphere is 100 000 Pa and the density of the seawater
is 1100 kg/m3. The gravitational field strength g is 10 N/kg.

(i) Define pressure.

…………………………………………………………………………….

…………………………………………………………………….……… [1]

(ii) Calculate the pressure exerted on hatch by the water when it is at


a depth of 30 m.

pressure = ……………………. [2]

(iii) Hence, calculate the force acting on the hatch.

force = ……………………. [1]

SNGS Sec 4 Physics Prelim P2 2019


www.KiasuExamPaper.com
189
21

(iv) Explain what happens to the volume of the trapped air when the
hatch is opened at 30 m below the surface of the sea. Assume that
the trapped air in the bell is at atmospheric pressure before the
hatch is opened.

…………………………………………………………………….………

…………………………………………………………………………….

…………………………………………………………………….……… [2]

(b) The same diving bell is lowered to the same depth of 30 m in another
region of the sea. It was observed that the volume of trapped air is more
than the volume in (a)(iv) when the hatch opens. Suggest a possible
reason for the observation.

…………………………………………………………………………….……...

…………………………………………………………………………………… [1]

(c) Using kinetic theory of matter, explain why the pressure of the trapped air
in the bell rises when its temperature increases.

……………………………………………………………………….…………...

……………………………………………………………………….…………...

……………………………………………………………………….…………...

……………………………………………………………………….…………...

……………………………………………………………………………………

…………………………………………………………………………………… [3]

------------------------------------------- END OF PAPER ---------------------------------------

SNGS Sec 4 Physics Prelim P2 2019


www.KiasuExamPaper.com
190
www.KiasuExamPaper.com
191
2019 Physics Prelim Paper 1

1 2 3 4 5 6 7 8 9 10
C C C C D D C B C C
11 12 13 14 15 16 17 18 19 20
A C B D B A D B D A
21 22 23 24 25 26 27 28 29 30
C B B A B B C B B B
31 32 33 34 35 36 37 38 39 40
C D D B C C B B D A

2019 Physics Prelim Paper 2

Qn Answer
1(a) The attachment is to provide the crane with better
tter stability
stabililiity / m
make
ake
ak e the ccrane more
stable.
It increases the base area of the crane lowers
e OR it lowe s tthe
ers he ccentre
entrre of gravity
gra
ravi of the
v ty o
crane.

1(b) Scale: 1 cm rep 10 kN


Accurate drawing of vector
vect
cttor diagram
or di
iagr m (incl.
agra ncll.. correct
(inc
ncl
nc ecctt labelling
corre
re labe
ellling forces
ng of fo
forc
rce
rces and direction
irectio of
dir
arrows):

Force
Fo
F orc
rce = 50
50 kN
kN

W eight
ht
Weight R
= 25
2 kN

T en
Te nssio
ion of
ion
Tension of tthe
he
he cable
cab
able = 56 kN (accept
56 (acce 55 kN to 57 kN)

2(a)(i) Q = mc
m
mc∆T
c∆T
∆T
= (22/10
(22/1000)
000
00)) x 2100 x 12.0
= 554.4
554.
4.4
4J
= 550 J (2 sf)
2(a)(ii) Q = ml
= (22/1000) x 3.3 x 105
= 7260 J
= 7300 J (2 sf)
2(b)(i) Q = mc∆T
554.4 + 7260 + (22/1000 x 4200 x 8.0) = m x 4200 x (22.0 – 8.0) *the mark is for
showing the concept of heat lost = heat gained
m = 0.15 kg (2 sf)

2(b)(ii) No heat gain from the surroundings

www.KiasuExamPaper.com
192
Qn Answer
3(a) The resistance of the LDR increases as the light intensity decreases.
The potential difference across LDR increases.

3(b) V across fixed resistor = 2000 x 0.004 = 8.0 V


Voutput = Routput / Rtotal x Vtotal
8.0 V = 2000 / (RLDR + 2000) x 12 V
RLDR = 1000 Ω

OR

V = IR
12 – 8 = (4/1000) x R
R = 1000 Ω

3(c)(i) Effective R = (1/4000 + 1/3000)-1


= 1700 Ω (2s.f)

3(c)(ii) The total resistance in the branch


anch with
witith
th point
poinnt P is
is higher
hig
igh
he er thann that
tha
h t with point Q
Q.
Since potential difference across
e acros each
o s eaachch b branch
rra
anncch is
is cconstant,
onst
on stan t, ccurrent
ant, urre
urren
re nt flowing thr
nt through
point P is lower than fflowing
n current fl
flow
lo
owwing ththrough
hro u h point
roug poiin
po nt QQ..

4(a) Electrical energy Æ He


Heat
eat

4(b) The
he exexternal
xtte
ernal
e surface
su
urrffa
acce
e off the
the tea
th teea kettle
kettle could
cou
ould
ld be
be modifiedd to
to one
one with a shiny silver
on
surface
su
urffac
a e which
which h is
is a p poor
oor em
oo e
emitter
miittte
ter ooff iinfrared
nfra
nf rarre
ed radiation
ra
adiation to the
the surroundings,
su so as to
minimize
mi
m ini
n mize
zeehheat lost
e t lo
ea ost to surroundings.
ssu
urr
rrou
rou
ound din
nggss.

4(c) Energy
E
En
ner
ergy
gy = PPower
ower
ow er x T
Time
im
ime
me
= 2.
2
2.88 kW
W x 10/60
10
1 0/6
/60 x 7 h
/60
= 3.27
3..27
27 kkWh
Wh
W h
Cost
C
Coost
st = 3
3.27
.2
27 kW
kWh
W h x $0.1
$0.18
18 x 1.03
1.0
=$$0
$0.61
0.6
0.6
61

4(d) I=P/V
= 28000 / 230
230
= 12.2
.2 A

Suitable fuse rating = 13 A

5(a) The paint droplets repel each other as they have the same charge / same charges
repel.

5(b) The positively charged paint droplets induced a negative charge on the surface of
the metal panel.

As opposite charges attract, most of the paint droplets are attracted to the metal
panel, reducing the amount of paint loss.

www.KiasuExamPaper.com
193
Qn Answer
5(c) The paint droplets are charged by friction.
The paint droplets lose electrons to the nozzle as they leave the nozzle.

6(a)(i) The long spring can be given a displacement such that the movement of each turn
on the long spring is parallel to the direction of the wave motion set up.

6(a)(ii)
wavelength

6(b)(i) Wavelength = 6 cm
v = fʎ
= 5.0 x 6
= 30 cm/s

6(b)(ii) 1. frequency
cy – remains
re
em
maain
ns un
u
unchanged
nch
chan
a ged d as
as source off wave
wav
ave iss the
the same
sam
me
2. wavelength
velength – becomes
be
be me
ecomees sm
smaller
maalllerr as
as speed decreases
deccre
reas
a es in the
as th
he shallow
shal
sh allo
all w region
lo re

7(a) Differen
Different
ennt colours
colo
ourrs off light
lig
ghtt ttravel
rave
ra el at
at the same
sam
am
ame speed
s ee
sp e d in a
air.
irr. However,
Howe
Ho ever ddifferent colours
of light
lig
ghtt ttravel
ravel at
ra at different
dififfe
ffe
fereent
n speeds
sppe
e
eed
ed
e ds in
n diamond.
dia
ammoond
nd.

Therefore,
Th
T herefore,
he e, tthere
he
ere
e are d different
di
iffer
ffferen
eren
er ent angles
an
angl
gless of re
refr
refraction
frac
fr acti
ac tion fo
ti for different colours / different
colours
cco
olo
lour
urs bend
nd
n d at
at different
difffer
eren
ren
ent angles.
an
angles.

OR
O R

There
Ther
ere ar
er are
a eddifferent
di
iff
fferentt re
refr
refractive
frac
fr actititive indices
ac ind for different colours, hence white light disperses
into
nto iits
ts various
ts var
a ious rrays.
ays.
ay s.

7(b)(i) ௦௜௡ସଽ
௦௜௡ସଽι
ସଽιι
݊ ൌ ௦௜௡ଵ଼ι
௡ଵ଼ι

n = 2.4 (2 sf)

7(b)(ii) v = 3 x 108 / (
௦௜௡ସଽι
)
௦௜௡ଵ଼ι
= 1.2 x 108 m/s (2 s.f.)

7(c) Blue light would bend towards the normal less at the water-diamond boundary as
compared to the air-diamond boundary.

This is due to the higher refractive index of water compared to that of air.

www.KiasuExamPaper.com
194
Qn Answer
8(a)
first metal bar second metal bar

third metal bar

Bring one end of the first metal bar near to both ends of the second metal bar, and
then to both ends of the third metal bar.

The repulsion between two metal bars will enable the student to
o id
identify them as the
two magnets.

8(b) Steel

8(c)(i) Magnetic field can be investigated by placing a plplot


plotting
otti
ot ting
ting ccompass
om
mpa
pass on
o the
th piece of
card and marking the positions of the
he needle of the
th
he compass.
co
ompmpass..

OR

Use iron filings to inves


investigate
essttiig
gaate
te tthe
he
he pattern
pattern
rn
n of
of magnetic
magn
g etic field
fie
ield
ld
d lines.
lin
nes
es..

8(c)(ii) x The
The innerr cicirc
circles
rcle
les are closer to
each
h other
othher
e
x Correct
Corr
Co rrec direction
ectt direc

9(a) In the
e first
s seco
second, the train travels at a constant speed of 8.3 m/s.
In the next two seconds, the train accelerates uniformly.

9(b)(i) Average acceleration = (v – u) / t


= (41.5 – 8.3) / (3.0 – 1.0)
= 16.6 m/s2 (3 sf)

Tension = m x a
= 1800 x 16.6
= 29 880 N
= 29 900 N (3 sf)

www.KiasuExamPaper.com
195
Qn Answer
9(b)(ii) The actual average tension is higher than the answer in (b)(i) as there is friction
between the moving parts and air resistance in actual situation.

9(c) Total distance travelled = (8.3 x 1) + ½ (8.3 + 41.5)(3-1)


= 58.1 m

9(d) The claim is false. The total mass is not halved as the mass of the empty train
remains constant. Thus, the acceleration will not be doubled.

10(a)(i)

3 cm

2 cm

10(a)(ii) When the coil rotates, ss,, itit cuts


cutts
cu s the the
e magnetic
magnetti tic flux which induces
ind
nduucess a
an
n emff ac
acro
across
ross
rosss the ttwo
ends of the
e coil.
When the co
coil
oilil changes
ch
haangeses position
es poossitittio
iio
on af
a
after
fte
ter half
ha
h alf a revolution,
rev
evolution
n, the
th emf
emmf induced
indu
in d ced is reversed.
When the
the
h coi
coililil is
is horizontal,
hori
hor zont n al
nt a , emfem mf induced
induce ed is
is mmaximum,
aximum,, when
when the
the
h coil is vertical, emf
induced
indu
du
uce
ed is zer
zero. ro.

10(a)(iii)
a)(iiii)
i)

3cm

1 cm
10(b)(i) Whenen tthe
he string
strings are plucked, their relative distances with the coil change
(vibration),
tion), and
a the magnetic flux linking the coil changes as a result.

The vibration thus causes an e.m.f. to be induced in the coil. Hence an induced emf
in the coil means a detection of the vibration in the string.

10(b)(ii) The changing induced current in the coil that causes the force/ vibration/ movement
of coil in loudspeaker [Fleming’s Left-hand Rule] is in sync with the variation of
current.

The vibration of coil will cause the cone in the loudspeaker and hence the air around
it to vibrate and produces sound. (produces regions of compressions and
rarefactions in the layer of air next to it)

www.KiasuExamPaper.com
196
Qn Answer
11 EITHER
11(a)(i) Anti-clockwise direction from the front

11(a)(ii) When current flows into the coil through the magnetic field of the magnet, it results
in a force produced such that the coil will turn.

The split-ring commutator reverses the current in the coil every half a cycle so that
the coil can turn continuously in one direction.

11(b)(i) When switch S is closed, the soft iron core becomes magnetised and attracts the
soft iron spring attached to the hammer.
The hammer strikes the metal plate and produces the first
st sound no
note.

When the swtich is released, the circuit is broken


ken and iron
on core
corre loses
lose magnetism
and hammer swings to the other metal plate,e, producing
ng the
producin the second
seccond
se co sousound note.

11(b)(ii) Yes, the iron core will be magnetized


etized and the
e spring
ssp
priing
ng will be attracted
att
ttra
raccted ssimilarly,
ra
although there is a change of polari
polarity.
ity
ty.

11(b)(iii) 1. Increase the current


ent flowin
flowing ng th
throughh th
the
he wire
wiire
re
2. Increase the numb
number
ber
er ooff tu
tturns
urn around
r s ar
a d tthe
round he soft iron ccore
he ore
or e

11 OR
11(a)(i) Pressure
Pressu
ure
re is th
the
he force
forc
fo ce acting
accti
ting
ng (on
ng (o
on
n an
an object)
obje
ecctt)
t) per
per unit area.
pe are
rea.
rea.

11(a)(ii)
(a)(iii) Pressure
P
Pr e sure
es e due
du
ue
e tto
o sea-wa
sea-water
atte
e at
er at depth
dep
ep
pth
th
h 30
30 m
hƿg
= hƿ
h ƿg =1100
=110
00 x 1100 x 30
30
= 330
33 kPa
30 kkP
Pa

11(a)(iii) Total
To
ota
tal pressure
tal pe
pr esssu
sure
e=3 330
30 k + 1
30 100
00 k = 430 k Pa
Force
Forc
rc
ce = prpressure
re sure x area
esssu area = 430 000
rea 0 x 0.50 = 215 kN

11(a)(iv) The
e volume
me ddecreases
ecrease so that the pressure of the trapped air becomes the same
ec
as the
he pressure
pre
resssure at 30 m / there is higher pressure exerted.

11(b) The density of the sea water in that region is lower resulting in lower pressure at
depth of 30 m. Hence the volume of the trapped air increases to give a lower
pressure that balances the external pressure.

11(c) When temperature increases, the air molecules gain kinetic energy and have greater
speed / average KE of molecules increases.
The molecules will collide with the wall more often and hence increasing the force
acting on the wall.
Since pressure is due to the force acting on the wall per unit area, larger force acting
on the wall results in a larger pressure (P = F/A).

www.KiasuExamPaper.com
197
www.KiasuExamPaper.com
198
Index
Name Class 4A
Number

DUNMAN HIGH SCHOOL


PRELIMINARY EXAMINATION 2019
GCE O LEVEL PHYSICS

Paper 1 6091/1
03 September 2019
Paper 1 Multiple Choice 1 hour

Additional Materials: Multiple Choice Answer Sheet

READ THESE INSTRUCTIONS FIRST

Write your name and index number on all the work you hand in.
Write in soft pencil.
Do not use staples, paper clips, glue or correction fluid.

There are forty questions on this paper. Answer all questions. For each question there are four possible
answers, A, B, C and D.
Choose the one that you consider correct and record your choice in soft pencil on the separate Answer
Sheet.

Each correct answer will score one mark. A mark will not be deducted for a wrong answer.
Any rough working should be done in this booklet.
The use of an approved scientific calculator is expected, where appropriate.

For teacher’s use:


Paper 1 /40

Paper 2 /80

Total /120

This document consists of 13 printed pages.

Page 1 of 13

www.KiasuExamPaper.com
199
1 A car travels at a speed of 65 km/h.

What is the speed of the car in m/s?

A 18.1 m/s B 55.4 m/s C 181 m/s D 1080 m/s

2 A light ray is reflected off two mirrored surfaces.

(not to scale)

What is the value of θ?

A 5° B 10° C 15° D 20°

3 A name tag reads “SAM”.

What is the image of the name tag in a plane mirror?

A B

C D

4 The refractive index for germanium is 4.01.

What is the speed of light in germanium?

A 7.5 × 107 m/s B 7.5 × 108 m/s C 1.2 × 108 m/s D 1.2 × 109 m/s

Page 2 of 13

www.KiasuExamPaper.com
200
5 A thin, converging lens of focal length f, is used to produce various types of images.

Which row is correct?

location of object type of image produced


A less than f inverted, virtual, diminished
B between f and 2f upright, real, enlarged
C at 2f inverted, real, same size
D beyond 2f upright, real, diminished

6 A rope is fixed to a wall at one end with the other end moved up and down to produce a wave.

wall

What is transferred along the rope due to this motion?

A mass
B energy
C frequency
D molecules

7 Which is not an application of microwaves?

A To detect structural flaws.


B To ionise biological molecules.
C To cook food in microwave ovens.
D To transmit communication signals.

8 The wavelength of an X-ray is 1.0 nm while the wavelength of a radio wave is 1.0 mm.

What is the ratio of the frequency of the X-ray to the frequency of radio wave?

A 1 : 1 00 000
B 1 : 1 000 000
C 1 00 000 : 1
D 1 000 000 : 1

9 A sound travels through a solid copper tube and then enters air.

Which of the following correctly compares the sound in the solid copper tube to air?

frequency speed (in copper tube)


A different higher
B different same
C same higher
D same lower

Page 3 of 13

www.KiasuExamPaper.com
201
10 A series of compressions and rarefactions of a sound wave are shown.

4.0 m (not to scale)

What is the wavelength of the sound wave?

A 2.0 m B 3.0 m C 4.0 m D 6.0 m

11 Which defines acceleration?

A ݄ܿܽ݊݃݁݅݊݀݅‫݁ܿ݊ܽݐݏ‬
‫݊݁݇ܽݐ݁݉݅ݐ‬

B ݄ܿܽ݊݃݁݅݊݀݅‫݊݋݅ݐܿ݁ݎ݅݀݀݁ݔ݂݅ܽ݊݅݁ܿ݊ܽݐݏ‬
‫݊݁݇ܽݐ݁݉݅ݐ‬

C ݄ܿܽ݊݃݁݅݊‫݀݁݁݌ݏ‬
‫݊݁݇ܽݐ݁݉݅ݐ‬

D ݄ܿܽ݊݃݁݅݊‫ݕݐ݅ܿ݋݈݁ݒ‬
‫݊݁݇ܽݐ݁݉݅ݐ‬

12 A stone falls freely from rest from the top of a building of height, 80 m.

The gravitational field strength g is 10 N/kg.

What is the speed of the stone when it hits the ground?

A 4.0 m/s B 8.0 m/s C 40 m/s D 80 m/s

13 Which quantity is a scalar?

A displacement B force C weight D work done

Page 4 of 13

www.KiasuExamPaper.com
202
14 A body is acted on by two forces, S and T. A frictional force, F keeps the body in equilibrium.

Which vector diagram shows the relationship between these forces?

A B

C D

15 A helicopter of mass 3.0 x 103 kg rises vertically with a constant speed of 25 m/s.

The gravitational field strength g is 10 N/kg.

What is the resultant force acting on the helicopter?

A 0N
B 3.0 x 104 N downwards
C 4.5 x 104 N upwards
D 7.5 x 104 N upwards

16 A parachutist is falling through air with terminal velocity.

Which quantity is changing?

A acceleration
B gravitational potential energy
C kinetic energy
D mass

17 Which object has the largest inertia?

A a 1.5 x 10-1 kg baseball pitched at 170 km/h


B a 1.2 x 103 kg sports car travelling at 100 km/h
C a 5.0 x 103 kg stationary helicopter
D a 3.0 x 106 kg falling tree

Page 5 of 13

www.KiasuExamPaper.com
203
18 The mass and volume of different sized samples of a newly discovered metal are measured.

Which graph shows how the mass varies with volume?

A B

C D

19 A uniform beam has a weight of 60 N and a length of 2.0 m. It is pivoted at a length of 0.60 m from
the end X.

At what distance from the pivot should a 160 N weight be placed in order to balance the beam?

A 0.150 m B 0.225 m C 0.375 m D 0.450 m

Page 6 of 13

www.KiasuExamPaper.com
204
20 A student of weight 500 N runs up a slope of length 30 m and height 15 m in 25 s.

What is the power developed by the student?

A 30 W B 60 W C 300 W D 600 W

21 The two cubes shown below are made from the same material. The bigger cube has sides that
are twice as long as the smaller cube. Standing on one face, the small cube exerts a pressure, P
on the floor.

2x
x

Given that the smaller cube has a weight, w and the bigger cube weighs eight times as much,
what is the pressure exerted by the larger cube standing on one of its faces?

A ½P B P C 2P D 4P

22 Equal masses of water are poured into four jars as shown.


Which jar has the least pressure exerted by the water on the base?
A B C D

23 When fine pollen grains suspended in water are viewed under a microscope, they are seen to be
making small, erratic movements.

Why is this?

A There are convection currents in the water.


B They are being hit by water molecules.
C They are moving and colliding with one another.
D They are living organisms so they move around.

Page 7 of 13

www.KiasuExamPaper.com
205
24 Which physical property is not suitable for defining temperature scales?

A e.m.f at the junction of two different metals


B mass of a solid object
C volume of a liquid column
D volume of trapped gas

25 A piece of wire has an electrical resistance of 980 Ω in ice of -10 °C, and 1750 Ω in boiling water
at 100 °C.

What is the resistance of the piece of wire at 30 °C, assuming resistance changes uniformly with
temperature?

A 280 Ω B 1190 Ω C 1260 Ω D 1370 Ω

26 A slice of bread is placed under a hot electric grill.

grill

slice of bread

How does thermal energy reach the bread?

A conduction only
B radiation only
C conduction and convection
D convection and radiation

27 Two identical blocks of metal are heated to 20°C and 80°C and placed in a vacuum. In which
scenario will thermal equilibrium between the two blocks be reached in the shortest time?

A B

80°C 80°C
20°C

20°C

C D
20°C

80°C 80°C
20°C

plastic block

Page 8 of 13

www.KiasuExamPaper.com
206
28 A piece of pure ice melts.

Which row describes the energy changes of the ice as it melts?

internal potential energy internal kinetic energy


A increases decreases
B decreases increases
C increases remains constant
D remains constant increases

29 A negatively charged rod is brought close to an isolated T-shaped piece of metal.

Initially, the metal is uncharged.

Which diagram shows the induced charge on the metal?

A B

C D

30 The diagram shows the electric field pattern between two isolated point charges.

Which two point charges produce this pattern?

A B

C D

Page 9 of 13

www.KiasuExamPaper.com
207
31 Identify the correct position of the switches such that all three bulbs are lit.

bulb 1

switch P

switch Q

bulb 2

switch R bulb 3

switch P switch Q switch R


A closed closed open
B open open closed
C open closed open
D open closed closed

32 In the diagram below, the potential difference across the battery is denoted by Vε and the potential
difference across R2 is denoted as Vout.

What is the ratio of Vε : Vout?

A R1 : R2
B R2 : (R1 + R2)
C (R1 + R2) : R1
D (R1 + R2) : R2

Page 10 of 13

www.KiasuExamPaper.com
208
33 Which graph shows the characteristics of a light dependent resistor?

A resistance B resistance

light intensity light intensity

C resistance D resistance

light intensity light intensity

34 Which of the following is not a safety feature found in a typical household electrical circuit?

A fuse
B earth wires
C lightning rod
D main circuit breaker

35 An electric oven is rated at 10 A.

What is a suitable fuse for the oven?

A 5.0 A B 8.0 A C 10 A D 13 A

36 To determine whether a material is magnetic, you should find out if it

A is a metal or a non-metal.
B is a conductor or an insulator.
C can be given an electric charge.
D affects the direction in which a compass needle points.

Page 11 of 13

www.KiasuExamPaper.com
209
37 The diagram shows an alarm system in which the switch S is shown closed.

What happens when the switch S is opened?

iron bell
A drops rings
B drops stops ringing
C moves up rings
D moves up stops ringing

38 Two parallel vertical wires P and Q are a small distance apart in air. There is a downwards electric
current in both wires. A force acts on Q due to the current in P. This force is perpendicular to the
wire Q.

What is the direction of the force on Q?

Page 12 of 13

www.KiasuExamPaper.com
210
39 The diagram shows a single-coil electric motor.

The split-ring commutator reverses the current in the coil as it rotates.

How many times is the current reversed if the coil is rotated for one complete revolution?

A 1 B 2 C 3 D 4

40 Electric power cables transmit electrical energy over large distances using a high voltage,
alternating current.

What are the advantages of using a high voltage and of using an alternating current?

advantage of using a high voltage advantage of using an alternating current


A a higher current is produced in the cable the resistance of the cable is reduced
the voltage can be changed
B a higher current is produced in the cable
using a transformer
C less energy is wasted in the cable the resistance of the cable is reduced
the voltage can be changed
D less energy is wasted in the cable
using a transformer

END OF PAPER

Page 13 of 13

www.KiasuExamPaper.com
211
Index
Name Class 4A
Number

DUNMAN HIGH SCHOOL


PRELIMINARY EXAMINATION 2019
GCE O LEVEL PHYSICS

Paper 2 6091/2
30 Aug 2019
Theory 1 hour 45 minutes
Candidates answer on the Question Paper.
No Additional Materials are required.

READ THESE INSTRUCTIONS FIRST

Write your name and index number on all the work you hand in.
Write in dark blue or black pen.
You may use an HB pencil for any diagrams or graphs.
Do not use staples, paper clips, glue or correction fluid.

Section A
Answer all questions.

Section B
Answer all questions. Question 17 has a choice of parts to answer.

Candidates are reminded that all quantitative answers should include appropriate units.
The use of an approved scientific calculator is expected, where appropriate.
Candidates are advised to show all their working in a clear and orderly manner, as more marks are
awarded for sound use of Physics than for correct answers.

At the end of the examination, fasten all your work securely together.
The number of marks is given in brackets [ ] at the end of each question or part question.

For teacher’s use:


Section A /50

Section B /30

Total /80

This document consists of 24 printed pages and 1 blank page.

Page 1 of 25

www.KiasuExamPaper.com
212
Section A
Answer all the questions in this section.

1 A student measures the diameter of a rod using a micrometer.

Fig. 1.1 shows the reading on the micrometer. Given that the micrometer has a zero error
of -0.04 mm, determine the actual diameter of the rod.

mm

Fig. 1.1

diameter = ……………………… [2]

2 The critical angle of glass is 41.2°.

(a) Calculate the refractive index, ߟ of glass.

ߟ = ……………………… [1]

(b) Fig. 2.1 shows a light ray incident on a glass block. Calculate r, the angle of
refraction.
air

glass
35°

Fig. 2.1 (not to scale)

r = ……………………… [2]

Page 2 of 25

www.KiasuExamPaper.com
213
3 Fig. 3.1 shows a thin converging lens that is used as a magnifying glass.

(a) On Fig. 3.1, draw two rays from the top of the object to locate the top of the image.
Draw in the whole of the image. [3]

F object F

thin converging lens

Fig. 3.1

(b) Other than the size of the image relative to the object, state another characteristic
of the image.

…………………………………………………………………………………………….... [1]

Page 3 of 25

www.KiasuExamPaper.com
214
4 Fig. 4.1 shows the displacement-distance graph of a transverse wave.

displacement /cm

30

0 40 80 120 160 200 240


distance /cm

-30

Fig. 4.1 (not to scale)

(a) Determine the amplitude of the wave.

amplitude = ……………………… [1]

(b) The wave propagates at a speed of 2.8 m/s.

Calculate the frequency of the wave.

frequency = ……………………… [2]

(c) Name a region of the electromagnetic spectrum that has a higher frequency than
ultraviolet radiation.

…………………………………………………………………………………………….. [1]

Page 4 of 25

www.KiasuExamPaper.com
215
5 Fig. 5.1 shows a sound source integrated with a microphone placed in the middle of a
rectangular room. The microphone is attached to an oscilloscope. A sound pulse is emitted
and the oscilloscope starts recording immediately after the entire pulse has been emitted.

Fig. 5.2 shows the oscilloscope trace of the first two returning echoes.

sound source and amplitude


microphone
first echo second echo

x 80 160
0 60 140 time /ms

Fig. 5.1 (not to scale)

Fig. 5.2 (not to scale)

(a) Explain why the second echo is of a smaller amplitude than the first echo.

……………………………………………….………………………………………………

……………………………………………….……………………………………………… [1]

(b) The speed of sound in air is 330 m/s.

Calculate the distance x, leaving your answer in metres.

x = ………………… m [2]

Page 5 of 25

www.KiasuExamPaper.com
216
6 Fig. 6.1 shows an empty double walled container of mass 235 g which can hold up to
235 cm3 of fluids.

Fig. 6.1

State and explain whether the empty double walled container will float or sink in water.
The density of water is 1.0 g/cm3.

……………………………………………….………………………………………………........

……………………………………………….………………………………………………........

……………………………………………….………………………………………………........ [2]

Page 6 of 25

www.KiasuExamPaper.com
217
7 Fig. 7.1 shows a simple balancing toy balanced on the tip of a finger.

Fig. 7.1 Fig. 7.2

(a) (i) Define the centre of gravity of an object.

……………………………………………………………………………………......

……………………………………………………………………………………...... [1]

(ii) On Fig. 7.1, mark the position of the centre of gravity of the simple balancing
toy with an ‘X’. [1]

(b) Fig. 7.2 shows the simple balancing toy displaced on the tip of the finger.
Explain how the simple balancing toy is able to return to its position in Fig. 7.1.

……………………………………………….……………………………………………….

……………………………………………….……………………………………………….

……………………………………………….………………………………………………. [2]

(c) Without any changes to the materials used to construct the simple balancing toy,
suggest a modification to make it more stable.

……………………………………………….……………………………………………….

……………………………………………….………………………………………………. [1]

Page 7 of 25

www.KiasuExamPaper.com
218
8 Fig. 8.1 shows a skier of mass 80 kg, ski down the slope from rest at X and reach a
speed of 5.0 m/s at Y.

Fig. 8.1

(a) Define work.

…………………………………………………………………..………………………….

…………………………………………………………………..…………………………. [1]

(b) Calculate the work done against friction from X to Y.

The gravitational field strength g is 10 N/kg.

work done = ……………………… [3]

Page 8 of 25

www.KiasuExamPaper.com
219
9 (a) Fig. 9.1 shows a gas in a sealed container at a pressure of 118 kPa connected to
the left arm of a manometer. The right arm of the manometer is open to the
atmosphere.

sealed container

118 kPa
45 cm

liquid Q

Fig. 9.1

Calculate the density of liquid Q.

The gravitational field strength g is 10 N/kg. The atmospheric pressure is 101 kPa.

density of liquid Q = ……………………… [2]

Page 9 of 25

www.KiasuExamPaper.com
220
(b) Fig. 9.2 shows a mercury barometer placed under a pressure of one
atmosphere. The density of mercury is 13 600 kg/m3.

vacuum

76 cm
mercury

19 cm

Fig. 9.2 (not to scale)

Liquid Q is used in place of mercury.

Calculate the height of the column of liquid Q when placed under a pressure of one
atmosphere.

[1]
height of column of liquid Q = ………………………

(c) (i) Define pressure.

……………………………………………………………………………………… [1]

(ii) Fig. 9.3 shows the caterpillar tracks found on excavators.

caterpillar tracks

Fig. 9.3

Explain why caterpillar tracks, instead of wheels, are fitted on excavators on


muddy ground.

………………………………………………………………………………………

……………………………………………………………………………………… [1]

Page 10 of 25

www.KiasuExamPaper.com
221
10 Fig. 10.1 shows a flask filled with air and covered with a rubber bung.

Fig. 10.1

When the flask is heated, the pressure of the air inside the flask increases and the rubber
bung flies out.

(a) State the kinetic theory of matter.

………...……...……………………………………………………………………................

…………...…………..……………………………………………………………………...... [1]

(b) Explain, using the kinetic theory of matter, why heating the air inside the flask causes
the air pressure to increase.

………...……...…………………………………………………………..………………......

………...……...…………………………………………………………..………………......

………...……...…………………………………………………………..………………......

………...……...…………………………………………………………..………………......

………...……...…………………………………………………………..………………...... [3]

Page 11 of 25

www.KiasuExamPaper.com
222
11 Fig. 11.1 shows petrol being pumped into a can. Electrostatic charges build up on the petrol
and the pipe.

pipe

petrol

Fig. 11.1

(a) Explain why this is dangerous.

……………………………………………………………………………………......……..

……………………………………………………………………………………......……..

……………………………………………………………………………………......…….. [2]

(b) Describe what can be done to stop the electrostatic charge building up in this way.

……………………………………………………………………………………......……..

……………………………………………………………………………………......…….. [1]

Page 12 of 25

www.KiasuExamPaper.com
223
12 Fig. 12.1 shows a circuit with three identical resistors R1, R2 and R3.

R1
A1

R2
A2

R3

Fig. 12.1

(a) The switch is closed.

State and explain what happens to the reading of A1 compared to when the switch
is opened.

……………………………………………………………………………………………..

…………………………………………………………………………………………….. [1]

(b) R2 is now replaced with a diode, , pointing to the right. The switch remains closed.

State the ratio of the reading of A1 to the reading of A2.

…………………………………………………………………………………………….. [1]

Page 13 of 25

www.KiasuExamPaper.com
224
13 Fig. 13.1 shows the three wires P, Q and R of an electrical appliance and their
corresponding wires on the mains socket.

mains socket electrical appliance

Q heating
element
R

metal casing
Fig. 13.1

(a) Name wire P.

…………………………………………………………………………………………….. [1]

(b) State the colour of wire R.

…………………………………………………………………………………………….. [1]

(c) Explain how wire Q serves as a safety feature.

……………………………………………………………………………………………..

…………………………………………………………………………………………….. [1]

(d) In some appliances, wire Q is not present.

State an additional safety feature that is present in place of wire Q.

…………………………………………………………………………………………….. [1]

Page 14 of 25

www.KiasuExamPaper.com
225
14 (a) State two conditions required for a particle to experience a force in a magnetic field.

1. ……………………………………………………………………………......…………..

……………………………………………………………………………………......……..

2. ……………………………………………………………………………......…………..

……………………………………………………………………………………......…….. [2]

(b) Microwaves and a beam of fast-moving electrons enter a strong magnetic field, as
shown in Fig. 14.1.
uniform magnetic
field out of page

fast-moving electrons

microwaves

Fig. 14.1

The magnetic field acts only in the shaded region shown in Fig. 14.1. The direction
of the magnetic field is out of the page.

On Fig. 14.1, sketch the paths of the microwaves and the electrons in the magnetic
field. [3]

Page 15 of 25

www.KiasuExamPaper.com
226
Section B
Answer all the questions in this section.
Answer only one of the two alternative questions in Question 17.

15 The marathon is a long-distance race of about 42 km. Water on the skin of a marathon
runner evaporates as he runs.

Data relevant to the marathon is given in the box.

mass of marathon runner: 70 kg


time taken by runner to complete the marathon: 4.5 hours
average body temperature before starting the marathon: 37 °C
body temperature above which overheating may cause serious damage: 40 °C
average production of thermal energy in one hour by the runner: 3.3 x 106 J

average loss of energy in one hour by evaporation from the skin: 2.2 x 106 J
average specific heat capacity of the human body: 3400 J/(kg °C)
specific latent heat of vaporisation of water at body temperature: 2.4 x 106 J/kg

The level of dehydration of a human body is measured by the percentage loss of body
mass caused by evaporation. The table below shows three levels of dehydration.

percentage of body mass


level of dehydration
lost by evaporation of water
mild dehydration <3%
moderate dehydration 3–5%
severe dehydration >5%

(a) Calculate the mass of water lost by evaporation from the skin of the runner in one
hour.

mass of water lost = ……………………… [2]

Page 16 of 25

www.KiasuExamPaper.com
227
(b) Assume that the runner only loses energy by evaporation from his skin.

Calculate the rise in temperature of his body in one hour of the race.

rise in temperature = ……………………… [3]

(c) Using your answer to (b), show that evaporation from his skin is not sufficient, on
its own, to prevent overheating during the race.

……………………………………………………………………………………………....

……………………………………………………………………………………………....

…………………………………………………………………………………………….... [1]

(d) One other mechanism for evaporation occurs in breathing. Water is vaporised in
the lungs and is then exhaled.

Assume that there is no increase in the runner’s body temperature during the race.

Calculate the mass of water vapour that the runner exhales during the whole race.

mass of water vapour = ……………………… [2]

(e) Assume that the runner does not drink any water during the race.

Using your answers to (a) and (d), determine the level of dehydration of the runner
at the end of the race.

…………………………………………………………………………………………….... [2]

Page 17 of 25

www.KiasuExamPaper.com
228
16 Fig. 16.1 shows a hovercraft which moves on a cushion of air trapped underneath it.

Fig. 16.1

(a) At an instant, the propellers produce a total forward force, F of 22000 N and
experiences a total resistive force of 2000 N. The mass of the hovercraft is 25000
kg.

Calculate the acceleration of the hovercraft at this instant.

acceleration = ……………………… [2]

(b) After some time, the hovercraft reaches a steady speed, even though the force F
is unchanged.

Explain, in terms of the forces acting on the hovercraft, why the hovercraft will reach
steady speed.

…………………………………………………………………...………………………….

…………………………………………………………………...………………………….

…………………………………………………………………...…………………………. [2]

(c) Explain how the propeller is able to produce a forward force, F.

…………………………………………………………………...………………………….

…………………………………………………………………...…………………………. [1]

Page 18 of 25

www.KiasuExamPaper.com
229
(d) Fig. 16.2 shows how the speed, v of the hovercraft varies with time, t for part of its
journey.
v/ m/s

Fig. 16.2

(i) State the acceleration of the hovercraft between t = 0 s and t = 2.5 s.

…………………………………………………………………...…………………. [1]

(ii) Calculate the total distance travelled by the hovercraft in 4.5 s.

distance = ……………………… [2]

(iii) On Fig. 16.3, sketch how the distance travelled, d of the hovercraft varies
with time, t. You are not required to make any calculations.

Fig. 16.3
[2]

Page 19 of 25

www.KiasuExamPaper.com
230
17 EITHER

Fig. 17.1 shows an electrical circuit used to determine the resistance of an unknown fixed
resistor, RT. Three fixed resistors of 100 Ω, 150 Ω and 450 Ω are used to provide varying
resistance values, simulating the rheostat shown in the circuit diagram.

three fixed resistors


A
used to provide
varying resistances

RT V

Fig. 17.1

To obtain each resistance value recorded in the first column of Fig. 17.2, all three fixed
resistors are used. The resistance values are varied by placing the three resistors in
different arrangements. The switch is then closed and the corresponding voltmeter and
ammeter readings are recorded.

resistance / Ω voltage / V current / mA


P 5.64 120.1
213 2.14 46.4
232 1.99 43.2
510 0.97 21.6
Q 0.71 16.1

Fig. 17.2

(a) Given that P and Q respectively represent the smallest and largest possible
resistance values obtainable,

(i) calculate P.

P = ……………………… Ω [2]

(ii) calculate Q.

Q = ……………………… Ω [1]

Page 20 of 25

www.KiasuExamPaper.com
231
(iii) Explain why the voltage measured across RT decreases as the resistance
values in the first column of Fig. 17.2 increases.

……………………………………………………………………………………..

…………………………………………………………………………………….. [1]

(iv) A student uses the results obtained for resistance of 232 Ω from Fig. 17.2
to correctly estimate the value of the unknown resistor, RT.

Calculate the resistance obtained by the student.

resistance of RT = ……………………… [1]

(v) Using your answer in part (iv), estimate the potential difference (p.d.)
across the terminals of the batteries in the circuit.

p.d across terminals of batteries = ……………………… [2]

(b) Fig. 17.3 shows the I/V characteristic graph of two light emitting diodes (LED).

I / mA

50

40 LED A

30

LED B
20

10

0 V/V
2.8 2.9 3.0 3.1 3.2 3.3 3.4 3.5 3.6 3.7

Fig. 17.3

(i) Describe how I varies with V for LED B.

……………………………………………………………………………………..

…………………………………………………………………………………….. [1]

Page 21 of 25

www.KiasuExamPaper.com
232
(ii) Assume that all the power supplied is converted to light for both LED A and
LED B.

Determine which LED will be brighter at its maximum power dissipation.


Show your working clearly.

…………………………………………………………………………………….. [2]

Page 22 of 25

www.KiasuExamPaper.com
233
OR

Fig. 17.4 represents the basic structure of a transformer.

Fig. 17.4

(a) The secondary coil is connected to a lamp. When there is an alternating current in the
primary coil, the lamp is lit.

When there is a direct current in the primary coil, the lamp is not lit.

(i) State one way in which an alternating current differs from a direct current.

……………………………………………………………………………………........

……………………………………………………………………………………........ [1]

(ii) Explain why the lamp is not lit when there is a direct current in the primary coil.

……………………………………………………………………………………........

……………………………………………………………………………………........

……………………………………………………………………………………........ [2]

(b) An alternating voltage of 240 V is applied to the primary coil and a voltage is induced
in the secondary coil. The primary coil has 560 turns.

Calculate the smallest number of complete turns in the secondary coil that would give
an induced voltage of at least 8.0 V in the secondary coil.

number of turns = ……………………… [2]

Page 23 of 25

www.KiasuExamPaper.com
234
(c) A student determines the input and output power of the transformer and calculates
the efficiency of the transformer.

(i) The student uses voltmeters and ammeters that measure alternating voltages
and currents.

On Fig. 17.4, draw two voltmeters and two ammeters that enable the input
power and the output power of the transformer to be determined. [2]

(ii) State what is meant by efficiency.

……………………………………………………………………………………........

……………………………………………………………………………………........

……………………………………………………………………………………........ [1]

(iii) The current in the primary coil is 0.033 A. The current in the secondary coil is
0.72 A and the output voltage of the transformer is 8.0 V.

Calculate the efficiency of the transformer.

efficiency = ……………………… [2]

END OF PAPER

Page 24 of 25

www.KiasuExamPaper.com
235
BLANK PAGE

Page 25 of 25

www.KiasuExamPaper.com
236
2019 Year 4A Physics Preliminary Exam Suggested Solution

Q1 Q2 Q3 Q4 Q5 Q6 Q7 Q8 Q9 Q10
A B D A C B B D C A
Q11 Q12 Q13 Q14 Q15 Q16 Q17 Q18 Q19 Q20
D C D A A B D B A C
Q21 Q22 Q23 Q24 Q25 Q26 Q27 Q28 Q29 Q30
C B B B C B C C A D
Q31 Q32 Q33 Q34 Q35 Q36 Q37 Q38 Q39 Q40
B D A C D D A D B D

1 5.5 mm + 0.37mm = 5.87mm


5.87mm – (-0.04mm) = 5.91mm

2 a ͳ
ߟ௚௟௔௦௦ ൌ
•‹ ‫ܥ‬
ߟ௚௟௔௦௦ ൌ ͳǤͷʹ
b Angle of incidence = 55○
•‹ ͷͷι
•‹ ‫ ݎ‬ൌ
ͳǤͷʹ
‫ ݎ‬ൌ ͵ʹǤ͸ι

3 a

image
im
mag
ge
F F

thin converging lens


b Image is upright / Image is virtual

4 a 30 cm
b ‫ ݒ‬ൌ ݂ߣ
2.8 = f (0.8)
f = 3.5 Hz
c X-rays, Gamma rays

5 a The pulse spreads out in all directions as it propagates across the room, not all of the first
echo will be picked up by the microphone when it returns as the second echo.

2019 DHP Physics Year 4A Preliminary Exam 1


www.KiasuExamPaper.com
237
OR
Some sound energy was absorbed by the medium as the pulse propagates and absorbed
by the wall / microphone when the pulse hits.
b †‹•–ƒ ‡ൌ•’‡‡†ൈ–‹‡
ൌ ͵͵Ͳ ൈͲǤͲͺ0
ൌ ʹ͸ǤͶ 

6 Float.
The volume of the container is larger than 235 cm 3, hence the density is less than
1.0 g cm-3.

7 ai The centre of gravity of any object is defined as the point through which its whole weight
appears to act.
aii

b When
Wh
he
en
nddisplaced,
i p
is plla
acced
ced
d, the
the liline
th ine off ac
acti
action
tion
tion of weig
weight produces an anti–clockwise moment about the
pivot
ot to
pivo to return
returrn itit to
t its original
ori
rigi nal position.
g na positi
c Decrease
ecrea
ase
se the
the angle
an
ngl
gle
e between
bettween the vertical and the satay stick.
be

8 a Work done is the product of the force applied and the distance moved by the object in the
direction of
o the force.
b Word done against friction = GPE – KE
= mgh – ½ mv2
= (80)(10)(108) – ½ (80)(5.0)2
= 85400 J

9 ai ͳͳͺ kPa ൌ ݄ߩ݃ ൅ ͳͲͳ kPa


17 kPa = 4.5 × ߩ
ߩ ൌ ͵͹ͺͲ kg mିଷ

aii ͳ͵͸ͲͲ
ൈ ͹͸ ൌ ʹ͹Ͷ cm
͵͹͹͹Ǥͺ

2019 DHP Physics Year 4A Preliminary Exam 2


www.KiasuExamPaper.com
238
bi Force acting per unit area.
bii Increased surface area for the weight of the excavators will cause the pressure with the
ground to decrease, prevent excavator from sinking in the mud.

10 a The kinetic theory of matter states that the tiny particles that make up matter are always in
continuous, random motion.
b Increase in average kinetic energy/ speed. Increase in collision frequency and average force
per collision (against the walls of the container). Average force of collisions to increase.

11 a Sudden discharge may produce sparks, which can ignite the petrol.
b Earth the can and/or pipe.

12 a A1 has increased. Current has bypassed R1, effective resistance of circuit


circ has decreased.
b 1:1

13 a Live
b Blue
c Provides a path of low resistance forr current to flow
flo
ow through
thrro
th oug
ugh in the
he e
event
vent
ve nt the me
metal casing
becomes live.
d Double insulation / The casing iss insulated
in
nsulate
ed

14 a The particle
e must have a charge.
c ha
ch arrg
ge
e.
The charged particle
pa e mustt be
artticcle be moving.
movvin
ng.
b Electrons
Electr
trron
ns deflected,
d fleccte
de ed,, u
upwards.
pw
warrd
dss.
o deflection
No deflect
ctio
ct on to
io o microwaves.
mic
icrrowa
ave
ves.
s.

15 a Q = m lv
m = (2.2 x 10
06) ÷ (2.4
(2.4
(2 .4 x 1
1006)
≈ 0.91667
0.91
0.916
66
67
=0
0.
0.92
.92
.92
92 kkg
g ((2
2 sf) orr 0.917
0.91
0. 917
91 7 kg (3 sf)
b amt.
t. off thermal
t erma
th mall energy
ma ene
energy retained
ret in body in one hour = (3.3 – 2.2) x 106
= 1.1 x 106 J
Q = m c Δθ
Δθ = (1.1 x 106) ÷ (70 x 3400)
= 4.6 °C (2 sf) or 4.62 °C (3 sf)
c The body temperature in one hour during the race is already 37 + 4.6 = 41.6 °C. This is
higher than the 40 °C where overheating may cause serious damage.
d Q = m lv
in 1 hour, m = (1.1 x 106) ÷ (2.4 x 106)
= 0.45833 kg (5 sf)
in 4.5 hours, m = 4.5 x 0.45833
≈ 2.0625 (5 sf)
= 2.1 kg (2 sf) or 2.06 kg (3sf)

2019 DHP Physics Year 4A Preliminary Exam 3


www.KiasuExamPaper.com
239
e total amt. of water lost in 4.5 hours = (0.91667 x 4.5) + 2.0625
≈ 6.1875 kg (5 sf)
level of dehydration = (6.1875 ÷ 70) x 100%
= 8.8% (2 sf) or 8.84% (3 sf)
or = (6.1875 ÷ 63.8125) x 100%
= 9.7% (2 sf) or 9.70% (3 sf)
He will experience severe dehydration.

16 a Fresultant = ma
a = (22000 – 2000) / 25000
= 0.80 m s-2
b As the speed of the hovercraft increases, the air resistance acting on it increases.
When the air resistance equals the forward force, resultant force is zero.
er
c The propellers exert a force on the air in the backward direction
n which rresults in a force by
the air on the propellers in the forward direction.
di Acceleration is zero or 0 m/s2.
ii Distance = Area under the speed – time graph
ph
= (12.5)(2.5) + (5.0)(2.0)
= 41.25 m
iii

17 ai ͳ ͳ ͳ ͳ
൅ ൅ ൌ
Eith ͳͲͲͲ ͳͷͷͲ Ͷ
ͳͷͲ Ͷͷ
ͷͲ ܴ௘௙௙
ͶͷͲ
er P = 53

aii Q = 700 Ω
aiii As effective
ective resistance
resi increases, the current drawn decreases. For a fixed resistor RT, the
potential difference
d across the fixed resistor will decrease.
aiv RT = 1.99 ÷ (43.2 x 10-3) = 46.1 Ω
av Total resistance = 232 + 46.1 = 278.1 Ω
p.d across batteries = 278.1 × 0.0432 = 12.0 V
bi I = 0 A up till V = 2.9 V, thereafter, I increases non-proportionally with V.
bii) LED A will be brighter.
PA = 45 x 3.54 = 159.3 mW; PB = 33 x 3.7 = 122.1 mW

Or ai a.c. changes direction or changes polarity / from positive to negative continually.


ii There is no change in magnetic flux and hence, no induced voltage / current in the secondary
coil.

2019 DHP Physics Year 4A Preliminary Exam 4


www.KiasuExamPaper.com
240
b Ns = Vs / Vp x Np
= 8.0 / 240 x 560
= 19 (whole no.)
ci Ammeters in series with each coil;
Voltmeters in parallel with each coil
ii It is the ratio (or percentage / proportion / fraction) of useful output power (or energy) to input
power (or energy)
iii Efficiency = (0.72 x 8.0) / (0.033 x 240) x 100%
= 73% (2 sf) or 72.7% (3 sf)

2019 DHP Physics Year 4A Preliminary Exam 5


www.KiasuExamPaper.com
241
www.KiasuExamPaper.com
242
(3XUH3K\VLFV3UHOLP0*6

Page 2 of 18

1 The following shows a set of Vernier caliper reading before and after a coin is placed
between its jaws.

What is the zero error and the corrected radius of the coin, in cm?

zero error/cm corrected radius/cm


A -0.04 2.33
B -0.04 2.44
C 0.06 2.28
D 0.06 2.38

2 Which line in the table correctly indicates the prefixes micro, nano and giga?

× 10-6 ×10-9 ×109

A giga micro nano


B giga nano micro
C nano micro giga
D micro nano giga

3 A car is traveling at constant speed. Brakes are applied for a short period of time and the
car continues at a lower constant speed.

Which displacement-time graph shows the motion of the car?

displacement displacement

displacement displacement

Methodist Girls’ School Physics Paper 1 Sec 4 Preliminary Examination 2019

www.KiasuExamPaper.com
243
Page 3 of 18

4 Which diagram correctly shows the addition of a 3 N force and a 4 N force at right angles
to each other?
A B

5N
5N
4N
4N

3N 3N

C D

5N 5N
4N 4N

3N 3N

5 Twenty-seven identical small cubes are arranged to form a big cube as shown in the
diagram.

The cubes are made from the same material and the density of each cube is ρ. If one
small cube is removed from the arrangement, what is the density of the remaining cubes?

A ρ
B ρ
C ρ
D ρ

Methodist Girls’ School Physics Paper 1 Sec 4 Preliminary Examination 2019

www.KiasuExamPaper.com
244
Page 4 of 18

6 The diagram shows the rest position of a balancing toy on the edge of a table.

Which position is most likely to be the centre of mass of the toy?

7 A rectangular block of wood rests on the ground as shown in the diagram.

Which of the following is the easiest way for a force F to topple the block?

A B

C D

Methodist Girls’ School Physics Paper 1 Sec 4 Preliminary Examination 2019

www.KiasuExamPaper.com
245
Page 5 of 18

8 Car X is travelling at half the speed of car Y. Car X has twice the mass of car Y.

Which statement is correct?

A Car X has half the kinetic energy of car Y.


B Car X has one quarter of the kinetic energy of car Y.
C Car X has twice the kinetic energy of car Y.
D The two cars have the same kinetic energy.

9 A mass is raised vertically. In time t, the increase in its gravitational potential energy is
Ep and the increase in its kinetic energy is Ek.

What is the average power input to the mass?

A −
B +
C −

D +

10 A barrel of mass 50 kg is loaded onto the back of a lorry 1.6 m high by pushing it up a
smooth plank 3.4 m long. The gravitational field strength, g = 10 N/kg.

What is the minimum work done on the barrel?

A 80 J B 170 J C 800 J D 1700 J

Methodist Girls’ School Physics Paper 1 Sec 4 Preliminary Examination 2019

www.KiasuExamPaper.com
246
Page 6 of 18
11 The diagram shows the energy transfer of a machine.

The machine is 50% efficient.

Which of the following statements is correct?

A input energy = useful output energy


B useful output energy = input energy + wasted energy
C wasted energy = input energy + useful output energy
D wasted energy = useful output energy

12 The graph shows how the pressure exerted by a liquid varies with depth below the
surface. The gravitational field strength, g = 10 N/kg.

What is the density of the liquid?

A 600 kg/m3 B 750 kg/m3 C 6000 kg/m3 D 7500 kg/m3

Methodist Girls’ School Physics Paper 1 Sec 4 Preliminary Examination 2019

www.KiasuExamPaper.com
247
Page 7 of 18
13 Bubbles of gas, escaping from the mud at the bottom of a deep lake, rise to the surface.

As the bubbles rise, they get larger.

Why is this?

A Atmospheric pressure on the bubble decreases.


B Atmospheric pressure on the bubble increases.
C Water pressure on the bubbles decreases.
D Water pressure on the bubbles increases.

14 According to the kinetic theory of matter, matter is made up of very small particles in a
constant state of motion.

Which of the following best describes the particle behaviour in the liquid state?

forces between particles motion of particles


A strong move randomly at high speed
vibrate and are free to move
B strong
position
vibrate to and fro about a fixed
C weak
position
D weak move randomly at high speed

Methodist Girls’ School Physics Paper 1 Sec 4 Preliminary Examination 2019

www.KiasuExamPaper.com
248
Page 8 of 18
15 A partially-inflated balloon is placed inside a bell jar. The bell jar is connected to an air
pump. The air pump is switched on and air is slowly removed from the bell jar, keeping
the temperature of the air constant.

What happens to the pressure and to the volume of the gas inside the balloon?

pressure volume
A decreases decreases
B decreases increases
C increases decreases
D increases increases

16 The lengths of the mercury thread in the stem of a mercury thermometer placed in three
conditions are provided below:

● length in melting ice = 10 mm


● length in steam above boiling water = 160 mm
● length in liquid L = 70 mm

What is the temperature of liquid L?

A 37.5 oC
B 40.0 oC
C 43.8 oC
D 46.7 oC

17 Expanded polystyrene is often used to make containers for storing ice-cream. Air is
trapped within the expanded polystyrene.

Which process(es) of thermal energy loss does the expanded polystyrene reduce?

A conduction and convection only


B conduction and radiation only
C conduction only
D convection only

Methodist Girls’ School Physics Paper 1 Sec 4 Preliminary Examination 2019

www.KiasuExamPaper.com
249
Page 9 of 18
18 The table gives the specific heat capacities of four materials.

material specific heat capacity /


J/(kgK)

aluminium 913

lead 130

steel 420

water 4200

Four samples of the above materials, of equal masses were heated by an identical heat
source. The graph below shows how the energy of the four samples varies with their
temperature.

Which graph best represents the energy-temperature graph of aluminium?


energy / J

A B C D

temperature / K

19 The graph shows how the displacement of a particle in a wave varies with time.

Which of the following is correct?

A The wave has an amplitude of 2 cm and could be either transverse or longitudinal.


B The wave has an amplitude of 2 cm and must be transverse.
C The wave has an amplitude of 4 cm and could be either transverse or longitudinal.
D The wave has an amplitude of 4 cm and must be transverse.

Methodist Girls’ School Physics Paper 1 Sec 4 Preliminary Examination 2019

www.KiasuExamPaper.com
250
Page 10 of 18
20 Which list shows electromagnetic waves in order of increasing frequency?

A gamma rays, X rays, visible light


B visible light, X rays, gamma rays
C visible light, gamma rays, X rays
D X rays, gamma rays, visible light

21 The four statements shown are about the uses of electromagnetic radiation.

1. Gamma rays are used in medical treatment.


2. Infra-red waves are used in sunbeds.
3. Microwaves are used in satellite television
4. X rays are used in intruder alarms.

Which of these statements are correct?

A 1 and 2 B 1 and 3 C 2 and 3 D 2 and 4

22 What happens to light as it passes from glass into air?

A Its frequency decreases because its speed decreases.


B Its frequency increases because its speed increases.
C Its wavelength decreases because its speed decreases.
D Its wavelength increases because its speed increases.

23 Four people, P, Q, R and S, are standing in front of a plane mirror as shown.

How many people (including herself) can Q see in the mirror?

A 0
B 1
C 2
D 3

Methodist Girls’ School Physics Paper 1 Sec 4 Preliminary Examination 2019

www.KiasuExamPaper.com
251
Page 11 of 18
24 An object placed at 28 cm from a thin converging lens produces an image of the same
size as the object.

When the object is moved to 12 cm from the same lens, the image produced will be

A real, inverted, diminished.


B real, inverted, magnified.
C virtual, upright, magnified.
D virtual, upright, diminished.

25 Four different rays are passing through a diverging lens as shown in the figure.

Which ray does not represent the path after passing through the diverging lens?

26 A girl, standing 150 m in front of a tall building, fires a shot using a starting pistol. A boy,
standing 350 m from the girl, hears two bangs 1 s apart.

From this information, what is the speed of sound in air?

A 300 m/s B 350 m/s C 500 m/s D 650 m/s

Methodist Girls’ School Physics Paper 1 Sec 4 Preliminary Examination 2019

www.KiasuExamPaper.com
252
Page 12 of 18
27 The diagram shows a light spherical conductor X that is positively charged and
suspended in between two insulated copper spheres, P and Q, which are connected to
an power supply.

What will happen to conductor X when the switch is closed?

A move to P and stay attached to P


B move to P and then oscillate between P and Q
C move to Q and stay attached to Q
D move to Q and then oscillate between P and Q

28 Two resistance wires made of the same material and of the same length are connected
in parallel to the same voltage supply. Wire P has a diameter of 2 mm. Wire Q has a
diameter of 1 mm.

What is the ratio of current in P to current in Q?

A 0.25 B 0.50 C 2.0 D 4.0

29 The resistance of a component in a circuit increases as the current through the


component increases.

Which of the following graphs best represent the I-V characteristics of the component?

Methodist Girls’ School Physics Paper 1 Sec 4 Preliminary Examination 2019

www.KiasuExamPaper.com
253
Page 13 of 18
30 Three identical heating elements are wired up to the mains supply in the three
arrangements shown.

In which arrangement is the current through the supply lowest and in which is the highest?

lowest current highest current


A X Z
B X Y
C Y X
D Y Z

31 Two lamps, X and Y are connected to a battery and a rheostat as shown in the
diagram.

What will happen to the brightness of the lamps if the resistance of the rheostat is
decreased?

X Y
A dimmer dimmer
B dimmer brighter
C brighter dimmer
D brighter brighter

Methodist Girls’ School Physics Paper 1 Sec 4 Preliminary Examination 2019

www.KiasuExamPaper.com
254
Page 14 of 18

32 A plug is wrongly wired as shown. It is connected to an old vacuum cleaner, which has
a metal case.

What is the effect of using the plug wired in this way?

A The fuse in the plug blows.


B The metal case is live.
C The neutral wire melts.
D The vacuum cleaner catches fire.

33 The diagram shows an unsafe use of an extension cable.

What is the electrical hazard?

A the danger of burning out the appliances


B the danger of melting the fuse
C the danger of overheating the cable
D the danger of the appliances not being earthed.

Methodist Girls’ School Physics Paper 1 Sec 4 Preliminary Examination 2019

www.KiasuExamPaper.com
255
Page 15 of 18

34 Four plotting compasses are placed in the magnetic field of two identical bar magnets
as shown in the diagram.

Which compass is shown pointing in the wrong direction?

35 A permanent magnet can be demagnetised by using a solenoid and switching the


current on then off.

Which diagram shows the most effective method of producing demagnetisation?

Methodist Girls’ School Physics Paper 1 Sec 4 Preliminary Examination 2019

www.KiasuExamPaper.com
256
Page 16 of 18

36 Two straight electrical conductors are parallel to one another. Each carries a current,
one into the plane of the paper and one out of the plane of the paper.

Which diagram shows the magnetic field around the two wires?

37 A d.c. motor consists of a coil of many turns rotating in a fixed magnetic field. The coil is
connected to a d.c. supply through a split-ring commutator.

Each of these changes are made, one at a time and then compared to the above
arrangement.
● The d.c. supply is reversed.
● The coil is rotated before switching on, so that P starts on the right and Q on the
left.
● The poles of the magnet are reversed
● The turns on the coil are increased in number.

How many of these changes make the coil rotate in the opposite direction?

A 1 B 2 C 3 D 4

Methodist Girls’ School Physics Paper 1 Sec 4 Preliminary Examination 2019

www.KiasuExamPaper.com
257
Page 17 of 18

38 A student pushes the N-pole of a bar magnet into the end Q of a long solenoid and
observes a deflection to the right on the sensitive ammeter.

What will produce a deflection in the same direction?

A pulling the N-pole out of end Q


B pulling the S-pole out of end P
C pushing the N-pole into end P
D pushing the S-pole into end P

39 A signal generator is connected to an oscilloscope (c.r.o) as shown the in the diagram.

The Y-gain setting is 20 mV/div and time base setting is 5 ms/div.

Which of the following indicates the correct amplitude and frequency of the actual signal?

Amplitude / mV frequency / Hz
A 40 50
B 40 100
C 400 50
D 400 100

Methodist Girls’ School Physics Paper 1 Sec 4 Preliminary Examination 2019

www.KiasuExamPaper.com
258
Page 18 of 18

40 The diagram shows a metal bar swinging like a pendulum across a uniform magnetic
field. The motion induces an e.m.f. between the ends of the bar.

Which graph represents this e.m.f. during one complete oscillation of the bar, starting
and finishing at P?

End of Paper

Methodist Girls’ School Physics Paper 1 Sec 4 Preliminary Examination 2019

www.KiasuExamPaper.com
259
Page 2 of 20
Section A [50 marks]

Answer all the questions in this section in spaces provided.

1 Fig. 1.1 shows a large tank containing water. The tank leaks and drops of water fall from
the tank at A to the ground at B.

The drops hit the ground at a regular rate.

Fig. 1.1

(a) A student measures the time interval between two drops of water hitting the
ground. She uses a stopwatch and repeats the procedure three times. She
recorded the readings as shown.

1.24 s 1.14 s 1.16 s

(i) Calculate the average time interval between two drops of water hitting the
ground.

average time = .................................................. [1]

(ii) The average time interval calculated in (a)(i) is not accurate due to human
reaction time error.

Describe a modification to the above procedure to obtain a more accurate


value of the average time interval.

……………………………………………………………………………………..…..

……………………………………………………………………………………..…..

……………………………………………………………………………………..…..

………………………………………………………………..……………………. [2]

Methodist Girls’ School Physics Paper 2 Sec 4 Preliminary Examination 2019

www.KiasuExamPaper.com
260
Page 3 of 20
(b) The mass of one drop of water is 1.0 × 10-3 kg.

The gravitational field strength g is 10 N/kg.

(i) Calculate the change in gravitational potential energy of one drop of water as
it falls from the tank at A to the ground at B.

change in gravitational potential energy = .................................................. [2]

(ii) Determine the velocity of the drop of water just before it hits the ground at B.
The effects of air resistance is negligible.

velocity = .................................................. [2]

(iii) When the hole on the tank at A is enlarged, every drop of water falling from
the tank will be of a much greater mass.

State and explain how the velocity of the drop of water will differ from that
calculated in (b)(ii).

……………………………………………………………………………………..…..

……………………………………………………………………………………..…..

……………………………………………………………………………………..…..

………………………………………………………………..……………………. [2]

Methodist Girls’ School Physics Paper 2 Sec 4 Preliminary Examination 2019

www.KiasuExamPaper.com
261
Page 4 of 20

2 A car accelerates from rest in a straight line. During the first 14 s, the acceleration is
uniform and the car reaches a velocity of 25 m/s.

(a) (i) Calculate the acceleration of the car.

acceleration = .................................................. [2]

(ii) After the first 14 s, the velocity of the car continues to increase but the
acceleration decreases gradually. From 70 s to 80 s after the start, the car
moves at a constant velocity of 55 m/s.

On Fig. 2.1, sketch a possible velocity-time graph for the car.

velocity /
m/s

Fig. 2.1
[2]

(b) At a later time, the driver applies the brakes to stop. As he is wearing a seat belt,
his body slows down in his seat. However, a bag on the seat next to him slides
forwards, across the seat towards the front of the car.

Using ideas about the forces acting, explain why

(i) the driver slows down,

……………………………………………………………………………………..…..

………………………………………………………………..……………………. [1]

(ii) the bag slides forwards.

……………………………………………………………………………………..…..

………………………………………………………………..……………………. [1]

Methodist Girls’ School Physics Paper 2 Sec 4 Preliminary Examination 2019

www.KiasuExamPaper.com
262
Page 5 of 20
3 Fig. 3.1 shows a rocket as it takes off with an initial acceleration of 1.25 m/s2. The total
mass of the rocket and fuel is 40 000 kg. The gravitational field strength g is 10 N/kg.

Fig. 3.1

(a) Determine the total weight of the rocket and fuel.

weight = ………………………….. [1]

(b) Calculate the magnitude of

(i) the resultant force acting on the rocket as it rises,

magnitude of resultant force = ………………………….. [2]

(ii) the upward thrust of the engine on the rocket as it rises.

magnitude of thrust = ………………………….. [2]

(c) As the rocket rises, the fuel will be used up.

Explain how this affects the acceleration of the rocket as it rises.

………………………………………………………………………………………………...

………………………………………………………………………………………………...

………………………………………………………………………………………………...

………………………………………………………………………………..…………... [2]

4 Fig. 4.1 shows a Galilean thermometer. This thermometer is used to measure the

Methodist Girls’ School Physics Paper 2 Sec 4 Preliminary Examination 2019

www.KiasuExamPaper.com
263
Page 6 of 20
approximate temperature of the surrounding air in an air-conditioned room.

Fig. 4.1

The glass cylinder contains water. When the temperature of the water changes, so does
its density.

Each bulb has a label printed with a temperature, as shown in Fig. 4.1. The bulbs have
different densities. At 22 °C, only bulb A is at the bottom of the cylinder.

(a) Explain, in terms of density, why bulb A is at the bottom of the cylinder while bulb E
is at the top.

………………………………………………………………………………………………...

………………………………………………………………………………..…………... [1]

(b) The temperature of the surrounding air increases to a temperature above 23°C.

(i) Suggest one reason why there is a delay before the temperature of the water
increases to 23 °C.

……………………………………………………………………………………..…..

………………………………………………………………..……………………. [1]

(ii) Explain why, after this delay, bulb B sinks. Assume that the density of the
bulbs remains the same.

……………………………………………………………………………………..…..

……………………………………………………………………………………..…..

……………………………………………………………………………………..…..

………………………………………………………………..……………………. [2]

Methodist Girls’ School Physics Paper 2 Sec 4 Preliminary Examination 2019

www.KiasuExamPaper.com
264
Page 7 of 20

5 (a) State the principle of moments.

………………………………………………………………………………………………...

………………………………………………………………………………………………...

………………………………………………………………………………..…………... [2]

(b) Fig. 5.1 shows a device for punching holes in a piece of paper. A person applies a
force F at the end of the arm. Just before the hole is made in the paper, the arm is
at rest.
1.5 cm 7.5 cm

7.2 N

Fig. 5.1

Just before the hole is made, the upward force acting on the steel rod by the paper
is 7.2 N.

Calculate

(i) the moment of the upward force acting on the steel rod by the paper about
the pivot,

moment = ………………………….. [2]

(ii) the magnitude of the applied force F.

F = ………………………….. [2]

Methodist Girls’ School Physics Paper 2 Sec 4 Preliminary Examination 2019

www.KiasuExamPaper.com
265
Page 8 of 20

6 Fig. 6.1 shows part of a hydraulic jack used to lift the front of a car.

Fig. 6.1

The operator pulls the handle and causes a force of 50 N to act on the small piston. A
force F is then exerted by the oil on the large piston.

The cross-sectional area of the small piston is 1.5 cm2.


The cross-sectional area of the large piston is 5.0 cm2.

(a) Calculate

(i) the pressure in the oil caused by the force on the small piston,

pressure = ………………………….. [2]

(ii) the magnitude of F.

F = ………………………….. [2]

(b) Explain why the large piston moves through a shorter distance than the small
piston.

………………………………………………………………………………………………...

………………………………………………………………………………..…………... [1]

Methodist Girls’ School Physics Paper 2 Sec 4 Preliminary Examination 2019

www.KiasuExamPaper.com
266
Page 9 of 20
7 Fig. 7.1 shows the relay connected in a circuit to a 12 V battery. A relay is an electrical
circuit used to open and close contacts in another circuit.

Fig. 7.1

(a) Explain why the bell rings when the temperature of X rises.

………………………………………………………………………………………………...

………………………………………………………………………………………………...

………………………………………………………………………………………………...

………………………………………………………………………………..…………... [2]

(b) At a particular temperature, the resistance of X is 2000 Ω and the current in the coil
is 1.5 mA. This causes the switch AB in the relay to close. The resistance of the
bell is 200 Ω.

Calculate

(i) the potential difference across X,

potential difference = ………………………….. [2]

(ii) the potential difference across the coil,

potential difference = ………………………….. [1]

(iii) the current through the battery.

current = ………………………….. [2]

8 Fig. 8.1 shows an electrical cooker hood used in some kitchens. The hood removes

Methodist Girls’ School Physics Paper 2 Sec 4 Preliminary Examination 2019

www.KiasuExamPaper.com
267
Page 10 of 20
cooking fumes from the kitchen.

fan light bulb

Fig. 8.1

The hood has a fan and a light bulb. Fig. 8.2 shows a simplified circuit diagram for the
cooker hood.

fan

Fig. 8.2

(a) Switch S is closed. Switch T is moved to position Y.


State all the components that are switched on.

………………………………………………………………………………..…………... [1]

(b) Switch T is moved from position Y to position Z.


Suggest how this change affects the fan.

………………………………………………………………………………………………...

………………………………………………………………………………..…………... [1]

(c) Electrical appliances with metal cases can become dangerous if there is a fault.
Suggest a hazard and describe a safety feature to reduce the danger due to this
hazard.

hazard ..……………………………………………………………………………………...

………………………………………………………………………………………………...

safety feature …………………………………..…………………………………………...

………………………………………………………………………………..…………... [2]

Methodist Girls’ School Physics Paper 2 Sec 4 Preliminary Examination 2019

www.KiasuExamPaper.com
268
Page 11 of 20

9 When a magnet is placed near a small cardboard box, paper clips on the other side of
the box are picked up as shown in Fig. 9.1.

When a small piece of soft iron is placed inside the box as shown in Fig. 9.2 the paper
clips fall off.

Magnetic field lines in each diagram are shown as thin lines.

Fig. 9.1 Fig. 9.2

(a) The lines in Fig. 9.1 are further apart compared to the lines in Fig.9.2.

State what this shows about the magnetic field in Fig. 9.1 compared to that in Fig.
9.2.

………………………………………………………………………………..…………... [1]

(b) Explain why placing the soft iron inside the box causes the paper clips to fall off.

………………………………………………………………………………………………...

………………………………………………………………………………………………...

………………………………………………………………………………..…………... [1]

Methodist Girls’ School Physics Paper 2 Sec 4 Preliminary Examination 2019

www.KiasuExamPaper.com
269
Page 12 of 20
Section B [30 marks]

Answer all the questions in this section in spaces provided.

10 An explosion is triggered on the surface of the earth to investigate a layer of rock


underground.

Fig 10.1 shows the paths of how the sound waves propagate after an explosion.

Fig. 10.1

Sound waves from the explosion travel to the detector through air (path 1) and through
earth (path 2).

Some waves are transmitted through earth into the layer of rock by path 4 with part of
wave being reflected at the boundary between the earth and the layer of rock as
indicated by path 3.

Some waves also travelled along path 5 undergoing total internal reflection.

The time taken for the sound to reach the detector is shown in Fig. 10.2.

path 1 path 2 path 3


Time taken (in seconds) for sound to
0.10 0.02 0.30
travel from the source to the detector

Fig. 10.2

(a) Sound is a longitudinal wave.


Describe how the particles in earth move as the sound passes.

………………………………………………………………………………………………...

………………………………………………………………………………..…………... [1]

(b) Suggest a reason why sound wave takes the shortest time to reach the detector
along path 2.

………………………………………………………………………………………………...

………………………………………………………………………………..…………... [1]

Methodist Girls’ School Physics Paper 2 Sec 4 Preliminary Examination 2019

www.KiasuExamPaper.com
270
Page 13 of 20

(c) (i) Given that the speed of sound in air is 330 m/s, calculate the distance
between the source of sound and the detector.

distance = ………………………….. [2]

(ii) Using your answer in part (c)(i), calculate the speed of sound in earth.

speed = ………………………….. [1]

(d) State and explain how the speed of sound changes when it travels from earth to
the layer of rock along path 4.

………………………………………………………………………………………………...

………………………………………………………………………………..…………... [2]

(e) State two conditions that allows total internal reflection to occur along path 5.

………………………………………………………………………………………………...

………………………………………………………………………………………………...

………………………………………………………………………………………………...

………………………………………………………………………………..…………... [2]

(f) There are small bubbles of gas in the earth.

Explain why the volume of the bubbles fluctuates as the sound passes through the
earth and the bubbles.

………………………………………………………………………………………………...

………………………………………………………………………………..…………... [1]

Methodist Girls’ School Physics Paper 2 Sec 4 Preliminary Examination 2019

www.KiasuExamPaper.com
271
Page 14 of 20

11 (a) Fig. 11.1 shows an open tray for storing water.

It is noticed that the level of water inside the tray slowly decreases as water
evaporates.
water
tray
Fig. 11.1

(i) Using ideas about molecules, explain how the temperature of the water is
affected when it evaporates.

[3]

(ii) Fig 11.2 shows a sheet of plastic used to cover half the surface of the water.

water plastic sheet

tray
Fig. 11.2

State how this affects the rate of evaporation.

[1]

Methodist Girls’ School Physics Paper 2 Sec 4 Preliminary Examination 2019

www.KiasuExamPaper.com
272
Page 15 of 20

(b) Liquid air contains a mixture of oxygen and nitrogen. The boiling point of nitrogen
is lower than oxygen. A sample of liquid air in a beaker is allowed to warm up
slowly.

Fig. 11.2 shows how the reading of a thermometer in the beaker varies with
time t.

Fig. 11.2

(i) On Fig 11.2, label the boiling point of nitrogen, N on the temperature axis.
[1]

(ii) The liquid air contains 200 g of liquid oxygen and 800 g of liquid nitrogen.
Table 11.3 shows the table of specific heat capacity and specific latent heat
of both gases.

specific heat capacity specific latent heat of


J/(goC) vaporisation J/g
oxygen 1.7 213
nitrogen 2.0 199

Table 11.3

Determine

(1) the thermal energy absorbed by the combined liquid air to reach the
temperature at the 1 min mark.

thermal energy = ……………………..[2]

Methodist Girls’ School Physics Paper 2 Sec 4 Preliminary Examination 2019

www.KiasuExamPaper.com
273
Page 16 of 20

(2) the total thermal energy absorbed by the combined liquid air to reach
the 4 min mark.

thermal energy = …………………...[3]

Methodist Girls’ School Physics Paper 2 Sec 4 Preliminary Examination 2019

www.KiasuExamPaper.com
274
Page 17 of 20
EITHER
12 Fig. 12.1 shows an electrical “wind-up” torchlight that operates through cranking a
mechanical handle. The crank is turned in one direction and energy is stored in the
spring internally. The torchlight then uses the energy stored in the wound-up spring to
light the light bulb.

Fig 12.2 shows the simplified diagram of the mechanism within the torchlight. The
spring is unwinding and N pole is moving away from the solenoid.

crank

solenoid with
soft iron core
to light bulb magnet
N

spring

Fig. 12.1 Fig. 12.2

(a) (i) On Fig 12.2, draw the current induced on the solenoid.
[1]

(ii) Explain how the direction of the induced current in (a)(i) is determined.

[1]

(b) As the spring is unwound, the magnet rotates together with the spring.

(i) Explain why electromotive force is induced in the solenoid.

[2]

(ii) Describe the various stages of how energy is converted when the spring is
unwound to the light bulb.

[1]

Methodist Girls’ School Physics Paper 2 Sec 4 Preliminary Examination 2019

www.KiasuExamPaper.com
275
Page 18 of 20

(iii) Explain why the induced current is an alternating current (A.C.).

[1]

(c) When the spring is tightly wound, the electrical signal from the wires is applied to
the input terminals of an oscilloscope. Fig 12.3 shows the trace obtained on the
screen of an oscilloscope of e.m.f. vs time.

e.m.f / V

time / s

Fig 12.3

(i) The position between the magnet and the solenoid affects the strength of
the e.m.f. induced in the coil.

On Fig. 12.3, indicate the position of the magnet poles by labelling N and S
to the corresponding trace on the oscilloscope.
[2]

(ii) After a period of time, the rotation of the unwinding spring slows down to
half.

On Fig. 12.3, sketch the new electrical signal trace produced by the spring.

[2]

Methodist Girls’ School Physics Paper 2 Sec 4 Preliminary Examination 2019

www.KiasuExamPaper.com
276
Page 19 of 20
OR
12 (a) Fig 12.1 shows a simple transformer.

Iron core

primary secondary

Fig. 12.1

(i) Explain how an alternating e.m.f. in the primary coil induces an e.m.f. in the
secondary coil.

[2]

(ii) State and explain one method to improve the efficiency of the transformer.

[2]

Methodist Girls’ School Physics Paper 2 Sec 4 Preliminary Examination 2019

www.KiasuExamPaper.com
277
Page 20 of 20

(b) Fig. 12.2 shows a consumer connected to a main electrical supply some distance
away. The electrical supplier generates electrical supply at 25 kV and transmits it
to the consumer that uses 20 kV. Transformer X steps up the output voltage to
275 kV.

electrical cable consumer


supply at
20 kV
25 kV
transformer X transformer Y
Fig. 12.2

(i) Explain why voltage is stepped up from the electrical supply and transmitted
at high voltage.

[1]

(ii) Determine the turns ratio for transformer X.

turns ratio = …………………[2]

(iii) 10 MW of power is transmitted through the cable of resistance 1 :/km at


275 kV.

Determine the power loss per kilometre as internal energy in the cable.

power loss per km = …………………[3]

End of Paper
Methodist Girls’ School Physics Paper 2 Sec 4 Preliminary Examination 2019

www.KiasuExamPaper.com
278
www.KiasuExamPaper.com
279
Answers:

1 2 3 4 5 6 7 8 9 10
C D B B A C D A D C
11 12 13 14 15 16
16 17
17 18
18 19
19 20
2
D B C B B B C B A B
21 22 23 24 25
5 26
6 27
27 28
28 29
29 30
B D C C B A B D B B
31 32 33 34 35
35 36
6 37 38
38 39
39 40
C B C D D C B D A A

Methodist Girls’ School Physics Paper 1 Sec 4 Preliminary Examination 2019

www.KiasuExamPaper.com
280
Methodist Girls’ School
Physics Sec 4 Preliminary Examination 2019
Marking Scheme

Section A
1(a)(i) (1.24 + 1.14 + 1.16) ÷ 3 = 1.18 s A1
1(a)(ii) - find time interval between 21 (appropriate no.) drops of water hitting the
ground, t B1
- find the average timing for the time interval between two drops using
T1 = t/20 = t / (no.of drops - 1) B1

1(b)(i) GPE = mgh = 0.001 x 10 x 12 C1


= 0.12 J A1
1(b)(ii) KE = GPE
½ mv2 = 0.12
½ (0.001) v2 = 0.12 C1
v = 15.5 m/s OR 15 m/s e.c.f. A1

1(b)(iii) Velocity will remain unchanged. B1

As ½ mv2 = mgh, m will be cancelled in this equation.n. As, height


heig
he ight
ht and g remain
unchanged, v calculated will be the same. B1
OR Acceleration due to free fall is constant, therefore,
t, therefor allll m
re, a masses
asse
as with the same
s s falls w
velocity.

2(a)(i) a = v–u / t = 25 / 14 C1
= 1.79 m/s2 OR 1.8 mm/ss2
//s A1

2(a)(ii) B1 – shape fromm 14 to


o 80
80 s
B1 – correct
ct coordinates
coordinat
ates
at es

(80,55)
(14,
(14,25)
4,25)
25
25) tto
o
(70,55)
(70,55
55)
55

2(b)(i) force backwards on driver / car B1


OR force produced by seat belt
2(b)(ii) mass of bag resists change from state of motion B1
OR bag has inertia

3(a) weight = mg = 40 000 x 10 = 400 000 N A1


3(b)(i) F = ma = 40 000 x 1.25 C1
= 50 000 N A1
3(b)(ii) resultant F = Thrust – weight
50 000 = Thrust – 400 000 C1
Thrust = 450 000 N e.c.f. A1

3(c) As the fuel is used up, the total mass (weight) of the rocket and fuel decreases,

www.KiasuExamPaper.com
281 1
thrust remains the same, resulting in an increase in resultant force. B1
Hence, resultant force and acceleration will increase. B1

4(a) Density of bulb A greater than the density of the water (and sinks)
AND Density of bulb E less than the density of water (and float) B1

4(b)(i) Water / Glass is a poor conductor of heat


OR water / glass conducts heat at a slow rate
OR water / glass has a high (specific) heat capacity B1

4(b)(ii) The water expands and becomes less dense when temp increase. B1
Bulb B now has a greater density than the water (and sinks)
OR Weight of bulb B more than buoyancy forces / upthrust B1

5(a) The principle of moment states that for an object in equilibrium, B1


the sum of clockwise moments about a point is equal to
the sum of anti-clockwise moments about the same point. B1

5(b)(i) Moment = 7.2 x 1.5 C1


= 10.8 Ncm (0.108 Nm) A1
5(b)(ii) F = 10.8 / 9.0 e
e.c.f. C1
= 1.2 N A1

6(a)(i) Pressure = Force / Area


= 50 / 1.5 C1
= 33.3 N/cm2 OR 3.33 3.3
33 x 10 05 N/m/m2 ((Pa)
N/m Pa) O
Pa ORR 333
333 kkPa
Pa A1
6(a)(ii) F = Pressure re x ar area
rea
e
= 33.33 x 5.0
5.0
0 e.c.f. C1
= 167 N OR OR 1 170
70 N
70 A1
6(b) the displaced
ced volume oililil rremains
e off o emai
em ns the
ains he ssame,
the a e, as the
am e larger
arger piston
la piston with
ton
to larger cross-
with larg
sectional
nal area a results
res u ts in
esul
es in a smaller
sma
sm er d
allle B1
OR Work
R Wororrk donee = Fd
Fd and
andd when
whe hen work
w rk done
wo don
onne iss constant,
con t, distance
o stant, dis tance is small when force is
ista
ta
large
larg
ge

7(a) rresistance
rees is ance (of
siist (of X) X) decreases
decrea ase
ses B1
ccurrent
cuurrrre
ennt (in
((iin coil)
cooilil)) increases
incrreaeas
se es or
or more voltage
voltltltag
vo age
ag e across
acros coil and either relay switch
ccloses
cl os or
los
oses or circuituiit (to
c rcui
ci ((tto bell)
ellll) complete
be
b comple
co ete B1

7(b)(i) V = 1.5
1.5 10––33 × 2000
5 ×10
×10 00 C1
= 3.
3 .0 V
3.0 A1
7(b)(ii) 12 – 3 = 9.0
9.0 V e.c.f. B1
7(b)(iii) wing th
I flowing hro
roug
ugh
ug
through h bell = 12 / 200 = 0.06 A or 60 mA C1
attery
ry
I in battery = 1.5 + 60 mA or 0.0015 +0.06 A
= 61.5 mA or 62 mA
= 0.0615 A or 0.062 A A1

8(a) motor / fan AND lamp B1


8(b) motor / fan speed decreases / slows down B1
8(c) hazard: live wire touching case AND
user gets electric shock / burns
OR electrical fire due overheating / wire gets hot B1
safety feature: case is earthed OR connect earth wire to the metal case B1

9(a) lines that are further apart shows weaker magnetic field strength
OR lines that are closer shows stronger magnetic field strength B1

9(b) magnetic field goes through soft iron OR no field through paper clips
paper clips lose their (induced) magnetism / cannot be magnetised B1

www.KiasuExamPaper.com
282 2
Section B
10(a) vibrate / oscillate in the same direction or parallel to transfer of energy or wave B1
10(b) sound travels faster in solid (earth) compared to air B1
10(c)(i) Distance = 330 x 0.10 C1
= 33 m A1
10(c)(ii) Speed = 33/0.02
=1650 m/s A1

10(d) As the sound wave bends away from the normal as it travels from earth to rock, B1
its speed in rock is faster than earth. B1

10(e) Wave must travel from a region of lower speed to a region of higher speed. B1
Angle of incidence is larger than critical angle from a region of lower speed to a region
of higher speed. B1

10(f) pressure increases and decreases B1

www.KiasuExamPaper.com
283 3
Page 14 of 20

11 (a) Fig. 11.1 shows an open tray for storing water.

It is noticed that the level of water inside the tray slowly decreases as water
evaporates.
water
tray
Fig. 11.1

(i) Using ideas about molecules, explain how the temperature of the water is
affected when it evaporates.

11 a i Faster moving molecules escape from the attraction of their neighbours B1


and leave surface of the liq.
Leaving behind slower moving molecules B1
Avg KE dec and temp dec B1

[3]

(ii) Fig
F
Fig 11.2
11
1.2
2 sshows
hows
how a sh
ssheet
hee
eet of
o plastic
plasttic
c used
us
seed to cover
covver half
hal
alff the
th
he surface
su of the water.

water
wa
watte
e
err plastic
plas
plasttic sheet
she
sh

tray
Fig. 11.2
Fi

State
St
Staatte ho
h
how
ow this a
affects
ffec
ff ectts
ec ts the rate of evaporation.

[1]

ii Rate of evaporation dec B1

Methodist Girls’ School Physics Paper 2 Sec 4 Preliminary Examination 2019

www.KiasuExamPaper.com
284
Page 15 of 20

(b) Liquid air contains a mixture of oxygen and nitrogen. The boiling point of nitrogen
is lower than oxygen. A sample of liquid air in a beaker is allowed to warm up
slowly.

Fig. 11.2 shows how the reading of a thermometer in the beaker varies with
time t.

Fig. 11.2
2

b i B1

(i)) On Fi
Fig
g 11.2
11.2, label the boiling point of nitrogen, N on the temperature axis.
[1]

(ii) The liquid air contains 200 g of liquid oxygen and 800 g of liquid nitrogen.
Table 11.3 shows the table of specific heat capacity and specific latent heat
of both gases.

specific heat capacity specific latent heat of


J/(goC) vaporisation J/g
oxygen 1.7 213
nitrogen 2.0 199

Table 11.3

Determine

Methodist Girls’ School Physics Paper 2 Sec 4 Preliminary Examination 2019

www.KiasuExamPaper.com
285
Page 16 of 20
(1) the thermal energy absorbed by the combined liquid air to reach the
temperature at the 1 min mark.

thermal energy = ……………………..[2]

(2) the total thermal energy absorbed by the combined liquid air to reach
the 4 min mark.

thermal ener
energy
rgy = ……
…………………...[3]

ii (1)Q=mc x theta
=200 x 1.7 x 9 + 800 x 2 x 9 C1
=17.5 x 10^3 J A1

(2) Q=ml B1
= 800 x 199
= 159 x 10^3
0^3 J A1
Total Q = 159 + 17
17.5
7.5 = 177
177 x 10^3
77 10^
0 3 J (ecf) A1

Methodist Girls’ School Physics Paper 2 Sec 4 Preliminary Examination 2019

www.KiasuExamPaper.com
286
Page 17 of 20
EITHER
12 Fig. 12.1 shows an electrical “wind-up” torchlight that operates through cranking a
mechanical handle. The crank is turned in one direction and energy is stored in the
spring internally. The torchlight then uses the energy stored in the wound-up spring to
light the light bulb.

Fig 12.2 shows the simplified diagram of the mechanism within the torchlight. The
spring is unwinding and N pole is moving away from the solenoid.

crank

solenoid with
soft iron core
to light bulb magnet
N

spring
spri
spring
ng

Fig. 12.1 Fig.


Fig. 12.2
12.2

(a) (i) On Fig 12.2, draw the current


curren
nt induced
induce
ce
ed on
on the
the
he solenoid.
solenoi
oid.
oid
[1]

a i B1

(ii)
(i
ii)
i) Explain
Exp
plai
pl
plai
ain how
ain h w the
ho th
he direction
dire
directio
e ioon of the
the
he induced
ind
nduc
uced ccurrent in (a)(i) is determined.
uc

[1]

ii Induced
ndu
uced cu
current is in direction to oppose the change causing it.
ORR S polarity induced in coil facing N pole of magnet to attract it. B1

(b) As the spring is unwound, the magnet rotates together with the spring.

(i) Explain why electromotive force is induced in the solenoid.

[2]

Methodist Girls’ School Physics Paper 2 Sec 4 Preliminary Examination 2019

www.KiasuExamPaper.com
287
Page 18 of 20
b i When magnet rotates, there is a changing magnetic field. B1
This induces an emf when magnetic field lines are “cut” by coil/magnetic
flux linkage B1

(ii) Describe the various stages of how energy is converted when the spring is
unwound to the light bulb.

[1]

ii KE of unwinding spring to electrical energy to light energy


ergy (b
(bulb) B1

(iii) Explain why the induced current is an alterna


alternating
ati
ting
ng ccurrent
u re
ur ren
nt (A.C.
(A.C.).

[1]

iiii Ever
Everytime
eryt
er ytime e th
tthe
he ma
m
magnet
ag
gnnet
e rot
rotates
ta
attes
es 180
180
0 deg,
deg
eg, the
e current
currrent induced
cu ind reverses its B1
direction
direct ctio
ct on to
io to the
the light
ht bulb
ht bu
ullb
O magnet
OR mag agne
ag moves
et moveve
v away
es aw
away tthen
he
h moves
en mo
move
ves
ve towards coil, the current reverses
s to
toward
itiits direction
ts di
direct
r ec
re cttion

(c) Whenen
en the
the spring
spr
prin
ing
ing is
is tightly
tigh wound, the electrical signal from the wires is applied to
the
e input
p t terminals
inpu
pu term
ter inals of an oscilloscope. Fig 12.3 shows the trace obtained on the
screen
een of an ooscilloscope of e.m.f. vs time.

Methodist Girls’ School Physics Paper 2 Sec 4 Preliminary Examination 2019

www.KiasuExamPaper.com
288
Page 19 of 20
e.m.f / V

time / s

Fig 12.3

(i) The position between n tthe


h ma
he magnet
agn
gnet
e an
and
nd th
tthe
he so
solenoid
ole
eno
n id a
affects
ffec
ff ects the strength
ec streng of
the e.m.f.. induced
induc
uced
uc ed in the
in th ccoil.
he co il.

On Fi
Fig.
ig. 12
1
12.3,
.3, in
iindicate
nd
diic
ca
atte
e th
the
he po
p
position
ossiittiion
o ooff th
tthe
e ma
magnet
agn
gneet po
poles
ole
less by llabelling
abelli N and S
to the
e ccorresponding
o re
or resp
s ondi d ng trace
trrac on the
ace thhe oscilloscope.
o cilloscope.
os
[2]

(ii)
(i
ii) After
Af
ftteer a period
p riod o
pe off time,
ttiime
me, the
tth
he rotation
ro
otta
ation
n of tthe
he unw
unwinding spring slows down to
half.
halflff.

On Fig.
Fig
g. 1
12
12.3,
2.3
.3,
3, sketch
sk
sketch
h the
the new
new
e electrical
ele signal trace produced by the spring.

[2]

Methodist Girls’ School Physics Paper 2 Sec 4 Preliminary Examination 2019

www.KiasuExamPaper.com
289
Page 20 of 20
b I
&
ii
N or S N or S

S or N S or N

B1 correct location of N
B1 correct location of S

mplitude
B1 correct amplitudede of
de o ggraph
raph
ra p
B1 correctt freq/sh
freq/shape
shap
sh ape of
of graph
grra
aph

OR
12 (a)
(a
(a) Fig
Fi
Fig 12.1
1 shows
show
show
ws a simple
simp
mple
mple transformer.
trra
anns
ssfo
forrm
fo mer.

Iron
Ir
ron
o core
cor
ore

primary
prima secondary

Fig. 12.1

(i) Explain how an alternating e.m.f. in the primary coil induces an e.m.f. in the
secondary coil.

Methodist Girls’ School Physics Paper 2 Sec 4 Preliminary Examination 2019

www.KiasuExamPaper.com
290
Page 21 of 20
[2]

a i AC in pri coil produces a


changing magnetic field in the soft iron core B1
magnetic field line “cut” at sec coil induces a changing emf in sec coil B1

(ii) State and explain one method to improve the efficiency of the transformer.

[2]

ii Laminated core B1
- reduce eddy currents, reduce thermal energy loss
oss B1

OR

Methods described in textbookk and substantiated


substant
nttia
iate
ate
ted
d with
w th correct
wi cor
orre
rect
rect effects.
eff
f ects

(b) Fig. 12.2 shows a consumer


conssu um
merr co
cconnected
onneccte
ted to a m
ma
main
ain e
electrical
le
ect
ctrica
al su
sup
supply
pply some
m dist
distance
away. The electrical
lectrical
all supplier
a sup
uppl
plie
i r generates
ge
g enerate
te
es electrical supply
sup
upp ply at 25
25 kVV and
and transmits
tra
r nsm it
to the consumer
onsumer that
tha
ha
h at uses
usse
ess 20
20 kV.
kV
k V. Tr
Transformer
T ra
annsformer X steps
ste
st eps up p the
the o
output
utput vo
ut voltage to
275 kV.

elecctrrical
electrical
e cable
e consumer
supply
supp
su pllyy at
20 kV
25
25 kV V
transformer
tran
ansf
sfor
orme
m rX transformer Y
Fig. 12.2

(i)) Explai
Explain
Expl ain
n why voltage is stepped up from the electrical supply and transmitted
high
at h igh voltage.
v

[1]

b i To reduce power loss as thermal energy B1

(ii) Determine the turns ratio for transformer X.

Methodist Girls’ School Physics Paper 2 Sec 4 Preliminary Examination 2019

www.KiasuExamPaper.com
291
Page 22 of 20

turns ratio = …………………[2]

ii Ns/Np = Vs/Vp
=275k/25k B1
=11 A1

(iii) 10 MW of power is transmitted through the cable of resistance 1 :/km at


275 kV.

Determine the power loss per kilometre as internal energy in the cable.

po
power loss
power ss p er km = …………………[3]
per

iiiii
ii P lo
lloss
oss
s = II^2R
^^2
2R
= (P/V)^2
(P/V
(P /V)^2
2xR B1
((10
= (1 0 x 10^6/275x10^3)^2
10
0^6^6/2
/275
75x10^3)
3)^2
3) ^2 x 1 B1
1320
= 13 320 W/km
20 W/
W /k
km
m A1

End of Paper

Methodist Girls’ School Physics Paper 2 Sec 4 Preliminary Examination 2019

www.KiasuExamPaper.com
292
www.KiasuExamPaper.com
293
1

NORTH VISTA SECONDARY SCHOOL

PRELIMINARY EXAMINATION 2019


40

NAME: ________________________________( ) CLASS: _________

SUBJECT: PHYSICS DATE: 18 SEP 2019

LEVEL / STREAM: SECONDARY 4 EXPRESS TIME: 1 HR

CODE : 6091/1

INSTRUCTIONS TO CANDIDATES

Write in soft pencil.


Do not use staples, paper clips, highlighters, glue or correction fluid.
Write your full name, index number and class on the Answer Sheet in the spaces
provided.

There are forty questions on this paper. Answer all questions. For each question there
are four possible answers A, B, C and D.
Choose the one you consider correct and record your choice in soft pencil on the
separate Answer Sheet.

Read the instructions on the Answer Sheet very carefully.

Each correct answer will score one mark. A mark will not be deducted for a wrong
answer.
Any rough working should be done in this booklet.
The use of an approved scientific calculator is expected, where appropriate.

This question paper consists of 15 printed pages.


[Turn over
www.KiasuExamPaper.com
294
2

1 What is the correct order of magnitude for the diameter of Earth and diameter of a strand of
human hair?

diameter of Earth diameter of human hair


A 10Gm 0.1mm
B 10Gm 0.1nm
C 10Mm 0.1mm
D 10Mm 0.1nm

2 Which is not a unit of a base quantity?

A ampere
B Kelvin
C kilogram
D Pascal

3 A vernier calipers is used to measure the diameter of a glass ball.

With the jaws closed and no glass ball, the vernier calipers reading is shown in Fig. 2.1.
With the jaws closed around the glass ball, the vernier calipers reading is shown in Fig. 2.2.

0 1 10 11

cm cm

0 1 0 1
Fig. 2.1 Fig. 2.2

What is the diameter of the glass ball?

A 10.07cm B 10.17cm C 10.19cm D 11.36cm

www.KiasuExamPaper.com
295
3

4 The bob of a simple pendulum is pulled to one side and released. The motion during its swing
is shown on the graph.

15

2.0 4.0 6.0 8.0

What is the period of the pendulum?

A 2.0s B 4.0s C 6.0s D 8.0s

5 A ball is falling at terminal velocity.

Which row best describes the acceleration of the ball and the velocity of the ball?

acceleration of ball velocity of ball


A downwards constant
B downwards zero
C zero constant
D zero zero

www.KiasuExamPaper.com
296
4

6 A rock was dropped on Earth and it accelerates at about 10m/s2. When the rock is dropped on
Jupiter, it accelerates at about 24.5m/s2.

Which graphs are the corresponding graphs of velocity against time for the rock?

A B
velocity velocity

Jupiter

Earth

time time

C D
velocity velocity

Jupiter

time time

7 A ball is released from the bottom of a pond. After a short time, it rises at a constant speed.

Which statement best describes the resultant force then acting on the ball?

A Upwards and equal to the ball’s weight


B Upwards and greater than the ball’s weight
C Upwards and less than the ball’s weight
D Zero

www.KiasuExamPaper.com
297
5

8 The diagram below shows a man walking along a road in the direction shown.

In which direction is the force of friction exerted by the road on the foot of the man?

direction of travel

road

9 What must change when a body is accelerating?

A the mass of the body


B the resultant force of the body
C the speed of the body
D the velocity of the body

10 The diagram shows a container filled with a liquid. There is a bubble in the liquid. The container
is moved forward with a constant velocity as shown.

forward motion

What will be the motion of the air bubble if the container suddenly decelerates?

A Air bubble will move in the backward direction.


B Air bubble will move in the forward direction.
C Air bubble will remain at the original position.
D Motion of air bubble cannot be determined as the total mass of the liquid is unknown.

www.KiasuExamPaper.com
298
6

11 Two cubes X and Y are made of iron.

X Y
Cube X has sides that are twice as long as cube Y.

Which statement is correct?

A The density of cube X is eight times that of cube Y.


B The density of cube X is four times that of cube Y.
C The density of cube X is two times that of cube Y.
D The density of cube X is equal to that of cube Y.

12 The diagrams show the cross-section of four solid objects.

Which object is the least stable?

A B C D

13 The diagram shows a hydraulic press being balanced by a 150N force and a 1500kg mass.

piston area of 10cm2 piston area A

The area of the smaller piston is 10cm2 and the area of the larger piston is A.

What is the value of A?

A 1.0cm2 B 10cm2 C 100cm2 D 1000cm2

www.KiasuExamPaper.com
299
7

14 A long tube, full of mercury, is inverted in a small dish of mercury.

The mercury level in the tube falls, leaving a vacuum at the top.

When the atmospheric pressure decreases, which length increases?

A QR B PQ C PR D PS

15 A workman exerts a force of 1300N to move a box of weight 2000N up a plank and onto a lorry.
The plank is 2.0m long and the back of the lorry is 0.80m above the horizontal surface of the
road.

The box also experiences a friction force of 500N.

1300N
500N 2000N

What is the useful work done on the box?

A 1600J B 2600J C 3000J D 4000J

16 A rocket of total mass M is travelling at a speed v. The engine of the rocket is fired and fuel is
used up. The mass of the rocket decreases to ½M and its speed increases to 2v.

What happens to the kinetic energy of the rocket?

A It doubles.
B It halves.
C It increases by a factor of 4.
D It remains the same.

www.KiasuExamPaper.com
300
8

17 Smoke particles are introduced into a glass container.

When they are viewed under a microscope, the smoke particles are seen to be moving in a
continuous and random motion.

Which row explains the motion of the smoke particles?

A collisions by air particles


B collisions with other smoke particles
C collisions against the walls of the glass container by the smoke particles
D motion due to high internal kinetic energy of the smoke particles

18 Hot water rises and cold water sinks due to changes in density.

Which statement explains the change in density?

A The water particles contract when heated.


B The water particles expand when heated.
C The water particles move further apart from each other when heated.
D The water particles have a smaller mass when heated.

19 The pressure of a gas in a container is the same at all points in the container.

Which statement explains this?

A The gas particles have the same size.


B The gas particles make the same number of collisions with the internal walls of the
container per unit time.
C The gas particles move at the same speed.
D The gas particles are all moving in a continuous and random motion.

20 Oxygen can be supplied to a fish tank by bubbling air into the water.

What row describes the changes in the pressure and volume of the air bubbles when they rise
to the surface of the water?

volume pressure
A increases increases
B increases decreases
C decreases increases
D decreases decreases

www.KiasuExamPaper.com
301
9

21 A pot is used to boil water.

How is thermal energy transferred from the base of a pot to the water?

A by conduction only
B by convection only
C by conduction and convection only
D by convection and radiation only

22 A piece of wire has a resistance of 0.50Ω in melting ice and 2.50Ω in steam above boiling
water.

What is the resistance of the wire at 40°C assuming that the resistance changes uniformly with
temperature?

A 0.40Ω B 0.80Ω C 1.30Ω D 1.50Ω

23 Two well-insulated copper blocks P and Q are heated at the same power. The diagram shows
the variation of temperature with time of the two blocks.

temperature / °C
P Q
80

60

40

20

0
time / s

What is the ratio of the specific heat capacity of P to Q?

A 0.67 B 1.00 C 1.33 D 1.50

24 An ice pack is used to cool 0.20kg of water from 25°C to 0°C. The specific heat capacity of
water is 4.20kJ/kg°C and the specific latent heat of fusion of ice is 334kJ/kg.

How much energy is removed from the water?

A 21kJ B 66.8kJ C 87.8kJ D 1670J

www.KiasuExamPaper.com
302
10

25 Water waves are travelling from deep region to shallow region.

Which statement is true?

A The frequency in shallow region is lower.


B The speed of the waves in both regions is the same.
C The speed of the waves in shallow region is higher.
D The wavelength in shallow region is shorter.

26 A vibrator generates a travelling wave on a string. The diagram shows the shape of the string
at a certain instant.

Which diagram shows the shape of the string between P and Q after half a period?

A B

C D

27 A ray of light passes from glass to air. In glass, the speed of light is 1.8 × 108m/s.

What is the critical angle for light passing from glass to air?

A 18.0° B 30.0° C 36.9° D 41.8°

www.KiasuExamPaper.com
303
11

28 The diagram shows a single ray of light being directed at a plane mirror.

65°

25°
30°
bench

What is the angle of reflection?

A 25° B 30° C 55° D 65°

29 An object is placed in front of a converging lens. The lens forms a magnified image of the object
on a screen.

Which statement is correct?

A The distance between the object and the lens is greater than the focal length.
B The image formed is a virtual image.
C The lens is acting as a magnifying glass.
D The image is upright.

30 Which statement about speed of sound is correct?

A Sound travels fastest in a vacuum.


B Sound travels fastest in gases.
C Sound travels fastest in liquids.
D Sound travels fastest in solids.

www.KiasuExamPaper.com
304
12

31 A man stands between two tall buildings, P and Q. The diagram is not drawn to scale.

The man is 50 m from P and 350 m from Q.

50m 350m

He fires a gun and the first two echoes he hears are 2.0 second apart.

What is the speed of sound calculated from this observation?

A 150m/s B 200m/s C 300m/s D 400m/s

Please refer to the diagram below for question 32 and 33.

The diagram shows a simple circuit consisting of three identical bulbs and a variable resistor.
The e.m.f. of the battery is 3.0V.

3.0V

32 When the variable resistor is adjusted to 0Ω, the current through the battery is 0.50A.

What is the resistance of each bulb?

A 2.0Ω B 4.0Ω C 6.0Ω D 18.0Ω

www.KiasuExamPaper.com
305
13

33 The resistance of the variable resistor is increased.

What happens to the brightness of bulb P and Q?

bulb P bulb Q
A decrease decrease
B decrease increase
C increase remains the same
D remains the same increase

Please refer to the following information for question 34 and 35.

A lighting system consists of 10 bulbs operating at their normal brightness. 5 bulbs are rated
230V 100W and the remaining 5 bulbs are rated 230V 120W.

34 What is the suitable fuse rating for the lighting system?

A 1A B 3A C 5A D 10A

35 The lighting system is switched on continuously for 1 week.

What is the total cost of using the lighting system if one unit of electrical energy cost $0.22?

A $1.69 B $5.81 C $8.13 D $40.66

www.KiasuExamPaper.com
306
14

36 The diagram shows a plug that is wired wrongly. The appliance has an external metal casing.

What will happen when the plug is used?

A The appliance will not work because there is a short circuit and causes the fuse to melt.
B The appliance will not work because the external metal casing is at high voltage and
causes the fuse to melt.
C The appliance will continue to work without any danger to users.
D The appliance will continue to work but the external metal casing is at high voltage.

37 The diagram below shows a positive charge travelling towards an electric field.

direction of electric field

What is the direction the positive charge will move when it first enters the electric field?

A into the paper


B out of the paper
C upwards
D downwards

www.KiasuExamPaper.com
307
15

38 The diagram shows a brass rod supported on two copper rails that are connected to a battery.
The north pole of a magnet is placed beneath the rails.

copper rails
D A

B
C

What is the direction of the induced force acting on the brass rod?

39 The diagram shows a compass placed between two solenoids.

P Q

The e.m.f. of the battery connected to solenoid P is larger than that of the battery connected
to solenoid Q.

Which is the correct direction of the compass needle.

A B C D

40 The primary coil of an ideal transformer has 200 turns and is connected to a 20V alternating
voltage supply. The secondary coil has 3200 turns and is connected to a 120Ω resistor.

Which row gives the correct secondary voltage and primary current?

secondary voltage / V primary


current / A
A 16 0.11
B 16 0.13
C 320 2.67
D 320 42.7
End of Paper

www.KiasuExamPaper.com
308
1

NORTH VISTA SECONDARY SCHOOL

PRELIMINARY EXAMINATION 2019


80

NAME: ________________________________( ) CLASS: _________

SUBJECT: PHYSICS DATE: 4 SEP 2019

LEVEL / STREAM: SECONDARY 4 EXPRESS TIME: 1 HR 45 MIN

CODE : 6091/2

INSTRUCTIONS TO CANDIDATES

Write your full name, index number and class on all the work you hand in.
Write your answers in dark blue or black pen.
You may use a HB pencil for any diagrams or graphs.
Do not use staples, paper clips, highlighters, glue or correction fluid.

The use of an approved scientific calculator is expected, where appropriate.


You may lose marks if you do not show your working or if you do not use appropriate
units.

Section A
Answer all questions.

Section B
Answer all questions. Question 11 has a choice of parts to answer.

Candidates are reminded that all qualitative answers should include appropriate units.
Candidates are advised to show all their working in a clear and orderly manner, as more
marks are awarded for sound use of Physics than for correct answers.

At the end of the examination, fasten all your work securely together.
The number of marks is given in brackets [ ] at the end of each question or part question.

This question paper consists of 21 printed pages.


[Turn over
www.KiasuExamPaper.com
309
2

Section A

Answer all the questions in the spaces provided.

1 Fig. 1.1 shows a block of wood moving at a constant speed down a slope.

W
30°
Fig. 1.1

The forces acting on the block are the weight W of the block, the normal reaction force N
exerted by the slope and the friction F between the block and the slope.

F is 10.0N and N is 17.4N.

(a) In the space below, draw a labelled diagram to show the resultant of F and N.

Determine the size of the resultant force and the direction between the resultant force
and the horizontal ground.

resultant force = …………………………………………

direction = …………………………………..… [3]

www.KiasuExamPaper.com
310
3

(b) State the weight of the block of wood.

weight = …………………………………..… [1]

(c) The resultant force in (a) and W are not a Newton’s Third Law action-reaction pair.

Describe the other force that is part of the action-reaction pair with W and state which
body it acts on.

……………………………………………………………….…………………………………..

…………………………………………………………….……………………………………..

……………..…………………………………………………….………………………….. [2]

www.KiasuExamPaper.com
311
4

2 Fig. 2.1 shows a student standing with his right foot and right shoulder touching a wall.

42N

180cm

20cm
20cm
m10cm
Fig. 2.1 Fig. 2.2

(a) The centre of gravity of the student is shown in Fig. 2.1.

State what is meant by centre of gravity.

……………..…………………………………………………….………………………….. [1]

(b) The student bends his right knee and raises his right foot off the ground.

Fig. 2.2 shows a 42N force exerted by the wall on his right shoulder to keep him
balanced. His left foot acts as the pivot.

By taking moments, determine the weight of the student.

weight = …………………………………..… [2]

(c) The student now raises his left foot off the ground instead of his right foot.

Using ideas about stability, state and explain what will happen to him.

……………………………………………………………….…………………………………..

……………………………………………………………….…………………………………..

…………………………………………………………….……………………………………..

……………..…………………………………………………….………………………….. [3]

www.KiasuExamPaper.com
312
5

3 Fig. 3.1 shows a small crack appearing in a vase of water and a stream of water is pushed
out through the crack. The water hits the table where a puddle of water starts to form.

vase of water

60cm
crack

stream of water
water

Fig. 3.1

The gravitational field strength g is equal to 10N/kg.

(a) The density of water is 1050kg/m3 and the crack is 60cm below the surface of the
water.

(i) Calculate the pressure due to the water at the level of the crack.

pressure = …………………………………..… [2]

(ii) Explain why the atmospheric pressure does not affect the rate at which the
water is pushed out through the crack.

……………………………………………………………………..……………….. [1]

(b) As time passes, the point where the water hits the table moves away from P and
towards the vase.

Explain why this happens.

……………………………………………………………….…………………………………..

…………………………………………………………….……………………………………..

……………..…………………………………………………….………………………….. [2]

www.KiasuExamPaper.com
313
6

4 Fig. 4.1 shows a syringe that has a sealed end. Air is trapped in the syringe and the piston
is free to move up and down. The piston has negligible weight.

Fig. 4.1

Using the movement of particles,

(a) explain how the trapped air exerts pressure in the syringe;

……………………….……………………………….………………………………………….

…………………………………………………….……………………………………………..

………………………………………………….………………………………………………..

……………..…………………………………………….………………………………….. [2]

(b) explain why the piston moves down when the temperature of the trapped air decreases.

……………………….………….……………………………………………………………….

……………………………………….…………………………………………………………..

…………………………………………….……………………………………………………..

……………..…………………………….………………………………………………….. [2]

www.KiasuExamPaper.com
314
7

5 Fig. 5.1 shows a ray of light incident at mid-point O of the plane surface AB of a semi-circular
diamond block. The angle of incidence at O is 60°

diamond air

A O B
60°

Fig. 5.1
The refractive index of diamond is 2.4.

(a) (i) State what is meant by refractive index of 2.4.

…………………………………………………………………………….……………..

……………..……………………………………………………………..………….. [1]

(ii) Calculate the angle of refraction of this ray at O.

angle of refraction = …………………………………..… [2]

(b) Draw the path of this ray from O on Fig. 5.1 and continue its path until it has emerged
into the air. [2]

(c) Calculate the critical angle for the diamond-air boundary.

critical angle = …………………………………..… [1]

(d) The semi-circular diamond block can be used to demonstrate total internal reflection at
surface AB.

Describe how this can be done.

……………………….…………………………………………………………………………..

……………………….…………………………………………………………………………..

……………..…………………………………………………………………………….….. [2]

www.KiasuExamPaper.com
315
8

6 Fig. 6.1 shows a positively charged metal sphere, P hanging from a string. It is placed near
an uncharged metal sphere, Q, supported on an insulating stand. P is attracted to Q but
both spheres are not in contact.

positively charged + + uncharged


metal sphere + metal sphere
+ P + Q
+ +

Fig. 6.1

(a) Define the term direction of electric field.

…………………………………………………………………………………………………...

……………………………………………………………………………………………….. [1]

(b) On Fig. 6.1, draw the charge distribution in sphere Q and the electric field pattern in
the space between the two spheres. [2]

(c) Explain why sphere P is less positively charged on the left side.

…………………………………………………………………………………………………...

………………………………………………………………………………………….……. [1]

(d) Sphere Q is moved towards the left until it makes contact with sphere P.

Describe and explain what happens to sphere P.

…………………………………………………………………………………………………...

…………………………………………………………………………………………………...

………………………………………………………………………………………….……. [2]

(e) When sphere P is earthed, 20C of charges flow to the sphere in 25s.

Calculate the current flowing in the earth wire.

current = ……………………. [2]

www.KiasuExamPaper.com
316
9

7 Fig. 7.1 shows a magnet attached to a paper cone and placed near a coil of wire that is
connected to a d.c. source. The magnet can vibrate horizontally about its rest position.

Fig. 7.1

(a) When the switch is closed, the magnet would move momentarily before it comes to a
stop.

When the switch is opened, the magnet would move back to its original position.

State the direction the magnet would move when the switched is closed.

…………………………………………………………………………………………….…. [1]

(b) When the battery is replaced by an a.c. source, the alternating current in the coil will
cause the magnet and the paper cone to vibrate continuously. Sound will be heard if
the frequency of the vibration is within the audible frequency.

(i) State the range of audible frequency of a normal healthy human.

……………………………………………………………………………………….. [1]

(ii) Explain how sound is produced by the cone and transmitted to the surrounding.

……………………………………………………………………………………………

……………………………………………………………………………………………

……………………………………………………………………………………………

……………………………………………………………………………………………

……………………………………………………………………………………….. [3]

(iii) The magnet is now replaced by a soft iron bar.

State and explain whether the alternating current in the coil will cause the soft
iron bar and the paper cone to vibrate continuously.

……………………………………………………………………………………………

……………………………………………………………………………………………

……………………………………………………………………………………….. [2]

www.KiasuExamPaper.com
317
10

8 Fig. 8.1 shows a torch that does not use batteries.

Fig. 8.1

To use the torch, the handle is turned to rotate a magnet near a coil of wire. This will
illuminate the torch bulb.

(a) Explain why the torch bulb is illuminated when the magnet rotates.

…………………………………………………………………………………………………...

…………………………………………………………………………………………………...

…………………………………………………………………………………………………...

……………………………………………………………………………………………….. [2]

(b) Describe the changes in light emitted, if any, by the torch bulb when the rate of rotation
of the magnet is decreased slowly.

…………………………………………………………………………………………………...

…………………………………………………………………………………………………...

……………………………………………………………………………………………….. [2]

(c) When the handle is turned, a force is induced that acts against the rotation of the
magnet.

Explain why there is an induced force and how it acts against the rotation of the magnet.

…………………………………………………………………………………………………...

…………………………………………………………………………………………………...

…………………………………………………………………………………………………...

…………………………………………………………………………………………………...

……………………………………………………………………………………………….. [2]

www.KiasuExamPaper.com
318
11

BLANK PAGE

Please Turn To
Page 12 for
Section B

www.KiasuExamPaper.com
319
12

Section B

Answer all the questions in the spaces provided.


Answer only one of the two alternative questions in Question 11.

9 Fig. 9.1 shows a circuit consisting of a thermistor and a 2.00kΩ fixed resistor connected
in series to an a.c. source with a peak voltage of 230V. A cathode ray oscilloscope (c.r.o.)
is connected in parallel to the fixed resistor. The circuit can be used to measure the room
temperature by studying the display on the c.r.o.

230V
o ~ o

2.00kΩ

to Y-plates
of c.r.o.

Fig. 9.1

Fig. 9.2 shows the resistance of the thermistor and peak voltage output to the c.r.o. at
different temperature. The peak voltage of the a.c. source and the resistance of the fixed
resistor remain constant.

temperature of resistance of resistance of peak voltage of peak voltage


thermistor thermistor fixed resistor a.c. source output to c.r.o
/ °C / kΩ / kΩ /V /V
5.0 2.60 2.00 230 100
10.0 1.75 2.00 230 123
15.0 1.25 2.00 230
20.0 1.10 2.00 230 148
25.0 1.00 2.00 230 153

Fig. 9.2

www.KiasuExamPaper.com
320
13

(a) The resistance of the thermistor is dependant of its temperature.

(i) Define the term resistance.

……………………..………………………………………………………………..

…………………………………………..………………………………………. [1]

(ii) Describe how the resistance of the thermistor changes with increasing
temperature.

……………………..………………………………………………………………..

…………………………………………..………………………………………. [1]

(iii) Using Fig. 9.2, state and explain one limitation of the above circuit in
measuring room temperature.

……………...……………………………………………………………………….

…………………………………………………………………………………… [1]

(b) Calculate the peak voltage output to c.r.o when the temperature of the thermistor is
15.0°C.

peak voltage output = ......................... [2]

www.KiasuExamPaper.com
321
14

(c) Fig. 9.3 shows the display on the c.r.o. at 5°C. The time base setting is 5.0ms/div.

Fig. 9.3

(i) Calculate the Y-gain setting of the c.r.o.

Y-gain setting = ……………………. [1]

(ii) Calculate the period of the a.c. source.

period = ……………………. [1]

(iii) Describe the changes to the display, if any, on the c.r.o. if both the time base
setting and the Y-gain setting are doubled.

…………………………………………………………………………………………

………………………………………………………….………………………….. [2]

www.KiasuExamPaper.com
322
15

(d) The circuit in Fig. 9.1 is modified by replacing the fixed resistor with the thermistor.
The new circuit is shown in Fig. 9.4

230V
o ~ o

to Y-plates
of c.r.o.
Fig. 9.4

State and explain if the modified circuit in Fig. 9.4 can be used to measure room
temperature.

………………………………………………………………………………………………..

………………………………………………………………………………………………..

……………………………………………………………………………………………. [1]

www.KiasuExamPaper.com
323
16

10 A student uses a long rope to demonstrate a transverse wave. A ribbon is tied at a point
P on the rope.

Fig. 10.1 (full scale)

The student’s hand moves up and down 10 times every 4.0 seconds. Fig. 10.1 shows, to
full scale, a side-ways view of the rope at one instant.

(a) Determine the

(i) amplitude of the wave,

amplitude = ………………………. [1]

(ii) wavelength of the wave

wavelength = ………………………. [1]

(iii) Calculate the speed of the wave. State clearly the equation used.

speed = ………………………. [2]

(b) On Fig. 10.1, mark a point on the rope that has the same vertical speed as point P,
but which moves in the opposite direction to P. Label this point Q. [1]

(c) Using the same rope, the student produces a wave of longer wavelength than that
shown in Fig. 10.1.

State how the student does this.

……………………………………………………………………..…………………………

……………………………………………………………………………………………. [1]

www.KiasuExamPaper.com
324
17

(d) Electromagnetic waves are also transverse waves.

(i) State one property of electromagnetic waves that differentiate them from all
other waves.

……………………………………………………………………..………………. [1]

(ii) X-rays are part of the electromagnetic waves.

Hospital uses them to discover whether the bones are broken. However, X-
rays can cause ionisation of living cells and tissue.

Explain what is meant by ionisation and describe the effects on living cells and
tissue.

……………………………………………………………………..…………………..

……………………………………………………………………..…………………..

……………………………………………………………………..…………………..

………………………………………………………………..……………………. [2]

(iii) Fig. 10.2 shows the relationship between the energy of electromagnetic waves
and the wavelength of the waves.

energy

wavelength
Fig. 10.2

State the component of the electromagnetic waves with the lowest energy.

……………………………………………………………………..………………. [1]

www.KiasuExamPaper.com
325
18

11 EITHER

Fig. 11.1 shows a pot of water placed above a flame.

Fig. 11.1

(a) State the thermometer you would use to measure the temperature of the flame.

……………………………………………………………………………………………….. [1]

(b) The pot, with the lid, is made of metal and has a shiny smooth external surface.

Explain how the above features of the pot minimises the time to boil the water

…………………………………………………………………………………………………...

…………………………………………………………………………………………………...

…………………………………………………………………………………………………...

………………………………………………………………………………………………. [3]

(c) On Fig. 11.2, sketch the temperature-time graph of the water when its temperature is
increased from room temperature to boiling point. [1]

temperature

boiling point -

room temperature -
time

Fig. 11.2

www.KiasuExamPaper.com
326
19

(d) Noodles are placed in water at boiling point. In order to cook the noodles in a shorter
time, the size of the fire is increased.

Explain why the above suggestion does not shorten the cooking time of the noodles.

…………………………………………………………………………………………………...

……………………………………………………………………………………………….. [1]

(e) The specific latent heat of vaporisation of water is 2200kJ/kg.

(i) State what is meant by the above statement.

………………………………………………………………………………….………..

……………………..……………………………………………………………..….. [1]

(ii) Using common apparatus found in school laboratory and the value of the specific
latent heat vaporisation of water, describe a method to determine the
approximate value of the rate of thermal energy supplied by the fire. State clearly
how the result is obtained.

……………………………………………………………………………………………

……………………………………………………………………………………………

……………………………………………………………………………………………

……………………………………………………………………………………………

……………………………………………………………………………………………

……………………………………………………………………………………………

……………………………………………………………………………………………

…………………………………………………………………………………………...

…………….…………………………………………………………………………. [3]

www.KiasuExamPaper.com
327
20

OR

A children’s ride consists of a steel cable that runs between two posts of different heights,
as shown in Fig. 11.3.

Fig. 11.3

A child starts and finishes the ride at rest. His horizontal motion can be taken as
x an initial decreasing acceleration for 5.0s, followed by
x a constant velocity of 1.6m/s for a further 3.0s and
x a final uniform deceleration that lasts for 1.0s.

(a) On Fig. 11.4, draw a velocity-time graph of the horizontal motion.

velocity
m/s

2 4 6 8 10
time/s
Fig. 11.4 [2]

(b) State how displacement is found from a velocity-time graph.

………………………………………………………………………………………………...…

…………………………………………………………………………………..…………… [1]

www.KiasuExamPaper.com
328
21

(c) (i) The child has a mass of 25kg and falls through a height of 2.0m during the ride.

The gravitational field strength g is 10N/kg.

Calculate the decrease in gravitational potential energy of the child.

decrease in potential energy = …………………….. [2]

(ii) Suggest why the loss in gravitational potential energy and the increase in kinetic
energy are different and explain how the law of conservation of energy applies in
this case.

……………………………………………………………………………………….……

……………………………………………………………………………………….……

………………………………………………………………………………………….[2]

(d) A group of students make measurements to show that the child’s velocity is constant
during the middle section of the ride.

Suggest what measurements are made and how they show that the velocity is
constant.

…………………………………………………………………………………...………………

…………………………………………………………………………………...………………

…………………………………………………………………………………...………………

…………………………………………………………………………………...………………

…………………………………………………………………………………...………………

……………………………………………………………………………………………….. [3]

END OF PAPER

www.KiasuExamPaper.com
329
www.KiasuExamPaper.com
330
16

Answer to P1

1 C Refer to textbook page 6 and7.


2 D Pascal is unit for Pressure
3 C Diameter = measurement – zero error
= 10.13 – (-0.06) = 10.19cm
4 B Period is the time taken for one complete oscillation (A to B and back to A again)
5 C Terminal velocity only happens when all forces are balanced (net force is zero), hence
zero acceleration and constant velocity.
6 C Higher acceleration means higher gradient (steeper vel-time graph).
7 D Constant speed = zero acceleration = zero resultant force
8 A Friction opposes the relative motion between 2 bodies in contact. The foot is pushing
backward in order for the man to move forward. Hence friction pushes the foot forward
(opposite direction)
9 D Things to note:
Acceleration is the rate of change of velocity.
Change in velocity can be either direction or numerical rical value
valuue (speed).
lu (s
speed)
When there is an acceleration, a resultant force m must
usst be
ust e present.
present. (not change in
resultant force).
10 A When the container decelerates, the he water has bigger biig
gg
g
ger
err mass,
e mass,s,, bigger
s big
gge
ger
er inertia
inertia as compare
with the bubble of air. Hence the water wa ateer willlll continue
con nue forward
ontiinu ffo
orward d and
and pushes the air bubble
an
backward.
11 D Both cubes are made de off tthe
he ssame
he am
a me mate
material.
te
eriial
a.
12 D Least stability y = highes
highest
e tC
es C.G
.G ((top
topph heavier)
e vier
ea er) and smalle
smallest
estt bbase
aseea area.
reaa.
13 D P1 = P2
150/100/10 = (1500 00
0 0x1 10)/A
0)/A
0)
2
A= 1000cm
100
0 00c
cm
14 B When
Wh hen
e atm
atmospheric
mosph
oss herric pressure
preess
ssu
ssururre drops,
drrop
d ops s,, m
mercury
e cury
er ry
y ccolumn
ollu
o
olum
um
mn QR decreases, PQ increases. R
rrises
ri ses as
se a more
mor
ore mercury
or
ore mercury
me y flows
flow
fl
low
ows outouut of
of mercury
me errcury
e curyy column,
cu col
o umn hence PR decreases. Height PS is
ffixed.
fixeed.
15 A Useful
U
Usef
Us sef
ef work done
eful do
don ne
e=g gain
aiin in m
a mghgh = 2
gh 2000
000 xx0.8
Or useful
use
sefuul work
wo
w k done
orrk do ne = resultant
on reesu a t force
sullttan forc x distance in direction of force = (1300-500)(2.0)
16 A Ek = ½ Mv Mv2
New w Ek = ½ (M )((2v)2 = 2 (½Mv2) = 2Ek
(M/2)(2v)
M/2/2)(2v
17 A collisions
ionss b
byy a
ai
air
ir particles
part
Key word “by” as air particles are moving in continuous and random motion at high
ord is “b
speed
18 C The water particles move further apart from each other when heated.
This will increase the volume of water and decrease the density.

B is wrong because particles do not expand when heated.


19 D The gas particles are all moving in a continuous and random motion. The probability of
the particles hitting at any point in the container with the same average speed (and force)
is the same.

B and C are wrong unless the options contain the word in bold: “average number of
collisions with the internal walls of the container per unit time” and “move at the same
average speed”.

www.KiasuExamPaper.com
331
17

20 B increases decreases
21 A by conduction only
C is wrong because the question did not ask for thermal energy transferred within the
liquid.
22 C 1.30Ω
40°C = [R - 0.50Ω]/[ 2.50Ω - 0.50Ω] × 100°C
R = 1.30Ω
23 B 1.00
Specific heat capacity is the same for objects from the same material
24 A 21kJ
energy removed = 0.20kg × 25°C × 4.20kJ/kg°C = 21kJ
25 D Speed of water waves decreases in shallow water as wavelength decreases.
Frequency is constant.
26 C Shift the wave to the right by half a waveform

27 C n = c/v = 3.0x108/ 1.8x108 = 1.67


c= sin-1 (1/1.667) = 36.9°
28 D Angle of reflection is the same as angle of incidence, ncidence,, i.e.i.e
.e. the
the angle
ang
an gle between
betw Normal
and incident ray.
29 A Key words from question “Magnified ed image ….. on on a screen”
sccrreen” [Real,
[Rea
[R eal,l, m
magnified
agnif image]
Case 4 from table 12.5 in textbook xtbook pg pg 242
2 2 where
24 wh
w ere object
her
ere obje
ob c is placed
ject
ject plac
placed
ac ed between f and 2f.
30 D Sound need medium to o travell a
an
and
nd is ffastest
aste
ast stt in
in solid
solilid wh
so w
where
her
ere
e pa
part
particles
rtic
rt icle
ic les are very closely
le c
packed.
31 C t diff = [2 x distance fro from
r m Q to
ro to man/
man
an
an
n// speed]
spee
eed]
ee d - [2 x distance
distatanc
ta nce from
nc om m P to to man/
ma speed]
spe
2.0 = [2x350
2x350 - 2x50]
2x50
2x 5 ] / speed
spee
sp eed
Speed
peed = 30 3
300m/s
0mm/ss
32 B 4.0Ω

0 Ω
5R = 3.0V
1.5R
5R 3.00V / 0.50A
0 50
0. 5 A = 6. 6.0Ω
6.0Ω

R=4 4.0Ω
4.
.0Ω
0
33 B Decrease
D e rease
ec e Increase
Incr
because
b
bee ause p.
eca p.d.
p d.. a
d across
oss P
crros
c because
b
be cau p.d. across Q
decreases
d
deec
crrea
eas se
es increase
34 C 5AA
current
rentt = 1100W
1100
00W
00 W / 230V
230V = 4.78A
35 D $40.6666
cost = (1.10kW × 7 × 24hrs) × $0.22 = $40.66
36 D The appliance
lia will continue to work but the external metal casing is at high voltage.

Both live and earth wire are at high voltage but are not connected to each other.

The fuse is in the live wire. The fuse and the live wire are not connected to the earth
wire unless the live wire touches the metal casing.
So fuse will not melt because current is flowing through the appliance as normal.
37 D downwards
Positive charge at the top and negative charge at the bottom.
This does not require Fleming’s Left hand Rule because the field is not a magnetic
field.
38 B Using Fleming’s Left hand Rule

www.KiasuExamPaper.com
332
18

39 B

Direction of the magnetic field of the two coils is the same. Using Right Hand Grip,
the magnetic field is towards the left.
40 D 320V 42.7A
secondary voltage power input
= [3200 / 200] × 20V = power input
= 320V = (320V)2 / 120Ω = 853.3W
primary current
= 853.3W / 20V = 42.7A

www.KiasuExamPaper.com
333
22

Solutions for Section A


1 (a)

(b) 20.0N (downward)


Allow ecf from (a)
(c) Force by the block of wood on Earth.
Acting in opposite direction (or upwards) pwards) and equal eqqua
ual in
in magnitude
magnitu tude
tude as
as W
2 (a) Centre of gravity is a point through rough whichwh
w
whic
hicch the
th
he whole
wh
w holle weight
we
w eig
i ht seems
see
eems
ms to act.
(b) Taking moments about the he leftt ffofoot,
oot,
ACW = CW
W × 10 = 42 × 180
W = 756N
(c) The student tudent willwiill be
be unstable,
u stab
un ble, causing
ca auussing him
him to
hi to topple/
topple/ lose
to se hiss balance.
bal
alan
a ce
The he line e ofof action
on of
on of weight
weig
we ight
ht fa
ht ffalls
alllls outside
o tsid
ou id
de the
tth
he baseba
b ase (right
(righht foot),
foot
foot)),
ot
resulting
resu ultltin
iin
ng inin a clockwise
cloockwi
ckwi
ck wise
wise
se moment
momo ent about abou
ab out his
hiis right
h rriight foot.
foot
ot.
ot
3 (a) ((i))
(i P = hρhρg
= (60/100)
(60
(6 0/1
0/ 1000) (1050)(10)
(10505
050)
0)(1
(10)
= 6300Pa
63300
0 Pa a
(ii)
(i
ii) Atmospheric
At tmo
m sp phe
heriric pressure
prres
p esssu
ure act acts
cts
ct s at
a both
both sides of the crack, i.e. at internal side and
tthe
th
he external
exxtte
e errn
naall o
off the crack.
crrac
ack. k
Accept
Ac
cce
c p
ptt th
tthere
he
erre is at
atmospheric
tmo
mosp
sphe
h ric pressure
p at the top of liquid as well outside the crack.
(b) The e height
he
eig
ight of
of water
wate er above
abov
ab ove the crack decreases.
Since e P = hρhρg,
ρg, tthe pressure due to the liquid at the crack decreases.
he pressu
4 (a) (i) Air
Aiir particles
part
partic
icles a
are travelling at high speed in a continuous and random motion.
They collide with the internal walls of the syringe. The average force exerted
e colli
per
er unit
u area is the air pressure.
(ii) KE (and speed) of the air particles decreases. They collide less frequently and
with smaller force with the internal wall.
Pressure of trapped air is lower than atmospheric pressure and a resultant force
acts downwards on the piston.
5 (a) (i) The speed of light in vacuum is 2.4 times faster than the speed of light in
diamond.
(ii) 2.4 = sin 60/sin r
r = 21.2°
(b)

www.KiasuExamPaper.com
334
23

(c) c = sin-1 (1/n)


= sin-1 (1/2.4)
= 24.6°
(d) Ensure the ray of light is incident at the curved surface so that light can enter the
diamond block and travel from diamond (more optically denser) towards the boundary
with air (optically less dense).

Make sure the angle of incidence in diamond more than critical angle of 24.6°.
6

positive charged +
+ uncharged
metal sphere - +-
+ m
metal sphere
+ P + - Q +-
+ + - +-

(a)
The direction of the electric force ce acting
actitiin
ng
g on a small
sma
sm alll positive
pos
po
pos
siiti
tive
i charge.
cha
hargge.
(b)
See above.
Correct charge distribution bution n
Correct field lines s
(c) The induced d nega
negative
g tive cchargeha
hargge on on tthe
he e lleft
efft
e ft si
sside
de of Q re repel
epe
pell the elec
electrons
ectr
tron
ons
s in P tto the left
side.
(d) The e elec
electrons
ctrtron
o s inin Q will
wilill be e attracted
attttra
tract
ra
act
cted
e byy th the
he p po
positive
osi
siti
t ve charged
cha
haarg
rged
ed P.
P. They
The will move into P
until b both
boothh spheres
sph
p erres sa are
ar
re eq e
equally
qua alllly positive
p siti
po tiive
ve ccharged.
harg
ha rged
ed.
P swings
sw win
ings away
awa
w y from
f om
fr om Q because
beeccau
ause se like
lik
ike charges
cha
ch arrges repel.
repe
pel.l.
pe
(e) cu
current
c rrrren
e t=Q/t
=2 20C
20 0C / 25s25s [1]
25 1] = 0.80A
[1 0.80 0A
7 (a) Itt will
wilill move
moove ve tto
o th
the
he lle
left.
efftt.
(b) (i) 20Hz
20 0Hzz toto 20
2
20kHz.
0kkHHz z..
(ii)
i) The co
The
Th cone
on ne
e vibrates
viibraatetess and
and collide
collid with the neighbouring air particles to vibrate. This
disturbance
di
d iist
s ur
st b ncce (or
urba
urba (or vibration)
vib
vibratio is passed on to other air particles.
The
Th
T he vibration
v br
vi brat
atio
at ion
io n of the particles is parallel to the propagation of the sound.
The e so
sou
sound
und is ttransmitted in a series of compressions and rarefactions.
(iii) The soft iiron bar will always be attracted to the coil regardless of the direction of
the em magnetic field of the current.
The soft iron bar will not vibrate (and no sound is produced).
8 (a) There is a change in the magnetic field lines linkage with the coil.
This induces an emf and thus a current in the coil.
(b) Light is less bright and blink less often (or frequency of blinking decreases or light is
emitted a shorter time).
(c) The interaction of the magnetic field of the induced current and the magnet will induce
the force (or the induced current in the coil will set up a magnetic field that will exert a
force on the magnet).
The direction of the induced force will oppose rotation of the magnet in accordance to
Lenz’s Law (a like pole will be induced when the magnet is moving towards the coil
and an unlike pole will be induced when the magnet is moving away from it).
Solutions for Section B
9 (a) (i) The ratio of the p.d. across it to the current flowing through it.
(ii) Resistance decreases at a decreasing rate.

www.KiasuExamPaper.com
335
24

(iii) It cannot measure high temperature because the decrease in resistance is


insignificant.
(b) peak voltage output = [2.00kΩ / (2.00 + 1.25)kΩ] × 230V
= 142V
(c) (i) Y-gain setting = 100V / 4div = 25V/div
(ii) period = 5.0ms/div × 5div = 25.0ms
(iii) time base setting: twice the no. of waveforms or 4 waves are seen
Accept: halved the period/number of divisions needed per waves
Y-gain setting: amplitude is halved or amplitude is 2 div
(d) No. The output voltage to c.r.o. is the same as the e.m.f. regardless of the resistance
of thermistor.
10 (a) (i) amplitude = height from crest to tough / 2 = 2.0cm /2 = 1.0cm
Accept 0.9 to 1.1 cm
(ii) wavelength = distance between crest to crest = 6.0cm
Accept 5.8 to 6.2 cm
(iii) v = f λ
= (10/4) (6.0)
= 15 cm/s
Accept 0.15m/s
(b) P ½λ

(c) The student move moves


oves s hhi
his
is hand
hand
nd
nd up p an
a
andd do d
down
own
w slower/ de
decrease
dec
crea
ase the
he ffrequency/
requ
requency/ lower
speed/ less times
ss timeme es per second
seco
se cond
nd
(d) (i) Accept one: onee:
Can
C
Ca travel
an trav el in
vel n vacuum
vac
va cuuum
um oror
travel
travvell at 3.0
3.00×1 1008 mm/s
/s in vacuum.
vac
acuu
uum.
m.
((ii)
(ii) Ionisation
Ionisasati
sa tion
tion
o n is
s the re
removal
emo
movavaall of
of electrons
ele
leccttrro
ons from
fro
rom
m atoms/molecules
ato
toms/m to form ions.
Causes
C
Ca useses d
es damage
am
amag ge to o living
livinng cells
cellllls and
and abnormal
abno cell divisions, e.g. cancer,
deformed
d
deeffo
orm
rme ed foetus.
d foeetu
tus
s..
(iii)
(iii
ii)
ii
i) Radi
Radiowave
d ow
di wavave

11 (a) Thermocouple
hermo
mocoup
moco ple
co e or
or data
a logger
logg
lo gger with temperature sensor.
E
(b) Metall (is a go
goodod condu
conductor of heat and) transfers thermal energy from the fire to the
water quic
quickly.
ckl
kly.
y
Shiny and smooth surface is a bad emitter of thermal energy.
nd sm
Thermal
mal energy is emitte
emitted to the surrounding at a slow rate.
(c) temperature

boiling point -

room temperature -
time

www.KiasuExamPaper.com
336
25

(d) The bigger fire does not increase the temperature of the boiling water. Noodles is still
cooked at the same temperature.
(e) (i) 2200kJ of thermal energy is needed to vaporise 1kg of water at boiling point.
(ii) When water is boiling, use a cold flat surface or the lid of the pot to condense
the steam.
The mass of steam, m, condensed is measured using a weighing machine.
The time, t, take to condense the steam is measured using a stop watch.

Approx. rate of thermal energy supplied = m × 2200kJ/kg / t

Also accept measuring the different in mass of the pot and boiling water using
weighing machine after a specific time.
11 (a)
O

(b) Area under velocity-time


ocity-time me g
me graph
ra aph
ph
(c) (i) Decrease
crease e in
in GPE
GP PE = mghmgh
mg
= (2((25)5) ((10)
100) (2.0))
= 50 5
500J
00 0J
J
(ii) Work
Wo orrk
k ddone
onne against
agaiinsnst the the friction
th ffrric
icttiion
ion
n present
pre
r sent between
bet
etwe
ween the moving parts in the ride/
we
work donedon
one
ne aga
against
ainstt aai
air ir rre
resistance/
esiis stta
an nce e/ co
conv
converted
nver
nverte
er ted to thermal
te t and sound energy
Energy
En
E ner
erggyy cannot
canno ot bebe created
crre eat
a ed or destroyed
dest
de stro
st roy
royed bu
but converted from one form to another
and d ttotal
otall energy
ene
en
ener gy is
errgy is the
he same,
tth sa
ame me, i.e. e the difference in loss of GPE and gain in KE is
the
thhe amount
amou
am unntt ooff thermal
ther
erma
er mall and
ma and sound s energy or work done against friction/air
rresistance.
re
resi
essiistan
an
a n
nce
ce.
ce
(d) Measurement
asurrem
ement of least lea
e st ttwo distances/displacement and corresponding times
mentioned.
ione ed.
d
Description
ption n of how
ho the actual measurement is made
- make marking on the ground every second and measure the
distances/displacements
- note video position every second and use a scale to find the
distance/displacement
- make mark on ground every metre and measure the time as the girl passes
Description of how constant speed/velocity using measurement is proven
- Same distance/displacement travelled between each position for the same time
interval
- Same time internal for same distance/displacement
- Constant gradient for distance-time graph/ displacement-time graph plotted.

www.KiasuExamPaper.com
337
www.KiasuExamPaper.com
338
SINGAPORE CHINESE GIRLS’ SCHOOL
PRELIMINARY EXAMINATION 2019
SECONDARY FOUR

CANDIDATE NAME

REGISTER
CLASS 4
NUMBER
CENTRE NUMBER INDEX NUMBER

PHYSICS 6091/1
Wednesday 4 September 2019 1 hour
Additional Materials: Multiple Choice Answer Sheet

READ THESE INSTRUCTIONS FIRST

Write in soft pencil.


Do not use staples, paper clips, glue or correction fluid.
Write your name, class and index number on the Question Paper and Answer Sheet in the
spaces provided.

There are forty questions in this paper. Answer all questions. For each question, there are four
possible answers, A, B, C, D.

Choose the one you consider correct and record your choice in soft pencil on the separate
Answer Sheet.

Read the instructions on the Answer Sheet very carefully.

Each correct answer will score one mark. A mark will not be deducted for a wrong answer.
Any rough working should be done in this booklet.
The use of an approved scientific calculator is expected, where appropriate.
Take g = 10 ms-2 or 10 Nkg-1 unless specified otherwise.

This question paper consists of 22 pages

www.KiasuExamPaper.com
339
2

1 A micrometer screw gauge is used to measure the diameter of a copper wire.

The reading with the wire in position is shown in diagram 1. The wire is removed and the
jaws of the micrometer are closed. The new reading is shown in diagram 2.

What is the diameter of the wire?

A 1.90 mm

B 2.45 mm

C 2.59 mm

D 2.73 mm

2 Two cylinders P and Q are made of copper.

P Q

The height of P is twice the height of Q. The diameter of P is half the diameter of Q.

Which statement is correct?

A The density of cylinder P is four times that of cylinder Q.


B The density of cylinder P is twice that of cylinder Q.
C The density of cylinder P is equal to that of cylinder Q.
D The density of cylinder P is half that of cylinder Q.

www.KiasuExamPaper.com
340
3

3 The diagram shows two objects on a beam balance.

pivot

The beam balance is in equilibrium.

Which quantities may be different?

A The masses of the two objects


B The moments about the pivot of the two objects
C The volumes of the two objects
D The weights of the two objects

4 A kite is in equilibrium at the end of a string, as shown.

The kite has three forces acting on it: its weight W, the tension T in the string, and the
force F from the wind.

Which vector diagram represents the forces acting on the kite?

www.KiasuExamPaper.com
341
4

5 A stone of mass m is dropped from a tall building. There is significant air resistance. The
acceleration of free fall is g.

When the stone is falling at a constant (terminal) velocity, which information is correct?

magnitude of magnitude of the magnitude of the


the acceleration of force of gravity force of air resistance
the stone on the stone on the stone

A g zero mg

B zero mg mg

C zero zero mg

D zero mg zero

6 The velocity-time graph for an object is shown.

How can the total displacement of the object be determined?

A area 1 – area 2

B (area 1 + area 2 ) y 2

C area 1 + area 2

D area 2 – area 1

www.KiasuExamPaper.com
342
5

7 A ball is released from rest above a horizontal surface. It strikes the surface and bounces
several times.
The velocity-time graph for the first two bounces is shown.

What is the maximum height of the ball after the first bounce?

A 0.20 m B 0.25 m C 0.45 m D 0.65 m

8 What is not the definition of power?

A force x displacement
B force x velocity
C voltage x current
D work done y time

www.KiasuExamPaper.com
343
6

9 A bus takes 25 s to reach a constant speed while travelling in a straight line. A graph of
speed v against time t is shown.

Which graph shows the variation of the resultant force F on the bus with t?

10 A stone is released at a great height in air and falls due to gravity. Each of the three
graphs below represents the variation of one of the three variables p, q and r with time.

Which row correctly identifies the three variables p, q and r?

p q r

A acceleration displacement velocity


B displacement velocity acceleration
C velocity acceleration displacement
D velocity displacement acceleration

www.KiasuExamPaper.com
344
7

11 An engine pulls a truck at constant speed on a level track.

engine
link
truck

track

The link between the engine and the truck breaks. The driving force on the engine
remains constant.

What effect does this have on the engine and on the truck?

engine truck
A speed stays constant slows down
B speeds up slows down
C speed stays constant stops immediately
D speeds up stops immediately

12 The diagram shows a uniform beam PQ. The length of the beam is 3.0 m and its weight
is 50 N. The beam is supported on a pivot 1.0 m from end P. A load of weight W is hung
from end P and the beam is in equilibrium.

What is the value of W?

A 25 N B 50 N C 75 N D 100 N

www.KiasuExamPaper.com
345
8

13 A uniform rectangular board is supported by a frictionless pivot at its centre point P.

Two forces act in the plane of the board. Force F acts at corner Q and a force 2.5 F acts
at corner R. The perpendicular distance between the line of action of the force F and the
point P is 20 cm. The board is in equilibrium.

What is the area of the board?

A 160 cm2 B 320 cm2 C 640 cm2 D 1600 cm2

14 A car of mass 500 kg is at rest at point X on a slope, as shown.

The car’s brakes are released and the car rolls down the slope with its engine switched
off. At point Y the car has moved through a vertical height of 30 m and has a speed of
11 m s–1.

What is the energy dissipated by frictional forces when the car moves from X to Y?

A 3.0 × 104 J B 1.2 × 105 J C 1.5 × 105 J D 1.8 × 105 J

www.KiasuExamPaper.com
346
9

15 In which situation is there no work done?

A A man carrying two luggage bags and walking up a slope

B A ball is dropped and falls to the ground

C A box moves at constant speed across a smooth horizontal surface

D A crane lifting a steel beam at constant speed

16 A rocket is fired vertically upwards.

As it accelerates upwards after leaving the launch pad, which forms of energy are
changing?

A Chemical energy, gravitational potential energy and kinetic energy

B Chemical energy and gravitational potential energy only

C Chemical energy and kinetic energy only

D Gravitational potential energy and kinetic energy only

17 A crane lifts a weight of 600 N through a vertical height of 30 m in 25 s. The efficiency of


the crane is 40%.

What is the total power input of the crane?

A 0.29 kW

B 0.72 kW

C 1.8 kW

D 1800 kW

www.KiasuExamPaper.com
347
10

18 A vertical tube, closed at one end, is immersed in water. A column of air is trapped inside
the tube.

The density of water is 1000 kgm-3.


What is the difference between the pressure of the air in the tube and the atmospheric
pressure?

A 2000 Pa B 3000 Pa C 5000 Pa D 8000 Pa

19 A U-tube closed at one end contains mercury. Air at a pressure of 5.0 × 104 Pa is
trapped at the closed end. The other end is open to the atmosphere and is fitted with a
piston of mass 5.0 kg and cross-sectional area 5.0 × 10–4 m2.

The density of mercury is 13 600 kg m–3 and atmospheric pressure is 1.01 × 105 Pa.

trapped air at
pressure 5.0
× 104 Pa

area
5.0 Ø 10 m2
–4 h

piston of mass
5.0 kg

mercury

What is the height h of the mercury column?


A 39 cm B 46 cm C 76 cm D 111 cm

www.KiasuExamPaper.com
348
11

20 Which lens does not show rays of light passing through a converging lens?

Lens Q Lens R
Lens P

A Lens Q only

B Lens P and Q only

C Lens Q and Lens R only

D Lens P and Lens R only

21 Vertical beams of light are incident on the horizontal faces of three plastic prisms, X, Y
and Z. The refractive index of plastic is 1.8.

In which prism(s) will total internal reflection occur at the surface PQ?

A X but not Y and Z


B X and Y but not Z
C Y and Z but not X
D X, Y and Z

www.KiasuExamPaper.com
349
12

22 Containers A and B are filled with equal amounts of hot water at the same
temperature. The temperature of the water in the containers are measured with a
thermometer some time later. It is observed that container A has a much lower
temperature than container B.

What are the possible reasons?

(i) Container A is painted black and container B is painted white


(ii) Container A has a lid and container B is not covered
(iii) Container A is made of aluminium and container B is made of plastic

A (i) and (ii) only


B (i) and (iii) only
C (ii) and (iii) only
D (i), (ii) and (iii)

23 A thin tube contains a thread of mercury which traps air at the end of the tube. The
other end of the tube is open to the atmosphere.

When the tube is turned upside down,

A the volume of the trapped air increases because the pressure in the trapped air is
reduced.
B the volume of the trapped air increases because the atmosphere pushes less when it
acts upwards on the mercury.
C the volume of the trapped air decreases because the pressure in the trapped air is
reduced.
D the volume of the trapped air decreases because gravitational force acting on the
mercury increases when the tube is turned upside down.

www.KiasuExamPaper.com
350
13

24 Equal masses of three liquids X, Y and Z are heated from room temperature. Energy is
supplied by heating at the same rate to each liquid.
The graph shows how the temperature of each liquid varies with time after heating
starts.

Z
temperature

0 time

What can be deduced from the graph?

A X has the highest melting point.

B X gains the most internal energy.

C Y has the largest specific heat capacity.

D Z has the smallest specific latent heat of vaporisation.

25 Using an electric kettle, 200 g of water at 100 °C is converted into steam at 100 °C in
300 seconds. The specific latent heat of steam is 2250 J/g.

What is the average electrical power used?

A 1.5 W
B 1500 W
C 3380 W
D 135 MW

www.KiasuExamPaper.com
351
14

26 A new liquid is tested to decide whether it is suitable for use in a liquid-in-glass


thermometer. It is found that the liquid does not expand uniformly with temperature.

What will be effect of this on the scale of the thermometer?

A It will have a short range.


B The markings will be too far apart.
C The markings will be too close together.
D The markings will be spaced unevenly.

27 An astronaut wishes to communicate with his fellow astronauts inside the space
shuttle some distance away.

Which two waves, in the correct nature and sequence, are being used during the
communication?

A transverse → longitudinal

B longitudinal→ transverse

C transverse → longitudinal →transverse

D longitudinal → transverse → longitudinal

www.KiasuExamPaper.com
352
15

28 Graph 1 shows how the displacement of one particular point of a wave varies with
time.
Graph 2 shows how the displacement of the same wave varies with distance along the
wave at one particular time.

Which expression gives the speed of the wave?

ࢄ ࢄ૛ ࢄ െࢄ૚ ࢄ૜ ିࢄ૛
A  ૚ B C  ૛ D
࢚૚ ࢚૛ ି࢚૚ ࢚૛ ࢚૛ ି࢚૚

29 A guitar player struck a note on a guitar string. The same string is then struck harder.

Which of the following correctly compares the speed and wavelength of the second
note with the first note?

Speed Wavelength
A same same

B same different

C different same

D different different

www.KiasuExamPaper.com
353
16

30 In normal light, the resistance of a light-dependent resistor (LDR) is R. It is connected


in the circuit with two resistors, each of resistance R. The currents in the two resistors
are ‫ܫ‬ଵ and‫ܫ‬ଶ as shown.

How do the currents change when the circuit is moved to a brighter place?

‫ܫ‬ଵ ‫ܫ‬ଶ

A increase increase

B increase decrease

C decrease decrease

D decrease increase

www.KiasuExamPaper.com
354
17

31 The graph shows the current-voltage (I-V) characteristics of two resistors R and X.

The resistors R and X are connected in series with a cell. The current in the circuit is
0.3A.

The resistors R and X are then connected in parallel with the same cell.

What is the e.m.f. of the cell and the current in the cell when the resistors are
connected in parallel?

e.m.f. / V current / A

A 1.0 0.3

B 1.5 0.7

C 2.5 0.5

D 2.5 1.0

www.KiasuExamPaper.com
355
18

32 Which is a consequence of connecting several electrical appliances to the same power


socket?

A Current drawn by each appliance is increased.


B Total resistance of all appliances is increased.
C Voltage drawn by each appliance is decreased.
D Total energy consumption is increased.

33 A current of 40 mA passes through a slice of semi-conducting material of dimensions


as shown.

The slice dissipates 400 mW of heat energy.


What is the resistivity of the semiconductor under these conditions?

A 0.25 Ωm
B 0.36 Ωm
C 56 Ωm
D 380 Ωm

34 Three conductors are placed close to each other. Conductor X is negatively-charged.


Both conductors Y and Z are neutral.
X Y Z

What will be the charge in conductor Z after it is being earthed momentarily?

A neutral B positive C negative D no charge

www.KiasuExamPaper.com
356
19

35 In which circuit is the voltmeter reading 7.2 V?

A B

3.0 Ƕ 4.0 Ƕ

12 V 12 V

5.0 Ƕ V 6.0 Ƕ V

C D

2.0 Ƕ 2.0 Ƕ

12 V 12 V

4.0 Ƕ V 6.0 Ƕ V

www.KiasuExamPaper.com
357
20

36 The diagram shows a simple d.c. motor.

Q
P

Which combination(s) will achieve the direction of rotation shown in the diagram?

Polarity Direction of current

1 X is S-pole, Y is N-pole P is +, Q is –
2 X is N-pole, Y is S-pole P is -, Q is +
3 X is N-pole, Y is S-pole P is +, Q is –

A 2 only

B 1 and 2 only

C 2 and 3 only

D 1 and 3 only

www.KiasuExamPaper.com
358
21

37 Copper rods P and Q are placed on top of rigid bare wires as shown.

Which observation is correct when the power supply is changed to a low frequency
alternating current ?

A P and Q attract each other.

B P and Q repel each other.

C P and Q repel then attract each other.

D P and Q both roll to the right and then to the left, keeping the same distance
apart.

38 The graph below shows how the e.m.f. of an A.C. generator varies with time.
e.m.f./V

time/s

The diagrams below show the front view of the coil of an A.C. generator. The coil is
being rotated about an axis through O in a uniform magnetic field. Which of them
shows the position of the coil when the value of the induced emf is at M?

A B
S O N S N
O

C D

S O N S N
O

www.KiasuExamPaper.com
359
22

39 The diagram shows an ideal transformer. An a.c. supply of 100 V is supplied to the
primary coil. A current of 0.5 A flows through it.

0.5 A current

100 V 1000 250 load


turns turns

What is the potential difference and current flowing through the load?

potential difference / V current / A


A 25 2
B 25 4
C 50 2
D 50 4

40 A teacher moves a magnet into and out of a coil of wire, as shown, in order to
demonstrate electromagnetic induction.

solenoid

magnet

A
sensitive ammeter

Which statement is correct?

A As the magnet is moved into the coil, the right-hand end of the coil becomes a S-pole.
B As the magnet is taken out of the coil, the right-hand end of the coil becomes a N-
pole.
C Increasing the speed at which the magnet enters the coil increases the induced
voltage.
D Increasing the speed at which the magnet leaves the coil decreases the induced
voltage.

END OF PAPER

www.KiasuExamPaper.com
360
1

SINGAPORE CHINESE GIRLS’ SCHOOL


PRELIMNARY EXAMINATION 2019
SECONDARY FOUR

CANDIDATE NAME

REGISTER
CLASS 4
NUMBER
CENTRE NUMBER INDEX NUMBER

PHYSICS 6091/2

Monday 2 September 2019 1 hour 45 mins


Candidates answer on the Question Paper.
No Additional Materials are required.

READ THESE INSTRUCTIONS FIRST

Section A
Answer all questions.
Section B

Answer all questions. Question 11 has a choice of parts to answer.

Candidates are reminded that all quantitative answers should include appropriate units.
The use of an approved scientific calculator is expected, where appropriate.
Candidates are advised to show all their working in a clear and orderly manner, as more marks are
awarded for sound use of Physics than for correct answers.

The number of marks is given in brackets [ ] at the end of each question or part question.

Take g = 10 ms-2 or 10 Nkg-1 unless stated otherwise.

For Examiner’s Use

Section A 50

Section B 30

Total 80

This question paper consists of 27 printed pages and 1 blank page.

www.KiasuExamPaper.com
361
2

SECTION A

Answer all the questions in this section.

1 A steel ball of mass 250 kg is suspended from the boom of a crane, a shown in Fig. 1.1.

boom

wall

steel ball
Fig. 1.1

(a) In order to demolish a wall, the ball is pulled from the wall at an angle and then released
and hits the wall. The variation of the speed v of the ball with time t is shown in Fig. 1.2.
6

v / ms-1

0.5 1.0 1.5 2.0


t/s

Fig. 1.2

Using Fig. 1.2, determine

(i) the magnitude of the acceleration of the ball at time t = 0.8 s.

Acceleration = …………….[2]

www.KiasuExamPaper.com
362
3

(ii) the total distance moved by the ball from the moment of release to when it comes to
rest.

Distance moved = ……………………….[2]

(b) Explain why the steel ball undergoes decreasing acceleration after it is released, and then
uniformly decelerates till it comes to rest.

……………………………………………………………………………………………………..…

…………………………………………………………………………………………….…….……

………………………………………………………………………………………………………..

………………………………………………………………………………………………………..

……………………………………………………………………………………………………..[2]

[Total: 6 m ]

www.KiasuExamPaper.com
363
4

2 A model rocket of initial mass 1.3 kg is fired vertically into the air. Its mass decreases at a
constant rate of 0.23 kgs-1 as the fuel burns. The final mass of the rocket is 0.38 kg.

(a) Calculate the weight of the fuel being burnt off.

Weight of fuel being burnt off = ……………………..[1]

(b) The variation with time t of the upward force on the rocket during the first 3 seconds after
firing is shown in Fig. 2.1. The dotted line from 3.0 – 3.5 s is the predicted variation of the
upward force on the rocket with time t.

Force / N

t/s

Fig. 2.1
On Fig. 2.1, draw a line drawn to represent the variation with time t of the total weight of
the rocket during the first 5 seconds after firing. [2]

www.KiasuExamPaper.com
364
5

(c) (i) From the graph drawn in Fig. 2.1, read off the time delay between firing the rocket and
lift-off.

Time delay = ……………………….. [1]

(ii) Determine the resultant force acting on the rocket at t = 2.5 s. Show clearly how you
arrived at your answer in the space below.

Resultant force = …………………….[2]

[ Total : 6 m ]

www.KiasuExamPaper.com
365
6

3 Fig. 3.1 shows a student sitting on a chair. Fig. 3.2 shows the same student with his chair
tilted backwards slightly. The four legs of the chair are identical.

Fig. 3.1 Fig. 3.2

(a) (i) State and explain how the pressure of the chair on the floor differs in the two positions.

…………………………………………………………………………………………….………….

..……………………………………………………………………...…………………….………...

……………………………………………………………………………………...……….………..

.…………………………………………………………………………………………………....[2]

(ii) The chair and student fall over if the chair is tilted backwards more than in Fig. 3.2. Explain
why.

..…………………………………………………………………………………………………….

.………………………………………………………………………...…………………………...

..……………………………………………………………………………………...……………..

…………………………………………………………………………………………………........

……………………………………………………………………………………………………….

…………………………………………………………………………………………………….[3]

www.KiasuExamPaper.com
366
7

(b) Fig. 3.3 shows a painter standing on a wooden plank, directly above the right-hand support.

wooden
plank
3.6 m
0.35 m 0.025 m

centre 1.3 m
support support
of gravity

Fig. 3.3

The plank has a length of 3.6 m and a mass of 23 kg. The centre of gravity of the plank is in
the middle of the plank at a distance of 1.3 m from each of the supports. The gravitational field
strength g is 10 N / kg.

(i) Calculate the moment of the plank about the right-hand support.

moment = ....................................................... [1]

(ii) The painter moves further to the right along the plank and the plank rotates about the right
hand support.

Explain why the plank rotates.

………………………………………………………………………………...…………………………...

……………………………………………………………………………………………...……………..

………………………………………………………………………………………………………....[1]

[ Total : 7 m ]

www.KiasuExamPaper.com
367
8

4 A lamp is positioned at the bottom of a small pool of water. The critical angle for light passing
from water into air is 49°.

(a) Explain what is meant by the term critical angle.

...................................................................................................................................................

...................................................................................................................................................

...............................................................................................................................................[1]

(b) The lamp sends light towards the surface of the pool.

Fig. 4.1 shows three rays of light that are at 30°, 60° and 90° to the horizontal.

water

60°
90°
horizontal
30°

lamp

Fig. 4.1

On Fig. 4.1, draw the path taken by each of the three rays after they strike the surface of
the water. [2]

www.KiasuExamPaper.com
368
9

(c) Determine nwater, the refractive index of water.

nwater = …………………..[2]

(d) The lamp is moved towards the right. It is observed that, at a certain position, a circular patch
of light is seen on the surface of the water.

Explain how this circular patch is formed.

...................................................................................................................................................

...................................................................................................................................................

...............................................................................................................................................[2]

[ Total : 7 m ]

www.KiasuExamPaper.com
369
10

5 Fig. 5.1 shows two glass containers, one painted black and one painted white, containing
gases A and B respectively. They are connected together by a tube containing mercury.

Fig.5.1

The density of mercury density is 13 600 kgm-3.

(a) State which of the two types of gases is at higher pressure.

…………………………………………………………………………………………………………[1]

(b) Given that H1 = 40.0 cm and H2 = 48.0 cm and Gas A is at 120 000 Pa, calculate the
pressure of Gas B.

Pressure= …………………………[2]

(c) The whole set up is then placed under strong sunlight. Describe and explain how H1 and H2
would change.

……………………………………………………………………………………………………………

……………………………………………………………………………………………………………

……………………………………………………………………………………………………………

……………………….……………………………………………………………………..…………….

…………………………………………………………………………………………………………[3]

[Total : 6 m ]

www.KiasuExamPaper.com
370
11

6 Fig. 6.1 shows a cylindrical copper kettle that contains cold water.

cold water

copper

cable heater

Fig. 6.1

(a) State and explain the advantage of heating the water from below.

......................................................................................................................................................

......................................................................................................................................................

.......................................................................................................................................................

.................................................................................................................................................. [2]

(b) As the water is heated, it expands.

(i) Explain, in terms of molecules, why water expands when it is heated.

...............................................................................................................................................

...............................................................................................................................................

...............................................................................................................................................

.......................................................................................................................................... [2]

(ii) Copper also expands when heated.

State what happens to level X of the water in the kettle. Explain your answer in terms of
the expansion of the copper and the water.

...............................................................................................................................................

................................................................................................................................................

................................................................................................................................................

….......................................................................................................................................... [2]

[Total : 6 m ]

www.KiasuExamPaper.com
371
12

7 Fig. 7.1 shows a design for a simple circuit breaker in a household circuit.

Fig. 7.1

(a) The circuit breaker opens the circuit when the current gets too high. Explain how the
circuit breaker works as a safety device in the household circuit.

......................................................................................................................................

.......................................................................................................................................

.......................................................................................................................................

……………………………………………………………………………………………….....

…………………………………………………………………………………………………..

…………………………………………………………………………………………….…[4]

(b) Explain what will happen if the current direction is reversed.

.........................................................................................................................................

.........................................................................................................................................

.......................................................................................................................................[1]

[Total : 5 m ]

www.KiasuExamPaper.com
372
13

8 Fig. 8 shows the cut-out section of the handle and cradle of an electric toothbrush. The
figure on the right of the cut-out section shows the actual handle and cradle.
1.2 V
battery
rechargeable cell
and motor
+ –

handle of
toothbrush

B handle
coil X
plastic cases
A

Soft iron bar coil Y


Charging
cradle
220
0V
a.c. Model No.EA 2004
a.c. 220 V 50 Hz
3W

Fig. 8

The handle consists of a 1.2 V rechargeable cell and a motor. The cell is connected to coil
X located at the bottom of the unit. The cradle consists of a short projection which houses a
coil Y wound round a soft-iron bar. The cradle is connected to a 230 V a.c. mains supply.
When the handle is inserted into this short projection, the battery is recharged.

The handle and the charging cradle are completely covered by plastic cases and there is no
metal contact between them.

A label is also pasted at the side of the charging cradle.

(a) When the toothbrush is in operation, the current flowing through the motor is 1.8 A. Calculate
the power consumed by the motor.

Power = …………………[1]

www.KiasuExamPaper.com
373
14

(b) It takes 16 hours to recharge the cell fully. Calculate the amount of energy needed.

Amount of energy = …………………….[2]

(c) Explain how an electromotive force (e.m.f.) is produced in the brush unit to recharge the
cell.

………………………………………………………………………………………………………….

……….…………………………………………………………………………………………..…….

………..………………………………………………………………………………...………………

………………………………………………………………………………………………………….

……….…………………………………………………………………………………………..…….

………….………………………………………………………………………………...…………[3]

(d) The charging unit is fitted with a two-pin plug.

Suggest one reason why it is safe for the charging unit to be fitted with a two-pin plug.

………………………………………………………………………………………………………….

……….…………………………………………………………………………………………..…….

……………………………………………………………………………………...………………[1]

[ Total : 7 m ]

END OF SECTION A

www.KiasuExamPaper.com
374
15

SECTION B

Answer all the questions in this section.


Answer any one of the two alternative questions in Question 11.

9 When a large earthquake occurs at a particular location near the surface of the Earth ( known
as the Epicentre) three types of seismic waves are produced. These waves are called
Primary Waves (P-waves), and Secondary Waves (S-waves) and Surface Waves.

Fig. 9.1 shows the characteristics of these three types of waves.

Primary Waves ( P-wave) Secondary Waves (S-wave) Surface Waves

x Longitudinal waves x Transverse waves x Transverse waves


x Travels through the x Travels through the x Travels on the
ground ground surface.
x Fastest waves x Medium speed x Slowest waves
x Can travel through waves
solid and liquid x Only travel through
solids

Fig. 9.1

(a) Explain the difference between a longitudinal wave and a transverse wave in terms of
particle motion.

………...…….….………………………………………………………………………………………

…………………………………………………………………………………………………………..

…………..……………………………………………………………………………………………[1]

www.KiasuExamPaper.com
375
16

(b) Seismic stations around the Earth detect these seismic waves using an instrument called a
seismograph. Two types of seismographs are shown in Fig. 9.2(a) and Fig. 9.2(b).

Fig. 9.2(a) Seismograph X Fig. 9.2(b) Seismograph Y

(i) Which type of wave does Seismograph X detect?

………………………………………………………………………………………………[1]

Fig. 9.3 shows how seismograph X works when an earthquake occurs. The bolts secure
the base of the seismometer to the ground. Fig. 9.3(a) shows the seismograph before an
earthquake occurs.

Spring Weight and pen

Rest position

(a) (b) (c)


Fig. 9.3

When an earthquake occurs, a seismic wave passes through the ground below the
seismograph. The weight moves down when the ground moves up and moves up when the
ground moves down. A trace of this motion, known as a seismogram, is recorded on rotating
graph paper.

www.KiasuExamPaper.com
376
17

(ii) Explain how the up-and-down movement of the ground results in the weight moving
up and down.

…….….…………………….…………………………………………………………………

...……………………….……………………………………………………………………..

………..…………………………………………………………………………………….…

………………………………………………………………………………………………...

…………………………………………………………………………………….…………..

.……………………………………………………………………………………………..[2]

www.KiasuExamPaper.com
377
18

(c) Fig. 9.4 shows the travel times for a P-wave and a S-wave with distance from the epicentre
of an earthquake.

Graph of P-Wave and S-Wave travel time versus distance from epicentre of earthquake
Travel time / min

Distance from epicentre x 103 / km

Fig. 9.4

www.KiasuExamPaper.com
378
19

(i) A seismographic station, A, detects the arrival of an S-wave 5 minutes 40 seconds


after the arrival of a P-wave. Using Fig. 9.4, state the distance of the seismographic
station from the epicentre of the earthquake. Mark clearly on the graph to show how
you arrive at your answer.

Distance = ………………………[2]

(ii) Determine the average speed of the P-waves arriving at Station A in kms-1.

Average speed = .......................................... [2]

(iii) Two other seismographic stations, B and C, are located 3.2 x 103 km and 7.8 x 103 km
from the epicentre. Determine the average speed of the S-waves in kms-1.

Average speed = ............................................. [2]

www.KiasuExamPaper.com
379
20

10 Fig. 10.1 shows a rotating magnet in an alternating current generator that is used to power
a lamp.

Fig. 10.1

(a) (i) Explain, in detail, how alternating current is produced by the apparatus
shown in Fig. 10.1.

.................................................................................................................................

..................................................................................................................................

…...............................................................................................................................

…..............................................................................................................................

…..............................................................................................................................

…...........................................................................................................................[3]

(ii) State two ways in which the current in the lamp may be increased.

1...............................................................................................................................

2............................................................................................................................[2]

www.KiasuExamPaper.com
380
21

(b) The generators at a power station produce a voltage of 25 000 V. This voltage is stepped
up to 400 000 V by a transformer for long-distance transmission on overhead power lines.
The voltage is later stepped down to 240 V.

(i) State and explain why the voltage is stepped up for long-distance transmission.

......................................................................................................................................

…..................................................................................................................................

…..............................................................................................................................[2]

(ii) Calculate the ratio of the number of turns in the primary coil of the step-up
transformer to the number of turns in its secondary coil.

ratio = ..................................................[1]

(iii) An electric drill of power 800 W is used in a country where the mains voltage is
240 V. State and explain the most appropriate fuse to use with this drill.
You should select a fuse from the following values: 1 A, 3 A, 4 A, 13 A.

......................................................................................................................................

.......................................................................................................................................

...................................................................................................................................[2]

www.KiasuExamPaper.com
381
22

11 EITHER

11(a) What do you understand by electrostatic induction ?

………………………………………………………………………………………………………….

…………………………………………………………………………………………………………..

……………………………………………………………………………………………..………...[1]

(b) Fig. 11.1 shows two identical light conducting spheres P and Q hanging vertically from two
points on insulating threads.

P Q

Fig. 11.1

Describe and explain what happens if

(i) P is negatively-charged and Q is neutral,

…………………………………………………………………………………………………

.………………………………………………………………………………………………..

….…………………………………………………………………………………..……….....

…………………………………………………………………………………………………

…………………………………………………………………………………………………

………………………………………………………………………………………………[2]

www.KiasuExamPaper.com
382
23

(ii) both P and Q have the same amount of negative charges.

…………………………………………………………………………………………………

…………………………………………………………………………………………………

…………………………………………………………………………………………………

…………………………………………………………………………………………………

…………………………………………………………………………………………………

…………………………………………………………………………………………. ….[3]

(c) When a balloon is rubbed on hair, the balloon becomes negatively charged. The balloon is
shown in Fig. 11.2.

Fig. 11.2

(i) Explain how rubbing causes the balloon to become negatively charged.

…....................................................................................................................................

…....................................................................................................................................

...……….........................................................................................................................

…………………………………………………………………………………………………. [2]

www.KiasuExamPaper.com
383
24

(ii) Explain why the hair is attracted towards the balloon.

.....................................................................................................................................................

.....................................................................................................................................................

................................................................................................................................................[1]

(iii) Explain why it is important that the balloon is made from an electrical insulator.

.....................................................................................................................................................

................................................................................................................................................[1]

www.KiasuExamPaper.com
384
25

11 OR

Newton’s third law of motion can be expressed in the following form.

“When body A exerts a force on body B, then body B exerts a force on body A. These
forces are

• equal in magnitude,
• opposite in direction,
• of the same nature.

(a) An object is undergoing free fall with no air resistance. Explain, using a labelled force
diagram, the application of Newton’s third law to this falling object.

……………………………………………………………………………………………………………

……………………………………………………………………………………………………………

……………………………………………………………………………………………………………

…………………………………………………………………………………………………………[2]

(b) An object is dropped out of a plane from 10,000 m. Air resistance increases as the object
speeds towards Earth.

(i) On Fig. 11.1, sketch a graph to show how the speed of the object falling from rest in
air varies with time.
[1]

speed

0
0
time

Fig. 11.1

www.KiasuExamPaper.com
385
26

(ii) Explain, using a labelled force diagram, the application of Newton’s third law to this
falling object at terminal velocity. You should exclude the answer you gave in (b)(i)
above if they are the same.

..................................................................................................................................

..................................................................................................................................

..................................................................................................................................

..................................................................................................................................

.................................................................................................................................

……………………………………………………………………………………………[2]

(c) A diver of height 1.80 m has his centre of gravity (C of G) 1.00 m above his feet when
standing on the springboard. Fig. 11.2 illustrates the diver leaving the springboard, moving
upwards and then entering the water.
C of G

0.80 m
C of G
1.00 m

3.00 m

C of G
water

Fig. 11.2 (not to scale)

The diver leaves the springboard with an upward velocity of 5.6 m s–1. The take-off point on
the board is 3.00 m above the water.

www.KiasuExamPaper.com
386
27

Assume that the centre of gravity (C of G) of the diver remains at the same position within
the diver throughout the dive and ignore air resistance.

(i) Explain what you understand by the centre of gravity ( C of G) of an object.

……………………………………………………………………………………………………

………………………………………………………………………………………………….[1]

(ii) Determine the maximum height of his centre of gravity above the water.

height = ..................................... [2]

(iii) Determine the speed at which the diver’s head reaches the water.

speed = ..................................... [2]

END OF PAPER

www.KiasuExamPaper.com
387
28

BLANK PAGE

www.KiasuExamPaper.com
388
www.KiasuExamPaper.com
389
1

2019 PRELIMINARY EXAMINATION


PHYSICS 6091

PAPER 1
1 B 2 C 3 C 4 A 5 B
6 A 7 A 8 A 9 A 10 D
11 B 12 A 13 C 14 B 15 C
16 A 17 C 18 C 19 D 20 A
21 B 22 B 23 A 24 C 25 B
26 D 27 D 28 D 29 A 30 B
31 D 32 D 33 A 34 B 35 B
36 B 37 A 38 C 39 A 40 C

PAPER 2

Mark
Ma
M
Mar
aarrkk
Qn Suggested solution
ution Remark
R
3 ms-2
1(a)(i) Acceleration = (5.6 – 2.2)/1.35 – 0.20 = 22.3
.3 [[2]]
[2 Evidenc (two
Evidence
coordinat
coordinates and
tang gen
e t shown
tangent sho
on
o n ggraph
raaph
(ii) Total distancee = area under
un
ndeer v-tt graph
ggrrap
aph [[2]
[2
2] Calculat
Calculation
C
؆66.0
.0
0m showing how
sh
area is derived
(b)(i)) ▪ Air
Air rresistance
Ai esiist
s ance inc increases
ncre
nc reaasseess w
reas with ith iin
it increasing
ncreaasi sin
ngg sspeed
peeeed
p d as GPEE is i [2]
converted
ccoonv
n erteed to to K
KE.From
E..FrF om Fnnet neett =ma,
=m
= m a
a,, t
the
he
h e n
net
e
ett f
force
o r ce acting
a ctin
ct i
inn g on
o n the ball
decreases
decr
de ccrreeaases and
a d it
an it w will
ill u
un
undergo
nddeerrggo de deceleration
ece
celeraation
ttiionn [[1] 1
1]
▪ Th
TThee wall’s
wallll’ss resistance
wa resistattaanc
nce iiss constant.
coonst
on stant. From
Fro
roomm Fresista
resistance = ma, the

deceleration
deece
d c leeration on iiss a co
on cconstant.
onnsstant. [[1] 1]
1]

Total
tal 6
2(a) Weight
ght = ((1.3– 0..3
0 38 )x10 Nkg-1
1.3– 0.38)x10
8)x [1] No marks for no
= 9.2
9.2
2N working/wrong
unit
Line sloping
ing from
f 13.0 N to 3.8 N [1]
Line parallel from t-axis from 4.0 s [1]

*Time taken for the fuel to blast off = 0.92 kg/0.23 kgs-1
= 4.0 s
*working optional
( c)(i) 0.5 s [1]
(ii) Resultant force acting at 2.5 s = 16.7 – 7.3* N [1] [1] *value as per
= 9.4 N [1] [1] graph drawn
Total 6
3(a)(i) Pressure exerted on the floor in Fig. 3.2 is greater than that in Fig. [2]
3.1. Weight of the boy and chair, W is distributed over two legs
T

compared to over four legs plus the shoes, so from P =W/A, the
smaller area of contact in Fig.3.2 will result in a larger pressure.

www.KiasuExamPaper.com
390
2

(a)(ii) The line of action of the combined weight of the student and the [3] Unstable
chair is in line with the pivot [1]in Fig. 3.2. If he tilted further equilibrium not
backwards, the line of action of this combined weight is not in line accepted unless
with the pivot [1] and this creates a resultant anticlockwise accompanied by
explanation
moment about the two hind legs[1].
(b)(i) Anticlockwise moment of plank = 230 N x 1.3 m [1]
= 299 Nm
= 300 Nm ( 2s.f.) [1]
3(c) The painter’s weight created a net clockwise moment about the [1]
right hand support / the clockwise moment > 299 Nm
Total 7
4(a) It is the incident angle in the optically less dense medium which [2]
resulted in a refracted angle of 90o in the optically less dense
medium
(b) [2] Three rays correctly
drawn [2]

TTwo rays correctly


air drawn [1]
dra
water
w
wa terr
te


60
90° 30°
30
horizon
h
ho
orriiz
zo
o
onn

lamp
p

(c)) nwat
aatterr = 1/ sin
water in
n cwwater
ater
e [2]
= 1/
1 sin 49o
n 49
= 1.33
1 33
1. 33
(b) ▪ Within
W th
Wi hin
i thee ccircular
ircula
ir culaar p
cu patch
attcch – the
the light
liligghtt is incident
in at the surface at [2]
an aangle
an nggle
le ooff in cciidence G49
iincidence
ncid G49 and
o
a d em
an emerge out of the water [1].
▪ Be Beyond
eyyo
ond d tthe
he circircular
rcu
cula
larr pa
la p
patch,
tch, tthe light is incident on at the water
surface
urface at an an of incidence > 49o, resulting in total internal
an angle
reflection.
lecti
tion
on. Light does not emerge out [1]. The edge of the
circular
lar patch
pa thus represents the boundary between total
internal reflection and no total internal reflection.

Total 7
5(a) Gas A [1]
(b) PB + (0.08)(13600)(10) = 120000 [1] [2]
5 5
PB = 1.09 x 10 Pa or 1.1 x 10 Pa. [1]
(c) H1 will drop and H2 will rise [1] resulting in a bigger difference [3]
between the two levels . Black surfaces are good absorbers of
radiation/thermal energy. Gas A receives the thermal energy,
resulting in a pressure build-up [1]. This increase in pressure
pushes the level of mercury down in the left arm and up in the
right arm, thus increasing the height difference between the two
levels. [1]

www.KiasuExamPaper.com
391
3

Total 6
6(a) Even heating throughout / Take less time / speed up heating / [2]
even temperature[1]
Heating the water from below creates a convection current due
to the displacement of cooler denser water at the top by warmer
but less dense water below[1] This continuous movement of water
will ensure that thermal energy is evenly spread throughout and
time taken for heating the water is less.

(b)(i) Molecules vibrate vigorously on receiving thermal energy. The [2]


increase in the amplitude of molecular vibration increases the
spacing between the molecules[1]. Layers of liquid molecules are
moving faster and move further apart[1]. Both factors produce an
increase in the volume of water. Thus water expands.
(ii) Level X drops and then rises [1] Copper expand faster than water. r. [2]
The increase in volume of copper will lower the water levell first.
After the copper ceases expanding, the continual expansion
pansion of
water will raise its level.
Total 6

7(a) ▪ When a high current passes through, hrough h, thtthee iron


on ccore
on orre is
is [[4]
4]
magnetized because a magne magnetic ettiic field d is sset
ett up
up in
in tthe
he ccoil
he oil [1]
oi [1]
▪ The magnetized core then thhen
en aattracts
ttrra
tt ract
actcts the ir iiron
ron
on lever,
lever
err, rotat
rotating
tingng iitt
about the pivot and lifting
iiffting it
d lif it upp [[1]
1]]
▪ This causes thtthe
he spring
springy gy m me
metal
ettaal to
to bebe released
rreele
leas
a edd aass it is pulled
puulllled
ed by
the spring
spri
sp
pr ng andnd
n d this
thi
his causes
c usses
ca es the
thee contacts
con
o tacts to to be
be opened.
op
▪ The
TTh
he spring
sppriing
n als
also
llsso pulls
pulllls th
pu the
he spspringy
pring
rii gy m metal
etal
et al towards
tow
owaarrds thee reset
res
eset et
button
button tthereby
bu heereby byy pushing
b ngg iitt o
n outwards[1]
ou utw
twaarrdsds[1 [1]

(b) The
Th
he workings
workings will
wilill not
no
n ot be
b affected
affecteed as tthe
he core is
i still magnetized [1]
and aattraction
d atttr
trac
a ti
actio
onn still
stiillll take
ake place.
tak plac
ace.
ac e.
Total
otal 5
8(a) P = 1.8 x 1.2
1.2
1.
.2 = 2.1616 W [1]

(b) E = Pt [2]
= 3 x 166 x 60 x 60
= 172800J
(c) ▪ The a.c. flowing in the coil in the charging unit produces a [3]
changing magnetic field in coil Y, which is concentrated by the
soft-iron bar [1].
▪ When the brush unit is placed on the charging unit, the
changing magnetic flux linking coil Y and X produces the
induced e.m.f. [1]
▪ The induced a.c. current in coil X will charge the cell connected
to it

(d) Because both the brush and charging unit are completely covered [1]
by plastic, the casing will not be ‘live’ even if there is a fault and

www.KiasuExamPaper.com
392
4

hence the earth wire is not necessary and a two-pin plug will
suffice.
Total 7
9(a) Difference is in the direction of oscillation of the particles. [1]
Longitudinal wave, the particles oscillate parallel to the direction
of wave propagation
Transverse wave, the particles oscillate perpendicular to the
direction of wave propagation.
(b)(i) S-wave / Secondary Wave and surface waves [1]
(ii) When the ground move up, the weight, due to its inertia, will tend [2]
to remain in its state of rest and move downwards. The spring is
stretched[1] When the ground move down, the stretched spring
will release its stored elastic potential energy and pull the weight
up [1]
(c)(i) 4 x 103 km [1 m] [2]
1m – clear marking on the graph
(ii) Average speed = 4000 km / (7x 60)s [2]
[2
= 9.52 kms-1
(iii) Average speed = (7.8 – 3.2)x 103 km÷( 20 mins 20 s – 10 0 min
min 40
40 s) [2]]
[2
= 4.6 x 103 ÷ 9 min 40 s
= 7.93 kms-1

Total 10

(a) ▪ TTh
10(a) The
he rotation
rotataatiion ofof the
the magnet
th magn
ma gnet
e induces
induc ucces
u e each
eac
a h end
en
nd of the he [3]
(i)) ▪ sso
soft
oft
f iron to to alternate
alter
eerrnate in polarity
po
p ola
larriittyy at
lari at every
evveerry halff rrotation.
otat
ot atio
at ion. [1]
io [
▪ Th
The
he strength
ssttre
rength th of
of the
t e magnetic
th maagn
m gnet
etic flux in the the soft
sof
oftt iron
iron increases
in and
decreases
d
decrease
dee ses as tthe
se hee m
h magnet
aaggne move
net mo
m ve ttowards
owaard
ow ds and away from the soft
iron.
ir
roonn
n.. [1] 1]]
1
▪ ThThe
he coil
coiill experiences
co exp
xpeerrie
xp i ncess a constant
con
onstant rate of change of magnetic flux
linkage
inkagagge w withith this
thi
his
is alternating
alte
al ternatin polarity and changing magnetic
te
field
ld strength.
stre
r ng
re ngthth. This
th T is induces
Th in an alternating e.m.f hence an
alternating
natin current in the coil. [1]
i g cur

(ii) more turns in coil/ thicker wires/ stronger magnet/ faster rotation [2]

(b)(i) To reduce power loss because with high voltage and low current is [2]
lowered [1] This reduces power loss through joule heating/heating
effect by the current [1]
(ii) N = 25/400 = 0.0625 ( 1:16) [1]
(iii) P = VI [2] Calculation
800W = 240 x I shown that
I = 800/240 warrant correct
= 3.33 A

www.KiasuExamPaper.com
393
5

Fuse : 4 A selection of fuse


rating.
Total 10
Either Charging without contact between a conductor and a [1]
11(a) charged body/ separation of charges in a conductor when
the conductor is placed in an electric field
(b)(i) ▪ P induces positive charges on Q on the side closer to P/repels [2]
electrons on Q to the right side leaving positive charges induced
on the side closer to P[1]
▪ P and Q are attracted to each other as opposite charges
attract[1].

(ii) ▪ P and Q will be repelled away from each other as like charges [3]
repel [1].
▪ Both P and Q will be displaced at the same angle from the
vertical and remain in that equilibrium position
tion [1].
▪ Both spheres have the same amount of charge and th tthe
he fo
forc
force
rcee of
rc
repulsion are action-reaction pair
air forc
forces
ces
es [ 1]

(c)(i) EITHER [2]


[2
Electrons
ctronss from f om the
fr thee hair
haiir aree st sstripped
tri
r pp
pped
ed off/trans
off/transferred
nssffeerrrred from th
n the
he ha
hai
hair
ir
atoms
atom ms an andnd d deposited
epossite ted onon the
thhee balloon
bal
allo [1].The
loon [1]1]].TTh
1 hee excess
excxcese s electrons
elecctronss on
tron
on
thee balloon
baalloon cause
n caausee itt to become
becoccoomeme negatively-charged
neg
egaattiv
iveeelllyy-charge g d [1[1]]
OR
OR
FFriction
Friiccttiio
onn between
beettwe
w en
en thee hair haaiir and
aannd th
tthee balloon
balloo on ge gene
generates
n rates thermal
energy[1].
en
nerrgy g [1]. TThe he wweakly-attracted
eeaakl
eakl
kly-aatttr
traccted elec electrons
ectr
ec tron
tr onss of the
on th atoms of the hair
gain n this
thiss tthermal
th herrm
he m
malall eenergy
neerg
n rgy to eescape
scap
sc apee and de
ap deposited on the balloon
thereby
hereb eb by making
makiin
ma ngg iitt negatively-charged
negaati
tive
vely
ve ly--ch
ly harged [1]

(c)(ii) EITHER R The


Th
he negatively-charges
negative
ne on the balloon and the polarized [1]
atoms on n the hair.
h Opposite charges attracts, causing the hair to
be attracted to the balloon.
OR The negatively-charged balloon attracts the positively-charged
hair / induces the positively-charge on the hair closer to the
balloon. As opposite charges attract, the hair is attracted to the
balloon.
(iii) Charges accumulated on the balloon will be retained on the balloon [1]
in and around the region where the balloon is being rubbed.

Total 10

www.KiasuExamPaper.com
394
6

11OR [1]
(a)(i)
Force exerted on object by
Earth
Force exerted on Earth by
object

Earth and object exerts an equal and opposite pull on earth other. [1]
The force exerted on the object is the weight. The object exerts an
amount of force equal to this weight on the Earth.
(b)(i) [1]

speed

0 ttime
ti
im
mee
0
(ii) [1]]
[1

Force
FFo
orce exerted
exxer
e ted on air by
by
object
ob
o bje
ject
ct

Force
orce exerted
exxeerrte
t d on
noobject
bjec
bj ect by
b aair
irr

At tterminal
A errminall velocity,
l ciityy, objectt exerts
vveelo rts a force
exeert forrcce on
fo on thee body
odyy of air as
bod a it
passes
pa es through
asssses through gh iit.
gh t The
t. The body ody of
bod
od of air
air exerts
exxeerts ann amount
amo
mounu t of fforce equal [1]
in magnitude
maggninitu de and
t de and
n oppopposite
pp
p po
ossit
ite in direction
in d irectionn to
to this
thiis force.
th forc
(c)(i) It iiss a point on n or
or outside
out
utsisid
dee a bod
d body
dy where
wheere the w
wh whole weight of the [1]
body dyy appears
app
ppea rs tto
eaars o act.
actt..
ac
(ii) From
om mghmgh
gh = ½ m mvv2 [2]
h = ½ (5.6)(5.6)2 ÷ 10
5.6)
= 1.57
1.57 m [1]
Height of CG above water = 1.57 + 4.00
= 5.57 m [1]
From v = √ ( 2gh ) [2]
(iii) = √ ( 2 x 10 x ( 5.57 – 0.8 ) [1]
= 9.8 ms-1 [1]
Total 10

www.KiasuExamPaper.com
395
www.KiasuExamPaper.com
396
TANJONG KATONG GIRLS’ SCHOOL
PRELIMINARY EXAMINATION 2019
SECONDARY FOUR

6091/01 PHYSICS
Paper 1 Multiple Choice

WEDNESDAY 4 SEP 2019 1 hour

Additional materials: OMR answer sheet

INSTRUCTIONS TO CANDIDATES
Do not open this booklet until you are told to do so.

Write your name, class and index number on the OMR Answer Sheet.

There are forty questions in this paper. Answer all questions. For each question, there are
four possible answers, A, B, C and D.

Choose the one you consider correct and record your choice in soft pencil on the OMR
Answer Sheet.

Take gravitational field strength to be 10 N/kg unless stated otherwise in the question.

Read the instructions on the OMR Answer Sheet very carefully.

At the end of the examination, hand in the OMR Answer Sheet.

INFORMATION FOR CANDIDATES

Each correct answer will score one mark. A mark will not be deducted for a wrong answer.
Any rough working should be done in this booklet.

This question paper consists of 17 printed pages including this cover page.

www.KiasuExamPaper.com
397
Answer ALL the questions in this paper on the OMR sheet provided.

1 Which of the following is a good estimate for the thickness of a fingernail?

A 1 dm B 1 cm C 1 mm D 1 μm

2 A student intends to determine the volume of a copper pipe which spans several
metres long. The pipe has a uniform cross-sectional area. The external diameter of
the copper pipe is estimated to be 5 cm.

Which pair of instruments will allow him to measure the necessary dimensions
accurately?

A vernier calipers and tape


B micrometre screw gauge and tape
C vernier calipers and rule
D micrometre screw gauge and vernier calipers

3 Three forces 3 N, 4 N and 8 N act on an object. Which of the following cannot be the
resultant force acting on the object?

A 0N B 1N C 8N D 14 N

4 A racing car is fitted with an on-board computer which can record the distance
travelled by the car for every one second. The computer starts recording when the car
passes the starting line and moves along a straight line.

Which set of data shows that the car is accelerating during the next 2 seconds?

A Time / s Distance / m B Time / s Distance / m


0 0 0 0
1 100 1 90
2 200 2 180

C Time / s Distance / m D Time / s Distance / m


0 0 0 0
1 100 1 80
2 180 2 190

Tanjong Katong Girls’ School 2 Sec 4 Preliminary Examination 2019

www.KiasuExamPaper.com
398
5 The diagram shows a person using a rope to pull a block on a rough surface to the
right. The block moves at a constant speed.

Which pair of forces is a pair of action and reaction force?

A Frictional force acting on the block by the ground and the pulling force on the
block
B The pulling force on the block and the tension force experienced by the rope
C The weight of the block and the normal reaction force acting on the block by the
ground
D The frictional force acting on the person by the ground and the frictional force
acting on the block by the ground

6 The diagram shows the velocity-time graph of a car. The total resistive force acting on
the car is 1000 N. The mass of the car is 1000 kg.

What is the resultant force acting on the car at t = 4.0 s?

A 0N B 4000 N C 5000 N D 6000 N

7 An astronaut lands on a planet where the acceleration of free-fall at its surface is


greater than that on Earth. Which of the following will be the same as that on Earth?

A The weight of the astronaut


B The period of oscillation of a simple pendulum
C The height reached by the astronaut when he jumps with the same initial velocity
D The acceleration of a block when being pushed horizontally by the same force on
a smooth surface

Tanjong Katong Girls’ School 3 Sec 4 Preliminary Examination 2019

www.KiasuExamPaper.com
399
8 A circular object P is lowered into a cylinder which contains 3 different layers of
immiscible liquids. The diagram shows the position of object P in the cylinder.

liquid Q
liquid R
object P
liquid S

Which of the following correctly shows the densities of the substances arranged in
increasing order?

A liquid Q, object P, liquid R, liquid S


B liquid Q, liquid R, object P, liquid S
C liquid S , liquid R, object P, liquid Q
D liquid S, object P, liquid R, liquid Q

9 The diagram shows a stationary uniform


ladder leaning against a smooth wall and
making an angle of 30o with the ground force, F
at point X.

The wall exerts a horizontal force F on


the ladder. The weight of the ladder is point X
500 N.

What is the magnitude of the force F?

A 144 N B 250 N C 433 N D 500 N

10 The diagram shows a bottle being filled up with water. water


What happens to the stability and the centre of the gravity of the
bottle (with water) as the bottle is filled with water?

Stability Centre of Gravity


A become less stable centre of gravity is raised
B become less stable centre of gravity is lowered
C become more stable centre of gravity is raised
D become more stable centre of gravity is lowered

Tanjong Katong Girls’ School 4 Sec 4 Preliminary Examination 2019

www.KiasuExamPaper.com
400
11 The diagram shows a pin being squeezed between a finger and the thumb.

Which statement is correct?

A The force of the pin is larger on the finger than on the thumb.
B The force of the pin is larger on the thumb than on the finger.
C The pressure of the pin is larger on the finger than on the thumb.
D The pressure of the pin is larger on the thumb than on the finger.

12 The diagram shows the levels X and Y in a liquid manometer with the gas tap open.

The gas supply is now replaced by a new gas supply which has a pressure that is 2 cm
of the liquid above the previous gas supply.

What will be the new difference in height between level X and Y?

A 16 cm B 18 cm C 20 cm D 22 cm

Tanjong Katong Girls’ School 5 Sec 4 Preliminary Examination 2019

www.KiasuExamPaper.com
401
13 The diagram shows a marshmallow placed inside a bell jar with the tap closed.

bell jar
marshmallow

tape
tube

The pressure of the gas in the bell jar is half of the atmospheric pressure.

The tap is then opened and the air from the surrounding rushes in through the tube.

What happens to the size of the marshmallow and the gas pressure in the
marshmallow?

Size of marshmallow Gas pressure in marshmallow


A decreases increases
B decreases decreases
C increases increases
D increases decreases

14 To calibrate a liquid-in-glass thermometer without using another thermometer, fixed


point(s) will be required.

Which statement is correct?

A Only one fixed point is required.


B Both a lower fixed point and an upper fixed point are required.
C Any temperature above the melting point of liquid can be used as fixed points.
D The melting point and boiling point of the liquid in the thermometer are always the
fixed points.

15 A resistance thermometer has a resistance value of 20 Ω and 80 Ω when the


temperature is 10 oC and 90 oC respectively. What will be the expected temperature if
the resistance of the thermometer is 100 Ω?

A 90 oC B 107 oC C 117 oC D 133 oC

Tanjong Katong Girls’ School 6 Sec 4 Preliminary Examination 2019

www.KiasuExamPaper.com
402
16 The diagram shows a thermocouple connected to a galvanometer. Two ends of the
wires are placed in junction P and Q respectively.

G
wire 1 wire 2

wire 3

P Q

However, the galvanometer does not show any deflection.

Which of the following is not a possible reason for the observation?

A Wire 1 and wire 2 are made of the same material.


B Wire 1 and wire 3 are made of the same material.
C Both junctions P and Q have same temperature.
D The galvanometer is not sensitive enough to detect the current.

17 The diagram shows a metal spoon in a cup of hot water.

metal spoon

a cup of hot water

Why does the top end of the metal spoon becomes hot after a while?

A Heat transfers from the hot water to the top of metal spoon by radiation.
B Heat transfers from the hot water to the top of metal spoon by convection.
C Heat transfers from the surrounding air to the top of metal spoon by conduction.
D Heat transfers from the hot water to the top of metal spoon by conduction.

Tanjong Katong Girls’ School 7 Sec 4 Preliminary Examination 2019

www.KiasuExamPaper.com
403
18 A solid object with a mass of 5.0 kg is heated from 30 oC to 40 oC. The heat capacity
of the object is 500 J K-1. Which expression gives the amount of thermal energy
required to raise the temperature of this object?

A 500 x 10
B 5.0 x 500 x 10
C 500 x (273 + 10)
D 5.0 x 500 x (273 + 10)

19 The diagram shows the change of state of matter for 1 kg of steam into ice.

steam water water ice ice

A B C D

100 oC 100 oC 0 oC 0 oC -10 oC

Which stage A, B, C or D involves the specific latent heat of vaporisation?

20 A student is investigating the rate of evaporation of water.

The student can change the following:

1. the depth of the water;

2. the atmospheric pressure;

3. the temperature of the water.

How many of these changes, if any, would alter the rate at which evaporation occurs?

A 0 B 1 C 2 D 3

Tanjong Katong Girls’ School 8 Sec 4 Preliminary Examination 2019

www.KiasuExamPaper.com
404
21 The graph shows how the height of a water surface at a point in a harbour varies with
time as waves pass the point.

Height

Time

What are p and q?

p q
A displacement period
B displacement wavelength
C amplitude period
D amplitude wavelength

22 A ripple tank is used to demonstrate refraction of plane water waves.

region X region Y

direction of
wave travel

boundary
Which statement is true?

A Region X is deeper than region Y and the direction of wave travel bends towards
the normal at the boundary.
B Region X is deeper than region Y and the direction of wave travel bends away
from the normal at the boundary.
C Region X is shallower than region Y and the direction of wave travel bends
towards the normal at the boundary.
D Region X is shallower than region Y and the direction of wave travel bends away
from the normal at the boundary.

Tanjong Katong Girls’ School 9 Sec 4 Preliminary Examination 2019

www.KiasuExamPaper.com
405
23 The diagram represents circular wavefronts coming from point S. The wavefronts are
about to strike a solid boundary.

Which diagram correctly shows the reflected wavefronts?

A B

C D

24 Which line shows the path of light ray after it passes through the glass block?

light ray C
B

glass block
A

Tanjong Katong Girls’ School 10 Sec 4 Preliminary Examination 2019

www.KiasuExamPaper.com
406
25 Which coloured light, red or violet, has a higher frequency and which one has a longer
wavelength?

Higher frequency Longer wavelength


A violet violet
B violet red
C red violet
D red red

26 The diagram shows a contactless payment system which involves the consumers
tapping their credit cards or debit cards on the card reader to make their payment. In
the process, a particular electromagnetic wave will be sent between the card and the
card reader.

What is likely to be this electromagnetic wave?

A Radio wave
B Ultrasound
C Ultra-violet radiation
D X-ray

27 A person strikes a tuning fork near a wall.

What will happen to the frequency and speed of the sound as it travels from air and
through the wall?

Frequency Speed of Sound


A increases increases
B decreases decreases
C unchanged decreases
D unchanged increases

Tanjong Katong Girls’ School 11 Sec 4 Preliminary Examination 2019

www.KiasuExamPaper.com
407
28 The diagram shows an engineer standing at P overlooking an explosion at X.

30 m 30 m

40 m 40 m

After the explosion, she hears two bangs. The speed of sound in the air is 300 m s-1.

What is the time lapse between the two bangs?

A 0.12 s B 0.17 s C 0.25 s D 0.34 s

29 The diagram shows a negatively charged object subjected to a constant pulling force
and is moving to the right on a rough insulated surface at a uniform speed before
entering a uniform electric field. The electric force exerted on the charged object due
to the electric field is greater than the weight of the charged object.

object electric field


force

rough insulated surface

Which statement describes the motion of the charge in the electric field?

A The object will decelerate along the rough surface.


B The object will accelerate along the rough surface.
C The object will move along a curved path towards the top of paper.
D The object will move in the direction that is perpendicular to the plane of this
paper.

Tanjong Katong Girls’ School 12 Sec 4 Preliminary Examination 2019

www.KiasuExamPaper.com
408
30 The diagram shows a positively charged acetate strip and a negatively charged
polythene strip that are freely suspended.

When an object is brought either to the acetate strip or polythene strip, the strip moves
towards the object.

What is the state of charge of the object?

A Neutrally charged
B Positively charged
C Negatively charged
D Cannot be determined

31 The diagram shows an electroshock gun that law enforcement officers use to
immobilise a person. The gun delivers electric current to a person and disrupts
voluntary control of muscles in the person.

During a single activation that last for 5.0 s, the electroshock gun can deliver a charge
of 100 mC with an average voltage of 350 V to the person.

What is the electrical energy transferred to the person?

A 7J B 35 J C 7000 J D 35 000 J

Tanjong Katong Girls’ School 13 Sec 4 Preliminary Examination 2019

www.KiasuExamPaper.com
409
32 A cell of e.m.f. 2.0 V is connected to a network of resistors shown.

What is the potential difference across P and Q?

A 0.2 V B 0.5 V C 0.8 V D 1.0 V

33 The diagram shows a 6.0 V battery connected to a transducer and a fixed resistor.

The following information is provided:

Transducer Range of Resistance / Ω


Light dependent resistor 4.0 to 10.0
Thermistor 3.0 to 7.0

What is the reading of the ammeter when the transducer is exposed to high
temperature?

A 0.50 A B 0.67 A C 1.0 A D 1.2 A

Tanjong Katong Girls’ School 14 Sec 4 Preliminary Examination 2019

www.KiasuExamPaper.com
410
34 What will happen to a bar magnet if it is dropped too often?

A The poles of the magnet will switch position.


B Both ends of the magnet will have the same pole.
C The strength of the magnet will increase.
D The strength of the magnet will decrease.

35 A positive charge is moving into the plane of this paper.

Which diagram shows the magnetic field produced by the positive charge when viewed
from the top of this paper?

A B

C D

36 What is the purpose of the split-ring commutator in a motor?

A To ensure that the current produced is a direct current.


B To ensure that the current produced is an alternating current.
C To ensure that the forces acting on the coil will always rotate the coil in the same
direction.
D To ensure that there is continuous electrical contact between the coil and the
external circuit at all times.

Tanjong Katong Girls’ School 15 Sec 4 Preliminary Examination 2019

www.KiasuExamPaper.com
411
37 Two circuits are set up as shown. The iron rods are placed close together, and are free
to move.

What happens to the size of the gap at X when switch S is closed?

A It decreases.
B It decreases and then increases.
C It increases.
D It does not change.

38 The North pole of a bar magnet is pushed into a solenoid via end X, as shown in the
diagram. An electromotive force is induced which moves the galvanometer needle to
the left.

Which action, using the same end of the solenoid, would produce the same deflection
in the galvanometer?

A Pulling a North pole out of the solenoid via end X


B Pushing a South pole out of the solenoid via end X
C Pulling the solenoid away from a North pole
D Pulling the solenoid away from a South pole

Tanjong Katong Girls’ School 16 Sec 4 Preliminary Examination 2019

www.KiasuExamPaper.com
412
39 An a.c generator is able to produce a peak voltage of V when the coil is rotated with a
frequency of F.

What will be the new peak voltage if the number of turns of the coil is now three times
as before and the coil is rotated with a frequency of 2F?

A 3V B 5V C 6V D 8V

40 A transformer has 50 turns on its primary coil and 100 turns on its secondary coil. An
alternating voltage of 25.0 V is connected across the primary coil.

What is the voltage across the secondary coil?

A 12.5 V B 50 V C 175 V D 200 V

 END OF PAPER 

Tanjong Katong Girls’ School 17 Sec 4 Preliminary Examination 2019

www.KiasuExamPaper.com
413
Class Register No.

Candidate Name ………………………………………………..

TANJONG KATONG GIRLS’ SCHOOL


PRELIMINARY EXAMINATION 2019
SECONDARY FOUR

6091/02 PHYSICS
Paper 2

MONDAY 2 SEP 2019 1 hour 45 minutes


INSTRUCTIONS TO CANDIDATES
Do not open this booklet until you are told to do so.

Write your name, class and register number in the spaces at the top of this page and on any separate
answer paper used.

Write in dark blue or black pen.


You may use a soft pencil for any diagrams or graphs.
Do not use staples, paper clips, highlighters, glue or correction fluid.

Take gravitational field strength to be 10 N/kg, unless specified in the question.

Section A
Answer all questions. Write your answers in the spaces provided on the Question Paper.

Section B
Answer all questions. Question 11 has a choice of parts to answer.

INFORMATION FOR CANDIDATES


The number of marks is given in brackets [ ] at the end of each question or part question.

Candidates are reminded that all quantitative answers should include appropriate units.

Candidates are advised to show all their working in a clear and orderly manner, as more marks are
awarded for sound use of physics than for correct answers.

The use of an approved scientific calculator is expected, where appropriate.

For Examiner’s Use


Section A
Section B
Setter : Mr Koh Meng Hong Total / 80

This question paper consists of 22 printed pages including this cover page.

www.KiasuExamPaper.com
414
SECTION A [50 marks]
Answer ALL questions from this section.

1 Fig 1.1 shows a hot-air balloon rising with a constant velocity of 15 m s-1. A sandbag For
was dropped by the pilot at time t = 0 s. Air resistance acting on the sandbag is Examiner’s
Use
considered to be negligible.

hot air balloon

Fig 1.1

(a) A person at ground level observed that the sandbag was moving up with an
initial velocity of 15 m s-1 at the time of drop. Explain this observation.

…………………………………………………………………………………….......

……………………………………………………………………………………...[1]

(b) Sketch the velocity-time graph of the sandbag for the first 3.0 s. [2]
velocity / m s-1

time / s
0

(c) Calculate the distance travelled by the sandbag between time t = 0 s and when
it reaches its highest point.

Distance travelled = ……………[1]

(d) Determine the distance between the sandbag and the hot air balloon at time
t = 3.0 s. Show your workings clearly.

Distance = ……………[2]

Tanjong Katong Girls’ School 2 Sec 4 Preliminary Examination 2019

www.KiasuExamPaper.com
415
2 Fig 2.1 shows a side view of a windmill. For
Examiner’s
Use
windmill
tower

wind force

steel cable
connected to
another structure
a
30.0 m

frictional force acting 20.0 m


on tower by the ground
ground

Fig 2.1

The windmill stands on a tower whose base is anchored into the ground. The centre
of the windmill is 30.0 m from the ground. The tower is held in place and connected
to another structure (not shown in the diagram) via a horizontal steel cable. The
steel cable is 20.0 m from the ground. The position of the combined centre of mass
of the tower and the windmill is within the tower.

(a) State the principle of moments.

..........................................................................................................................

..........................................................................................................................

......................................................................................................................[1]

(b) It can be modelled that the wind force acts through the centre of the windmill.
When a wind force of 2000 N is acting horizontally on the windmill, the windmill
and the tower remains in equilibrium.

Calculate the magnitude of the frictional force acting on the tower by the
ground.

Magnitude of frictional force = ……………[2]


Tanjong Katong Girls’ School 3 Sec 4 Preliminary Examination 2019

www.KiasuExamPaper.com
416
(c) The combined weight of the tower and the windmill is 5000 N. The frictional For
force acting on the tower by the ground is 3000 N. Using a scaled diagram, Examiner’s
Use
determine the magnitude and the direction of resultant force acting on the
tower by the ground.

Magnitude = ……………

Direction = …………………………

[3]

Tanjong Katong Girls’ School 4 Sec 4 Preliminary Examination 2019

www.KiasuExamPaper.com
417
3 In the movie Spiderman, Spiderman attempted to use two flexible flag poles to shoot For
himself into the sky as if like a catapult. Fig 3.1 shows the set-up which Spiderman Examiner’s
Use
attaches his web to the top of the flag poles. He then moves backwards to his final
position (as shown in Fig 3.2) just before shooting up into the sky.

Fig 3.1 Fig 3.2

(a) Describe the energy changes that have occurred when Spiderman moves
backward from his original to his final position as shown in Fig 3.1 and Fig 3.2
respectively. Assume the web is not stretchable.

..........................................................................................................................

..........................................................................................................................

..........................................................................................................................

..........................................................................................................................

..........................................................................................................................

......................................................................................................................[3]

(b) The mass of Spiderman is 65.0 kg. When he released his foot grip from the
ground, he shot up to the sky with an initial speed of 40.0 m s-1. When he is
at his highest point, he had a speed of 15.0 m s-1. The work done against air
resistance during this journey is 5000 J.

(i) Determine the loss in kinetic energy from the time he leaves the ground
to the time he reaches the highest point.

Loss in kinetic energy = ……………[2]


Tanjong Katong Girls’ School 5 Sec 4 Preliminary Examination 2019

www.KiasuExamPaper.com
418
(ii) Determine the distance of Spiderman from the ground when Spiderman For
is at his highest point. Examiner’s
Use

Height = ……………[2]

4 (a) Fig 4.1 shows a plastic container with a lid cover. A lid tightener is used to
control the movement of the base of the lid. As a person flips down the lid
tightener, the base of the lid moves up and increases the volume of air in the
container. The container is air tight and it becomes difficult to remove the lid
cover.
lid tightener

lid cover
base of the lid

Fig 4.1

(i) Using the kinetic model of matter, explain why the air pressure in the
container decreases as the base of the lid moves up.

...............................................................................................................

...............................................................................................................

...............................................................................................................

...............................................................................................................

...............................................................................................................

...............................................................................................................

...........................................................................................................[2]

Tanjong Katong Girls’ School 6 Sec 4 Preliminary Examination 2019

www.KiasuExamPaper.com
419
(ii) Explain in terms of pressure, why it is difficult to remove the lid cover For
when the base of the lid moves up. Examiner’s
Use

...............................................................................................................

...............................................................................................................

...........................................................................................................[1]

(b) Fig 4.2 shows a hydraulic press that is used to lift up the body of the car during
the replacement of a car tyre. A force of 30 N is exerted on piston A.

handle

piston B with
a diameter of
15.0 cm
piston A with
oil a diameter of
5.0 cm

Fig 4.2

(i) Using the idea of molecules, explain why a liquid, such as oil, is used
in the hydraulic press.

……………………………………………………………..……………….…

……………………………………………………………..……………….[1]

(ii) Calculate the force exerted on piston B.

Force = ……………[2]
Tanjong Katong Girls’ School 7 Sec 4 Preliminary Examination 2019

www.KiasuExamPaper.com
420
5 Fig 5.1 shows a light dependent resistor (LDR) connected to a circuit. Fig 5.2 shows For
the relationship between the potential difference V across and the current I flowing Examiner’s
Use
through the LDR.

resistor A
V/V

resistor B

0
I/A

Fig 5.1 Fig 5.2

(a) Describe how the resistance of the LDR changes with light intensity.

………………………………………………………………………………………...

……………………………………………………………………………………...[1]

(b) Explain how the graph in Fig 5.2 shows that the LDR is a non-ohmic
conductor.

………………………………………………………………………………………...

………………………………………………………………………………………...

……………………………………………………………………………………...[1]

(c) The resistances of resistor A and B are 1000 Ω and 3000 Ω respectively.
When the resistance of the LDR is 2000 Ω,

determine, by showing detailed workings,

(i) the effective resistance of resistor B and LDR.

Effective resistance = ……………[2]

Tanjong Katong Girls’ School 8 Sec 4 Preliminary Examination 2019

www.KiasuExamPaper.com
421
(ii) the potential difference across resistor A. For
Examiner’s
Use

Potential difference = ……………[2]

(iii) the current through the LDR.

Current = ……………[2]

Tanjong Katong Girls’ School 9 Sec 4 Preliminary Examination 2019

www.KiasuExamPaper.com
422
6 A 2.4 kW water heater is connected to a 240 V main supply. For
Examiner’s
Use
(a) Calculate the current in the heating element of the water heater when it is
working normally.

Current = ……………[2]

(b) The water heater is protected by a 13 A fuse.

Explain how the fuse works.

………………………………………………………………………………………...

……………………………………………………………….………………………..

………………………………………………………………………………………...

……………………………………………………………….………………………..

……………………………………………………………….……………………..[2]

(c) The water heater has double insulation. Explain whether it is necessary for
the water heater to have an earth wire connected to the casing.

………………………………………………………………………………………...

…………………………………………………………………………….…………..

………………………………………………………………………………………...

…………………………………………………………………………….…………..

…………………………………………………………………………….…………..

…………………………………………………………………….………………..[2]

Tanjong Katong Girls’ School 10 Sec 4 Preliminary Examination 2019

www.KiasuExamPaper.com
423
7 Fig 7.1 shows the use of an electromagnetic relay switch to switch on another For
secondary circuit that is connected to a high voltage power supply. Examiner’s
Use

pivot point
A high voltage
power supply

soft iron core


Fig 7.1

(a) Explain how, by adjusting the resistance of a variable resistor, the motor in
the secondary circuit can be switched on.

……………………………………………………………………………….……......

………………………………………………………………………….……………..

………………………………………………………………………….……………..

………………………………………………………………………….……………..

………………………………………………………………………….……………..

…………………………………………………………………….………………..[2]

(b) State the pole at end A of the soft iron core when current is flowing through
the coil.

……………………………………………………………………………………...[1]

Tanjong Katong Girls’ School 11 Sec 4 Preliminary Examination 2019

www.KiasuExamPaper.com
424
(c) Fig 7.2 shows a simplified diagram of the motor which is connected to the high For
voltage supply. Points P and Q are two corners of a copper coil. Examiner’s
Use

copper coil
Q

permanent
magnet

Fig 7.2

(i) Name one electrical device that can be included into the circuit to
ensure that the coil in the motor will only turn in one direction.

……………………………………………………………………………...[1]

(ii) At a particular instant, the magnetic force acting on the wire between
points P and Q is in the upward direction. Using the idea of magnetic
fields, explain why this is so.

…………………………………………………………………………………

……………………………………………………………………….………..

……………………………………………………………………….………..

……………………………………………………………………….………..

……………………………………………………………………….………..

………………………………………………………………….…………..[2]

Tanjong Katong Girls’ School 12 Sec 4 Preliminary Examination 2019

www.KiasuExamPaper.com
425
8 Fig 8.1 shows a set-up with two coils of wire wound around a soft iron core. For
Examiner’s
User

coil B

soft iron
G
core
coil A

Fig 8.1

(a) State Faraday’s law of electromagnetic induction.

………………………………………………………………………………………...

………………………………………………………………………………………..

…………………………………………………………………….………………..[1]

(b) Explain why the galvanometer shows a deflection when the switch is just
closed.

………………………………………………………………………………………...

…………………………………………………………………….…………………..

…………………………………………………………………….…………………..

……………………………………………………………………….………………..

…………………………………………………………………….………………..[2]

(c) Suggest one change to be made to the set-up if

(i) the galvanometer is to have a continuous deflection at all times.

……………………………………………………………………………...[1]

(ii) the galvanometer is to show a larger deflection with the same power
input.

……………………………………………………………………………...[1]

Tanjong Katong Girls’ School 13 Sec 4 Preliminary Examination 2019

www.KiasuExamPaper.com
426
Section B [30 marks]

Answer all the questions in this section.


Answer only one of the two alternative questions in Question 11.

9 I-Fly is an indoor skydiving facility which uses high air speeds to keep a person For
floating in the air. Fig 9.1 shows a simplified setup of how high air speeds are Examiner’s
Use
projected onto the person (flyer). Air is drawn into the chamber using the wind
blade. The shape of the chamber allows air to move upward at high speed. The
speed of the air is regulated by the speed of the wind blade.

top chamber

Fig 9.1

Fig 9.2 shows the power of the wind blade and the average wind force acting on a
person.

Average Wind
200 400 640 800 900
Force / N
Power of Wind
2.0 3.0 4.0 5.0 8.0
Blade / MW

Fig 9.2

Tanjong Katong Girls’ School 14 Sec 4 Preliminary Examination 2019

www.KiasuExamPaper.com
427
(a) State Newton’s 1st Law of Motion. For
Examiner’s
User
...........................................................................................................................

.......................................................................................................................[1]

(b) Draw the free body diagram acting on the flyer when he is floating in the air.
Label and name all the forces. [2]

(c) Using the data shown in the Fig 9.2, plot a graph of the average wind force
acting on the flyer against the power of the wind blade in Fig 9.3 . [2]

average wind force / N

power / MW

Fig 9.3

Tanjong Katong Girls’ School 15 Sec 4 Preliminary Examination 2019

www.KiasuExamPaper.com
428
(d) A person with a mass of 75 kg has signed up to be a flyer. For
Examiner’s
(i) State the weight of the person. User

Weight = ……………[1]

(ii) Using the plotted graph in Fig 9.3, determine the minimum power of the
wind blade required to keep the person floating in the air.

Minimum power = ……………[1]

(iii) The power of the wind blade is adjusted to 5.0 MW. Using the data in
Fig 9.2, calculate the initial acceleration of the person.

Acceleration = ……………[2]

(e) Explain why it is important to have a safety net installed at the base of the top
chamber.

...........................................................................................................................

.......................................................................................................................[1]

Tanjong Katong Girls’ School 16 Sec 4 Preliminary Examination 2019

www.KiasuExamPaper.com
429
10 (a) Fig 10.1 shows a light ray travelling from diamond to medium X. The diamond For
has a refractive index of 2.4. Examiner’s
Use

medium X
boundary
diamond

normal
light ray
Fig 10.1

(i) Compare the refractive index of medium X to that of the diamond. Use
Fig 10.1 to explain how you reach the answer.

…………………………………………………….…………………………..

……………………………………………………….………………………..

…………………………………….…………………………………………..

…………………………………….………………………………………..[2]

(ii) If medium X is glass, the critical angle is found to be 39o.

1. State what is meant by critical angle.

…………………………………………………………………………………

………………………………………………………………………….…..[1]

2. A light ray strikes the boundary with an angle of incidence 42 o. State


and explain what will happen to this light ray.

…………………………………………………………………………………

…………………………………………………………………………………

…………………………………………………………………………………

………………………………………………………………………….…..[2]

3. If medium X is water which is optically less dense than glass, state, if


any, the changes to the critical angle.

……………………………………………………………………………...[1]

Tanjong Katong Girls’ School 17 Sec 4 Preliminary Examination 2019

www.KiasuExamPaper.com
430
(b) Fig 10.2 shows a slanted lens with a focal length of 2.0 cm. The height of the For
object is 1.5 cm. The intersection point between the horizontal line and the Examiner’s
Use
lens is the optical centre of the lens. The diagram is drawn to scale.

object horizontal
line

Fig 10.2

(i) Define principal axis.

………………………………………………………………………….……..

………………………………………………………………………….……..

…………………………………………………………………………..….[1]

(ii) In Fig 10.2,

1. sketch the principal axis and label the principal focal points of the
lens as f1 and f2.
[1]

2. sketch two rays from the tip of the object to locate the image of the
tip of the object. Mark this point with “X”. [2]

Tanjong Katong Girls’ School 18 Sec 4 Preliminary Examination 2019

www.KiasuExamPaper.com
431
Either
11 Fig 11.1 shows a car equipped with a reverse parking sensor at the car’s back For
bumper. Examiner’s
Use
back bumper reverse parking sensor

side view of the car back view of the car

Fig 11.1

The reverse parking sensor uses ultrasound to measure the distance between the
car’s bumper and nearby objects. The sensor is connected to a sound buzzing
system which will provide beep sounds in the car to alert the driver of the obstacles
while parking the car.

(a) Define ultrasound.

………………………………………………………………………………………..

……………………………………………………………………….……………..[1]

(b) Fig 11.2 shows a screen displaying the signal of one set of pulses picked up
by the reverse parking sensor.

emitted pulse returning pulse

Fig 11.2

The time interval between the two pulses is 800 μs. The ultrasound has a
frequency of 25 kHz and the speed of ultrasound is 330 m s-1.

(i) Using the idea about the molecular motion in air, describe what is
meant by “a frequency of 25 kHz”.

………………………………………………………………………………...

……………………………………………………………………………...[1]

Tanjong Katong Girls’ School 19 Sec 4 Preliminary Examination 2019

www.KiasuExamPaper.com
432
(ii) Calculate the distance (in cm) between the back bumper and the For
obstacle. Examiner’s
Use

Distance = ……………cm [2]

(iii) On Fig 11.2, sketch the returning pulse for the same emitting pulse if
the distance between the back bumper and the obstacle is smaller than
the value calculated in part (b)(ii). [1]

(c) Sound wave comprises regions of compression and rarefaction.

(i) Explain, in terms of pressure, the meaning of region of rarefaction.

………………………………………………………………………………...

……………………………………………………………………………...[1]

(ii) In Fig 11.3, line X represents the position of the air molecules in a
sound wave at a particular instant. The sound wave is travelling to the
right.

Fig 11.3

1. In Fig 11.3, mark a distance equal to the wavelength of the sound wave
on line X. [1]

2. In Fig 11.3, mark the position of the centre of all compressions after
another 1½ period on line Y. [1]

3. Describe how you obtained your answer in part 2.

……………..……………………………………………………………….…

……………………………………………………..……………………….…

………………………………………………………………………..…….…

…………………………………………………………………….…………..

Tanjong Katong Girls’ School 20 Sec 4 Preliminary Examination 2019

www.KiasuExamPaper.com
433
………………………………………………………………………..….…[2]
Or
11 (a) Fig 11.1 shows a simplified diagram of a power station that involves the For
combustion of natural gas to generate electricity. Water is directed into Examiner’s
Use
combustion chamber via a pipe.

steam out
combustion
chamber

pipe cold water in


Fig 11.1

(i) Suggest an appropriate material for the pipe. Explain your answer.

………………………………………………………………………………...

……………………………………………………………………………..[1]

(ii) “Using natural gas to generate electricity is an environmentally friendly


method”.

Do you agree with this statement? Justify your answer.

………………………………………………………………………………...

…………………………………………………………………….………..[1]

(iii) In 1.0 minute, 2.0 kg of water at 30oC is converted to steam at 100oC.


The specific heat capacity of the water and the specific heat latent heat
of vaporisation are 4200 J kg-1 oC-1 and 2.1 MJ kg-1 respectively.

1. Calculate the amount of thermal energy required to change 2.0 kg of


water at 30oC to steam in 1.0 minute.

Leave your answer to the appropriate number of significant figures.

Tanjong Katong Girls’ School 21 Sec 4 Preliminary Examination 2019

www.KiasuExamPaper.com
434
Amount of thermal energy = ……………[3]
2. The efficiency of the system is 80%. For
Examiner’s
Use
Calculate the power input.

Power input = ……………[2]

(b) Nuclear energy is considered as a good source of energy to generate


electricity, however, the gamma radiation produced is extremely dangerous.

(i) Explain why some countries continue to use nuclear energy to generate
electricity despite the danger.

………………………………………………………………………………...

…………………………………………………………………….………..[1]

(ii) Gamma radiation belongs to a family of waves. Name this family.

…………………………………………………………………….………..[1]

(iii) Suggest why gamma radiation is extremely dangerous.

………………………………………………………………………………...

………………………………………………………………………………...

…………………………………………………………………….………..[1]

 END OF PAPER 

Tanjong Katong Girls’ School 22 Sec 4 Preliminary Examination 2019

www.KiasuExamPaper.com
435
www.KiasuExamPaper.com
436
Solution to Sec 4 Physics Prelim Exam 2019 Paper 1

1 C 6 C 11 D 16 A 21 A 26 A 31 B 36 C
2 A 7 D 12 C 17 D 22 D 27 D 32 A 37 A
3 A 8 B 13 A 18 A 23 D 28 C 33 D 38 D
4 D 9 C 14 B 19 A 24 B 29 A 34 D 39 C
5 B 10 A 15 C 20 C 25 B 30 A 35 B 40 B

Qn Solution Ans
1 Thickness of a fingernail is approximately 1 mm.

Note: A student can use his existing ruler to estimate that the C
fingernail must be around 1 mm.

2 To measure a length of several metres, the measuring


uring instrument
ins
nstr
trum
ument will
have to be measuring tape. Other measuring g instruments
instrumen
entsts can
an only
measure up to 1 m.

To determine the volume of the pipe,, both external


exxte ern
rna
all and
and
n internal
intter
erna
nall
na A
diameter of the pipe is needed.ed. Thu us,
Thus,s the
he
e best
be es
st measuring
mea
me as
suring
g instrument
inst
in stru
str ment
will be vernier caliper. Labora
Laboratory
ato
tory
r micrometer
micr
c om
cro ete te
err sscrew
cre
cre
crewwg gauge
auge
au ge ccan
an only
up to 25.0 mm and onlyy for
fo
orr external
ext
xter
en na
al diameter.
diam
amet
am e er.

3 Maximum resultant
resultlttan
ant force
forcce occurs
occc
o cu
urrs
s when
whe
hen all
a l the
al th
he three forces
forrcess are
fo are in
in the
e direction
same direc
eccti
t on aandnd
n d tthe
he val
he a ue
al u iis
value s 3+
3 4+8++15
15 N
3+4+8+15 N..

Th
T hus,, option
Thus, o tion
op n D is
s po
p ssib
ble
possiblele rresultant
es
e sul
ulta
tant
nt fforce.
orrce
o c .

Byy e liim
miina
nati
tion
elimination, o , op
o tion
on
o
option n A wi
will
will b
bee the not
not be a possible
po value of the
resultant
resuulttant force.
fo
orc
rce.
A
e: We
Note: W e ccan
an a
an llw
wayys pa
always pairr u
upp two forces and determine the maximum
mini
niim
and minimummuum value.
valu ue. Thus,
T us if we pair up 3 N and 4 N forces, their
Th
ant forc
resultant ce wi
force willll be in the range of 1 N to 7 N. When this resultant
force is commbibined w
combined with 8 N force, then the total resultant force will be
ge of 1 N and 15 N.
in the range

4 When the car is accelerating, this means that the speed of the car is
increasing. This implies that the distance travelled by the car per unit
time (in this case, for every one second interval) should be
increasing.
D
Note: For option D,
the distance travelled in the 1st second (from t=0 s to t =1 s) = 80 m
the distance travelled in the 2nd second (from t=1s to t=2 s) = 110 m

www.KiasuExamPaper.com
437
5 Action-reaction forces must be acting on mutually opposite bodies
and cannot be on the same body.

For option A, the pulling force on the block is due to the rope. The
tension (always acting away from the body of interest) experienced
by the rope is due to the block. See free body diagram below
illustrating the two forces

pulling force

tension force

6 Acceleration of the car (through the journey)


= rate of change in velocity
= 10/2.0
= 5.0 m s-2

Using Newton’s 2nd Law,


C
Resultant force, Fnet
=mxa
= 1000 x 5.0
00 N
= 5000

7 The
T
Thhe question
question
qu on
n is
is fo
ffocus
ocuc s on
n the
tth
he effect
effffe
e ect
c of
of gravitational
gravittat
atio
iona
io nall field strength on
na
ma
m asss and
ass
mass and
n weight.
wei
eigh
ei gh
ghtt..

Mass
ss o
off a bo
b dy w
body wiillll b
will bee co
cconstant
onstaant a nd hence
and hen with the same amount of
plie
ed force
applied forc
fo rc
ce a
annd hence
and he
h ence resultant
resu
re sultltltant force
su fo acting on the block, the
D
elera
ra
ati
tion
accelerationo ooff tth
he bl
the b ock
oc
block k wi
will be constant.

Note: Althou
o gh tthe
ou
Although he peri
period of pendulum T is not dependent on the
dependent on gravitational field strength g. T2 = 4 S2 l / g.
mass, it is de
depend

8 Concept of flotation: Object or liquid of a smaller density will float


above the liquid of larger density.

Thus, liquid Q must have the smallest density and liquid S must have
the largest density. B

Since liquid R floats on top of object P, density of object P must be


larger than liquid R.

www.KiasuExamPaper.com
438
9 Let the length of ladder by l m. By principle of moment, taking
moment about point X,

anti-clockwise moment due = clockwise moment due to weight


to force F of the ladder

F x l sin 30o = 500 x ½ l cos 30o


F
F sin 30o = 250 cos 30o
C
F = 433 N

Note:
(1) Perpendicular distance between line of action of the force F and
the pivot point X is l sin 30o
(2) Since the c.g of the ladder is at the mid-point of the ladder,
perpendicular distance between line of action of the force F and
an the
pivot point X is ½ l cos 30 o

10 As more water is added to the bottom, the combinedbined centre


cenntre of gravity
grav
e water will
will be raised as the centre of gravity of the wililll be raised.
rai
aise
sed.
se

There is no change in the base area. rea. Thus, with h a higher


wiith gher centre
hiig cent
ce ntre
nt re of
A
gravity, the bottle with water will be less
less stable
le stab
tta ble as
abl as the
he line
the e of aaction
c ion of
ct
the weight of the bottle with water
waatete is likely
ter like
ikely to
ik be
to b outside
e ou
o uts
tsid
idee th
the base
e ba
basse area
of the bottle when displaced
isplac ed slightly.
acced
a sliigh
ghtl
t y.

11 The pin is not moving.


mov
oving. Th T
Thus,
hus
us, th
tthe
he pin
piin is
p is a
att eq
equilibrium.
qui
u libriu
um. The
e fo
forc
force
rce
rc e of
the pin
n is equ
equal on
q al o n th
tthe
he finger
fing
ng
ger
e and nd on the
nd e th
tthumb.
hum
umb.

Sincee pr
p
pressure
essu
su
s ure
r = fforce
orrce / area
o arre
ea of
of ccontact,
on
o nta
tacctt, and th
thee area
a ea of contact
ar D
b
beetw
twee
een the pin
between piin and
p an
and the thumbth
hum
umb is is ssmaller
maller
ma er a s co
as compa
compared to that at the
fing
nger
ngerr, th
e
finger, tthe
he p
prresss
su
ure o
pressure off tth
he pi
the p in is llarger
pin arge
gerr on tthe
ge he th
thumb.

12 The
he pressure
p re
pr es
sssu
ure
re o
off the
tth
he gas
gas supp
ga supply
pply
ly
= pressure
resssu
sure
ed due
ue tto
ue o a liqu
liquid
id ccolumn
quid
qu olumn of 18 cm + atmospheric pressure.

The new
ew pressure
presssu re of the gas supply
sure
= pressure due
ure d ue to a liquid column of 20 cm + atmospheric pressure.
C
Thus, the difference
d in level X and Y = 20 cm

Note: Level X will have dropped by 1 cm and level Y have risen by 1


cm.

13 As air rushes in, the external pressure acting on the marshmallow


increases.

The resultant pressure hence produces a resultant force acting


inward towards the centre of marshmallow. The size of the
marshmallow decreases. A

With a reduction in the size of the marshmallow, the area of contact


between the gas particles in the marshmallow and the internal
surface of the marshmallow decreases. Pressure in the
marshmallow increases.
www.KiasuExamPaper.com
439
14 Fixed points must be easily reproducible and obtainable through any
physical processes. Two fixed points are required so that the length
of the liquid can be divided into equal intervals.
B
Note: Option C is not entirely correct as the fixed point must be below
the boiling point so that the substance remains in liquid state.

15 The two given temperatures are not 0oC and 100oC. Thus, the
temperature formula cannot be used.

Using the underlying concept that the change in the thermometric


property is directly proportional to the change in the temperature.

change in resistance ‫ ؠ‬change in temperature


( 80 – 20) Ω ‫ ؠ‬90oC - 10oC
60 Ω ‫ ؠ‬80oC

Using 10oC as the reference temperature, C

When resistance changes from 20 Ω to 100


00 Ω, chan
change
ange
an ge in
in resistance
res
re sistance
= 80 Ω.

mperatur
urre
u
ure
Corresponding change in temperature
= 80 / 60 x 80
= 107 oC

Thus, the expect


expected
ce
ct ed
d tem
temperature
mpe
pera
rattu
ure
r =1
107 0 = 117 oC
07 + 10
07

16 For
or ther
thermocouple,
errmo
oco
co ple
coup l , to
o have
hav
ave deflection
de
d effllec
e tiio
on
n for
for
or galvanometer,
gal
alva
vanome
ete
terr, two
two
w
condit
iittio
conditionsons m u st b
must beem ett:
met:

(1
1) T
(1) Th he
Thereerre is
is a ttemperature
empera
em ratu
ra ture
re d ifferencce be
difference betw
tween tthe hot and the cold
tw
between
A
juncctiions.
junctions.
2) Th
(2) Thehe wiw
wire
ire
es cco
wires on
nnnec
ecttiing att th
connecting thee tw
two ju
junctions must be of different
tal.
metal.

17 The focus
cus of tthe
he q
question
uestio is the metal spoon which is gaining heat
from thee hot
hoot water
water which
w has a higher temperature. Since the spoon
is a metall which a good conductor of heat and the spoon is in
physical contact with the water, the main process of heat transfer is
conduction. D

Note: The initial temperature of the metal spoon is assumed to be the


same as the surrounding air.

18 In this case, the heat capacity of the object is given, rather than the
specific heat capacity.

Using Q = C Δθ, A

Amount of heat, Q = 500 x (40 – 30) = 500 x 10

www.KiasuExamPaper.com
440
19 Specific latent heat of vaporisation
= amount heat absorbed by 1.0 kg of liquid as it changes from liquid
to gas at boiling point
= amount of heat absorbed by 1.0 kg of gas as it changes from gas to
A
liquid at condensation point.

Boiling point = condensation point.

20 Rate of evaporation depends on how readiness the liquid molecules


at the surface of the liquid is able to leave the liquid, and this
depends on whether the liquid molecules have sufficient energy to
escape from the surface of the liquid.
C
In this case, atmospheric pressure and temperature of the liquid are
the only two factors that affect the rate of evaporation.

21 In this case, it is noted that p is not the maximum height of the w


water
surface at a particular point. Thus, p is the displacement
ementt of the
the
h
water.
A
This is the displacement-time graph, and hence the
the graph
grap
gr a h only
only
provide the information on period (the time taken
ta
ake
en for
ffo
or the
t e particle
th pa
art
rtic
icle
ic le tto
o
complete one oscillation).

22 The wavelength hass increased


inccrre
ea
asse
edd as
as it travels
tra
avve
els
l from
fro
om regions
regi
gion
gi ons X to Y.
on Y.
rema
mains constant
ma
Frequency of wave remains consstaant (as
as
a s tthere
here is no
o cchange
hangge in
n tthe
he
source).

Using v = f x λ, this
th
h s iimplies
mplil es
mp es tthat
ha
hat thhe s
the sp
peee
ed of
speed of the
the wave
wav
ave
e increases.
increa
in
D
Thus,, region
r gion
re on
n X iss shallower
shallo
sh ow
we e
err an
a
andd region
rre
eggiion
on Y.
Y.

Us
sin
ng th
Using tthe
he concept
conccep
co ept off refraction,
ref
efra
ract
ction, the direction
dir
irec
ecti
ec tion of wave travel will bend
ti
away
ay from
ay fromm the
the normal
th no
orm
rmaall as
as speed
sp
s peed of the
the wave
wav increases.

23 Strategy:
ategy
gy:
gy
(1) Sket
tcch
Sketchh a di
d rect
ctio
ction
io
direction n of wav
wave travel and use the concept of reflection
(2) Wavefronts
aveffron
nts aalways
lways p
lw perpendicular to the direction of wave travel.

corresponding
rres reflected
direction of wave travel

one of the original


directions of wave

www.KiasuExamPaper.com
441
24 Light ray bends towards the normal as it travels from optically less
dense medium to more dense medium, and vice versa.

25 Using infra-red radiation and ultra-violet as reference, red light will


have a larger wavelength and smaller frequency than the violet light. B

26 The electromagnetic waves must be safe for usageage and h


hence
ence
ce must
mu
quency).
not have high ionisation energy (or high frequency).
A
magnetic wav
Note: Ultrasound is not an electromagnetic avve.
e.
wave.

27 Due to atoms or moleculeses are e closer


c oser
cl err tto
oeea
each
acch
h other
oth
ther
er in
in solid
solililid
so d as
he air,
compared to that in the aiir,
r, the
th
hee speed
spe
p ed ofof sound
s un
so nd is
is larger
lar
arrge
gerr in ssolid
olid
ol id than
n
in the air.
D
er, as ther
However, erre
e re is
there is no change
h e in
in th
he source,
the so
our
urc
cee, the frequency
f eq
freque
uenc
ue ncy
nc y of the
und remains
sound re
ema
mains uunnchhan
ange
ged
unchanged. d..

28 The
Th
T he first
ffiirs
rst sound
soun
un
und
nd heard
heear
a d byy the
the
h engineer
en
nggin
neee
er is the
he sound
sou
ounnd that
th travels
di
irre
ect
ctlyy ffrom
directly rom th
ro the
he so
ssource
ource
ce tto
ce o hi
h im.
him.

Distan
an
a nce ttravelled
Distance raveellle
eddb
byy tth
his sou
this und tto
sound o reac
reach the engineer
2 2
= ඥ30 +40
+ 40
40
= 50 m

ond
The second d sound
sou
o nd heard
h by the engineer is the sound that reflected
rtical wall on the right.
on the vertical

Distance travelled by this sound to reach the engineer (See below)


2 2
= 3 x ඥ10 +40 C
≈ 124 m

10 m

10 m
10 m

www.KiasuExamPaper.com
442
Time lapse between the two sounds
= ( 124 – 50 ) / 300
= 0.25 s

29 Initially, the force applied on the object is equal to the frictional force
(since the object is moving with constant speed).

When the negative charged object enters the electric field, there will
be a downward electric force acting on the object and pressing the
object against the rough surface. The frictional force acting on the
object will increase.
A
By Newton’s 2nd Law, as there is a now resultant force acting on the
object opposing the motion. The object will decelerate.

e
Note: The direction of electric field shows the direction of the electric
force acting on the positively charged object.

30 When an object is neutrally charged, induced separation n of


of charges
cha
h rge
harged object.
can cause an object to be attracted to a charged obj
bjec
bj ect.
ec t. On the
the other
othe
rged can also
hand, an object that is oppositely charged als
lsso be attracted
attra
act
cted
ed to
to a
charged object. A

Thus, only repulsion can


n determine
determ
rmine the
rm th
he state
s at
st ate of
of charge
cha
harrg
hargge of an
an object.
ob

31 Using definition
on of elec
electromotive
ctrromot
otiv
ivve ffo rce,, ε = W / Q
force,
orc

Amount
unt of
of electrical
e ectr
el tricca
tr all energy
ene
n rg ttransferred,
rgy trran
ansf
s erred,
d,, W
=εxQ B
= 35
3
3500 x 1000x1100--33
= 355J

32

+2.0
0V

Potential at P
A
= 2.0 – potential difference across 5.0 kΩ
= 2.0 – 1.0
= 1.0 V

Potential at Q
= 2.0 – potential difference across 2.0 kΩ
2.0
= 2.0 – x 2.0
2.0 + 3.0
= 1.2 V

Potential difference across P and Q = 1.2 – 1.0 = 0.2 V


www.KiasuExamPaper.com
443
33 The symbol shown is the symbol for thermistor. At high temperature,
thermistor will have low resistance.

Total effective resistance in the circuit = 3.0 + 2.0 = 5.0 Ω


D
Using R = V / I,

ammeter reading = R / V = 6.0 / 5.0 = 1.2 A

34 This is one of the method to demagnetise a magnet. When the


magnet is dropped onto the floor, the energy absorbed by the magnet
will disorientate the atomic magnets and hence the atomic magnets
D
will no longer align themselves in the same direction. Thus, the
magnet will lose magnetism.

35 This is a recall question. Flow of positive charge is equivalent ent to the


ositive charge
flow of conventional current and hence the motion of positive ch
wiith the
will create a magnetic field. Using right hand grip rule with the thumb
thu
B
etic field will
pointing into the paper, the direction of magnetic willl be
b in
in clock-
cloc
wise direction (as given by the fingers).

36 This is a recall question. The ultimate


timate function
functio io
on of
of ssplit
pliti ring
pl g
commutator is to ensure the e direction
directio
on of
o current
cu
urrre
rent in
in the
the coilill is
th is al
alwa
w ys
always
in the same direction soo that th
he coil w
the illl al
will lwa
ways
alwaysys tturn
urn
ur
rn in tthehe ssame
ame
am
s.
direction at all times.
C
Note: A motor iss to
to convert
conv
nvve
errt el
e lec
ectr
tricca
electrical all e nerg
ne gy to
energy t mechanical
meccha
hanica
al energy,
ener
en ergy,
er
ratherr than
n producing
prod
duccin
ing elec
ecctr
trical
electricalal e
al nergy..
energy.

37 Direction
D
Di
Direct
i c
cttio
on off ccurrent
urre
enntt when
en
n switch
sw
wiitch
ttc
ch S iss closed:
cllo
osed:

N S N S

Both iron rods will be magnetised with opposite polarities as shown


above. Since unlike poles attracts, the spacing at X will decrease.

38

N
The induced current in the solenoid will flow such that it can induce a
www.KiasuExamPaper.com
444
N-pole at X when the magnet moves into the solenoid at X.

Thus, to create the same deflection, the cause should be able to


create a N-pole at X, that is, pulling a S-pole of the magnet out from
X and pulling a solenoid away from a S-pole.

39 By Faradays’ Law of electromagnetic induction, the magnitude of the


induced emf is directly proportional to the rate of cutting of magnetic
field lines by the conductor.

Thus, when the number of coils increases to three times as before,


C
the induced emf will be 3V.

When the frequency of rotation is now 2F (twice of the original rate of


rotation), the induced emf will be 2 times of 3V, that is 6V.

40 Ns Vs
Using = ,
Np Vp
B
Voltage across secondary coil, Vs = 100
0 / 50 x 25
5 = 50 V

www.KiasuExamPaper.com
445
Solution to 2019 Sec 4 Physics Prelim Paper 2

Section A

Qn Solution Marking Scheme


1(a) Due to inertia, the sandbag will have a reluctancy to change B1
its state of motion and will appear to move with an initial
velocity of 15 m s-1.

(b)
B1 – straight line
with gradient of 10
m s-2

B1 – coordinates of
a
al
alll the
th critical points.

(c) Distance travelled


= area under the v-t graph
ph from t = 0 s to t = 1.5
5s
= ½ x 15 x 1.5
≈ 11 m (also accept
cept 11.3 m
m)) B1

(d)

distance
di
d is
sttaan
n
nce
ce ttravelled
ce rave
ra velled
dbbyy hot
hot air
ba
b alllloo
balloon on
balloon
balloo
oon
oo
sandbag
sa ndba
ndbag
part (c)’s answer
pa
t=0s t=1
1.5
.5 s t=3
3.0 s
At t = 3.0
0 s, th
tthe
he sand
sandbag
ndba
nd bag
ba g is ba
back at the original drop off point.

tweeen the sandbag and the hot air balloon


Distance between
= distance travelled
vel by the hot air balloon from the original M1 – student shows
drop off point attempts to find this
= 3.0 x 15 value
= 45 A1

2(a) Principle of moment states that when an object is in B1


equilibrium, the total clockwise moments about a point is
equal to the total anti-clockwise moments about the same
point.

www.KiasuExamPaper.com
446
(b) By principle of moment, taking moment about the point at
which the steel cable is connected to the windmill,

fr x 20.0 = 2000 x (30.0 – 20.0) M1


A1
fr = 1000 N

(c) Normal reaction force on the tower = 5000 N

Frictional force on the tower = 3000 N (to the left)

Scale: 1.0 cm to 500 N

M – correct
M1
drawing
dra

A
A11 – correct
magnitude
magnit

A1 – correct
corre
direction
di
Rf
((11.7
(1
11.
1.7 cm
cm)
m)
Normal al
fforce
fo rce
acting
a
ac ting oonn
tower,
towe er,
r
5000
50 00 N
((10.0
10.0 cm)

59o

fricti
frictional force, 3000 N
(6.0 cm)

Resultant force = 11.7 x 500 = 5850 N


(also accept 5550 N to 6150 N)

Direction = 59o clockwise from the frictional force


(also accept 56o to 62o)

www.KiasuExamPaper.com
447
3(a) The chemical potential energy possessed by Spiderman is B1
converted to work done on the ground by his foot as
Spiderman is walking backwards.

This energy is then converted to elastic potential energy of the


flag pole. B1

The total energy in the system remains a constant. The loss in


chemical potential energy of Spiderman is equal to the gain in
the elastic potential energy of the pole. B1

(b) Loss in kinetic energy

= initial kinetic energy – final kinetic energy M1

= ½ x 65.0 x 40.02 - ½ x 65.0 x 15.02

= 44687.5

≈ 44 700J A1 – acce
accept only 3
s
sf

(c) By conversation of energy,


ergy,,

loss in kineticc energy


gy = gain
gy gaiin in
in GPE
GPE
PE
+ wo
w
workrrk
kddone
on
ne agains
against
st air
air re
ai resistan
resistance
nce M1
M

4468
687 5 = 6
687.
68
44687.5 5.0 x 10
65.0 10 x h + 5000
500
50000

h≈6
61
1..1
61.11m A1

4(a) As the
e base
bas
ase of
as o tthe
he llid
he id
dmmoves
ovess u
up,
p, tthis
h s inc
hi increases the surface B2 – student’s
(i) wh
hiic
ch the
area at which the air
th aiir molecules
a moole
ecu
cule
le
es hitting
hittin the internal surface of answer contains all
aineer.
the container.r. the three points

The numberr off air


ai molecules
mole hitting per unit area of the internal B1 – student’s
ntain (that is, the frequency of collisions of the
surface of container answer contains all
air molecules with the internal surface of the container) one to two points
decreases.
B0 – student’s
Since the amount of force exerted by each air molecules on the answer does not
internal surface of container remains unchanged during contains any of
collision, the pressure of the air in the container decreases. these three points

Note:
‘–ƒŽˆ‘” ‡‡š‡”–‡†„›‘އ —އ•
”‡••—”‡ ൌ
‘–ƒŽ•—”ˆƒ ‡ƒ”‡ƒ

‘” ‡‡š‡”–‡†„›‘‡‘އ —އ ൈ ‘–ƒŽ‘Ǥ ‘ˆ‘އ —އ•
”‡••—”‡ ൌ
‘–ƒŽ•—”ˆƒ ‡ƒ”‡ƒ

www.KiasuExamPaper.com
448
”‡••—”‡ ൌ  ‘” ‡‡š‡”–‡†„›‘‡‘އ —އ• ൈ ˆ”‡“—‡ ›‘ˆ ‘ŽŽ‹•‹‘

(ii) The atmospheric pressure acting on the lid is much larger than B1
the pressure of the air in the container. A large upward force is
needed to overcome this pressure difference.

www.KiasuExamPaper.com
449
(b) Molecules are closely packed due to strong intermolecular B1
(i) forces of attraction, however, the molecules are still free to
move.

(ii) Pressure on piston A = Pressure on piston B


FA / AA = FB / AB
30 FB M1
=
S(2.5)2 S (7.5)2
FB = 9 x 30
FB = 270 N A1

5(a) As the light intensity increases, the resistance of LDR B1


decreases or vice versa.

(b) The graph does not shows a straight line passing through the B1
origin, that is, potential difference across LDR is not directlyy
proportional to the current flowing through it.

(c) Resistor B and LDR are connected parallel


allel to each o
other.
ther
th er..
er M1
(i)
1 1 1
= +
Reff 3000 22000
000

1 1
= A1
Ref
efff 1200
e 1200
12

Ref
efff = 12
e 1
1200
200 Ω

(ii) Using
sing
gppo
potential
ote
tent
ntial
iia
a divider
diivviide
der concept,
co
onc
n ce
ep
pt,
t,

acro
osss
p.d acrosss resistor
res
es
e
esisto
s or A

1000
00
= ×9
×9.0
×9 M1
1000+1200
200
0

≈ 4.1 V A1 - accept 2 sf
only

(iii) p.d across LDR = 9.0 – 4.1 = 4.9 V M1

Using R = V / I,

Current through LDR


=V/R
= 4.9 / 2000 A1 – sf must be
= 0.00245 A consistent with part
≈ 0.0025 A (ii)

www.KiasuExamPaper.com
450
6(a) Using P = I V,

Current through the heating element


=P/V
= ( 2.4 x 103 ) / 240 M1
= 10 A A1

(b) When there is a current exceeding 13 A, the fuse will be B1


heated up and melt when the melting point is reached.

This cause the circuit to be an open circuit and current will no B1


longer flow through the appliance. Hence, this protect the
appliance which can be damaged due to over-heating.

(c) When the water heater has double insulation, this means that B1
the outer-casing is made of non-electrical conducting
g material.
materi
rial
al.

Hence, even if there is an electrical fault such thatt llive


ive
iv e wire
w re
wi e
touching the casing, the casing remains ns at low potential.
potten
enttial
tial
al. A B1
e ectriccl shock.
el
person touching the casing will not get electricl shoc
sh o ck
k.. Thus,
Thuss,
earth wire is not required to be connect
ted
ed to
connected o the
tth
he casing.
casi
ca sing
ng.

7(a) By adjusting the resistance


tanceeooff vvariable
aria
ar abl
ble resistor
resiis
sttor to a smaller
smal
alle
al er B2
B 2 – student’s
stud
st uden
e t’s
crease the
value, this will increase e current
currenen
e nt flowing
flow
fl
low
owing
g through
through the
th e coil
co answ
an swe
answerer contains
cont all
ound the
wounding around th
he soft
s ft iron
so iro
ron core.
co
corre
e. the three
th thre points

Thee increase
increa
ea
e ase iin
n the
t e current
th cu
urr
rre
ren
ent will
willll iincrease
wi ncreas
nc asse th
tthe
he mmaagenttic ffield
magentic ield
ie d B1 – student’s
streng
ngth in
ng
strength in the coil
co
coili and
and
d the soft
soft
ft iron
iro
ron
on core
co
c ore
e will
will be
e magnetised
mag
agnen tised answer contains all
nd become
and be
b ecco
ome
m a st stronger
trro
ongger
e mag
magnet.
ag
gneet. t. one to two points

Due to induced
ind
nduced d magnetism,
magaggn
ne e
eti
tiis
sm
m, one end ennd of the
the
h iron
ir armature will B0 – student’s
tract
cted
ct
be attracted ed towards
tow
owa arrdss tthe
he
h e sof
softft ir
iron
on ccore
ore a
and the turning effect answer does not
on the iron naar
rmattuure
ure
armature re will
willl close
clos
cl ose
os e the ccontacts and close the contains any of
onne ecti
ec
circuit connectingting
n to the
the mo m
motortor and motor is switched on. these three points

(b) South pole. B1

Note: Student needs to first establish the direction of current


flowing in the coil and then apply right hand grip rule.

(c) Diode B1
(i)
Note: Diode only allows current to flow in a single direction.

www.KiasuExamPaper.com
451
(ii) The magnetic force on the wire PQ is in the upward direction B1
as the magnetic field strength below the wire PQ is larger than
that above the wire PQ.

The difference in the magnetic field strength is due to the B1 – student is


interaction of the magnetic field due to the current that is required to state the
flowing from Q to P and the magnetic field due to the direction of the
permanent magnet. current in wire PQ

8(a) Faraday’s law of electromagnetic induction states that the


magnitude of the induced emf is directly proportional to the B1
rate of cutting of magnetic field lines by the conductor.

or

Faraday’s law of electromagnetic induction states thatt the


onal to the
magnitude of the induced emf is directly proportional he
rate of change magnetic flux linkage with the conduct
conductor.
tor
or.

(b) When the switch is first closed, the increase


incre
eas
a e in the
the
e current
cu
urrre
rent
n B1
flowing through coil A result in an increaase
increase e in
n th
tthe
he ma
m
magn
agn
gnete ic
magnetic i
field strength experienced by coil BB..

By Faraday’s Law, w, there iss aan


n in induced
ndu
uceed emf
emf in
n coil B. Since
Sinc
Si ncce it B1
is a closed circuit between
rcuit be
b etw
tween co ccoil and
oiill B a d the
nd he galvanometer,
the gal
a vaano
nomete er, there
theree
is an induced
ucedd current
c rren
cu ennt flowing
flow
flowing g through
thro
th oug
ugh galvan
galvanometer
an
a nom
ometer which
whiich
shows
ws a de
deflection.
eflec
ecti
ection
tio .

(c) The batteries


battterriie
ba es is rreplaced
ep
e pla
acce
ed byy a
ann alternating
a te
al errn
nat
atin
ng current
curr
curren
rrent supply.
en sup B1
(i)
o acc
Also ccep
cc ep
ept any ch
accept chan
an
a nges
changes ge
g es thha
att can
that a p rodu
roduce a continuous
du
produce
ging
changingg magnetic
mag
agn
agne
ne
eti
tic field
ffiie
elld in
i coil
coi
oill A.

www.KiasuExamPaper.com
452
(ii) One of the following: B1

- decreasing the number of turns in coil B


(explanation: use Ns/Np = Vs/Vp and P = IsVs, since Vs
decreases)

- increasing the number of turns in coil A


(expalanation: use Ns/Np = Vs/Vp and P = IsVs, since Vs
decreases)

- Use a more sensitive galvanometer


(explanation: greater deflection for the same amount of
current flow through the galvanometer)

- Use connecting wire of a lower resistance


(explanation: will give a larger amount of current flow for the
e
same power output, use P = I R) 2

(Do not accept open and close the switch ch faster as this thi
h s will
will
not affect the rate of cutting of magnetic
netic field lin
lines
ness bby the
y tth
he
conductor. The mangetic field is produc ced a
produced att th
tthe
he in
iinstant
nst
stan
ant whw
whenen
the switch is just close)

www.KiasuExamPaper.com
453
Section B

Qn Solution Marking Scheme


9(a) Newton’s 1st law of motion states that an object will remain at B1
rest or continue to move with a constant speed along a
straight line unless a resultant force acts on the object.

(b)
upthrust / upward B1 – students
force / wind force name the two
forces correctly

B1 – both arrows
m
must have the
sam
same length
weight of person
erson

(c)

B1
B 1 – correct
co
orr
r ect
plotting
plot
otti
ting
ng of poi
points

B1 – correct shape
of graph (no marks
if student draws a
straight line)

(d) 750 N B1
(i)
(ii) From the graph, B1 – allow ecf from
part (c)
the power required = 4.6 MW

www.KiasuExamPaper.com
454
(iii) Corresponding wind force = 800 N

Using Newton’s 2nd Law,

Fnet = m x a

800 – 750 = 75 x a M1

a ≈ 0.067 m s-2 (2 sf) A1 – accept only 2


sf

(e) As the wind force can be smaller than the weight of the
person, the person will descend vertically and a safety net will B1
be able to prevent the person from any impact with the base off
the chamber.

10(a) The light ray is moving away from the normal as it acrosss the
the M1
(i) ght ray in tthe
boundary. This implies that the speed of light he
he light ray in
medium X is higher than the speed of the n ththe
e
diamond.

Thus, the refractive index of medi


diium
d
dium
mediumm X is
s smal
ssmaller
smmallle
ler th
tthan
tha
han
an o
off th
thee A
A11
diamond.

(ii) 1. Critical anglee is the


e an
a
angle
nglg e of
of iincidence
ncciid
n deen
nce
ce wwhen
he
h en the
th
he light ray
ray is B1
B 1
ng from
travelling m an o ptic
pt ica
allly
optically ly d ense
en
denserser medium
ser m dium
me m to
to a less
less
le
e dense
den
ense
en se
dium and
medium and
n the
the angle
ang
ng
n gle
le ofof refraction
re
refr
efr
f ac 90o.
acttiion iss 90

(ii) 2. As tthe
he
h e angle
ang
ngle
l of incidence
inc
in ciide
c dence iss larger
lar
a rggeer than
tth
han
a the
the
he critical
cri
riti
tica
tic l an
angle, the B1
ht ray
light ray w
will
wi
illll g
go
o th
tthrough
hro
rro
ough h to
ttotal
ota
tal internal
tal iin
ntte
ernal reflection.
ref
efle
ef le
ect
ctio
i n.

Usingg la
law
aw of rreflection,
effle
e ect
ctiio
on
n,, tthe
h a
he angle
ngle
ngle o
off refl
reflection will be equal to B1
gle of
the angle of incidence.
inc
n iddeennce.

(ii) 3. Critical angle


ngle is
s sm
smal
smaller than 39o.
alller th
al B1

Note: For thee sam


same angle of incidence, the light ray will bend
more away from the normal if medium X is water. Thus, this
implies that the critical angle will not need to reach 39o for
angle of refraction to be 90o.

(b) Principal axis is defined as a straight line passing through the B1


(i) optical centre and perpendicular to the vertical plane of the
lens.

www.KiasuExamPaper.com
455
(ii)

f2

f1

Marking points:

1. Correct construction of principal


ipal axis and
d la
labelling
abe
bellllin
ing of B1
B 1 mark
focal points

B2 marks
mark
ma rks
s –all 4
2. - Ray
y from obj
object
bjjec
b e t pa
passin
passing
ing
ing thro
through
oug
ugh optical centre
cen
centre correct
corr
co rrec
ectt marking
mar
point
po
- Ra
R
Rayy parallel
pa
p ara
rallllel
rall e to
o principal
prrinci
p cip
ci pa
al axis a
and
nd
n d passing
passing
g
through
th gh focal
hroug foca
fo
occa point.
al po
poinnt.
t B1 marks – only 2
or 3 correct
- Diirre
ecttio
Direction on of rays
ray
ayys from
ays ffrrom
m object
obj
bjec
ectt
(allows ecf from
- Lab
a eelllliing
Labelling ng of
of iimage
ma
m age X part (ii)1.)

www.KiasuExamPaper.com
456
Either
11(a) Ultrasound is defined as sound with a frequency that is greater B1
than 20 000 Hz.

(b) It means that for every one second, the particle will make B1
(i) 25000 complete oscillations about its undisturbed position.

Note: Do not accept the source generates 25000 completes


waves in one second.

(ii) Distance between the obstacle and the back bumper


= ½ x total distance travelled by the ultrasound M1
= ½ x ( speed of sound x time taken )
= ½ x 330 x 800 x 10-6
= 0.132 m
= 13.2 cm A1

(iii)
B1 – th the returning
p ulse sho
pulse should have a
larger
la a
amplitude
a nd nearer to the
and
emititte
terr pu
emitter p lse.
pulse.

(c) Rare
Rarefaction
rre
efa
f ctctio
on is tthe
he
he reg
region
eg
e gio
i n in tthe
he ssound
he ound
ound w wave
av
a ve wh
whic
which
ich
ic h ha
has th
the B1
(i) owe est
lowestst air
airr pressure
pre
r ssur
urre (the
u ((tthe
h adjdja
djaccen
ent particles
adjacent part
pa rticle
es ar
are
e th
the fu
furthest
apart).
art)t)).

(ii) one
o
onne wavelength
wavele
eng
ngth
th
1. B1

2.
B1

3. After 1 period, all the particles will have returned back to the
original position. When another ½ period has progressed, all B1
the rarefaction zone will be changed to compression zone.

The particle at the centre of rarefaction would have moved to B1


its furthest point and returned back to its undisturbed position
and will now be at the centre of compression zone.

www.KiasuExamPaper.com
457
Or
11(a) Copper metal.
(i)
Copper is a good conductor of heat and resistant to corrosion. B1

(ii) Agree. Comparing to generate electricity using burning of Either one.


fossil fuel, the by-products generated are less harmful.
B1
Disagree. Natural gas is a non-renewable source of energy
and will be depleted. There are by-products released to the
environment during the combustion and this can pollute the
environment.

(iii) Amount of thermal energy to change 2.0 kg of water to steam


1. in 1.0 minute

= amount of energy to increase the temperaturee of wate er


water
00oC to ssteam
+ amount of energy to convert water at 100 team
te am

= m c ∆θ + m lv

= 2.0 x 4200 x (100 – 30) + 2.0 x 2


2.
2.1 06
.1 x 10
M1 – co
correct
corrrect m c ∆θ
≈ 590 000 (2 sf) + 4 200 00
0
000
00 M1 – corre
correct m lv

= 4 790 000 J A1 - accept 3 sf


only.

2. Efficiency
Effic
cie
cienncy
cy = use
useful
effu
ul p
po
power
owe
w r ou
o
output
utp
put
ut / p
power
ow
o werr iinput
nput
np ut x 1
100%
00%

80%
80
0% = us
useful
sef
e ul
u ppower
ow
o wer
er o
output
uttpu
u putt / po
powe
power
w r inpu
we input x 100% M1 – finding useful
power output
Therefore,
fore,
e,,

ut = ( 100
power input 100 / 80
80 ) x useful power output

= (10
(100 / 80) x ( 4 790 000 / 60 )

≈ 100 000 W (2 sf) A1 – 2 sf

(b)(i) A small amount of the raw material (radioactive substance) B1


can help to generate a large amount of electrical energy.

(ii) Electromagnetic wave B1

(iii) Gamma radiation has extremely high frequency and hence B1


ionising power. Exposure of gamma radiation can lead to
mutation of cells, cause cancer and lead to death.

www.KiasuExamPaper.com
458
www.KiasuExamPaper.com
459
Name Class Index Number

UNITY SECONDARY SCHOOL


PRELIMINARY EXAMINATION 2019

SECONDARY FOUR EXPRESS

PHYSICS 6091/01 18 SEPTEMBER 2019

PAPER 1 1 HOUR

Additional Materials : Optical Answer Sheet

READ THESE INSTRUCTIONS FIRST


1. This paper consists of 40 Multiple Choice Questions.
2. Answer all questions on the Optical Answer Sheet (OAS).
3. Write your name, class and shade your register number in the spaces on
the OAS
4. Do not fold nor use any correction fluid on the OAS. Read the instructions on
the OAS carefully.
5. The total mark for this paper is 40 marks.

This paper consists of 19 printed pages, including this cover page.

Sec 4E Physics P1 Page 1 of 19 USS Prelim 2019

www.KiasuExamPaper.com
460
1 Which of the following statements is CORRECT?

A The length of a bus is about 1 × 102 m.


B The diameter of an atom is about 1 × 10í5 m.
C The diameter of the Earth is about 1 × 107 m.
D The thickness of a human’s hair is about 1 × 10í2 m.

2 Figure I shows the reading of the zero error of the micrometer screw gauge.
Figure II shows the reading of the same instrument when it is used to measure
the width of an object.

Figure I Figure II

What is the actual width of the object?

A 4.63 mm
B 4.73 mm
C 4.83 mm
D 4.85 mm

3 A car travels for 10 m in 2 seconds.

What can be deduced from the information given?

A The average speed of the car is 5 m/s.


B The car travels at a constant speed.
C The maximum speed of the car is 5 m/s.
D The minimum speed of the car is 5 m/s.

Sec 4E Physics P1 Page 2 of 19 USS Prelim 2019

www.KiasuExamPaper.com
461
4 The diagram shows a trolley that starts rolling down a sloping runway
connected to a flat floor. Assume all the surfaces are frictionless.

Which of the following velocity-time graphs best illustrates the motion of the
trolley?

A B C D

5 The total mass of a skydiver and her equipment is 80.0 kg when she jumps
from an aircraft. When the parachute just opens, she decelerates at 2.5 m/s2.

What is the tension in the cord at this instance?

A 200 N B 600 N
C 800 N D 1000 N

6 A hollow rectangle metal block has the dimensions shown. The hole in the
middle goes all the way through the block. The density of the metal is 10 g/cm3.

5 cm
3 cm
2 cm
4 cm

10 cm

What is the mass of the block?

A 14 g
B 400 g
C 1400 g
D 2000 g

Sec 4E Physics P1 Page 3 of 19 USS Prelim 2019

www.KiasuExamPaper.com
462
7 The figure below shows a metre rule hanging from the ceiling using two ropes.
Two weights, 12 N and 20 N, are attached to the metre rule.

rope 1 rope 2

T1 T2

20 cm 45 cm 20 cm

12 N
20 N

Assuming the weight of the metre rule is negligible, what is the tension T 2 ?

A 11 N B 12 N
C 21 N D 32 N

8 The diagram shows the first step in an experiment to determine the position of
the centre of gravity of mass of an irregular shaped card.

irregular PP
shaped card
R
string
Q

bob

What is the next step in this experiment?

A find the mid-point of PQ


B hang the card from R
C measure the mass of the card
D measure the thickness of the card

Sec 4E Physics P1 Page 4 of 19 USS Prelim 2019

www.KiasuExamPaper.com
463
9 Which of the following objects is in neutral equilibrium?

A a pencil balanced on its sharp tip


B a playground swing that is at rest
C a traffic cone that is upright
D an ice cream cone resting on its slant edge

10 The diagram shows a mercury manometer connected to a gas container. The


density of mercury is 13 600 kg/m3. The gravitational field strength g is 10 N/kg.

What is the pressure difference between the gas in the container and the
atmosphere?

A 27 200 Pa B 40 800 Pa
C 54 400 Pa D 81 600 Pa

11 A lift of mass 1000 kg rises 50 m in 2.0 minutes.

If the efficiency of the lift is 80 %, what is the power supplied to the motor?
(The gravitational field strength on Earth, g, is 10 N/kg.)

A 3.33 kW
B 5.00 kW
C 5.21 kW
D 250 kW

Sec 4E Physics P1 Page 5 of 19 USS Prelim 2019

www.KiasuExamPaper.com
464
12 A tungsten wire has a resistance of 30 : at -10 qC and 50 : at 90 qC.

What is the working temperature of the wire if its resistance is 70 :?

A 100 qC B 170 qC
C 190 qC D 210 qC

13 Diagram I shows a thermocouple with junction P and Q. The millivoltmeter


shows a deflection that depends on the temperature difference between the
junctions P and Q. Diagram II shows how the temperature difference between
junctions P and Q corresponds to the deflection in the millivoltmeter.

temperature difference between P and Q / qC

V 120

80

40
P Q
Diagram I
0 10 20 30
0
deflection on meter / mV
de

Diagram II

If P is placed in a substance of 20 qC and Q is placed in another substance of


higher temperature, a deflection of 30 mV is seen.

What is the temperature of Q?

A 20 qC B 100 qC
C 120 qC D 140 qC

14 A gas in a sealed cylinder with fixed volume is heated.

Which of the following does NOT increase as the gas is heated?

A The average distance between the gas molecules.


B The average kinetic energy of the gas molecules.
C The average number of gas molecules hitting the cylinder walls per
second.
D The average speed of the gas molecules.

Sec 4E Physics P1 Page 6 of 19 USS Prelim 2019

www.KiasuExamPaper.com
465
15 The diagram below shows the cross-section of a plastic container that a
manufacturing company has created.
plastic cover
and box

black
inner walls vacuum

The company claims that the container can keep food warm for a duration that
surpasses other brands of container. It offers the following explanations to
justify their claims:

1. The plastic cover will reduce heat loss through conduction as plastic is
a poor conductor of heat.
2. The black inner walls will absorb heat from the environment to keep food
warm since black surfaces are good absorbers of heat.
3. The vacuum between the interior and exterior walls of the container will
prevent heat losses to the surroundings through conduction, convection
and radiation.

Which of the statements is/are CORRECT?

A 1 only B 1 and 2 only


C 2 and 3 only D All of the above

16 The graph shows the change in temperature when heat is supplied at 200 W to
1 kg of the substance.

Temperature / qC

100

0 50 100 Time / min

The specific latent heat of vaporisation of the substance is _________ J/kg.

A 4 000 B 6 000
C 240 000 D 360 000

Sec 4E Physics P1 Page 7 of 19 USS Prelim 2019

www.KiasuExamPaper.com
466
17 A pin is placed in front of a plane mirror as shown.

Where is the image of the pin?

18 The diagram below shows an image of an object formed by a thin converging


lens.

The lens is replaced with another lens of twice the focal length.

What are the properties of the new image?

A real, inverted, diminished


B real, inverted, same size
C real, inverted, enlarged
D virtual, upright, enlarged

Sec 4E Physics P1 Page 8 of 19 USS Prelim 2019

www.KiasuExamPaper.com
467
19 The diagram below shows circular wavefronts radiating from a point source P.

The point source is then set to vibrate with a gradually decreasing frequency.

Which of the following shows the possible resulting wavefronts?

20 The wavelength of X-rays is roughly the size of an atom.

What is the frequency of the X-rays?

A 1 u 10-10 Hz B 3 u 108 Hz
C 1 u 1010 Hz D 3 u 1018 Hz

Sec 4E Physics P1 Page 9 of 19 USS Prelim 2019

www.KiasuExamPaper.com
468
21 Sonar waves are emitted from a surface vessel to determine the depth of the
sea. The emitted signal and its reflection from the sea bed are displayed on the
screen of a cathode-ray oscilloscope as shown below.

transmitted signal reflected signal

6 cm

Given that the speed of sound in water is 1200 m/s and that the time-base of
the oscilloscope is 8 cm/s. What is the depth of the sea at this point?

A 450 m
B 800 m
C 900 m
D 7200 m

22 The frequencies of two musical notes X and Y are 256 Hz and 512 Hz
respectively.

If X and Y have the same amplitude, which of the following statements is/are
TRUE?

1. Y has a higher pitch than X.


2. The loudness of X is larger than that of Y.
3. The wavelength of Y is longer than that of X.

A 1 only
B 3 only
C 1 and 3 only
D 2 and 3 only

Sec 4E Physics P1 Page 10 of 19 USS Prelim 2019

www.KiasuExamPaper.com
469
23 An electron is projected at right angles to a uniform electric field E as shown in
the diagram.

In the absence of other fields, in which direction is the electron deflected?

A to the left
B to the right
C into the plane of the paper
D out of the plane of the paper

24 A bird is seen standing safely on a high voltage transmission line.

Which statement best explains this?

A The body of the bird has a high resistance.


B The scaly feet of the bird are good insulators.
C The trapped air in the feathers of the bird acts as an electrical insulator.
D There is negligible potential difference between the two feet.

25 The diagram below shows two copper wires X and Y. Both wires have the
same volume and wire Y is four times as long as wire X.

What is the ratio of the resistance of wire Y to resistance of wire X?

A 4:1 B 8:1
C 16:1 D 64:1

Sec 4E Physics P1 Page 11 of 19 USS Prelim 2019

www.KiasuExamPaper.com
470
26 The diagram shows a circuit containing five resistors connected to a battery.

Through which resistor is the current the smallest?

C D

27 The diagram below shows a circuit with a variable resistor and light dependent
resistor (LDR) connected in series.

6V
When light shines brighter on the LDR, what will happen to the reading on the
ammeter and voltmeter?

ammeter reading voltmeter reading


A increases increases
B increases decreases
C decreases increases
D decreases decreases

28 An electrical circuit uses insulated copper wire and the wire overheats.

Which of the following correctly explains how the wire should be replaced to
prevent it from overheating again?

A Use thicker copper wire which has less resistance.


B Use thicker insulation to reduce conduction.
C Use thinner copper wire which has more resistance.
D Use thinner insulation to reduce conduction.

Sec 4E Physics P1 Page 12 of 19 USS Prelim 2019

www.KiasuExamPaper.com
471
29 Ben connects a fuse along the neutral wire of a fan.

Which of the following statement(s) is/are CORRECT?

1. The fan will be safe to touch after the fuse blows.


2. When there is a large current, the fuse will blow and prevents the fan
from overheat.
3. There is no current passing through the neutral wire after the fuse
blows.

A 3 only
B 1 and 2 only
C 1 and 3 only
D 2 and 3 only

30 The ends of three metal rods are tested by holding end Q of rod 1 close to the
others in turn.

R T

Q S U

rod 1 rod 2 rod 3

The results are as follows:

End Q attracts end R.


End Q attracts end S.
End Q attracts end T.
End Q repels end U.

Which metal rod is a magnet?

A rod 1 only
B rod 1 and rod 2
C rod 1 and rod 3
D rod 3 only

Sec 4E Physics P1 Page 13 of 19 USS Prelim 2019

www.KiasuExamPaper.com
472
31 A piece of steel can be magnetised by stroking it with a magnet.

magnet

X steel Y

When the magnet is moved in the direction shown, which poles are produced
at X and at Y?

X Y

A North North

B South North

C North South

D South South

32 A soft iron rod is attracted by a bar magnet as shown the diagram below. When
a plotting compass is placed at X, the needle points in a direction as shown.

bar magnet

soft iron rod


Y
Which of the following shows the correct direction of the compass needle when
it is moved from position X to Y. (Ignore earth’s magnetic field)

Sec 4E Physics P1 Page 14 of 19 USS Prelim 2019

www.KiasuExamPaper.com
473
33 The diagram shows a wire AB that is bent into a U-shape and placed along the
earth’s north-south direction. Two plotting compasses are placed at the
positions X and Y. The compass at X is above the wire.
N

A B
What will happen when a current flows from A to B in the wire?

compass at X compass at Y

A deflects to the right remains in the position shown

B deflects to the right deflects to the left

C deflects to the left deflects to the right

D remains in the position shown remains in the position shown

34 Two iron bars P and Q are placed inside two solenoids as shown below.

P Q

What will happen to P and Q, when the solenoids are connected to an a.c.
power supply?

A repel each other


B attract each other
C oscillate towards and away from each other.
D oscillate upwards and downwards

Sec 4E Physics P1 Page 15 of 19 USS Prelim 2019

www.KiasuExamPaper.com
474
35 Two long parallel wires PQ and RS are connected to batteries as shown in the
diagram. PQ can move freely in any direction but RS is fixed.
P R

free to
move fixed

Q S
When both switches are closed, what happens to wire PQ?

A PQ moves away from RS.


B PQ moves towards RS.
C PQ moves in a direction into the plane of the paper.
D PQ moves in a direction out of the plane of the paper.

36 The diagram below shows the circular anti-clockwise path of a charged particle
in a field. The direction of the field is out of the paper.

Ignoring the effect of gravity, which of the following correctly describes a


possible state of charge of the particle and the nature of the field?

charge field

A negative magnetic

B positive electric

C negative electric

D positive magnetic

Sec 4E Physics P1 Page 16 of 19 USS Prelim 2019

www.KiasuExamPaper.com
475
37 A bar magnet is dropped through a loop of copper wire as shown.

bar magnet

N
copper loop

Which of the following statement(s) is/are CORRECT?

1. When the magnet approaches the copper loop, a current is induced in the
loop that flows in clockwise as seen by the observer from the top of the loop.

2. When the magnet moves through the copper loop, the current induced in
the copper loop sets up a magnetic field that always repel the magnet.

3. Heat is produced in the copper loop.

A 2 only
B 1 and 2 only
C 2 and 3 only
D 3 only

38 The diagrams show three generators, X, Y and Z.

X Y Z

Which is/are alternating current generator(s)?

A X only
B Y only
C X and Y only
D X and Z only

Sec 4E Physics P1 Page 17 of 19 USS Prelim 2019

www.KiasuExamPaper.com
476
39 The diagram shows a metal bar swinging like a pendulum across a uniform
magnetic field. The motion induces an e.m.f. between the ends of the bar.

pivot

magnetic field

Which graph represents this e.m.f. during one complete oscillation of the bar,
starting and finishing at P?

A B

C D

Sec 4E Physics P1 Page 18 of 19 USS Prelim 2019

www.KiasuExamPaper.com
477
40 The number of turns in a primary coil and secondary coil of a transformer is N p and
N s respectively.

Which pairs of values for N p and N s will result in an output voltage of 240 V when
the input voltage is 20 000 V?

Np Ns

A 2 000 240

B 10 000 240

C 10 000 480

D 30 000 360

***END OF PAPER***

Sec 4E Physics P1 Page 19 of 19 USS Prelim 2019

www.KiasuExamPaper.com
478
Name Class Index Number

UNITY SECONDARY SCHOOL


PRELIMINARY EXAMINATION 2019

SECONDARY FOUR EXPRESS

PHYSICS 6091/02 18 SEPTEMBER 2019

PAPER 2 1 HOUR 45 MINUTES

Additional Materials : NIL

READ THESE INSTRUCTIONS FIRST


1. Answer ALL questions in Section A on the question paper.
2. In Section B, answer Questions 10 and 11, and either part of Question 12. Write
your answers in the spaces provided on the question paper.
3. All workings and constructions must be shown clearly. Omission of essential
working will result in loss of marks
4. The number of marks is given in brackets [ ] at the end of each question or part
question.
5. You are expected to use an electronic calculator to evaluate explicit numerical
expression.
6. The total mark for this paper is 80 marks.

This paper consists of 19 printed pages, including this cover page.

Sec 4E Physics P2 Page 1 of 19 USS Prelim 2019

www.KiasuExamPaper.com
479
Section A [50 Marks]

Answer all the questions in this section.

1 A fisherman’s buoy is held submerged in sea water by a rope anchored to the sea
bed. Currents in the sea cause the buoy to be displaced so that the rope makes an
angle of 30q with the vertical as shown in Fig. 1.1.

The buoy may be considered to be acted upon by three forces: the tension T in the
rope which is 600 N, a horizontal force of sea current D and a vertical force known
as upthrust V.
V

direction
of D
current
T

sea bed
Fig. 1.1

By means of a scale drawing, determine the value of the upthrust, V and horizontal
force, D.

scale ………………………….. [1]

upthrust, V = ………………………….. [2]

force, D = ………………………….. [1]


Sec 4E Physics P2 Page 2 of 19 USS Prelim 2019

www.KiasuExamPaper.com
480
2 Fig. 2.1 shows a diver working below the surface of a lake. The density of the water
in the lake is 1000 kg/m3, the atmospheric pressure at the surface is 1.0 x 105 Pa.

Fig. 2.1

The diver inflates a balloon with air at a depth of 15 m and attaches the balloon to a
tray of objects.

(a) Calculate the total pressure acting on the balloon at 15 m below the surface
of the lake.

pressure = ………………………….. [2]

(b) The diver releases the tray and the balloon, and they begin to rise. The
temperature of the air in the balloon does not change. The volume of the
balloon is 0.3 m3 at 15 m depth.

Calculate the volume of the balloon when it reaches the surface.

volume = ………………………….. [2]

(c) Explain, in terms of the air molecules inside the balloon, why the air pressure
in the balloon is less at the surface.

……………………………………………………………………………………......

……………………………………………………………………………………......

…………………………………………………………………………………….. [2]
Sec 4E Physics P2 Page 3 of 19 USS Prelim 2019

www.KiasuExamPaper.com
481
3 Fig. 3.1 shows a cooler box packed with food and a coolant pack is placed on top.
Fig. 3.2 shows the temperature-time graph of the contents over a period of six hours.

coolant pack
Manufacturer’s information on coolant pack:

mass = 0.20 kg

specific heat capacity


of liquid coolant= 3820 J/(kg °C)

specific latent heat of fusion


of coolant =2.1 x 105 J/kg

cooler box Fig. 3.1

Fig. 3.2

(a) Explain why the coolant pack must be placed on top of the food in order to
bring down the temperature.

……………………………………………………………………………………......

……………………………………………………………………………………......

…………………………………………………………………………………….. [2]

Sec 4E Physics P2 Page 4 of 19 USS Prelim 2019

www.KiasuExamPaper.com
482
(b) Explain why coolant at solid state is better in keeping the content cold for a
longer period of time as compare to coolant at liquid state.

……………………………………………………………………………………......

……………………………………………………………………………………......

……………………………………………………………………………………......

…………………………………………………………………………………….. [2]

(c) Using the information given in Fig 3.1 and 3.2, calculate the total energy lost
by the food to the coolant pack during the period of six hours. Assume that
there is no energy lost to the surrounding outside the cooler box.

total energy lost = ………………………….. [3]

Sec 4E Physics P2 Page 5 of 19 USS Prelim 2019

www.KiasuExamPaper.com
483
4 Fig. 4.1 shows a narrow laser beam directed towards a point A on a vertical wall.
A semicircular glass block G is placed symmetrically across the path of the beam.
1.50 m
O
A
laser
G

wall

1.68 m

Fig. 4.1
B

When the beam strikes the wall at A, a bright spot is formed at A. The glass block is
then rotated about the centre, O, and the bright spot moves down from A to B and
then disappears.

(a) State the direction of rotation (clockwise or anticlockwise) of the glass block
G in order to obtain the bright spot to move down from A to B.

…………………………………………………………………………………….. [1]

(b) Explain why the bright spot disappears as it moves beyond B.

..……………………………………………………………………………………….

..……………………………………………………………………………………….

…………………………………………………………………………………….. [2]

(c) Given that OA = 1.50 m and AB = 1.68 m,

(i) calculate the critical angle of the glass block,

critical angle = ………………………….. [2]

(ii) and hence determine the refractive index of the glass block.

refractive index = ………………………….. [1]


Sec 4E Physics P2 Page 6 of 19 USS Prelim 2019

www.KiasuExamPaper.com
484
5 Fig. 5.1, not drawn to scale, shows water waves generated by a dipper vibrating in
a ripple tank with the wave travelling from the deep region to the shallow region.

9.0 cm

shallow region
dipper deep region

Fig. 5.1

(a) State what happens to the frequency and speed of the water waves as it
moves from the deep region to the shallow region.

…………………………………………………………………….………………......

…………………………………………………………………………………….. [2]

(b) The vibrating dipper makes 10 vibrations in 2 s.


Calculate the speed of the water waves in the shallow region.

speed = ………………………….. [2]

(c) The rate of vibration of the vibrating dipper is doubled.

State what will happen to the speed and the wavelength of the wave in the
shallow region.

………………………………………………………………………………….…......

…………………………………………………………………………………….. [1]

Sec 4E Physics P2 Page 7 of 19 USS Prelim 2019

www.KiasuExamPaper.com
485
6 Fig. 6.1 shows a positively charged sphere held with an insulating handle. When the
sphere is brought near the metal plate, the galvanometer needle deflects momentarily.

galvanometer
insulating metal
handle earth
plate

Fig. 6.1

(a) Explain clearly why there is a momentary deflection in the galvanometer


needle.

…………………………………………………………………………………………

…………………………………………………………………………………………

…………………………………………………………………………………………

…………………………………………………………………………………………

…………………………………………………………………………………………

…………………………………………………………………………………….. [3]

(b) Suggest a method that would increase the magnitude of the deflection.

…………………………………………………………………………………………

…………………………………………………………………………………….. [1]

(c) State clearly what can be observed when the positively charged sphere is
removed quickly.

…………………………………………………………………………………………

…………………………………………………………………………………….. [1]

Sec 4E Physics P2 Page 8 of 19 USS Prelim 2019

www.KiasuExamPaper.com
486
7 Fig. 7.1 below shows a circuit connected to a batteryy of unknown e.m.f.

A
2.0 : 4.0 :

P R

1.0 : 2.0 :

S
Fig. 7.1

(a) When the switch is opened, the ammeter reads 3.0 A.

(i) Calculate the e.m.f. of the battery.

e.m.f. = ………………………….. [2]

(ii) Calculate the current passing through PSR.

current = ………………………….. [1]

(b) The switch is now closed. Explain whether a current will flow through QS.

………………………………………………………………………………….…......

…………………………………………………………………………………………

………………………………………………………………………………….…......

…………………………………………………………………………………….. [2]
Sec 4E Physics P2 Page 9 of 19 USS Prelim 2019

www.KiasuExamPaper.com
487
8 A student has made a battery tester shown in Fig. 8.1. It uses a magnet, wire that is
flexible and springy, and a pointer. With it, she can check whether a small battery is
“live” or “dead”. When she connects a battery to the tester, the pointer moves.

pointer
small
battery
S
+
coil

D
C
-
N viewing
Fig. 8.1 direction

(a) Fig. 8.2 shows the magnet and coil as viewed in the direction shown in
Fig. 8.1. In Fig. 8.2, draw

(i) magnetic field lines around the cross-section of wires AB and CD. [1]

(ii) arrows to indicate the direction of motion of wires AB and CD. [1]

Fig. 8.2

(b) State the observation on the pointer if

(i) the battery delivers less current;

…………………………………………………………………………….. [1]
(ii) the battery terminals are reversed;

…………………………………………………………………………….. [1]
(iii) the battery is replaced by an alternating current source.

…………………………………………………………………………….. [1]
Sec 4E Physics P2 Page 10 of 19 USS Prelim 2019

www.KiasuExamPaper.com
488
9 A bar magnet is suspended by a spring so that it can oscillate vertically and freely in
and out of a coil as shown in Fig. 9.1. The coil is connected to an an oscilloscope.

bright
spot

spring

magnet

coil

Fig. 9.1

(a) The time-base is switched off, and the oscilloscope is adjusted so that the
bright spot is in the middle of the screen when the magnet is not oscillating.

(i) Describe and explain what is seen on the screen as the magnet
oscillates.

….…………………………………………………………………………......

….…………………………………………………………………………......

….…………………………………………………………………………......

….…………………………………………………………………………......

….…………………………………………………………………………......

….……………………………………………………………………….... [3]

(ii) Describe the changes that will be observed on the screen when the
magnet oscillates at a faster rate.

…………………………………………………………………………………

…………………………………………………………………………….. [1]

Sec 4E Physics P2 Page 11 of 19 USS Prelim 2019

www.KiasuExamPaper.com
489
(b) With the magnet still in motion, the time-base of the oscilloscope is switched
on. The controls are suitably adjusted until the trace in Fig. 9.2 is seen on the
screen.
1 cm

1 cm

Fig. 9.2

On Fig. 9.2, mark with “X” one position of the bright spot that corresponds to
the position of the magnet when the magnet is at the lowest point. [1]

(c) The trace in Fig. 9.2 is obtained when the time-base control is set to 0.50 s/cm.
Determine the frequency of the oscillation of the magnet.

frequency = ……………………….. [2]

Sec 4E Physics P2 Page 12 of 19 USS Prelim 2019

www.KiasuExamPaper.com
490
Section B [30 Marks]

Answer all the questions in this section.


Answer only one of the two alternative questions in Question 12.

10 Fig. 10.1 shows two of the towers that support a single cable of total length 5.0 km,
which links a factory to the electrical grid.

Fig. 10.1

If the weight of the cable between two towers is larger than a given force F, then the
cable breaks. The value of F depends on the material from which the cable is made.
The table gives the value of F and other data for three cables made from different
materials.

resistance of 1 km density cross-sectional area


cable material F/N
: kg/m3 m2
1 aluminum 6 000 0.075 2 700 3.0 x 10-4
2 copper 9 000 0.050 8 900 3.0 x 10-4
3 steel 27 000 210 7 800 3.0 x 10-4

(a) Assuming that g is 10 N/kg, calculate

(i) the mass of a copper cable of length 5.0 km,

mass = …………………………….. [2]

(ii) the minimum number of towers needed between the grid and the
factory to support this copper cable.

number = ……………..…………….. [2]


Sec 4E Physics P2 Page 13 of 19 USS Prelim 2019

www.KiasuExamPaper.com
491
(b) The cable used is actually made from aluminium and the current in it is 500 A.

For this cable, calculate

(i) the potential difference (p.d.) between the grid and the factory,

p.d. = ………………………….. [1]

(ii) the power loss in the cable,

power = .………………………… [2]

(iii) the cost of this power loss in 1 day. 1 kWh costs 22 cents.

cost = …………………………. [2]

(c) Using data from the table, explain why

(i) the aluminium is more suitable material than copper for the cable,

…………………………………………………………………………………

…………………………………………………………………………………

…………………………………………………………………………………

…………………………………………………………………………………

…………………………………………………………………………..… [2]

(ii) steel is an unsuitable material for the cable.

…………………………………………………………………………………

…………………………………………………………………………..… [1]
Sec 4E Physics P2 Page 14 of 19 USS Prelim 2019

www.KiasuExamPaper.com
492
11 At a sharp corner on a car racing circuit there is an escape lane, as shown in
Fig. 11.1.

direction of
racing car
escape lane

corner

Fig. 11.1

The escape lane is a bed of small stones. The escape lane slopes upwards. A car
of mass 700 kg approaches at a speed of 40 m/s. The brakes fail and the car stops
in the escape lane.

(a) Describe what happens to the kinetic energy of the car as it stops.

…………………………………………………………………………………………

…………………………………………………………………………………………

…………………………………………………………………………………….. [2]

(b) The car comes to rest 40 m along the escape lane, having risen through a
vertical distance of 3.0 m. The acceleration of free fall is 10 m/s2.
Calculate

(i) the change in gravitational potential energy of the car,

change = ……………………………[2]

(ii) the average frictional force on the car in the escape lane.

force = ……………………………[3]

(c) The frictional force on the car in the escape lane is not constant. Suggest one
factor, apart from the car’s speed, that affects the value of the frictional force.

…………………………………………………………………………………………

…………………………………………………………………………………….. [1]
Sec 4E Physics P2 Page 15 of 19 USS Prelim 2019

www.KiasuExamPaper.com
493
12 EITHER

An appliance is connected to the live, neutral and earth conductors of the mains
supply. The current in the circuit is 4.0 A and the rating of the fuse is 5 A.

(a) Explain what is meant by

(i) live,

…………………………………………………………………………………

…………………………………………………………………………….. [1]

(ii) neutral.

…………………………………………………………………………………

…………………………………………………………………………….. [1]

(b) When a fault occurs in the appliance, no damage or injury is caused provided
that the correct fuse is used and the metal case is connected to earth.

(i) The 5 A fuse is replaced by a 30 A fuse.

Explain why this presents a risk of damage or injury.

…………………………………………………………………………………

…………………………………………………………………………………

…………………………………………………………………………………

…………………………………………………………………………………

…………………………………………………………………………….. [2]

(ii) The earth is not connected to the metal case.

Explain why this present a risk of damage or injury.

…………………………………………………………………………………

…………………………………………………………………………………

…………………………………………………………………………………

…………………………………………………………………………………

…………………………………………………………………………….. [2]

Sec 4E Physics P2 Page 16 of 19 USS Prelim 2019

www.KiasuExamPaper.com
494
(c) State one advantage of using a circuit breaker rather than a fuse to protect
the appliance.

…………………………………………………………………………………………

…………………………………………………………………………………….. [1]

(d) Describe an experiment to check that a fuse blows at 5 A.

In your account

x draw a diagram of the apparatus,


x describe the procedure to be taken. [3]

…………………………………………………………………………………………………

…………………………………………………………………………………………………

…………………………………………………………………………………………………

…………………………………………………………………………………………………

…………………………………………………………………………………………………

…………………………………………………………………………………………………

…………………………………………………………………………………………………

…………………………………………………………………………………………………

…………………………………………………………………………………………………

Sec 4E Physics P2 Page 17 of 19 USS Prelim 2019

www.KiasuExamPaper.com
495
12 OR

Fig. 12.1 shows a thermistor connected to a battery of e.m.f. 6.0 V in a circuit.

Fig. 12.2 shows the variation of the voltmeter reading with the temperature of the
thermistor.

Fig. 12.1

Fig. 12.2
(a) Explain why the voltmeter reading increases as the temperature rises.

…………………………………………………………………………………………

…………………………………………………………………………………………

…………………………………………………………………………………………

…………………………………………………………………………………………

…………………………………………………………………………………………

…………………………………………………………………………………….. [2]

Sec 4E Physics P2 Page 18 of 19 USS Prelim 2019

www.KiasuExamPaper.com
496
(b) Determine the resistance of the thermistor when its temperature is 40qC.

resistance = ……………………………. [3]

(c) On Fig. 12.2, draw a graph to show how the voltage across the thermistor
varies with the temperature. [2]

(d) Describe an experiment, using the circuit of Fig. 12.1, to produce the graph
of the voltmeter reading in Fig. 12.2.

In your account

x List all the apparatus you use, apart from the apparatus shown in
Fig. 12.1,
x Describe the procedure you use to obtain the readings. [3]

…………………………………………………………………………………………………

…………………………………………………………………………………………………

…………………………………………………………………………………………………

…………………………………………………………………………………………………

…………………………………………………………………………………………………

…………………………………………………………………………………………………

…………………………………………………………………………………………………

…………………………………………………………………………………………………

…………………………………………………………………………………………………

…………………………………………………………………………………………………

…………………………………………………………………………………………………

…………………………………………………………………………………………………

…………………………………………………………………………………………………

*** End of Paper ***

Sec 4E Physics P2 Page 19 of 19 USS Prelim 2019

www.KiasuExamPaper.com
497
www.KiasuExamPaper.com
498
Sec 4E Express (Physics) Prelim Exam Marking Scheme 2019

P1 MCQ:

Qn Ans Qn Ans Qn Ans Qn Ans Qn Ans Qn Ans Qn Ans Qn Ans


1 C 6 C 11 C 16 D 21 A 26 C 31 C 36 A
2 A 7 A 12 C 17 D 22 A 27 A 32 B 37 C
3 A 8 B 13 D 18 D 23 A 28 A 33 A 38 D
4 A 9 D 14 A 19 B 24 D 29 D 34 B 39 A
5 D 10 C 15 A 20 D 25 C 30 C 35 B 40 D

P2 Section A:

1 D
scale 1 cm : 50 N [A1]
Correct diagram [M1]
upthrust, V = 520 N ± 10 N [A1]
force, D = 300 N ±10 N [A1]

2(a) Pressure = (1.0 x 105 ) + (15 x 1000 x 10) [M1]


= 250 000 Pa [A1]

2(b) P1V1 = P2V2


V2 = P1V1 / P2
= (250 000)(0.3) / (100 000) [M1]
= 0.75 m3 [A1]

2(c) As the balloon increases in volume, this causes the number of air molecules per unit
volume decreases. [A1]
The frequency of the air molecules colliding with the inner wall decreases. [A1]

www.KiasuExamPaper.com
499
3(a) The air at the top is cooled and the cold air contracts, becomes more dense and sink.
[A1]
The warmer air at the bottom rises up to replace the sunken cool air. This cycle
continues to setup a convection current to bring the temperature down quickly and
uniformly. [A1]

3(b) The coolant at solid state absorb a lot more heat from the content to change to liquid
state. It will then absorb more heat to increase its temperature. [A1]
Coolant at liquid state only absorb a limited amount of heat from the content to increase
its temperature. Therefore, coolant at solid state absorb more heat from the content.
[A1]

3(c) total energy lost by food = total energy absorb by coolantt


= ml f +mc'T
= (0.2 x 2.1 x 105) [M1]
1] + (0.2 x 382
3820
20 x 1
13) [M1]
= 51 932 J
= 51 900 J [A1]

4(a) anticlockwise [A1]

4(b) When the glass block iss rotated d further,


fu
u the
th
hee angle
an
ng e of
glle of incidence
in
nci
cide
d ncce be
de becomes s more than the
th
he laser
critical angle [A1] and the lla
asse
er undergoes
unde
un d rgoe es total internal
internnal
al reflection.
refle
le
ect
ctiio
ion. [A
[A1]
1]

4(c)(i) ngle AOB


Angle AO
OB n -11 (1
= tan ((1.68
1.6
68 / 1
1.50)
.50
50)
48.2q
= 48
4 2q
.2 [M1]
[M1
[M1]]
80q – 9
180q
c=1
18 0q – 48
90q
90 48.2q
4 2q = 41
8..2 8q [A1]
41.8q
1.8 [A
A1]
1]

4(c)(ii)
ii) n = 1/sin
1s
1/ siin c
sin
= 1 / si 8q
41.8q
in 4
411.8
=11.
.550
1.50 0 [A1]
[A
A1]]

5(a) Frequency
que cy remains
enc re
ema ins the
main he same.
th m . [A1]
same
sa [A
Speedd decreases.
de
ecr
cre ses.. [A
crea [A1]
1]

5(b) f = 10 / 2 = 5 Hz
Hz
O = 9 / 3 = 3 cm
Speed = fO = 5 x 3 [M1]
= 15 cm/s [A1]

5(c) Speed remains the same.


Wavelength reduce by half. [A1]

6(a) Electrons are attracted from earth by the positively charged sphere as unlike charges attract.
[A1]
The flow of electrons which carry negative charges is detected by the galvanometer and hence
there is a deflection. [A1]
The electrons remain attracted by the positively-charged sphere and they stay in the metal
plate. There is no flow of electrons and the galvanometer needle return to zero. [A1]

www.KiasuExamPaper.com
500
6(b) Use a positively-charged sphere with higher magnitude of charge. [A1]
OR
Bring the sphere closer to the plate.

6(c) The galvanometer deflect momentarily to the other side. [A1]

7(a)(i) R eff = (1/6 + 1/3)-1


=2:
Emf = 3A x 2: [M1]
= 6V [A1]

7(a)(ii) I = 6V / 3: = 2A [A1]

7(b) The potential difference across PQ is 1A x 2: = 2V. So potential at Q iis 6V -2V = 4V


The potential difference across PS is 2A x 1: = 2V. So potential att S is 6
ential a 6V - 2V = 4V. [M1]
Since Q and S has same potential, potential difference is zero, so no ccurrent
o cu rre flow. [A1]

er apart (far
8(a)(i) concentric circles – closer (near to wire), further (fa
ar fr
from
om wir
ire)
ir e)
wire)
8(a)(ii)

8(b)(i) deflect less to right [A1]

8(b)(ii) deflect to left [A1]

8(b)(iii) vibrate to and fro between left and right. [A1]

www.KiasuExamPaper.com
501
9(a)(i) A vertical line is seen on the screen. [A1]
When magnet moves down into the coil, there is cutting of magnetic field lines and an emf is
induced in the coil. This causes the y-plates to be charged which moves the electron beam
either up or down. [A1]
When the magnet moves up out of the coil, there is cutting of magnetic field lines in the
opposite direction and an emf is induced in the opposite direction. This causes the y-plates to
move the electron beam the other direction. [A1]

9(a)(ii) The vertical line becomes longer. [A1]

9(b)
1 cm

1 cm

9(c) T = 6 x 0.5 = 3 s [M1]


[M1]
[M 1]
f = 1/3 = 0.
0.333
0 333 Hz [A
33 [A1]
A1]]

Section
ion B

10(a)(i) m =88900
89
90000x5 5000
000 x 3.0
00 3.0 x 1
100-4 [M
[M1]
= 13
13 3 50
5
3500 kg
=113
3 400
00 kg
kg [A1]
[A

10(a)(ii) mber
e = 133500
number 13 N / 9000N [M1]
= 14 8 = 15 [A1] (accept 16) ecf 10a(i)
14.83

10(b)(i) R = 0.075 x 5 = 0.375:


pd = 500A x 0.375:
= 187.5 = 188 V [A1]

10(b)(ii) power loss = I2R


= 500A x 500A x 0.375: [M1{
= 93 750 W
= 93 800 W [A1]

10(b)(iii) cost = 24h x 93.75kW x $0.22 [M1]


= $495 [A1]

www.KiasuExamPaper.com
502
10(c)(i) Aluminium wire has a much lower density than copper, about 3 times lower.
[A1] The number of towers used to support 5km of the wire can be reduced by 3
times. [A1]

10(c)(ii) The resistance of steel is too high compare to aluminium and copper. This will
incur lots of power loss. [A1]

11(a) The kinetic energy is converted into thermal, sound [A1] and gravitational potential
energy [A1] as it stops.

11(b)(i) GPE = 700 x 10 x 3 [M1]


= 21 000 J [A1]

11(b)(ii) KE = ½ x 700 x 402


= 560 000 J [M1]

KE = GPE + loss
Loss = KE – GPE
Friction x distance = KE – GPE E
Friction = (KE – GPE) / d distance
ista
is tanc
ta ncce
= (560 000
0 – 21
2 000)00)) / 40
000
00 [[M1]
0 [MM1]
1]
= 13
34475
75 N
75
= 13 500
0 N [[A
500 A11]]
[A1]

11(c) Texture (size


sizze of tthe
ure (s he ssmall
he a sstones)
ma
mall tone
to es) off the e
s) o escape
sc
sca
ap weight
pe lane,, we ightt of car,
weig
ig ca [A1]

EITHE
HE
HER
12 EITHER

i)
12(a)(i) havi
ha ving
ng a h
having igh p
high poote
tent
ntia
ial
potentialal [A1]

12(a)(ii) ma
m ain
ain
inta
taiin
maintain n at
at zero
ze
zerro
eroo potential
pot
oten
ot entititial
en al [A1]
[A

12(b)(i) When
W he
Wh enn a current
currre nt of
rent of larger
larg than 4.0 A passes through the appliance, the large
currentt is n
not
ot large enough to blow the 30 A fuse. [A1]
he la
The arg
rge cur
large current can then cause over heating in the appliance and may lead to
fire. [A1]

12(b)(ii) When there is a fault in the appliance, the metal case may become live. With the
absence of earth wire, the current is not able to flow between the metal case to
the earth terminal to blow the fuse or trip the ELCB. [A1]
If a user touches the metal case, large current will flow through the user between
the metal case and the earth and may lead to electric shock. [A1]

12(c) When large current trip a circuit breaker, we do not need to replace the circuit breaker.
But we will need to replace a fuse if it is blown by a large current. [A1]

www.KiasuExamPaper.com
503
12(d)

battery

A Ammeter

Rheostat

5 A fuse

Diagram [A1]

nce and m
1. Adjust rheostat to give the largest resistance inim
in imum
im
minimumum rea
eadi
ea d ng in ammeter.
reading a
he resistance
2. Adjust rheostat slowly to decrease the e and
a d increase
an in
ncrea
ase tthe
he ammeter
amm reading to
4 A.
crease the
3. Adjust rheostat slowly to increase e ammeter
ammet
mmmeettter
err reading
e rea
eadi
ding by 0 .1A
.1
0.1AA each tim
time. [A1]

4. Repeat step 3 untill bulb


bggooe
es
goes s off.
off
f . Wh
W
Whenen
n the
the bulb goes
go
oes
e off,
off, this
this indic
cat
ates
es that the
indicates t fuse
has blown.
ing on the
5. The reading the
he ammeter
ammmet
ete
err just
jusst before
befo
be orre the
t e bulb
th bu
bulb goes
goe
es off
off is
s tthe
he current
current which blows the
fuse. [A1]]

12 OR

hen
12(a) When n temperature
te
emmpe
perarattu
urre
e increases,
inc
ncre
reasses
es,, the
th resistance
resis of thermistor decreases. [A1]
The pootte
ent
ntiia
potentialal di
d iff
fference
difference ce across
acr
cros
o s the thermistor decreases.
Sincee th
he emf
the em
e mf remains
remamain
ma ins
in s constant,
c nsta the potential difference across the resistor increases.
co
[A1]
Thereforere th
thee vvoltmeter
oltme reading increases.

12(b) At 40qC, pd across resistor = 3.6V

Current = 3.6V / 1600: = 0.00225 A [M1]

R of thermistor = (6V-3.6V) / 0.00225 A [M1]


= 1070 : [A1]

www.KiasuExamPaper.com
504
12(c)

www.KiasuExamPaper.com
505
12(d)

Apparatus:
a. Retort stand
b. Beaker
c. Plastic sheet
d. Laboratory thermometer
e. Wire gauze
f. Tripod
g. Bunsen burner

[A1]

Procedures:
1. Setup as shown in diagram. Use water use temperature rature off abou ou
aboutut 5 qC
2. Wrap the thermistor in a plastic sheet to prevent ent waterr ffrom rom
ro m enente
tering the
te
entering t thermistor and
immerse it into the beaker of water. [A1]]
3. Heat the water.
4. When thermometer shows 10qC, C, recor
orrd the v
o
record vo olttme
voltmetermeteter rreading.
eadiing
ng.
crease
se
5. Repeat step 4 for every increasee in 10 qC until
0 qC untitil th
un tthermometer
herermo
momemete h s 100 qC
terr reaches
te reac
reache
ac qC.[A1]
6. Tabulate Temperature e of Therm
he missto
t r (T
Thermistor T) an
(T) a nd vo
and oltlttm
mete
voltmeter er re
readin
readingng of resisto or (V).
resistor
againsst T.
7. Plot a graph of V against T.

www.KiasuExamPaper.com
506
www.KiasuExamPaper.com
507
Name: ______ ( ) Class: ___

WHITLEY SECONDARY SCHOOL


A Caring and Learning Community
Discipline * Integrity * Respect * Responsibility

PRELIMINARY EXAMINATION 2019


SUBJECT : PHYSICS Paper 1 (6091/01)

LEVEL : Sec 4 Express

DATE : 2 Sep 2019 (Mon)

DURATION : 1 hr

TOTAL MARKS: 40

INSTRUCTIONS TO CANDIDATES

Do not open this booklet until you are told to do so.


Write your name, index number and class in the spaces on the Multiple Choice Answer Sheet and at
the top of this cover page.
Do not use staples, paper clips, highlighters, glue or correction fluid/tape.

There are forty questions in this section. Answer all questions. For each question, there are four
possible answers, A, B, C and D.
Choose the one you consider correct and record your choice in soft pencil on the separate answer
sheet provided.

Each correct answer will score one mark. A mark will not be deducted for a wrong answer.
Any rough working should be done in this booklet.
The use of an approved scientific calculator is expected, where appropriate.

Take the value of the acceleration due to free fall on earth to be 10 m / s2.

This document consists of 17 printed pages including the cover page.

www.KiasuExamPaper.com
508
Answer all questions.

1 Three groups of quantities are shown below.

I mass, force, weight


II weight, work done, acceleration
III weight, force, displacement

Which group of quantities consists of only vectors?


A I only B I and II only C III only D All of them

2 A sphere runs along a smooth rail from P to Q as shown.

Which of the following graphs best represents the variation of the distance d travelled by the
sphere with time t?

A B

distance / m distance / m

t/s t/s

C D

distance / m distance / m

t/s t/s

www.KiasuExamPaper.com
509
3 A car, which was travelling due east at a speed of 3.6 m / s initially, changes direction and travels
due west at a speed of 6.2 m / s.

Taking the direction to the east as positive, determine the change in speed and velocity.

change in speed (m / s) change in velocity (m / s)


A 2.6 – 2.6
B 9.8 2.6
C 2.6 9.8
D 2.6 – 9.8

4 A wooden block that is pushed along a horizontal flat surface moves at constant speed.

Which statement is correct?


A The frictional force is greater than the pushing force.
B The frictional force is equal and opposite to the pushing force.
C The frictional force is less than the pushing force.
D The frictional force increases as the block moves at constant speed.

5 A magician pulled a tablecloth swiftly off a table top. An empty glass which was set on the
tablecloth remained on the table top when the table cloth was removed.

Which of the following modifications would make this performance easier?


A use a rough table cloth
B use a glass of a lighter mass
C wet the table cloth
D fill the empty glass with water

6 Suppose some aliens landed on several planets.

alien mass / kg weight / N


P 40 80
Q 20 200
R 10 200
S 20 400

From the information given, which two aliens are likely to have landed on the same planet?
A P and S
B Q and S
C Q and R
D R and S

www.KiasuExamPaper.com
510
7 The following density experiment was set up.

gas jar

oil ( 800 kg / m3)

water ( 1000 kg / m3)


iron

mercury ( 13 600 kg / m3)

Which of the following is a possible density for iron?


A 600 kg / m3 B 1000 kg / m3 C 8000 kg / m3 D 14 000 kg / m3

8 The diagram below shows a uniform 5.0 m beam. The beam is supported at P and Q. A man of
weight 800 N stands at M such that QM = 2.0 m. Assume that the mass of beam is negligible.

2.0 m

M
P Q

5.0 m

What are the reaction forces at P and Q due to the weight of the man?

reaction force at P reaction force at Q


A 300 N 500 N
B 320 N 480 N
C 400 N 400 N
D 480 N 320 N

www.KiasuExamPaper.com
511
9 A die is unbiased when its centre of gravity (CG) is at its geometrical centre whereas a biased
die has its centre of gravity nearer to one of its six faces.

The diagram below shows an unbiased dice.

CG

Which one of the following biased dice has a higher chance of getting a ‘6’ on top after it is
rolled?

A B

C D

www.KiasuExamPaper.com
512
10 The diagram shows a simple mercury barometer. The height of mercury, h, was recorded as
76 cm.
vacuum

h = 76 cm

mercury reservoir

Which of the changes will result in a smaller h being observed?


A conduct the experiment under a shelter
B conduct the experiment below sea level
C tilt the mercury barometer
D replace the mercury with a liquid of a greater density

11 The diagram shows a forked tube containing mercury, air in one branch and a vacuum in the
other.
vacuum air

mercury

What is the pressure exerted by the air?


A 50 mm Hg B 150 mm Hg C 600 mm Hg D 750 mm Hg

12 A ball of mass 0.50 kg is released from rest at a height of 3.0 m above the ground.

Assuming that air resistance is negligible, what is the kinetic energy of the ball when it is 2.0 m
above the ground? Take the acceleration due to gravity to be 10 m/s2.
A 5.0 J B 10.0 J C 15.0 J D 25.0 J

www.KiasuExamPaper.com
513
13 When a liquid evaporates, its temperature is lowered.

Which of the following is the most appropriate explanation for this observation?
A The liquid lost transferred heat to the surroundings.
B The average internal kinetic energy of the molecules in the liquid decreased.
C The total internal energy of the liquid decreased.
D The total internal kinetic energy of the molecules in the liquid decreased.

14 A match would ignite if held 10 cm above a Bunsen flame but not if held 10 cm away to one side
of the flame.

This is because the match above the Bunsen flame gains more thermal energy through
A convection.
B conduction.
C radiation.
D diffusion.

15 The diagrams show the scale of a voltmeter connected to a thermocouple thermometer.

ice point steam point unknown


temperature
What is the temperature of the liquid?
A 12.5 oC B 100 oC C 125 oC D 150 oC

16 18 000 J of energy is required to increase the temperature of 2 kg of liquid by 4 °C.

What is the heat capacity of the liquid?


A 2250 J / K B 4500 J / K C 9000 J / K D 12 000 J / K

www.KiasuExamPaper.com
514
17 A solid substance is placed in a boiling tube and heated steadily. The temperature-time graph of
the substance is shown below.

temperature / °C

S T

Q R

time / min

At which portion(s) do the substance gain internal potential energy?


A PQ and QR only
B PQ and RS only
C RS and ST only
D QR and ST only

18 The graph shows the displacement-distance graph of a sound wave. The sound wave is
travelling to the right. Three of the particles X, Y and Z in the sound wave are marked below.

displacement / cm

distance / cm
X Z

Which particle(s) in the graph above is/ are centre(s) of compression?


(Assume that a displacement to the right is positive displacement and a displacement to the left
is negative displacement.)
A particle X
B particle Y
C particle Z
D particles X and Z

www.KiasuExamPaper.com
515
19 In a ripple tank experiment, a dipper is connected to a motor to generate water waves.

If the motor rotates with higher speed, what is the effect on frequency, wavelength and speed of
the water waves generated?

frequency wavelength speed


A decreases increases decreases
B decreases increases no change
C increases decreases increases
D increases decreases no change

20 A ray of light is incident normally at the curved surface of a semicircular transparent block. It is
found that the light deviates by 55˚ from its original path and emerges as shown below.

55˚

What is the refractive index of the material of the block?


A 1.00 B 1.22 C 1.74 D 2.00

21 The diagram shows how light travels through the optical fiber of an endoscope used to look into
stomach of ulcer patients. X represents the inner material of the optical fiber while Y represents
the outer material.

Y
X

Which of the following statements is false?


A No light is lost through the optical fiber.
B The refractive index of X is greater than the refractive index of Y.
C The refractive index of Y is greater than the refractive index of X.
D The light in the optical fiber obeys the Laws of reflection.

www.KiasuExamPaper.com
516
22 The table shows the properties of some waves.

Which of the following correctly describes the properties of the waves?

waves types of waves speed of wave in vacuum


A gamma rays longitudinal 3.0 x 108 m / s
B X-rays transverse 3.0 x 108 m / s
C radio waves transverse 340 m / s
D sound waves longitudinal 340 m / s

23 The statements below describes the property of a kind of electromagnetic wave.

I It causes chemical reactions and causes many substances to glow or fluoresce.


II It causes human skin to have sunburn due to prolonged exposure to the Sun.
III It has a wavelength shorter than that of visible light.

Which electromagnetic wave is best described by the statements?


A ultraviolet ray
B infra-red radiation
C microwave
D gamma rays

24 A boy strikes a rigid metal fence with a stick to create a sound along the fence. A girl listens with
her ear against the fence. One second after the fence is struck, the girl hears a sound through
the air.

boy girl

stick
How long will it take for the sound to reach the girl through the fence?
A 0.0 s
B less than 1.0 sec
C 1.0 s
D more than 1.0 sec

10

www.KiasuExamPaper.com
517
25 The figure below shows a light and neutral conducting sphere suspended vertically by an
insulated thread near a charged conductor. The sphere moves towards and touches the charged
conductor.

In which position will the light conducting sphere come to rest?

26 The diagram shows a rectangular block with dimensions x, 2x and 3x.

3x

2x

P, Q and R mark the opposite faces on the block across which a potential difference is applied.

Across which two faces would there be maximum electrical resistance?


A the faces labelled P
B the faces labelled Q
C the faces labelled R
D the resistance is the same, whichever pair of faces is used

11

www.KiasuExamPaper.com
518
27 A handphone battery requires 900 C of charge before it is 100% charged.

The following diagram shows the characteristic voltage-current graph of the charging circuit.
It initially displays ohmic behaviour at low voltages but its gradient increases as the circuit heats
up.

The phone is plugged into a 255 V supply.

Which of the following shows the time required to charge the phone to 100%?
A 1800 s
B 1.8 hours
C slightly more than 1800 s
D slightly less than 1800 s

28 The diagram shows a circuit in which all the switches are open.

5.0 
S1
1.5 
X Y
5.0 
S2 S3

Which switch positions give a resistance of 4.0  between X and Y?

S1 S2 S3
A closed closed closed
B closed closed open
C closed open closed
D closed open open

12

www.KiasuExamPaper.com
519
29 The diagram shows a thermistor and a light-dependent resistor connected in series.

X Y

Which of these conditions will result in the maximum resistance between X and Y?

temperature lighting
A warm bright
B warm dim
C cool bright
D cool dim

30 The diagram below shows a potential divider circuit.

P
lamp 1

X
lamp 2
Q

What happen to the brightness of the lamps as the contact X moves from Q to P?

lamp 1 lamp 2
A dimmer brighter
B brighter brighter
C brighter remains the same
D brighter dimmer

31 The cost of a unit of electricity is $0.10.

appliance power rating time used (hours)


lamp 100 W 5
heater 1.5 kW 3
cooker 3 kW 0.5

What is the total cost when all these appliances are used for the durations shown above?
A $0.065 B $0.65 C $2.65 D $50.60

13

www.KiasuExamPaper.com
520
32 The diagram shows the three wires of an electrical supply connected to a water heater.

brown wire

blue wire

yellow and green


striped wire
What is the amount of current that flows through each of the wires when the water heater is
switched on?

brown wire blue wire yellow and green striped wire


A 8A 8A 8A
B 8A 0A 0A
C 8A 0A 8A
D 8A 8A 0A

33 The diagram shows a magnetic field lines pattern.

Which pair of bar magnets will produce the magnetic field as shown above?

A B

C D

34 Which one of these statements best describes magnetic induction?


A Magnetic induction is the reason why unmagnetised material may be attracted
by a magnet.
B Induced magnetism is always permanent.
C Magnetic induction is the same as electromagnetic induction.
D The material to be induced must be in physical contact with the magnet.

14

www.KiasuExamPaper.com
521
35 Three identical compasses are placed over a wire loop as shown in the diagram below.

YY

X Z
X Z

The switch is now closed.

Which row shows the correct orientation of each compass after some time?

compass X compass Y compass Z


A no change down left
B no change up right
C left down left
D left up right

36 Two current-carrying wires X and Y are arranged in parallel as shown below.

wire X wire Y

What is the direction of the electromagnetic force on each wire?

wire X wire Y
A to the left to the left
B to the left to the right
C to the right to the left
D to the right to the right

15

www.KiasuExamPaper.com
522
37 One end of a wire Y is immersed in a conducting liquid while the other end X is connected to a
battery and is free to rotate. The direction of the current in the circuit is indicated. A cylindrical
magnet is placed in the centre of the conducting liquid with the North Pole facing upwards.

magnetic field lines


current

conducting liquid

d.c. power source


metal dish

When viewed from the top, which direction will the wire XY move?
A clockwise
B anti-clockwise
C towards the magnet
D away from the magnet

38 The following diagram shows three electrical generators.

X Y Z

Which generator(s) provide an alternating voltage?


A X only B Y only C X and Y only D X and Z only

16

www.KiasuExamPaper.com
523
39 The circuit below shows a resistor R connected to a 20 V d.c. power supply through a
transformer of turns ratio 1 : 10.

20 V d.c. R

power supply
1 : 10

What is the voltage across R after some time?


A zero B 20 V a.c C 200 V d.c D 200 V a.c

40 An alternating supply with a period of 0.040 s is connected to a cathode-ray oscilloscope (c.r.o).

What is the time-base setting of the c.r.o?


A 0.4 ms / cm B 1 ms / cm C 4 ms / cm D 10 ms / cm

–– End of paper ––

17

www.KiasuExamPaper.com
524
Name: ______ ( ) Class: ___

WHITLEY SECONDARY SCHOOL


A Caring and Learning Community
Discipline * Integrity * Respect * Responsibility

PRELIMINARY EXAMINATION 2019


SUBJECT : PHYSICS Paper 2 (6091/02)

LEVEL : Sec 4 Express

DATE : 29 Aug 2019 (Thur)

DURATION : 1 hr 45 mins

TOTAL MARKS: 80

INSTRUCTION TO CANDIDATES

Do not open this booklet until you are told to do so.


Write your name, class and index number in the spaces at the top of this page.
Write in dark blue or black pen in the spaces provided on the Question Paper.
You may use a soft pencil for any diagrams, graphs or rough working.
Do not use staples, paper clips, highlighters, glue or correction fluid/tape.

Section A [50 marks]


Answer all questions. Write your answers in the spaces provided on the question paper.

Section B [30 marks]


Answer all questions. Question 10 has a choice of parts to answer.
Write your answers in the spaces on the question paper.

Candidates are reminded that all quantitative answers should include appropriate units.
Candidates are advised to show all their working in a clear and orderly manner, as more marks are
awarded for sound use of physics than for correct answers.

At the end of the examination, fasten separate writing For Examiner’s Use
papers (if any) securely to the Question Paper.
The number of marks is given in brackets [ ] at the end Section A / 50
of each question or part question.
Section B / 30
Take acceleration due to gravity, g as 10 m / s .
2
Total / 80

The document consists of 19 printed pages including this cover page.

www.KiasuExamPaper.com
525
Section A [50 marks]
Answer all questions in this section in the spaces provided.

1 A ball is given a push to start it rolling freely up a slope. The velocity-time graph in Fig. 1.1
shows the change in the velocity of the ball with time.

Fig. 1.1

(a) State the time when the ball reaches the highest point on the slope.

time = …….………………….. [1]

(b) Determine the acceleration of the ball at the highest point on the slope.

acceleration = …….………………….. [2]

(c) Determine the average speed of the ball between 2.0 and 12.0 s.

average speed = …….………………….. [2]

www.KiasuExamPaper.com
526
(d) On Fig. 1.2, sketch the displacement-time graph of the ball between 0.0 s to 12.0 s. [2]

Fig. 1.2

2 A manhole cover is a plate to cover the opening of a manhole or a hole on the ground leading
to a sewer. A manhole cover is typically round in shape as shown in Fig. 2.1.

Fig. 2.1

Fig. 2.2 shows the side view of a manhole cover hinged at X and with a handle at Y.

Fig. 2.2

(a) On Fig. 2.2, draw and identify all the forces acting on the manhole cover. [2]

www.KiasuExamPaper.com
527
(b) The manhole cover has a weight of 45.0 N and the centre of gravity is 30.0 cm from the
hinge at X. The handle is 55.0 cm from X.

(i) Explain the term the moment of a force about a point.

…..…..….…………………..………...…...…………………………………….…………...

…..…..….…………………..………...…...…………………………………….…………... [1]

(ii) A pulling force is applied at Y to lift the manhole cover.

Explain why it is easier to lift the manhole cover if the pulling force at Y is normal to
the manhole cover.

…..…..….…………………..………...…...…………………………………….…………...

…..…..….…………………..………...…...…………………………………….…………...

…..…..….…………………..………...…...…………………………………….…………... [2]

(iii) Calculate the minimum pulling force applied at Y to lift manhole cover.

minimum pulling force = …….………………….. [2]

www.KiasuExamPaper.com
528
3 Fig. 3.1 shows a set of traffic lights supported by two cables, A and B, which hang from a pole.
The set of traffic lights is in equilibrium.

Fig. 3.1

The weight of the set of traffic lights is 350 N. The weight of the cables and pole is negligible.
The tensions in the cables A and B are T1 and T2 respectively.

In the space below, draw a labelled vector diagram to show the resultant of the two
tensions. State the scale used and determine the magnitudes of T1 and T2. [2]

scale = …….………………….. [1]

T1 = …….………………….. [1]

T2 = …….………………….. [1]

www.KiasuExamPaper.com
529
4 Fig. 4.1 shows the plan of a bedroom and part of the main room of a house and their
respective temperatures. Other rooms are not shown.

Fig. 4.1

Fig. 4.2 shows all the thermal energy inputs to the bedroom in one hour.

thermal energy input to bedroom energy / J

through the door and walls from main room 4.5 ×104

through the walls from outside of house 2.3 × 106

through the window 1.1 × 106

from the person sleeping in bedroom 2.0 ×105

Fig. 4.2

(a) Explain why more thermal energy enters the bedroom from the outside of the house than
from the main room.

…..…..….…………………..………...…...…………………………………….………………..

…..…..….…………………..………...…...……………………………………….…………….. [1]

(b) An air conditioner keeps the temperature constant in the bedroom by removing thermal
energy.

(i) Identify a suitable location of the air conditioner in the bedroom for maximum
efficiency.

…..…..….…………………..………...…...…………………………………….…………... [1]

(ii) Explain how the location in (b)(i) cools the bedroom efficiently.

…..…..….…………………..………...…...…………………………………….…………...

…..…..….…………………..………...…...……………………………………….………...

…..…..….…………………..………...…...…………………………………….…………...

…..…..….…………………..………...…...……………………………………….………...

…..…..….…………………..………...…...……………………………………….………... [3]

www.KiasuExamPaper.com
530
(iii) A person sleeping in the bedroom.

Calculate the power of the air conditioner required to keep the temperature in the
bedroom constant.

power = …….………………….. [2]

(iv) State an assumption made in the calculation for (b)(iii).

…..…..….…………………..………...…...…………………………………….…………...

…..…..….…………………..………...…...……………………………………….………... [1]

www.KiasuExamPaper.com
531
5 An experiment is conducted to determine the specific latent heat of fusion of ice.

Fig. 5.1 shows two set-ups in the same room. The immersion heater in setup 1 is connected to
a 12 V power supply and the current is 10.0 A. The heater in setup 2, which serves as a control
in the experiment, is not connected to any power supply.

12 V control
power (no power)
immersion
supply
heater

crushed ice

funnel

beaker

melted ice

setup 1 setup 2
Fig. 5.1

The immersion heater in setup 1 is switched on until water flows at a steady rate from the
funnel, for a duration of 5.0 minutes.

Fig. 5.2 shows the data collected from the experiment after 5.0 minutes.

setup 1 setup 2
mass of empty beaker / g 60 60
mass of beaker with melted ice / g 192 85
mass of melted ice / g

Fig. 5.2

(a) Define specific latent heat of fusion.

…..…..….…………………..………...…...…………………………………….………………..

…..…..….…………………..………...…...……………………………………….…………….. [1]

(b) Fill in the blanks for Fig. 5.2. [1]

(c) Setup 2 is known as a control set.

Explain the purpose of having a control set in this experiment.

…..…..….…………………..………...…...…………………………………….………………..

…..…..….…………………..………...…...……………………………………….…………….. [1]

www.KiasuExamPaper.com
532
(d) Calculate the heat energy provided by the immersion heater for 5.0 minutes.

heat energy = …….………………….. [2]

(e) Calculate the value of the specific latent heat of fusion of ice.

specific latent heat of fusion = …….………………….. [3]

(f) The actual value of specific latent head of fusion of ice is smaller than that calculated in (e).

Suggest and explain why.

…..…..….…………………..………...…...…………………………………….………………..

…..…..….…………………..………...…...……………………………………….…………….. [1]

www.KiasuExamPaper.com
533
6 A plastic rod is initially electrically neutral. It is rubbed with a cloth and becomes positively
charged. After charging, the rod is held close to a suspended table-tennis ball shown in
Fig. 6.1. The table-tennis ball is covered with metal paint and is initially uncharged.

Fig. 6.1

(a) Describe what happens to the charges on the electrically neutral plastic rod when it is
rubbed with a cloth.

…..…..….…………………..………...…...…………………………………….………………..

…..…..….…………………..………...…...……………………………………….…………….. [1]

(b) Describe what happens to the charges on the metal-painted table-tennis ball as the
positively-charged rod is brought close to the ball.

…..…..….…………………..………...…...…………………………………….………………..

…..…..….…………………..………...…...……………………………………….…………….. [1]

(c) The ball swings towards the positively charged rod.

Explain why this happens.

…..…..….…………………..………...…...…………………………………….………………..

…..…..….…………………..………...…...……………………………………….……………..

…..…..….…………………..………...…...……………………………………….…………….. [2]

(d) When it is a few centimetres away from the rod, the ball is briefly touched by a wire
connected to earth.

In terms of the movement of charges, describe what happens to the charge on the ball.

…..…..….…………………..………...…...…………………………………….………………..

…..…..….…………………..………...…...……………………………………….……………..

…..…..….…………………..………...…...……………………………………….…………….. [2]

10

www.KiasuExamPaper.com
534
7 A 600 Ω resistor and a thermistor are connected in series with an ammeter and a power supply
of 20 V d.c. (direct current). A voltmeter is in parallel with the resistor.

Fig. 7.1 shows the circuit diagram.

Fig. 7.1

The ammeter reads 0.025 A.

(a) Calculate the reading on the voltmeter.

voltmeter reading = …….………………….. [2]

(b) Calculate the resistance of the thermistor.

resistance = …….………………….. [2]

(c) Temperature of the thermistor increases.

(i) State and explain what happens to the ammeter reading.

…..…..….…………………..………...…...……………………………………….………...

…..…..….…………………..………...…...……………………………………….………...

…..…..….…………………..………...…...……………………………………….………... [2]

(ii) State and explain any change to the voltmeter reading.

…..…..….…………………..………...…...…………………………………….…………...

…..…..….…………………..………...…...……………………………………….………...

…..…..….…………………..………...…...……………………………………….………... [2]

11

www.KiasuExamPaper.com
535
Section B [30 marks]
Answer all questions in this section in the spaces provided.
Question 10 has a choice of parts to answer.

8 Fig. 8.1 shows a cylindrical tank with two taps at P and Q. The tank, which contains oil, is
resting on a horizontal surface. An empty horizontal tube is attached to tap P and an empty
U-tube is attached to the tap Q. The other ends of the tubes are open. Both taps are initially
turned off. Taps P and Q may be replaced with steel, copper or aluminium taps.

vacuum

oil surface level


Q
oil

Fig. 8.1

Fig. 8.2 consists of information related to the operation of the taps.

atmospheric pressure 1 x 105 Pa


density of oil 800 kg/m 3
gravitational field strength 10 N/kg
height difference between tap P and tap Q 20 m
height difference between oil surface level and tap Q 15 m
base area of tank 18 m2
cross-sectional area of horizontal tube 0.05 m2
cross-sectional area of U-tube 0.03 m2
cross-sectional area of steel tap replacement 0.02 m2
cross-sectional area of copper tap replacement 0.03 m2
cross-sectional area of aluminium tap replacement 0.04 m2
Fig. 8.2

(a) Calculate the oil pressure acting on tap Q.

oil pressure = …….………………….. [2]

12

www.KiasuExamPaper.com
536
(b) Tap Q is a copper tap.

Calculate the net force acting on tap Q when it is turned on.

net force = …….………………….. [3]

(c) When only tap P is turned on, the oil starts to flow into the horizontal tube.

Suggest why the rate of flow of oil into the horizontal tube is not constant as the level of oil
falls in the tank.

…..…..….…………………..………...…...…………………………………….………………..

…..…..….…………………..………...…...…………………………………….………………..

…..…..….…………………..………...…...……………………………………….……………..

…..…..….…………………..………...…...……………………………………….……………..

…..…..….…………………..………...…...……………………………………….…………….. [3]

(d) State whether steel tap, copper tap or aluminium tap should be installed at P for greater
rate of flow of oil. Explain your answer.

…..…..….…………………..………...…...……………………………………….……………..

…..…..….…………………..………...…...……………………………………….……………..

…..…..….…………………..………...…...……………………………………….…………….. [2]

13

www.KiasuExamPaper.com
537
9 Fig. 9.1 shows a ray of light incident on an interface of air and corn oil at an angle, i equals to
35°. The ray is transmitted through parallel layers of corn oil and glycerol and is then reflected
at the surface of a plane mirror, located below and parallel to the glycerol layer. The ray then
emerges from the corn oil back into the air. The refractive index of corn oil is 1.48.
normal

i
air

corn oil

glycerol

plane mirror

Fig. 9.1

(a) Calculate the angle of refraction of the light ray when it travels from air to corn oil.

angle of refraction = …….………………….. [2]

(b) Explain why the light ray did not bend at the corn oil and glycerol interface.

…..…..….…………………..………...…...…………………………………….………………..

…..…..….…………………..………...…...…………………………………….………………..

…..…..….…………………..………...…...……………………………………….…………….. [2]

(c) Calculate the critical angle of light in the corn oil.

critical angle = …….………………….. [2]

14

www.KiasuExamPaper.com
538
(d) Explain why the reflected ray from the mirror will not undergo total internal reflection at the
corn oil and air interface, regardless of the values of i.

…..…..….…………………..………...…...…………………………………….………………..

…..…..….…………………..………...…...…………………………………….………………..

…..…..….…………………..………...…...……………………………………….…………….. [2]

(e) Complete the ray diagram in Fig. 9.1 to show the path of the refracted light ray until it [2]
returns to air.

15

www.KiasuExamPaper.com
539
EITHER
10 Fig. 10.1 shows a d.c. motor that is designed to rotate anti-clockwise. A rheostat is used in the
circuit to adjust the motor speed.

permanent magnet permanent magnet

Fig. 10.1

(a) Label the polarities of the permanent magnet in the 2 boxes provided in Fig. 10.1. [1]

(b) Name the component C and states its function.

…..…..….…………………..………...…...…………………………………….………………..

…..…..….…………………..………...…...…………………………………….………………..

…..…..….…………………..………...…...……………………………………….…………….. [2]

(c) Explain how the current causes the coil to rotate.

…..…..….…………………..………...…...…………………………………….………………..

…..…..….…………………..………...…...…………………………………….………………..

…..…..….…………………..………...…...…………………………………….………………..

…..…..….…………………..………...…...…………………………………….………………..

…..…..….…………………..………...…...……………………………………….…………….. [3]

16

www.KiasuExamPaper.com
540
(d) Fig. 10.2 shows the rheostat that is connected to the d.c. motor.
The sliding contact is shifted to the right towards terminal B.

sliding contact
terminal B

terminal A

Fig. 10.2

State the effect of shifting the sliding contact to the right on the speed of the d.c. motor.
Explain your answer.

…..…..….…………………..………...…...……………...………………………….…………...

…..…..….…………………..………...…...……………...………………………….…………...

…..…..….…………………..………...…...……………...………………………….…………...

…..…..….…………………..………...…...……………...………………………….…………...

…..…..….…………………..………...…...……………...………………………….…………...

…..…..….…………………..………...…...……………...………………………….…………...

…..…..….…………………..………...…...……………...………………………….…………... [4]

17

www.KiasuExamPaper.com
541
OR
10 Fig. 10.3 shows the compact cassette which is widely used to record and playback audio from
the 1960s to the 1990s.

Fig. 10.3

When the cassette is inserted into the audio recorder, the recording head is positioned at the
cassette opening.

During recording, as shown in Fig. 10.4, an audio signal is sent to the recording head in the
form of an electric current which changes direction.

Fig. 10.4

The arrows on the tape represents the direction of magnetisation where the arrow head
represents North Pole and the arrow tail represent South Pole.

(a) From the direction of current shown in Fig. 10.4, deduce the direction in which the
tape at A will be magnetised.

Draw an arrow in the box given in Fig. 10.4 to represent this direction at A. [1]

(b) (i) State a difference between magnetic materials that form temporary magnets and
permanent magnets.

…..…..….…………………..………...…...…………………………………….…………...

…..…..….…………………..………...…...…………………………………….…………...

…..…..….…………………..………...…...……………………………………….………... [2]

(ii) Deduce the type of magnetic material used in the tape.

…..…..….…………………..………...…...…………………………………….…………... [1]

18

www.KiasuExamPaper.com
542
(c) When playing back the tape, the same recording head is used to read the tape. As shown
in Fig. 10.5, when the tape moves over the recording head, an electrical signal is produced
in the coil of wire.

Fig. 10.5

An audio signal is then transmitted from the coil of wires to the speakers in the form of a
current. This audio signal matches the audio signal initially used for recording.

Explain why there is an electrical signal being produced.

…..…..….…………………..………...…...…………………………...…………….…………...

…..…..….…………………..………...…...…………………………...…………….…………...

…..…..….…………………..………...…...…………………………...…………….…………...

…..…..….…………………..………...…...…………………………...…………….…………... [2]

(d) The cassette player also comes with an erase function which activates the recording head
to erase the recorded audio on the tape.

Suggest and explain how the recording head achieves this function.

…….....…..….…………………..………...…...…………………………………….…………...

…..…..….…………………..………...…...……...………………………………….…………...

…..…..….…………………..………...…...………………...……………………….…………...

…..…..….…………………..………...…...………………...……………………….…………... [2]

(e) If a cassette is not properly stored, the recorded audio on the tape will gradually be lost
over time.

State two reasons why this happens.

1 …….....…..….…………………..………...…...…………………………………….……...

…..…..….…………………..………...…...……...………………………………….…………...

2 …..…..….…………………..………...…...………………...……………………….……...

…..…..….…………………..………...…...………………...……………………….…………... [2]

– End of Paper –

19

www.KiasuExamPaper.com
543
www.KiasuExamPaper.com
544
Whitley Secondary School
Marking Scheme

Assessment: 2019 Prelim


Level and Paper: 4E Pure Phy

Paper 1

1 C 11 B 21 C 31 B
2 C 12 A 22 B 32 D
3 D 13 B 23 A 3
333 C
4 B 14 A 24
24 B 3
34 A
5 D 15 D 25
25 D 35 A
6 D 16
16 B 26
26 C 36
3 B
7 C 17
17 D 27
27 D 37
37 A
8 B 18
8 A 28
28 B 3
38 C
9 B 19
19 D 29
29 D 39 A
10 D 20
2 0 C 30 A 40 D

www.KiasuExamPaper.com
545
Paper 2

Deduct 1 mark for the following errors:


 Wrong / missing units
 Numerical ans not expressed in 3 s.f
 Answers expressed in fractions

Maximum of 2 marks deduction in a paper (due to any error above).

Section A

Q No. Answers Marks


1 (a) 6.0 s 1
(b) 0  15.0 1
acc 
6.0  0
 2.5 m / s 2 1
(c) 1 1
Total distance travelled  10  4.0   15  6.0 
2 2
 65 m
1

65
Average speed 
10.0
 6.5 m / s 1
(d)

 Correct sshape
ct shhape wiwith max
th m displacement at t = 6.0 s
ax dis 1
 Decreasing
singg gradient
gra dient from
rad fr t = 0.0 s to t = 6.0 s and increasing gradient from
t = 6.0 s to t = 12
12.0 s 1

Q No. Answers Marks


2 (a)

www.KiasuExamPaper.com
546
3 correct forces – B2
1 or 2 correct forces – B1
0 correct force – B0 1+1
(b) (i) The product of the force and its perpendicular distance from the pivot to the line 1
of action of the force
(ii) The perpendicular distance from the pivot to the line of action of the force is the 1
longest when the pulling force at Y is normal to the manhole cover
Force required to apply to the manhole cover will hence be the smallest, in 1
order to produce the same anti-clockwise moment as the clockwise moment
due to the weight of the manhole cover
(iii) CW moment  ACW moment
30.0  45.0  55.0  F 1
F  24.54545
1
 24.5 N upward

Q No. Answers Marks


3 Correctly drawn parallelogram method 1+1
- Solid lines with arrows and labels for forces
- Dotted lines for construction
- Double-headed arrow for resultant forcerce
- ording to
Length of arrows drawn for according to stated
ed
d sscale
cale
ca e
- Correct measurement of anglengle between
en the
the
e fforces
orce
orces

Suitable scale - 1 cm : 50 N o
orr 1 cm
cm : 25 N – B
B1
1 1
T1 = 275 N (260 N ~ 290 N)) 1
T2 = 175 N (160
60 N ~ 1
190
9 N))
90 1

Q No. An nsswwerrs
Answers Marks
4 (a) Th
T her
Theree e is gr rea
eate
a er tte
greater emperrat aturre d
temperature di
iff
fferen
ere
erennce
difference ce of 1
14 C between
4 C b twe outside the house
be 1
and
a
an nd the
the bedroom
th b dr
be droo
oom than
oo th
haan
n that
th
haat between
between n the
th
he ma
mai roo
main room and the bedroom which is a
ttemperature
te
emperat attu
a urre difference
diiffffe
erren
ence eo off 4 C / ThThe eg greater
reate the temperature difference, the
faster
fa err the
asstte the rrate
he ate of
at of ttransfer
ra
annssfer off tthermal
herm
he rmal en
rm energy
(b) (i) It neeeed
ee
needs dss tto
o be
be placed
placed d at tthe
he top of the bedroom 1
(ii) Ass thehee air
air around
aroun nd thethe air conditioner
co cools, it contracts and becomes denser 1
and d sinks to tthe he bottom
bottom of the room

armer air,
The warmer ai being less dense, rises to the top of the room to be cooled by 1
the air co
conditioner

A convection current is created from top to bottom of room which helps to cool 1
the room efficiently
(iii) total thermal energy  4.5  104  2.3  106  1.1 106  2.0  105 1
 3 645 000 J

E
P
t
3 645 000

60  60
 1012.5
 1010 W 1

www.KiasuExamPaper.com
547
(iv) Any reasonable assumption: 1
 There is no thermal energy entering or leaving the room other than what is
stated.
 The window and door is kept closed throughout.
 The temperature outside the bedroom remains as stated.

Q No. Answers Marks


5 (a) The amount of thermal energy required to change unit mass (1 kg) of the substance 1
from solid state to liquid state, without a change in temperature
(b)

1
(c) m
Heat energy supplied by the surroundings to melt the ice can be determined / 1
temperature changes in the environment affect both setups in the same wway
(d) E  IVt 1
 10 12  5  60s 
 36 000 J 1
(e) Mass of melted ice due to power supply  132  25 1
 10
107
1 07 g

E  ml
36 000  107  l 
l  336.44859
44859
9
 336
6J/g 1

6 J / g  336
336 336 000
000 J / kg
k 1
(f) Thee ma
m asss o
mass off melt ltted
melted ed ice
ce due
ce e tto
o th
he po
the owwe
er su
power upp
pply
supplyly sshould
hould be higher than 107 g. There
ho 1
was additional
a di
ad d ttiion
onaall transfer
tra
ransnsfer ooff tthermal
he
h ermmal
al energyy fr
from
om setup 1 to setup 2, causing more ice
to mel
meltlt in
n setup p2/M asss o
a
Mass off m ellted ic
melted ce in ssetup
ice ettup 1 should be more

Q No. Answers rs Marks


6 (a) The electrons
tronns from
om the
the plastic rod are transferred to the cloth. Hence there are now 1
more electrons
tronss tthan
han
han prot
protons, the rod therefore becomes positively charged
(b) The negative e charges
charge (electrons) move towards the rod, leaving positive charges on 1
the left
(c) The negative charges in the ball are attracted to the positively charged rod 1
The forces of attraction between the unlike charges are stronger than the forces of 1
repulsion between like charges, hence the ball swings towards the charged rod due
to the net force to the right
(d) Negative charges (electrons) flow up from earth to the ball through the wire to 1
neutralise the induced positive charges
The ball becomes negatively charged 1

www.KiasuExamPaper.com
548
Q No. Answers Marks
7 (a) V  IR
  0.025  600  1
 15 V 1
(b) V  IR
20  15   0.025  R  1
R  200  1
(c) (i) Ammeter reading increases 1
Resistance of thermistor decreases when its temperature increases which 1
decreases the overall effective resistance of the circuit, hence current
increases
(ii) voltmeter reading increases 1
Resistance of thermistor decreases when its temperature increases hence its 1
potential difference decreases and the potential difference off fixed resistor
increases

www.KiasuExamPaper.com
549
Section B

Q No. Answers Marks


8 (a) P  h g
 15  800 10  1
 120 000 Pa 1
(b) Atmospheric pressure = 100 000 Pa

Net pressure acted at tap Q  120 000  100 000 1


 20 000 Pa

F
P
A
F
20 000  1
0.03
F  600 N 1
(c) Pressure due to the oil depends on the height of the oil column ab bove the
above th tap
th ta P 1
As level of oil falls in the tank decreases, the pressure re due to the
the ooilil de
dec
decreases
crease 1
The difference in pressure between oil and the atmospher
atmosphereere
er e at P de
d
decreases
cre
creases 1
Hence rate of flow of oil decreases
(d) Aluminium tap 1
The larger the cross-sectional area rea of the tap,
tap, the
the
e larger
larg
la rger
er is
is the
the
he fforce
orce
ea ppliled
pp
applied 1
F
(due to P  )
A

Q No. Answers s Marks


9 (a) sin
si in i
n
ssin
si nr
ssin35
si n 35
8
1.48
48 1
ssiin r
sin
r  22
22.80224
2.8022424
2 4
 22
22.8
2 2.8
8o 1
(b) Corn oil and
ndd glycerol
gly
lyce
cero
ol have
ve the
the same
s me refractive index
sa 1
There iss noo change
cha
hange in n the
the speed of light ray as it travels from corn oil to glycerol 1
(c) 1
sin c 
n
1
c  sin1 1
1.48
 42.50664
 42.5 o 1
(d) Angle of incidence will not be greater than critical angle / angle of incidence will be 1
smaller than critical angle
The maximum angle of incidence at corn oil-air interface is equal to the maximum 1
angle of refraction at air-corn oil interface which has a maximum value of 42.5°

www.KiasuExamPaper.com
550
(e)

 correct reflected ray at mirror 1


(i = r with no bending at glycerol-corn oil interface)
 correct refracted ray
(r for ray leaving corn oil is 35°) 1

Q No. Answers Marks


10E (a) 1

(b) Split ring commutator utator 1


Function: Changeshange es the
tth
he dire ect
ctiio
direction on of
of thethe
he ccurrent
urrrre
u ent n flo
ow in the coil
flow coil ever
eryy ha
er
every halflf a revolution
revo 1
so thatt coil cancan
a rotate rottatte continuously
co
ontn inuouo ous
u lyy
(c) Currentt inin ccoil
oil prp od
produces ducces es a magnetic
maag gneti
ne
n ettiic field
fiel
elld 1
This
Thhis field
fie
eld
d inte
interacts
erar ct
c s wi w
with
ith
t the permanent
per
erma
ermamane nentnt magnetic
magagnetic fieldfiel
eld
eld to produce
produc a force at the side 1
of ccoil
ol
oi
The
Thhe forces
forc
fo ess at
rce at twtwo
wo ssides id
de es a are
arre acting
accti
a ting
n in opp opposite
pos
osititite directions hence produces a moment
e di
directio 1
about
abou
ouut the
the axle
axle
ax le toto rotate
ro
ota
tate e ccoil
oil
oi
oil
(d) Ass thee sslider
lilde
er shifts
shifififttss to
to the
tth
he rirright,
g t, ccurrent
gh urre
ur rent
re nt h
nt hasas to flow through longer section of the 1
resistance
stan nce
ce wire
wiiire
w rre
e
resistance
tancece in
ce increases
incr
crreea ases an andnd cu current
currrent dedecreases 1
The forcerce onon thethe
h sides siddes of of coil due d to the current and magnetic field decrease 1
Hence speedpeed of rotation rot
otat
ation decreases
at de 1

Q No. Answers Marks


10O (a) 1

(b) (i) Soft magnetic materials form temporary magnets while hard magnetic 1
materials form permanent magnets
Soft magnetic materials are easily magnetised and demagnetised while hard 1
magnetic materials are hard to be magnetised and demagnetised

www.KiasuExamPaper.com
551
(ii) Hard magnetic material 1
(c) As the magnetised tape approaches the recording head, there is a change 1
in magnetic flux linking through the magnetic core to the coil of wire
By Faraday’s Law, an emf is induced in the coil of wire which is proportional 1
to the rate of change of the magnetic flux (magnetic field lines linking the
coil)
Hence an electrical signal in the form of an induced current is produced
(d) The cassette player sends a strong alternating current to the coil of wire 1
This produces a strong alternating magnetic field which causes the 1
magnetisation on the tape to be disrupted as the tape passes the recording
head
OR OR
The cassette player sends a strong direct current to the coil of wire 1
This produces a strong magnetic field which causes the magnetisation on 1
the tape to be reset to a single direction as the tape passes the recording
head
(e)  The cassette is exposed to heat causing the tape inside de to be 1+1
demagnetised
 The cassette has been dropped / subjected to physic physical cal impact
mpact causing
imp
mp
the tape inside to be demagnetised
 Over time, Earth’s magnetic field causes
auses the d irec
irectitition
ec
direction o of
on of magnetisation
magnetis
to change
 Different parts of the tape with differentt d direction
ire
irec
ireccti
tion
on o
on off mamagnetisation
agn
gnet
etis
et isa
is ation will
wi
affect one another, causing the
ausing th directions
he di
d re
eccttions tto
ons o be
be a altered
ltered ed

Any two

En
E nd of
– End of P ap
pe
err –
Paper

www.KiasuExamPaper.com
552
www.KiasuExamPaper.com
553

You might also like